From neoajg en gmail.com Mon Oct 1 06:51:12 2012 From: neoajg en gmail.com (aj gutierrez) Date: Sun, 30 Sep 2012 23:51:12 -0500 Subject: [Python-es] [python-es] | conectar python con vfp Message-ID: Buenas noches a todos, Saben si existe o se puede conectar python a una base de datos en visual fox pro? ------------ próxima parte ------------ Se ha borrado un adjunto en formato HTML... URL: From clatorreg en dphuesca.es Mon Oct 1 09:06:31 2012 From: clatorreg en dphuesca.es (Carlos Latorre) Date: Mon, 1 Oct 2012 09:06:31 +0200 Subject: [Python-es] [python-es] | conectar python con vfp In-Reply-To: References: Message-ID: Fox Pro utiliza bases las antiguas bases de datos dBase ( http://es.wikipedia.org/wiki/DBase), actualmente conocidas como xBase ( http://es.wikipedia.org/wiki/XBase). Yo trabaje con ellas mucho en el pasado, utilizando Clipper, pero de eso hace ya casi 20 años. Tras una mirada rápida por S. Google aparecen un par de librerías en el sourceforge: - dbfpy (http://dbfpy.sourceforge.net/ ) - xbase-py (http://sourceforge.net/projects/xbase-py/) Espero que te sea de ayuda para comenzar, Saludos: Carlos El 1 de octubre de 2012 06:51, aj gutierrez escribió: > Buenas noches a todos, > > Saben si existe o se puede conectar python a una base de datos en visual > fox pro? > > > > > > _______________________________________________ > Python-es mailing list > Python-es en python.org > http://mail.python.org/mailman/listinfo/python-es > FAQ: http://python-es-faq.wikidot.com/ > > -- \|/ (QQ) ( O ) ==oo=======oo============================================================ Carlos Latorre Galicia mail:clatorreg en dphuesca.es Tecnico de Sistemas Telefono: +34 974 294 110 Diputacion Provincial de Huesca http://www.dphuesca.es/ Porches de Galicia, 4 Telefono: +34 974 294 100 22002 Huesca Fax: +34 974 294 149 ========================================================================= ------------ próxima parte ------------ Se ha borrado un adjunto en formato HTML... URL: From clatorreg en dphuesca.es Mon Oct 1 09:07:19 2012 From: clatorreg en dphuesca.es (Carlos Latorre) Date: Mon, 1 Oct 2012 09:07:19 +0200 Subject: [Python-es] [python-es] | conectar python con vfp In-Reply-To: References: Message-ID: Por si no estas familiarizado con las bbdd dBase: Los datos están distribuidos en ficheros de la siguiente forma: - *.dbf - los ficheros de datos (los importantes) - *.dbt - contenido de los campos "memo" (estos ficheros, de extensión dbt y el mismo nombre que el pdf son muy importantes, pues sin ellos no se abre correctamente el .pdf, guardan el contenido de los campos "memo", equivalente al text de sql) - *.ndx ó *.ntx - ficheros de indice (se pueden reconstruir a partir de los datos) - tienen una importancia relativa, pues deben abrirse y utilizarse de forma explicita, por lo que, si no conoces la aplicación de antemano, tal vez sea mejor que crees y utilices los que vayas necesitando. Además puede haber mas ficheros, propios de foxpro, conteniendo formularios, vistas, etc, ... pero estos son muy dependientes. Saludos, Carlos L. El 1 de octubre de 2012 08:54, Carlos Latorre escribió: Fox Pro utiliza bases las antiguas bases de datos dBase ( > http://es.wikipedia.org/wiki/DBase), actualmente conocidas como xBase ( > http://es.wikipedia.org/wiki/XBase). > Yo trabaje con ellas mucho en el pasado, utilizando Clipper, pero de eso > hace ya casi 20 años. > Tras una mirada rápida por S. Google aparecen un par de librerías en el > sourceforge: > - dbfpy (http://dbfpy.sourceforge.net/ ) > - xbase-py (http://sourceforge.net/projects/xbase-py/) > > Espero que te sea de ayuda para comenzar, > Saludos: > Carlos > > El 1 de octubre de 2012 06:51, aj gutierrez escribió: > >> Buenas noches a todos, >> >> Saben si existe o se puede conectar python a una base de datos en visual >> fox pro? >> >> >> >> >> >> _______________________________________________ >> Python-es mailing list >> Python-es en python.org >> http://mail.python.org/mailman/listinfo/python-es >> FAQ: http://python-es-faq.wikidot.com/ >> >> > > > -- > \|/ > (QQ) > ( O ) > ==oo=======oo============================================================ > Carlos Latorre Galicia mail:clatorreg en dphuesca.es > Tecnico de Sistemas Telefono: +34 974 294 110 > > Diputacion Provincial de Huesca http://www.dphuesca.es/ > Porches de Galicia, 4 Telefono: +34 974 294 100 > 22002 Huesca Fax: +34 974 294 149 > ========================================================================= > > -- \|/ (QQ) ( O ) ==oo=======oo============================================================ Carlos Latorre Galicia mail:clatorreg en dphuesca.es Tecnico de Sistemas Telefono: +34 974 294 110 Diputacion Provincial de Huesca http://www.dphuesca.es/ Porches de Galicia, 4 Telefono: +34 974 294 100 22002 Huesca Fax: +34 974 294 149 ========================================================================= ------------ próxima parte ------------ Se ha borrado un adjunto en formato HTML... URL: From diego.uribe.gamez en gmail.com Mon Oct 1 18:57:57 2012 From: diego.uribe.gamez en gmail.com (Diego Uribe Gamez) Date: Mon, 1 Oct 2012 11:57:57 -0500 Subject: [Python-es] =?iso-8859-1?q?Como_expiro_la_sesi=F3n_del_usuario=3F?= In-Reply-To: References: Message-ID: Hola!!! El 29 de septiembre de 2012 18:52, Diego Uribe Gamez < diego.uribe.gamez en gmail.com> escribió: > Hola, > > El problema que tengo es que no se como hacer que la sesión del usuario > expire, se que tengo que usar una cookie pero el problema es que el login > de usuario es el default de django y es desde esta vista de login (la que > no hay en mis views.py porque es el login default del admin django con mi > template) que es donde tengo que setear la cookie: > > url(r'^login/$', 'django.contrib.auth.views.login'), > > busque en Internet y encontré que me toca colocar una SESSION_COOKIE_AGE? > esto se coloca en setings.py? como se coloca? SESSION_COOKIE_AGE = 1000? > > en la vista también seto una cookie en el response: > > response.set_cookie("time_now",datetime.now()) > > intentando entender solucionar el problema. > > pero cuando el usuario entra a esta vista esto ya tiene que haber sido > seteado desde el login? y verificado en la vista antes del response para > sacarlo si la cookie ya caduco? creo que esto no funciona? > > La verdad ya mire muchos de los ejemplos en Internet pero no entiendo ni > pio!! > > Necesito una ayuda! > > Saludos. > > -- > *Diego Alonso Uribe Gamez* > ------------------------------ > > *Desarrollador web* > > Twitter: @DiegoUG > > Google+: http://gplus.to/diegoug > ------------------------------ > > > -- *Diego Alonso Uribe Gamez* ------------------------------ *Desarrollador web* Twitter: @DiegoUG Google+: http://gplus.to/diegoug ------------------------------ ------------ próxima parte ------------ Se ha borrado un adjunto en formato HTML... URL: From jza en oooes.org Mon Oct 1 19:10:22 2012 From: jza en oooes.org (Alexandro Colorado) Date: Mon, 1 Oct 2012 12:10:22 -0500 Subject: [Python-es] =?utf-8?q?Como_expiro_la_sesi=C3=B3n_del_usuario=3F?= In-Reply-To: References: Message-ID: 2012/10/1 Diego Uribe Gamez > Hola!!! > Sera bueno que incluyas los vinculos a los ejemplos, que pegues tu codigo en pastebin y que incluyas el mensaje de error. > > El 29 de septiembre de 2012 18:52, Diego Uribe Gamez < > diego.uribe.gamez en gmail.com> escribió: > > Hola, >> >> El problema que tengo es que no se como hacer que la sesión del usuario >> expire, se que tengo que usar una cookie pero el problema es que el login >> de usuario es el default de django y es desde esta vista de login (la que >> no hay en mis views.py porque es el login default del admin django con mi >> template) que es donde tengo que setear la cookie: >> >> url(r'^login/$', 'django.contrib.auth.views.login'), >> >> busque en Internet y encontré que me toca colocar una SESSION_COOKIE_AGE? >> esto se coloca en setings.py? como se coloca? SESSION_COOKIE_AGE = 1000? >> >> en la vista también seto una cookie en el response: >> >> response.set_cookie("time_now",datetime.now()) >> >> intentando entender solucionar el problema. >> >> pero cuando el usuario entra a esta vista esto ya tiene que haber sido >> seteado desde el login? y verificado en la vista antes del response para >> sacarlo si la cookie ya caduco? creo que esto no funciona? >> >> La verdad ya mire muchos de los ejemplos en Internet pero no entiendo ni >> pio!! >> >> Necesito una ayuda! >> >> Saludos. >> >> -- >> *Diego Alonso Uribe Gamez* >> ------------------------------ >> >> *Desarrollador web* >> >> Twitter: @DiegoUG >> >> Google+: http://gplus.to/diegoug >> ------------------------------ >> >> >> > > > -- > *Diego Alonso Uribe Gamez* > ------------------------------ > > *Desarrollador web* > > Twitter: @DiegoUG > > Google+: http://gplus.to/diegoug > ------------------------------ > > > > _______________________________________________ > Python-es mailing list > Python-es en python.org > http://mail.python.org/mailman/listinfo/python-es > FAQ: http://python-es-faq.wikidot.com/ > > -- Alexandro Colorado PPMC Apache OpenOffice http://es.openoffice.org ------------ próxima parte ------------ Se ha borrado un adjunto en formato HTML... URL: From lgmerino en yahoo.com Mon Oct 1 19:13:36 2012 From: lgmerino en yahoo.com (Luis Garcia Merino) Date: Mon, 1 Oct 2012 18:13:36 +0100 (BST) Subject: [Python-es] =?iso-8859-1?q?Como_expiro_la_sesi=F3n_del_usuario=3F?= In-Reply-To: References: Message-ID: <1349111616.79642.YahooMailNeo@web171205.mail.ir2.yahoo.com> Hola, no me queda muy claro qué es exactamente lo que pretendes hacer, así que te doy varias opciones: - Si quieres que la sesión expire pasado cierto tiempo después de que el usuario hace login, inicializa la variable SESSION_COOKIE_AGE en el settings.py: SESSION_COOKIE_AGE = numero_de_segundos_que_quieres_mantener_viva_la_sesion - Si quieres que la sesión expire una vez que el usuario cierre el navegador, inicializa la variable SESSION_EXPIRE_AT_BROWSER_CLOSE en el settings.py: SESSION_EXPIRE_AT_BROWSER_CLOSE = True - Si quieres que el usuario pueda hacer logout: [https://docs.djangoproject.com/en/dev/topics/auth/#how-to-log-a-user-out] Espero que te sea útil. Un saludo,    Luis ________________________________ De: Diego Uribe Gamez Para: La lista de python en castellano Enviado: Lunes 1 de octubre de 2012 18:57 Asunto: Re: [Python-es] Como expiro la sesión del usuario? Hola!!! El 29 de septiembre de 2012 18:52, Diego Uribe Gamez escribió: Hola, > > >El problema que tengo es que no se como hacer que la sesión del usuario expire, se que tengo que usar una cookie pero el problema es que el login de usuario es el default de django y es desde esta vista de login (la que no hay en mis views.py porque es el login default del admin django con mi template) que es donde tengo que setear la cookie: > > >url(r'^login/$', 'django.contrib.auth.views.login'), > > >busque en Internet y encontré que me toca colocar una SESSION_COOKIE_AGE? esto se coloca en setings.py? como se coloca? SESSION_COOKIE_AGE = 1000? > > >en la vista también seto una cookie en el response: > > >response.set_cookie("time_now",datetime.now()) > > >intentando entender solucionar el problema. > > >pero cuando el usuario entra a esta vista esto ya tiene que haber sido seteado desde el login? y verificado en la vista antes del response para sacarlo si la cookie ya caduco? creo que esto no funciona? > > >La verdad ya mire muchos de los ejemplos en Internet pero no entiendo ni pio!! > > >Necesito una ayuda! > > >Saludos. >-- > > Diego Alonso Uribe Gamez >________________________________ > >Desarrollador web >Twitter: @DiegoUG >Google+: http://gplus.to/diegoug >________________________________ > > > --  Diego Alonso Uribe Gamez ________________________________ Desarrollador web Twitter: @DiegoUG Google+: http://gplus.to/diegoug ________________________________ _______________________________________________ Python-es mailing list Python-es en python.org http://mail.python.org/mailman/listinfo/python-es FAQ: http://python-es-faq.wikidot.com/ ------------ próxima parte ------------ Se ha borrado un adjunto en formato HTML... URL: From diego.uribe.gamez en gmail.com Mon Oct 1 19:23:11 2012 From: diego.uribe.gamez en gmail.com (Diego Uribe Gamez) Date: Mon, 1 Oct 2012 12:23:11 -0500 Subject: [Python-es] =?iso-8859-1?q?Como_expiro_la_sesi=F3n_del_usuario=3F?= In-Reply-To: <1349111616.79642.YahooMailNeo@web171205.mail.ir2.yahoo.com> References: <1349111616.79642.YahooMailNeo@web171205.mail.ir2.yahoo.com> Message-ID: La cuestión es que no es un mensaje de error, lo que pasa es que no se como hacerlo, o por donde comenzar? como dije antes: tengo el login default del admin django en mi pagina web url(r'^login/$', 'django.contrib.auth.views.login'), para activar el hecho de que se cierre la sesión después de algún tiempo: busque en Internet y encontré que me toca colocar una SESSION_COOKIE_AGE? esto se coloca en setings.py? como se coloca? SESSION_COOKIE_AGE = 1000? Como activo el tiempo en que caduca la cesión? como le doy un tiempo para que se cierre? es obligatorio usar la cookie? como la uso para cerrar la sesión del navegador después de un tiempo determinado? El 1 de octubre de 2012 12:13, Luis Garcia Merino escribió: > Hola, > > no me queda muy claro qué es exactamente lo que pretendes hacer, así que > te doy varias opciones: > > - Si quieres que la sesión expire pasado cierto tiempo después de que el > usuario hace login, inicializa la variable SESSION_COOKIE_AGE en el > settings.py: > > SESSION_COOKIE_AGE = numero_de_segundos_que_quieres_mantener_viva_la_sesion > > - Si quieres que la sesión expire una vez que el usuario cierre el > navegador, inicializa la variable SESSION_EXPIRE_AT_BROWSER_CLOSE en el > settings.py: > > SESSION_EXPIRE_AT_BROWSER_CLOSE = True > > - Si quieres que el usuario pueda hacer logout: [ > https://docs.djangoproject.com/en/dev/topics/auth/#how-to-log-a-user-out] > > Espero que te sea útil. > > Un saludo, > > Luis > > ------------------------------ > *De:* Diego Uribe Gamez > *Para:* La lista de python en castellano > *Enviado:* Lunes 1 de octubre de 2012 18:57 > *Asunto:* Re: [Python-es] Como expiro la sesión del usuario? > > Hola!!! > > El 29 de septiembre de 2012 18:52, Diego Uribe Gamez < > diego.uribe.gamez en gmail.com> escribió: > > Hola, > > El problema que tengo es que no se como hacer que la sesión del usuario > expire, se que tengo que usar una cookie pero el problema es que el login > de usuario es el default de django y es desde esta vista de login (la que > no hay en mis views.py porque es el login default del admin django con mi > template) que es donde tengo que setear la cookie: > > url(r'^login/$', 'django.contrib.auth.views.login'), > > busque en Internet y encontré que me toca colocar una SESSION_COOKIE_AGE? > esto se coloca en setings.py? como se coloca? SESSION_COOKIE_AGE = 1000? > > en la vista también seto una cookie en el response: > > response.set_cookie("time_now",datetime.now()) > > intentando entender solucionar el problema. > > pero cuando el usuario entra a esta vista esto ya tiene que haber sido > seteado desde el login? y verificado en la vista antes del response para > sacarlo si la cookie ya caduco? creo que esto no funciona? > > La verdad ya mire muchos de los ejemplos en Internet pero no entiendo ni > pio!! > > Necesito una ayuda! > > Saludos. > > -- > *Diego Alonso Uribe Gamez* > ------------------------------ > *Desarrollador web* > Twitter: @DiegoUG > Google+: http://gplus.to/diegoug > ------------------------------ > > > > > > -- > *Diego Alonso Uribe Gamez* > ------------------------------ > *Desarrollador web* > Twitter: @DiegoUG > Google+: http://gplus.to/diegoug > ------------------------------ > > > > _______________________________________________ > Python-es mailing list > Python-es en python.org > http://mail.python.org/mailman/listinfo/python-es > FAQ: http://python-es-faq.wikidot.com/ > > > > _______________________________________________ > Python-es mailing list > Python-es en python.org > http://mail.python.org/mailman/listinfo/python-es > FAQ: http://python-es-faq.wikidot.com/ > > -- *Diego Alonso Uribe Gamez* ------------------------------ *Desarrollador web* Twitter: @DiegoUG Google+: http://gplus.to/diegoug ------------------------------ ------------ próxima parte ------------ Se ha borrado un adjunto en formato HTML... URL: From lgmerino en yahoo.com Mon Oct 1 19:31:04 2012 From: lgmerino en yahoo.com (Luis Garcia Merino) Date: Mon, 1 Oct 2012 18:31:04 +0100 (BST) Subject: [Python-es] =?iso-8859-1?q?Como_expiro_la_sesi=F3n_del_usuario=3F?= In-Reply-To: References: <1349111616.79642.YahooMailNeo@web171205.mail.ir2.yahoo.com> Message-ID: <1349112664.96627.YahooMailNeo@web171202.mail.ir2.yahoo.com> Casi todas las preguntas que haces tienen respuesta en el correo que envié antes. Si quieres que la sesión tenga una vida de una hora añade a tu settings.py: SESSION_COOKIE_AGE = 3600 Django se encargará de que expire pasado ese tiempo, no tienes que hacer nada más. Si quieres saber más sobre sesiones échale un vistazo a [https://docs.djangoproject.com/en/dev/topics/http/sessions/] ________________________________ De: Diego Uribe Gamez Para: Luis Garcia Merino ; La lista de python en castellano Enviado: Lunes 1 de octubre de 2012 19:23 Asunto: Re: [Python-es] Como expiro la sesión del usuario? La cuestión es que no es un mensaje de error, lo que pasa es que no se como hacerlo, o por donde comenzar? como dije antes: tengo el login default del admin django en mi pagina web url(r'^login/$', 'django.contrib.auth.views.login'), para activar el hecho de que se cierre la sesión después de algún tiempo: busque en Internet y encontré que me toca colocar una SESSION_COOKIE_AGE? esto se coloca en setings.py? como se coloca? SESSION_COOKIE_AGE = 1000? Como activo el tiempo en que caduca la cesión? como le doy un tiempo para que se cierre? es obligatorio usar la cookie? como la uso para cerrar la sesión del navegador después de un tiempo determinado? El 1 de octubre de 2012 12:13, Luis Garcia Merino escribió: Hola, > > >no me queda muy claro qué es exactamente lo que pretendes hacer, así que te doy varias opciones: > > >- Si quieres que la sesión expire pasado cierto tiempo después de que el usuario hace login, inicializa la variable SESSION_COOKIE_AGE en el settings.py: > > >SESSION_COOKIE_AGE = numero_de_segundos_que_quieres_mantener_viva_la_sesion > > >- Si quieres que la sesión expire una vez que el usuario cierre el navegador, inicializa la variable SESSION_EXPIRE_AT_BROWSER_CLOSE en el settings.py: > > >SESSION_EXPIRE_AT_BROWSER_CLOSE = True > > > >- Si quieres que el usuario pueda hacer logout: [https://docs.djangoproject.com/en/dev/topics/auth/#how-to-log-a-user-out] > > >Espero que te sea útil. > > >Un saludo, > > >   Luis > > > >________________________________ > De: Diego Uribe Gamez >Para: La lista de python en castellano >Enviado: Lunes 1 de octubre de 2012 18:57 >Asunto: Re: [Python-es] Como expiro la sesión del usuario? > > > >Hola!!! > > >El 29 de septiembre de 2012 18:52, Diego Uribe Gamez escribió: > >Hola, >> >> >>El problema que tengo es que no se como hacer que la sesión del usuario expire, se que tengo que usar una cookie pero el problema es que el login de usuario es el default de django y es desde esta vista de login (la que no hay en mis views.py porque es el login default del admin django con mi template) que es donde tengo que setear la cookie: >> >> >>url(r'^login/$', 'django.contrib.auth.views.login'), >> >> >>busque en Internet y encontré que me toca colocar una SESSION_COOKIE_AGE? esto se coloca en setings.py? como se coloca? SESSION_COOKIE_AGE = 1000? >> >> >>en la vista también seto una cookie en el response: >> >> >>response.set_cookie("time_now",datetime.now()) >> >> >>intentando entender solucionar el problema. >> >> >>pero cuando el usuario entra a esta vista esto ya tiene que haber sido seteado desde el login? y verificado en la vista antes del response para sacarlo si la cookie ya caduco? creo que esto no funciona? >> >> >>La verdad ya mire muchos de los ejemplos en Internet pero no entiendo ni pio!! >> >> >>Necesito una ayuda! >> >> >>Saludos. >>-- >> >> Diego Alonso Uribe Gamez >>________________________________ >> >>Desarrollador web >>Twitter: @DiegoUG >>Google+: http://gplus.to/diegoug >>________________________________ >> >> >> > > > >-- > > Diego Alonso Uribe Gamez >________________________________ > >Desarrollador web >Twitter: @DiegoUG >Google+: http://gplus.to/diegoug >________________________________ > > > > >_______________________________________________ >Python-es mailing list >Python-es en python.org >http://mail.python.org/mailman/listinfo/python-es >FAQ: http://python-es-faq.wikidot.com/ > > > >_______________________________________________ >Python-es mailing list >Python-es en python.org >http://mail.python.org/mailman/listinfo/python-es >FAQ: http://python-es-faq.wikidot.com/ > > --  Diego Alonso Uribe Gamez ________________________________ Desarrollador web Twitter: @DiegoUG Google+: http://gplus.to/diegoug ________________________________ ------------ próxima parte ------------ Se ha borrado un adjunto en formato HTML... URL: From reroes799 en gmail.com Mon Oct 1 19:38:17 2012 From: reroes799 en gmail.com (=?ISO-8859-1?Q?Ren=E9_Rolando_Elizalde_Solano?=) Date: Mon, 1 Oct 2012 12:38:17 -0500 Subject: [Python-es] [python-es] | conectar python con vfp In-Reply-To: References: Message-ID: Lo que debes usar es pymssql para poder conectarte a la base en fox pro. funciona bien.. !! saludos El 30 de septiembre de 2012 23:51, aj gutierrez escribió: > Buenas noches a todos, > > Saben si existe o se puede conectar python a una base de datos en visual > fox pro? > > > > > > _______________________________________________ > Python-es mailing list > Python-es en python.org > http://mail.python.org/mailman/listinfo/python-es > FAQ: http://python-es-faq.wikidot.com/ > > -- René Rolando Elizalde Solano *Ingeniero en Sistemas* *Analísta - *Programador *PYTHON, HTML, CSS, JQUERY, SQLALCHEMY* * * *http://rrelizalde en utpl.edu.ec* http://reroes79.wordpress.com http://reroes799.blogspot.com/ http://twitter.com/reroes http://www.facebook.com/reroes799 reroes799 en gmail.com reroes79 en hotmail.com reroes79 en yahoo.es ------------ próxima parte ------------ Se ha borrado un adjunto en formato HTML... URL: From francesc en continuum.io Mon Oct 1 19:55:46 2012 From: francesc en continuum.io (Francesc Alted) Date: Mon, 01 Oct 2012 19:55:46 +0200 Subject: [Python-es] [ANN] PyData NYC 2012 Message-ID: <5069D922.2080408@continuum.io> Hola a todos, Todo el equipo de PyData en Nueva York y Continuum Analytics nos complacemos en anunciar la lista completa de charlas y ponentes de la conferencia PyData NYC 21012. Estamos seguros que la excelente lista de talleres, prácticas, tutoriales y charlas van a ser de gran utilidad para todos aquellos interesados en cómo usar Python para análisis de datos de manera cotidiana: http://nyc2012.pydata.org/schedule Asímismo, nos congratulamos de ser parte de la DataWeek de Nueva York: http://oreilly.com/dataweek/?cmp=tw-strata-ev-dr ¡Desde luego, la semana del 22 de Octubre va a ser muy movida en Nueva York! ¡También buscamos espónsors para este evento! Si quieres que tu empresa figure delante de centenares de hackers y desarrolladores 'duros' de Python, ésta es una excelente oportunidad para publicitar productos o reclutar grandes talentos. Por favor, visita: http://nyc2012.pydata.org/sponsors/becoming/ para saber como conseguir ser espónsor. ¡Por favor, ayúdanos a promocionar la conferencia! Díselo a amigos, envia mensajes a listas de interés en Python y/o análisis de datos, y seguidnos en @PyDataConf en Twitter. ¡El registro temprano acaba en un par de semanas, así que regístrate ahora! http://pydata.eventbrite.com/ La PyData pasada tuvo lugar en los cuarteles generales de Google en Mountain View, con un considerable éxito, agotándose el aforo en unas pocas horas. Con esta edición en la ciudad que nunca duerme pretendemos, no sólo continuar con el nivel alcanzado, sino superarlo, con unos locales con más aforo y con más calidad en los contenidos. Así que, ¡nos vemos en Nueva York! -- Francesc Alted Continuum Analytics From neoajg en gmail.com Tue Oct 2 09:55:22 2012 From: neoajg en gmail.com (aj gutierrez) Date: Tue, 2 Oct 2012 02:55:22 -0500 Subject: [Python-es] [python-es] | conectar python con vfp In-Reply-To: References: Message-ID: muchas gracias, estoy probando lo que me han comentado, download las librerias de source y la pymssql le comento los resultados De antemano muchas gracias 2012/10/1 René Rolando Elizalde Solano > Lo que debes usar es > pymssql > > para poder conectarte a la base en fox pro. > > funciona bien.. !! > > saludos > > El 30 de septiembre de 2012 23:51, aj gutierrez escribió: > >> Buenas noches a todos, >> >> Saben si existe o se puede conectar python a una base de datos en visual >> fox pro? >> >> >> >> >> >> _______________________________________________ >> Python-es mailing list >> Python-es en python.org >> http://mail.python.org/mailman/listinfo/python-es >> FAQ: http://python-es-faq.wikidot.com/ >> >> > > > -- > René Rolando Elizalde Solano > *Ingeniero en Sistemas* > *Analísta - *Programador > *PYTHON, HTML, CSS, JQUERY, SQLALCHEMY* > * > * > *http://rrelizalde en utpl.edu.ec* > http://reroes79.wordpress.com > http://reroes799.blogspot.com/ > http://twitter.com/reroes > http://www.facebook.com/reroes799 > reroes799 en gmail.com > reroes79 en hotmail.com > reroes79 en yahoo.es > > > _______________________________________________ > Python-es mailing list > Python-es en python.org > http://mail.python.org/mailman/listinfo/python-es > FAQ: http://python-es-faq.wikidot.com/ > > -- Alfredo de Jesús Gutiérrez Gómez Lic. en Informática Cedula <5219429> msn «Cuando tu encuentres el Camino otros te encuentran a ti» ------------ próxima parte ------------ Se ha borrado un adjunto en formato HTML... URL: From decastro en netvision.com.py Mon Oct 1 18:43:40 2012 From: decastro en netvision.com.py (decastro) Date: Mon, 01 Oct 2012 12:43:40 -0400 Subject: [Python-es] [python-es] | conectar python con vfp In-Reply-To: References: Message-ID: <5069C83C.9000601@netvision.com.py> Hola a todos. Esto va a ser un poquito largo, pero creo que hace falta que se haga una explicación más específica, aunque superficial. He trabajado con Visual FoxPro (VFP) por muchos años y hasta hoy le doy manutención a algunos sistemas que he hecho con ese lenguaje en su versión 9.0. En verdad, VFP maneja el tema de las tablas en dos formas básicas: 1. Tablas libres (prácticamente igual a los otros sistemas xBase). Estas casi ya no son utilizadas por los desarrolladores que usan VFP. 2. Tablas que pertenecen a una "base de datos". Las extensiones utilizadas por los archivos que manejan datos son: *.dbf - como en todos los lenguajes xBase, son los archivos de datos propiamente dichos. *.fpt - son los archivos de campos "memo". Llevan el mismo nombre de la tabla principal y están "conectados" a la tabla de datos por medio de su cabecera. *.cdx - son los archivos de índices compuestos (que pueden manejar más de un ordenamiento para la misma tabla por medio de etiquetas) y también llevan el mismo nombre del DBF, además de también estar "conectados" a la tabla de datos por medio de su cabecera. *.idx - son los archivos de índices independientes (que solo soportan un tipo de ordenamiento y tienen que ser abiertos manualmente), actualmente prácticamente no son utilizados. Cuando digo "conectados por medio de la cabecera" quiero decir que no se puede abrir dichos archivos independientemente y que la tabla a la que pertenecen no podría ser abierta correctamente, si los mismos no están presentes. Cuando se crea una "Base de datos" de Visual FoxPro, se generan 3 archivos a modo de repositorio que tienen las siguientes extensiones: *.dbc - es el archivo de datos que reúne las definiciones de todas las tablas, campos, vistas, etc. que pertenecen a la base de datos. *.dct - son los archivos de campos "memo" de la base de datos. *.cdx - son los archivos de índices de la base de datos. En realidad NO ES una base de datos como las normalmente conocidas, sigue usando las tablas DBF y todo lo demás, por lo que carece de una mayor seguridad específica. Sin embargo, las tablas que son marcadas como pertenecientes a una base de datos pueden ser manejadas como las tablas de cualquier otra base de datos. Es decir: - Sus campos pueden tener nombres largos (hasta 255 caracteres) en tablas libres su límite es 10 caracteres. - Se pueden agregar descripciones de campos a modo de diccionario de datos. - Se pueden crear Reglas de Integridad, Claves primarias, Campos autoincrementables, etc., etc., etc. Por otro lado, dichas tablas también quedan "conectadas por su cabecera" al archivo repositorio de la base de datos. Tengo entendido que existen drivers ODBC y OLE DB que permiten el acceso a esas "Bases de Datos" desde otros lenguajes. Infelizmente no tengo a mano ningún link que te pueda ayudar, pero creo que si buscas en San Google los encontrarás. Espero haber aclarado algo. Cualquier otra duda, es solo preguntar. Respecto a conectarse desde Python... Lo siento, yo estoy aun muy crudo en Python como para ayudarte con eso. Aun estoy en el "BeABa" del lenguaje, pero estoy estudiando de a poquito para llegar a conocerlo un poco más... =) Saludos aclaratorios. Ricardo De Castro Aquino Asunción - Paraguay El 01/10/2012 03:07, Carlos Latorre escribió: > Por si no estas familiarizado con las bbdd dBase: > Los datos están distribuidos en ficheros de la siguiente forma: > - *.dbf - los ficheros de datos (los importantes) > - *.dbt - contenido de los campos "memo" (estos ficheros, de > extensión dbt y el mismo nombre que el pdf son muy importantes, pues > sin ellos no se abre correctamente el .pdf, guardan el contenido de > los campos "memo", equivalente al text de sql) > - *.ndx ó *.ntx - ficheros de indice (se pueden reconstruir a partir > de los datos) - tienen una importancia relativa, pues deben abrirse y > utilizarse de forma explicita, por lo que, si no conoces la aplicación > de antemano, tal vez sea mejor que crees y utilices los que vayas > necesitando. > Además puede haber mas ficheros, propios de foxpro, conteniendo > formularios, vistas, etc, ... pero estos son muy dependientes. > > Saludos, > Carlos L. > > El 1 de octubre de 2012 08:54, Carlos Latorre > > escribió: > > Fox Pro utiliza bases las antiguas bases de datos dBase > (http://es.wikipedia.org/wiki/DBase), actualmente conocidas como > xBase (http://es.wikipedia.org/wiki/XBase). > Yo trabaje con ellas mucho en el pasado, utilizando Clipper, pero > de eso hace ya casi 20 años. > Tras una mirada rápida por S. Google aparecen un par de librerías > en el sourceforge: > - dbfpy (http://dbfpy.sourceforge.net/ ) > - xbase-py (http://sourceforge.net/projects/xbase-py/) > > Espero que te sea de ayuda para comenzar, > Saludos: > Carlos > > El 1 de octubre de 2012 06:51, aj gutierrez > escribió: > > Buenas noches a todos, > > Saben si existe o se puede conectar python a una base de datos > en visual fox pro? > > > > > > _______________________________________________ > Python-es mailing list > Python-es en python.org > http://mail.python.org/mailman/listinfo/python-es > FAQ: http://python-es-faq.wikidot.com/ > > > > > -- > \|/ > (QQ) > ( O ) > ==oo=======oo============================================================ > Carlos Latorre Galicia mail:clatorreg en dphuesca.es > > Tecnico de Sistemas Telefono: +34 974 294 110 > > > Diputacion Provincial de Huesca http://www.dphuesca.es/ > Porches de Galicia, 4 Telefono: +34 974 294 100 > > 22002 Huesca Fax: +34 974 294 149 > > ========================================================================= > > > > > -- > \|/ > (QQ) > ( O ) > ==oo=======oo============================================================ > Carlos Latorre Galicia mail:clatorreg en dphuesca.es > > Tecnico de Sistemas Telefono: +34 974 294 110 > > > Diputacion Provincial de Huesca http://www.dphuesca.es/ > Porches de Galicia, 4 Telefono: +34 974 294 100 > > 22002 Huesca Fax: +34 974 294 149 > > ========================================================================= > > > > > _______________________________________________ > Python-es mailing list > Python-es en python.org > http://mail.python.org/mailman/listinfo/python-es > FAQ: http://python-es-faq.wikidot.com/ > > > > __________ Información de ESET NOD32 Antivirus, versión de la base de datos de firmas de virus 7534 (20121001) __________ > > El mensaje fue verificado por ESET NOD32 Antivirus. > > http://www.eset-la.com > ------------ próxima parte ------------ Se ha borrado un adjunto en formato HTML... URL: From pych3m4 en gmail.com Wed Oct 3 01:36:21 2012 From: pych3m4 en gmail.com (Chema Cortes) Date: Wed, 3 Oct 2012 01:36:21 +0200 Subject: [Python-es] [Anuncio] PythonBrasil[8] - Llamada de charlas y tutoriales In-Reply-To: <5065cb9c.145b650a.6351.7a77@mx.google.com> References: <5065cb9c.145b650a.6351.7a77@mx.google.com> Message-ID: Informar que este correo estaba retenido entre los "pendientes de moderar", lo que explica el que haya llegado bastante tarde a la lista. Si a álguien le interesa la PythonBrasil, recomiendo ponerse en contacto directo en las direcciones indicadas el mensaje y no a través de la lista python-es. El día 28 de septiembre de 2012 18:09, Álvaro Justen escribió: > Estimados(as) Pythonistas Hispanos, > > Me gustaría invitarles a participar del mayor encuentro de la comunidad Python > brasileña! La PythonBrasil[8] se dará en Rio de Janeiro, la ciudad maravillosa, > 20 al 24 de noviembre. El día 20 se llevará a cabo los sprints; en los días 21 > y 22, tutoriales y en los días 23 y 24, la conferencia (charlas, keynotes, open > spaces y lightning talks). > > El evento consiste en actividades de la comunidad para la comunidad. Por lo > tanto, invitamos a los interesados a presentar propuestas de charlas y > tutoriales. La llamada de trabajos está abierta hasta el 30 de septiembre > (domingo) y las actividades aprobadas se dará a conocer el 8 de octubre. Si > quieres proponer algo, inscríbase en: > http://2012.pythonbrasil.org.br/ > > Cualquier pregunta, no dude en ponerse en contacto con el equipo de > organización a través del correo electrónico . > > Esperamos verlos pronto en Rio! ;-) > > Abrazos, > Álvaro Justen "Turicas" > Organización PythonBrasil[8] > http://2012.pythonbrasil.org.br > Twitter: @PythonBrasil > http://facebook.com/pythonbrasil8 > > _______________________________________________ > Python-es mailing list > Python-es en python.org > http://mail.python.org/mailman/listinfo/python-es > FAQ: http://python-es-faq.wikidot.com/ > -- Hyperreals *R: http://ch3m4.org/blog Quarks, bits y otras criaturas infinitesimales From decastro en click.com.py Wed Oct 3 15:21:57 2012 From: decastro en click.com.py (Ricardo De Castro Aquino) Date: Wed, 03 Oct 2012 09:21:57 -0400 Subject: [Python-es] Fwd: Re: [python-es] | conectar python con vfp In-Reply-To: <5069C83C.9000601@netvision.com.py> References: <5069C83C.9000601@netvision.com.py> Message-ID: <506C3BF5.1090701@click.com.py> Hola a todos. Esto va a ser un poquito largo, pero creo que hace falta que se haga una explicación más específica, aunque superficial. He trabajado con Visual FoxPro (VFP) por muchos años y hasta hoy le doy manutención a algunos sistemas que he hecho con ese lenguaje en su versión 9.0. En verdad, VFP maneja el tema de las tablas en dos formas básicas: 1. Tablas libres (prácticamente igual a los otros sistemas xBase). Estas casi ya no son utilizadas por los desarrolladores que usan VFP. 2. Tablas que pertenecen a una "base de datos". Las extensiones utilizadas por los archivos que manejan datos son: *.dbf - como en todos los lenguajes xBase, son los archivos de datos propiamente dichos. *.fpt - son los archivos de campos "memo". Llevan el mismo nombre de la tabla principal y están "conectados" a la tabla de datos por medio de su cabecera. *.cdx - son los archivos de índices compuestos (que pueden manejar más de un ordenamiento para la misma tabla por medio de etiquetas) y también llevan el mismo nombre del DBF, además de también estar "conectados" a la tabla de datos por medio de su cabecera. *.idx - son los archivos de índices independientes (que solo soportan un tipo de ordenamiento y tienen que ser abiertos manualmente), actualmente prácticamente no son utilizados. Cuando digo "conectados por medio de la cabecera" quiero decir que no se puede abrir dichos archivos independientemente y que la tabla a la que pertenecen no podría ser abierta correctamente, si los mismos no están presentes. Cuando se crea una "Base de datos" de Visual FoxPro, se generan 3 archivos a modo de repositorio que tienen las siguientes extensiones: *.dbc - es el archivo de datos que reúne las definiciones de todas las tablas, campos, vistas, etc. que pertenecen a la base de datos. *.dct - son los archivos de campos "memo" de la base de datos. *.cdx - son los archivos de índices de la base de datos. En realidad NO ES una base de datos como las normalmente conocidas, sigue usando las tablas DBF y todo lo demás, por lo que carece de una mayor seguridad específica. Sin embargo, las tablas que son marcadas como pertenecientes a una base de datos pueden ser manejadas como las tablas de cualquier otra base de datos. Es decir: - Sus campos pueden tener nombres largos (hasta 255 caracteres) en tablas libres su límite es 10 caracteres. - Se pueden agregar descripciones de campos a modo de diccionario de datos. - Se pueden crear Reglas de Integridad, Claves primarias, Campos autoincrementables, etc., etc., etc. Por otro lado, dichas tablas también quedan "conectadas por su cabecera" al archivo repositorio de la base de datos. Tengo entendido que existen drivers ODBC y OLE DB que permiten el acceso a esas "Bases de Datos" desde otros lenguajes. Infelizmente no tengo a mano ningún link que te pueda ayudar, pero creo que si buscas en San Google los encontrarás. Espero haber aclarado algo. Cualquier otra duda, es solo preguntar. Respecto a conectarse desde Python... Lo siento, yo estoy aun muy crudo en Python como para ayudarte con eso. Aun estoy en el "BeABa" del lenguaje, pero estoy estudiando de a poquito para llegar a conocerlo un poco más... =) Saludos aclaratorios. Ricardo De Castro Aquino Asunción - Paraguay El 01/10/2012 03:07, Carlos Latorre escribió: > Por si no estas familiarizado con las bbdd dBase: > Los datos están distribuidos en ficheros de la siguiente forma: > - *.dbf - los ficheros de datos (los importantes) > - *.dbt - contenido de los campos "memo" (estos ficheros, de > extensión dbt y el mismo nombre que el pdf son muy importantes, pues > sin ellos no se abre correctamente el .pdf, guardan el contenido de > los campos "memo", equivalente al text de sql) > - *.ndx ó *.ntx - ficheros de indice (se pueden reconstruir a partir > de los datos) - tienen una importancia relativa, pues deben abrirse y > utilizarse de forma explicita, por lo que, si no conoces la aplicación > de antemano, tal vez sea mejor que crees y utilices los que vayas > necesitando. > Además puede haber mas ficheros, propios de foxpro, conteniendo > formularios, vistas, etc, ... pero estos son muy dependientes. > > Saludos, > Carlos L. > > El 1 de octubre de 2012 08:54, Carlos Latorre > > escribió: > > Fox Pro utiliza bases las antiguas bases de datos dBase > (http://es.wikipedia.org/wiki/DBase), actualmente conocidas como > xBase (http://es.wikipedia.org/wiki/XBase). > Yo trabaje con ellas mucho en el pasado, utilizando Clipper, pero > de eso hace ya casi 20 años. > Tras una mirada rápida por S. Google aparecen un par de librerías > en el sourceforge: > - dbfpy (http://dbfpy.sourceforge.net/ ) > - xbase-py (http://sourceforge.net/projects/xbase-py/) > > Espero que te sea de ayuda para comenzar, > Saludos: > Carlos > > El 1 de octubre de 2012 06:51, aj gutierrez > escribió: > > Buenas noches a todos, > > Saben si existe o se puede conectar python a una base de datos > en visual fox pro? > > > > > > _______________________________________________ > Python-es mailing list > Python-es en python.org > http://mail.python.org/mailman/listinfo/python-es > FAQ: http://python-es-faq.wikidot.com/ > > > > > -- > \|/ > (QQ) > ( O ) > ==oo=======oo============================================================ > Carlos Latorre Galicia mail:clatorreg en dphuesca.es > > Tecnico de Sistemas Telefono: +34 974 294 110 > > > Diputacion Provincial de Huesca http://www.dphuesca.es/ > Porches de Galicia, 4 Telefono: +34 974 294 100 > > 22002 Huesca Fax: +34 974 294 149 > > ========================================================================= > > > > > -- > \|/ > (QQ) > ( O ) > ==oo=======oo============================================================ > Carlos Latorre Galicia mail:clatorreg en dphuesca.es > > Tecnico de Sistemas Telefono: +34 974 294 110 > > > Diputacion Provincial de Huesca http://www.dphuesca.es/ > Porches de Galicia, 4 Telefono: +34 974 294 100 > > 22002 Huesca Fax: +34 974 294 149 > ========================================================================= > > > > > _______________________________________________ > Python-es mailing list > Python-es en python.org > http://mail.python.org/mailman/listinfo/python-es > FAQ:http://python-es-faq.wikidot.com/ > > > > __________ Información de ESET NOD32 Antivirus, versión de la base de datos de firmas de virus 7534 (20121001) __________ > > El mensaje fue verificado por ESET NOD32 Antivirus. > > http://www.eset-la.com > ------------ próxima parte ------------ Se ha borrado un adjunto en formato HTML... URL: From decastro en click.com.py Wed Oct 3 15:31:56 2012 From: decastro en click.com.py (Ricardo De Castro Aquino) Date: Wed, 03 Oct 2012 09:31:56 -0400 Subject: [Python-es] [python-es] | conectar python con vfp In-Reply-To: <5069C83C.9000601@netvision.com.py> References: <5069C83C.9000601@netvision.com.py> Message-ID: <506C3E4C.7030702@click.com.py> Hola a todos. Antes que nada, quiero disculparme si se duplica el mensaje anterior, ya que recibí un aviso de que estaba retenido por el moderador porque usé otra cuenta de correo que no era la que usé para inscribirme. Como no se publicaba, repetí el envío con la cuenta correcta. Minutos después me llega la respuesta original. Bueno, cosas que pasan. En segundo término quiero rectificar la extensión de los archivos de índices de las bases de datos de VFP que he posteado como siendo CDX y en realidad son los de extensión .DCX. --- Saludos rectificativos. Ricardo De Castro Aquino Asunción - Paraguay El 01/10/2012 12:43, decastro escribió: > Hola a todos. > Esto va a ser un poquito largo, pero creo que hace falta que se haga > una explicación más específica, aunque superficial. He trabajado con > Visual FoxPro (VFP) por muchos años y hasta hoy le doy manutención a > algunos sistemas que he hecho con ese lenguaje en su versión 9.0. > > En verdad, VFP maneja el tema de las tablas en dos formas básicas: > > 1. Tablas libres (prácticamente igual a los otros sistemas xBase). > Estas casi ya no son utilizadas por los desarrolladores que usan VFP. > 2. Tablas que pertenecen a una "base de datos". > > Las extensiones utilizadas por los archivos que manejan datos son: > > *.dbf - como en todos los lenguajes xBase, son los archivos de > datos propiamente dichos. > > *.fpt - son los archivos de campos "memo". Llevan el mismo nombre > de la tabla principal y están "conectados" a la tabla de datos por > medio de su cabecera. > > *.cdx - son los archivos de índices compuestos (que pueden manejar > más de un ordenamiento para la misma tabla por medio de etiquetas) > y también llevan el mismo nombre del DBF, además de también estar > "conectados" a la tabla de datos por medio de su cabecera. > > *.idx - son los archivos de índices independientes (que solo > soportan un tipo de ordenamiento y tienen que ser abiertos > manualmente), actualmente prácticamente no son utilizados. > > Cuando digo "conectados por medio de la cabecera" quiero decir que no > se puede abrir dichos archivos independientemente y que la tabla a la > que pertenecen no podría ser abierta correctamente, si los mismos no > están presentes. > > Cuando se crea una "Base de datos" de Visual FoxPro, se generan 3 > archivos a modo de repositorio que tienen las siguientes extensiones: > > *.dbc - es el archivo de datos que reúne las definiciones de todas > las tablas, campos, vistas, etc. que pertenecen a la base de datos. > *.dct - son los archivos de campos "memo" de la base de datos. > *.cdx - son los archivos de índices de la base de datos. > > En realidad NO ES una base de datos como las normalmente conocidas, > sigue usando las tablas DBF y todo lo demás, por lo que carece de una > mayor seguridad específica. > Sin embargo, las tablas que son marcadas como pertenecientes a una > base de datos pueden ser manejadas como las tablas de cualquier otra > base de datos. Es decir: > - Sus campos pueden tener nombres largos (hasta 255 caracteres) en > tablas libres su límite es 10 caracteres. > - Se pueden agregar descripciones de campos a modo de diccionario de > datos. > - Se pueden crear Reglas de Integridad, Claves primarias, Campos > autoincrementables, etc., etc., etc. > > Por otro lado, dichas tablas también quedan "conectadas por su > cabecera" al archivo repositorio de la base de datos. > > Tengo entendido que existen drivers ODBC y OLE DB que permiten el > acceso a esas "Bases de Datos" desde otros lenguajes. Infelizmente no > tengo a mano ningún link que te pueda ayudar, pero creo que si buscas > en San Google los encontrarás. > > Espero haber aclarado algo. Cualquier otra duda, es solo preguntar. > > Respecto a conectarse desde Python... Lo siento, yo estoy aun muy > crudo en Python como para ayudarte con eso. Aun estoy en el "BeABa" > del lenguaje, pero estoy estudiando de a poquito para llegar a > conocerlo un poco más... =) > > Saludos aclaratorios. > Ricardo De Castro Aquino > Asunción - Paraguay > ------------ próxima parte ------------ Se ha borrado un adjunto en formato HTML... URL: From zodman en gmail.com Thu Oct 4 16:57:34 2012 From: zodman en gmail.com (zodman) Date: Thu, 4 Oct 2012 09:57:34 -0500 Subject: [Python-es] [Job] Developer Python Message-ID: ¿Te gusta python? ¿Te gustaria programar software en python y en la web ? sabes que son tuplas y diccionarios ? www.interalia.net esta buscando developers web ... Si tienes mas o menos nocion de python nosotros te damos training!!! para que seas NINJA Mandame tu cv.... El trabajo es requerido onsite y es en mexico -- Andres Vargas www.zodman.com.mx From packo en assamita.net Fri Oct 5 14:49:41 2012 From: packo en assamita.net (Francisco Moreno) Date: Fri, 5 Oct 2012 14:49:41 +0200 Subject: [Python-es] =?iso-8859-1?q?Aplicaci=F3n_escritorio_para_windows?= In-Reply-To: References: <50445381.2080401@gmail.com> <505D52E3.6050201@gmail.com> Message-ID: Yo he hecho algunas aplicaciones visuales en Windows con Python y he usado Glade para generar las interfaces y py2exe para crear la aplicación windows con sus dependencias y todo. No es complicado aunque tampoco es un VisualBasic. Un saludo. El 22 de septiembre de 2012 13:52, RadicalEd escribió: > Para wxPython existe uno bueno llamado VisualWX > > El 22/09/2012 1:42, "Alexandro Colorado" escribió: > > > Hola, > estas haciendo preguntas diferentes. Una cosa es hacer herramientas > visuales para windows. > > Otra cosa es tener un diseñador visual para estas herramientas. > > Las herramientas visuales para windows pueden ser hechas con Python y > algun toolkit que tenga compatibilidad en windows como lo es: > - Tk http://i1-win.softpedia-static.com/screenshots/Trebuchet-Tk_1.png > - Gtk http://www.emcken.dk/screenshots/GTK%20i%20windows.jpg > - Qt4 http://sourceforge.net/projects/ncreport/screenshots/107225 > - WxWidgets http://www.wxwidgets.org/images/screens/gamedevelop_win.jpg > > Estos toolkits corren en windows como podras ver en las imagenes. Sin > embargo la programación es hecha dentro del codigo. > > El diseñador grafico es algo diferente el cual usualmente es un > programa dedicado a dibujar la interfaz. El diseñador para el toolkit > es algo que parece que es lo que estas buscando. No todos los toolkits > tienen un diseñado. La forma mas tradicional es que el diseñador > genera un XML copuesto de los componentes del toolkit y el codigo coje > estos elementos para hacer el rendering de instrucciones. > > http://doc.qt.digia.com/4.7-snapshot/designer-manual.html > > GTK usa Glade, TK y wxWidgets ignoro si tienen un diseñador o varios. > Tambien no se haya un diseñador universal que aplique una conversion a > todos los toolkits. > > > On 9/22/12, kausdiv wrote: > > Hola de nuevo. > > > > No consigo encontrar un sistema... > > -- > Alexandro Colorado > PPMC Apache OpenOffice > http://es.openoffice.org > > _______________________________________________ > Python-es mailing list > Python-es en python.org > http://m... > > > _______________________________________________ > Python-es mailing list > Python-es en python.org > http://mail.python.org/mailman/listinfo/python-es > FAQ: http://python-es-faq.wikidot.com/ > > -- Francisco Moreno ------------ próxima parte ------------ Se ha borrado un adjunto en formato HTML... URL: From jeancarlosr_ve en yahoo.com Fri Oct 5 15:33:29 2012 From: jeancarlosr_ve en yahoo.com (=?iso-8859-1?Q?Jean_Carlos_Rodr=ECguez?=) Date: Fri, 5 Oct 2012 06:33:29 -0700 (PDT) Subject: [Python-es] =?iso-8859-1?q?Aplicaci=F3n_escritorio_para_windows?= In-Reply-To: References: <50445381.2080401@gmail.com> <505D52E3.6050201@gmail.com> Message-ID: <1349444009.16611.YahooMailNeo@web124901.mail.ne1.yahoo.com> Yo utilizo GUI2EXE y va muy bien, te anexo el link http://code.google.com/p/gui2exe/ ________________________________ De: Francisco Moreno Para: La lista de python en castellano Enviado: Viernes, 5 de octubre, 2012 8:19 A.M. Asunto: Re: [Python-es] Aplicación escritorio para windows Yo he hecho algunas aplicaciones visuales en Windows con Python y he usado Glade para generar las interfaces y py2exe para crear la aplicación windows con sus dependencias y todo. No es complicado aunque tampoco es un VisualBasic. Un saludo. El 22 de septiembre de 2012 13:52, RadicalEd escribió: Para wxPython existe uno bueno llamado VisualWX >El 22/09/2012 1:42, "Alexandro Colorado" escribió: >> >> >>Hola, >>estas haciendo preguntas diferentes. Una cosa es hacer herramientas >>visuales para windows. >> >>Otra cosa es tener un diseñador visual para estas herramientas. >> >>Las herramientas visuales para windows pueden ser hechas con Python y >>algun toolkit que tenga compatibilidad en windows como lo es: >>- Tk http://i1-win.softpedia-static.com/screenshots/Trebuchet-Tk_1.png >>- Gtk  http://www.emcken.dk/screenshots/GTK%20i%20windows.jpg >>- Qt4 http://sourceforge.net/projects/ncreport/screenshots/107225 >>- WxWidgets http://www.wxwidgets.org/images/screens/gamedevelop_win.jpg >> >>Estos toolkits corren en windows como podras ver en las imagenes. Sin >>embargo la programación es hecha dentro del codigo. >> >>El diseñador grafico es algo diferente el cual usualmente es un >>programa dedicado a dibujar la interfaz. El diseñador para el toolkit >>es algo que parece que es lo que estas buscando. No todos los toolkits >>tienen un diseñado. La forma mas tradicional es que el diseñador >>genera un XML copuesto de los componentes del toolkit y el codigo coje >>estos elementos para hacer el rendering de instrucciones. >> >>http://doc.qt.digia.com/4.7-snapshot/designer-manual.html >> >>GTK usa Glade, TK y wxWidgets ignoro si tienen un diseñador o varios. >>Tambien no se haya un diseñador universal que aplique una conversion a >>todos los toolkits. >> >> >>On 9/22/12, kausdiv wrote: >>> Hola de nuevo. >>> >>> No consigo encontrar un sistema... >>-- >>Alexandro Colorado >>PPMC Apache OpenOffice >>http://es.openoffice.org >> >>_______________________________________________ >>Python-es mailing list >>Python-es en python.org >>http://m... >_______________________________________________ >Python-es mailing list >Python-es en python.org >http://mail.python.org/mailman/listinfo/python-es >FAQ: http://python-es-faq.wikidot.com/ > > -- Francisco Moreno _______________________________________________ Python-es mailing list Python-es en python.org http://mail.python.org/mailman/listinfo/python-es FAQ: http://python-es-faq.wikidot.com/ ------------ próxima parte ------------ Se ha borrado un adjunto en formato HTML... URL: From gmourinopardo en yahoo.es Fri Oct 5 16:12:38 2012 From: gmourinopardo en yahoo.es (=?iso-8859-1?Q?Gonzalo_Mouri=FFfffffffffff1o_Pardo?=) Date: Fri, 5 Oct 2012 15:12:38 +0100 (BST) Subject: [Python-es] Variables wxpython Message-ID: <1349446358.90812.YahooMailNeo@web29601.mail.ird.yahoo.com> Buenas llevo mas o menos 2 semanas aprendiendo a programar en python, y me encuentro en una encrucijada. Necesito sacar información de Ventana_New (tc1) y llevarlo al Ventana.SetTitle, no se como podría hacerlo, gracias. Ahí va mi código: import wx class Aplicacion(wx.App):     def OnInit(self):         self.frame = Ventana(None, -1, title = "Control Horario")         self.SetTopWindow(self.frame)         self.frame.Show()         return True class Ventana(wx.Frame):          def __init__(self, parent, id, title):         super(Ventana, self).__init__(parent, id, title, size = (800, 275)) panel = wx.Panel(self) menubar = wx.MenuBar() arcm = wx.Menu()         anmi = wx.MenuItem(arcm, ID_NUEVO, "&Nuevo\tCtrl+N")         arcm.AppendItem(anmi)         menubar.Append(arcm, '&Archivo')         self.SetMenuBar(menubar)         self.Bind(wx.EVT_MENU, self.OnMenu)     def OnMenu(self, event):  evt_id = event.GetId ()   if evt_id == ID_NUEVO:       self.frame = Ventana_New(None, -1, title = "Nuevo Usuario")  class Ventana_New(wx.Frame):          def __init__(self, parent, id, title):         self.padre = parent         wx.Frame.__init__(self, parent, id, title, size = (295, 357))         self.panel = Panel_New(self)         self.Show(True) class Panel_New(wx.Panel):     def __init__(self, parent, *args, **kwargs):         self.padre = parent         wx.Panel.__init__(self, parent, *args, **kwargs)         gs = wx.FlexGridSizer(3, 2, 9, 9)         vbox = wx.BoxSizer(wx.VERTICAL)         hbox = wx.BoxSizer(wx.HORIZONTAL)         nombre = wx.StaticText(self, -1, "Nombre:")         self.tc1 = wx.TextCtrl(self, -1, size = (150, -1)) gs.AddMany([(nombre), (self.tc1, 1, wx.LEFT, 10)]         self.btn = wx.Button(self, -1, 'Aceptar', size = (-1, 30))         hbox.Add(self.btn)         vbox.Add(hbox, 0, wx.ALIGN_CENTER | wx.BOTTOM, 10)         self.Bind(wx.EVT_BUTTON, self.OnAceptar, id = self.btn.GetId())         self.SetSizer(vbox)     def OnAceptar(self, event):         padre = self.GetParent()         padre.Close(True) ------------ próxima parte ------------ Se ha borrado un adjunto en formato HTML... URL: From jcaballero.hep en gmail.com Fri Oct 5 17:42:49 2012 From: jcaballero.hep en gmail.com (Jose Caballero) Date: Fri, 5 Oct 2012 11:42:49 -0400 Subject: [Python-es] encapsular llamadas a metodos arbitrarios desde una clase container Message-ID: Hola, pido disculpas por adelantado por lo criptico del "subject". Intentare explicar mejor lo que quiero hacer. Quiero crear una clase Container que contiene una lista de objetos de una clase X arbitraria. ------------------------------------------------------------------------------ class Container: def __init__(self, list_obj): self.list_obj = list_obj class X: blah x1 = X() x2 = X() x3 = X() container = Container( [ x1, x2, x3] ) ------------------------------------------------------------------------------ Me gustaria poder llamar a un metodo cualquiera en Container, y que internamente haga un loop de llamadas a ese mismo metodo para todos los objectos de la lista. Por ejemplo, si hago container.f() que internamente se haga un loop de llamadas: x1.f() x2.f() x3.f() Lo mas cerca que estoy de conseguirlo es con lo siguiente: ------------------------------------------------------------------------------ class Container: ... ... def __getattr__(self, atr): outs = [] for o in self.list_obj: out = getattr(o, atr)() outs.append(out) return outs ------------------------------------------------------------------------------ con ese codigo puedo hacer container.f container.g container.h pero no container.f() container.g() container.h() Sigo investigando, pero si mientras tanto alguien me ofrece una pista, sera mas que bienvenida. Saludos cordiales, Jose P.S. perdon por la ausencia de tildes. ------------ próxima parte ------------ Se ha borrado un adjunto en formato HTML... URL: From sanreikaj.foros en gmail.com Fri Oct 5 18:17:48 2012 From: sanreikaj.foros en gmail.com (Alvaro Manrique) Date: Fri, 5 Oct 2012 11:47:48 -0430 Subject: [Python-es] encapsular llamadas a metodos arbitrarios desde una clase container In-Reply-To: References: Message-ID: Para hacer dinámico lo quieres mi opción es que crees el atributo en la clase container con setattr() y luego asignar a este nuevo atributo el objeto que necesitas, bien sea una clase o un atributo de ella. El 05/10/2012 11:13, "Jose Caballero" escribió: > Hola, > > > > pido disculpas por adelantado por lo criptico del "subject". Intentare > explicar mejor lo que quiero hacer. > Quiero crear una clase Container que contiene una lista de objetos de una > clase X arbitraria. > > > > ------------------------------------------------------------------------------ > class Container: > def __init__(self, list_obj): > self.list_obj = list_obj > > class X: > blah > > x1 = X() > x2 = X() > x3 = X() > > container = Container( [ x1, x2, x3] ) > > ------------------------------------------------------------------------------ > > > Me gustaria poder llamar a un metodo cualquiera en Container, y que > internamente haga un loop de llamadas a ese mismo metodo para todos los > objectos de la lista. > Por ejemplo, si hago > > container.f() > > que internamente se haga un loop de llamadas: > > x1.f() > x2.f() > x3.f() > > > Lo mas cerca que estoy de conseguirlo es con lo siguiente: > > > > ------------------------------------------------------------------------------ > class Container: > ... > ... > def __getattr__(self, atr): > outs = [] > for o in self.list_obj: > out = getattr(o, atr)() > outs.append(out) > return outs > > ------------------------------------------------------------------------------ > > > con ese codigo puedo hacer > > container.f > container.g > container.h > > pero no > > container.f() > container.g() > container.h() > > > Sigo investigando, pero si mientras tanto alguien me ofrece una pista, > sera mas que bienvenida. > > > Saludos cordiales, > Jose > P.S. perdon por la ausencia de tildes. > > > > > > > _______________________________________________ > Python-es mailing list > Python-es en python.org > http://mail.python.org/mailman/listinfo/python-es > FAQ: http://python-es-faq.wikidot.com/ > > ------------ próxima parte ------------ Se ha borrado un adjunto en formato HTML... URL: From carlos.zun en gmail.com Fri Oct 5 18:19:38 2012 From: carlos.zun en gmail.com (Carlos Zuniga) Date: Fri, 5 Oct 2012 11:19:38 -0500 Subject: [Python-es] encapsular llamadas a metodos arbitrarios desde una clase container In-Reply-To: References: Message-ID: 2012/10/5 Jose Caballero : > Hola, > > > > pido disculpas por adelantado por lo criptico del "subject". Intentare > explicar mejor lo que quiero hacer. > Quiero crear una clase Container que contiene una lista de objetos de una > clase X arbitraria. > > > ------------------------------------------------------------------------------ > class Container: > def __init__(self, list_obj): > self.list_obj = list_obj > > class X: > blah > > x1 = X() > x2 = X() > x3 = X() > > container = Container( [ x1, x2, x3] ) > ------------------------------------------------------------------------------ > > > Me gustaria poder llamar a un metodo cualquiera en Container, y que > internamente haga un loop de llamadas a ese mismo metodo para todos los > objectos de la lista. > Por ejemplo, si hago > > container.f() > > que internamente se haga un loop de llamadas: > > x1.f() > x2.f() > x3.f() > > > Lo mas cerca que estoy de conseguirlo es con lo siguiente: > > > ------------------------------------------------------------------------------ > class Container: > ... > ... > def __getattr__(self, atr): > outs = [] > for o in self.list_obj: > out = getattr(o, atr)() > outs.append(out) > return outs > ------------------------------------------------------------------------------ > > > con ese codigo puedo hacer > > container.f > container.g > container.h > > pero no > > container.f() > container.g() > container.h() > > > Sigo investigando, pero si mientras tanto alguien me ofrece una pista, sera > mas que bienvenida. Tal vez algo como: class Cont: ... def __getattr__(self, atr): def foo(): outs = [] for o in self.list_obj: out = getattr(o, atr)() outs.append(out) return outs return foo > > Saludos cordiales, > Jose > P.S. perdon por la ausencia de tildes. UTF existe, ya no es excusa ;-) Saludos -- A menudo unas pocas horas de "Prueba y error" podrán ahorrarte minutos de leer manuales. From carlos.zun en gmail.com Fri Oct 5 18:53:42 2012 From: carlos.zun en gmail.com (Carlos Zuniga) Date: Fri, 5 Oct 2012 11:53:42 -0500 Subject: [Python-es] Variables wxpython In-Reply-To: <1349446358.90812.YahooMailNeo@web29601.mail.ird.yahoo.com> References: <1349446358.90812.YahooMailNeo@web29601.mail.ird.yahoo.com> Message-ID: On Fri, Oct 5, 2012 at 9:12 AM, Gonzalo Mouriÿfffffffffff1o Pardo wrote: > Buenas llevo mas o menos 2 semanas aprendiendo a programar en python, y me > encuentro en una encrucijada. > Necesito sacar información de Ventana_New (tc1) y llevarlo al > Ventana.SetTitle, no se como podría hacerlo, gracias. > Ahí va mi código: > > import wx > > class Aplicacion(wx.App): > > def OnInit(self): > self.frame = Ventana(None, -1, title = "Control Horario") > self.SetTopWindow(self.frame) > self.frame.Show() > return True > > class Ventana(wx.Frame): > > def __init__(self, parent, id, title): > super(Ventana, self).__init__(parent, id, title, size = (800, 275)) > panel = wx.Panel(self) > menubar = wx.MenuBar() > arcm = wx.Menu() > anmi = wx.MenuItem(arcm, ID_NUEVO, "&Nuevo\tCtrl+N") > arcm.AppendItem(anmi) > menubar.Append(arcm, '&Archivo') > self.SetMenuBar(menubar) > self.Bind(wx.EVT_MENU, self.OnMenu) > > def OnMenu(self, event): > evt_id = event.GetId () > if evt_id == ID_NUEVO: > self.frame = Ventana_New(None, -1, title = "Nuevo Usuario") > > class Ventana_New(wx.Frame): > > def __init__(self, parent, id, title): > self.padre = parent > wx.Frame.__init__(self, parent, id, title, size = (295, 357)) > self.panel = Panel_New(self) > self.Show(True) > > class Panel_New(wx.Panel): > > def __init__(self, parent, *args, **kwargs): > self.padre = parent > wx.Panel.__init__(self, parent, *args, **kwargs) > gs = wx.FlexGridSizer(3, 2, 9, 9) > vbox = wx.BoxSizer(wx.VERTICAL) > hbox = wx.BoxSizer(wx.HORIZONTAL) > nombre = wx.StaticText(self, -1, "Nombre:") > self.tc1 = wx.TextCtrl(self, -1, size = (150, -1)) > gs.AddMany([(nombre), (self.tc1, 1, wx.LEFT, 10)] > self.btn = wx.Button(self, -1, 'Aceptar', size = (-1, 30)) > hbox.Add(self.btn) > vbox.Add(hbox, 0, wx.ALIGN_CENTER | wx.BOTTOM, 10) > self.Bind(wx.EVT_BUTTON, self.OnAceptar, id = self.btn.GetId()) > self.SetSizer(vbox) > > def OnAceptar(self, event): > padre = self.GetParent() > padre.Close(True) > Hola, tu ejemplo no corre así que no lo puedo probar... pero lo que puedes hacer es añadir un evento close: def OnMenu(self, event): evt_id = event.GetId() if evt_id == ID_NUEVO: self.frame = Ventana_New(None, -1, title = "Nuevo Usuario") self.frame.Bind(wx.EVT_CLOSE, self.OnCloseMenu) self.frame.Show(True) def OnCloseMenu(self, event): print self.frame.panel.tc1.GetValue() self.frame.Destroy() También tendrías que chequear si solo se ha cerrado la ventana o se ha presionado el boton Aceptar. Añade un atributo aceptado con valor False a Ventana_New y lo cambias a True en OnAceptar. Luego en OnCloseMenu chequeas `if self.frame.aceptado: ...` Saludos -- A menudo unas pocas horas de "Prueba y error" podrán ahorrarte minutos de leer manuales. From neoajg en gmail.com Fri Oct 5 18:58:27 2012 From: neoajg en gmail.com (aj gutierrez) Date: Fri, 5 Oct 2012 11:58:27 -0500 Subject: [Python-es] [python-es] | conectar python con vfp In-Reply-To: <506C3E4C.7030702@click.com.py> References: <5069C83C.9000601@netvision.com.py> <506C3E4C.7030702@click.com.py> Message-ID: Gracias Castro por la ilustracion!! no sabia El 3 de octubre de 2012 08:31, Ricardo De Castro Aquino < decastro en click.com.py> escribió: > Hola a todos. > > Antes que nada, quiero disculparme si se duplica el mensaje anterior, ya > que recibí un aviso de que estaba retenido por el moderador porque usé otra > cuenta de correo que no era la que usé para inscribirme. Como no se > publicaba, repetí el envío con la cuenta correcta. Minutos después me llega > la respuesta original. Bueno, cosas que pasan. > > En segundo término quiero rectificar la extensión de los archivos de > índices de las bases de datos de VFP que he posteado como siendo CDX y en > realidad son los de extensión .DCX. > > --- > Saludos rectificativos. > > Ricardo De Castro Aquino > Asunción - Paraguay > > El 01/10/2012 12:43, decastro escribió: > > Hola a todos. > Esto va a ser un poquito largo, pero creo que hace falta que se haga una > explicación más específica, aunque superficial. He trabajado con Visual > FoxPro (VFP) por muchos años y hasta hoy le doy manutención a algunos > sistemas que he hecho con ese lenguaje en su versión 9.0. > > En verdad, VFP maneja el tema de las tablas en dos formas básicas: > > 1. Tablas libres (prácticamente igual a los otros sistemas xBase). > Estas casi ya no son utilizadas por los desarrolladores que usan VFP. > 2. Tablas que pertenecen a una "base de datos". > > Las extensiones utilizadas por los archivos que manejan datos son: > > *.dbf - como en todos los lenguajes xBase, son los archivos de datos > propiamente dichos. > > *.fpt - son los archivos de campos "memo". Llevan el mismo nombre de la > tabla principal y están "conectados" a la tabla de datos por medio de su > cabecera. > > *.cdx - son los archivos de índices compuestos (que pueden manejar más de > un ordenamiento para la misma tabla por medio de etiquetas) y también > llevan el mismo nombre del DBF, además de también estar "conectados" a la > tabla de datos por medio de su cabecera. > > *.idx - son los archivos de índices independientes (que solo soportan un > tipo de ordenamiento y tienen que ser abiertos manualmente), actualmente > prácticamente no son utilizados. > > Cuando digo "conectados por medio de la cabecera" quiero decir que no se > puede abrir dichos archivos independientemente y que la tabla a la que > pertenecen no podría ser abierta correctamente, si los mismos no están > presentes. > > Cuando se crea una "Base de datos" de Visual FoxPro, se generan 3 archivos > a modo de repositorio que tienen las siguientes extensiones: > > *.dbc - es el archivo de datos que reúne las definiciones de todas las > tablas, campos, vistas, etc. que pertenecen a la base de datos. > *.dct - son los archivos de campos "memo" de la base de datos. > *.cdx - son los archivos de índices de la base de datos. > > En realidad NO ES una base de datos como las normalmente conocidas, sigue > usando las tablas DBF y todo lo demás, por lo que carece de una mayor > seguridad específica. > Sin embargo, las tablas que son marcadas como pertenecientes a una base de > datos pueden ser manejadas como las tablas de cualquier otra base de datos. > Es decir: > - Sus campos pueden tener nombres largos (hasta 255 caracteres) en tablas > libres su límite es 10 caracteres. > - Se pueden agregar descripciones de campos a modo de diccionario de datos. > - Se pueden crear Reglas de Integridad, Claves primarias, Campos > autoincrementables, etc., etc., etc. > > Por otro lado, dichas tablas también quedan "conectadas por su cabecera" > al archivo repositorio de la base de datos. > > Tengo entendido que existen drivers ODBC y OLE DB que permiten el acceso a > esas "Bases de Datos" desde otros lenguajes. Infelizmente no tengo a mano > ningún link que te pueda ayudar, pero creo que si buscas en San Google los > encontrarás. > > Espero haber aclarado algo. Cualquier otra duda, es solo preguntar. > > Respecto a conectarse desde Python... Lo siento, yo estoy aun muy crudo en > Python como para ayudarte con eso. Aun estoy en el "BeABa" del lenguaje, > pero estoy estudiando de a poquito para llegar a conocerlo un poco más... =) > > Saludos aclaratorios. > Ricardo De Castro Aquino > Asunción - Paraguay > > > > _______________________________________________ > Python-es mailing list > Python-es en python.org > http://mail.python.org/mailman/listinfo/python-es > FAQ: http://python-es-faq.wikidot.com/ > > -- Alfredo de Jesús Gutiérrez Gómez Lic. en Informática Cedula <5219429> msn «Cuando tu encuentres el Camino otros te encuentran a ti» ------------ próxima parte ------------ Se ha borrado un adjunto en formato HTML... URL: From nadaird en gmail.com Fri Oct 5 19:07:34 2012 From: nadaird en gmail.com (G V) Date: Fri, 5 Oct 2012 19:07:34 +0200 Subject: [Python-es] encapsular llamadas a metodos arbitrarios desde una clase container In-Reply-To: References: Message-ID: si no te molesta tocar un poco de programación funcional, lambdas y demas: a = ['hola', 'adios'] map(lambda x:x.swapcase(), a) esto da un resultado de: ['HOLA', 'ADIOS'] espero que te sirva para orientar lo que quieres hacer ;) 2012/10/5 Carlos Zuniga : > 2012/10/5 Jose Caballero : >> Hola, >> >> >> >> pido disculpas por adelantado por lo criptico del "subject". Intentare >> explicar mejor lo que quiero hacer. >> Quiero crear una clase Container que contiene una lista de objetos de una >> clase X arbitraria. >> >> >> ------------------------------------------------------------------------------ >> class Container: >> def __init__(self, list_obj): >> self.list_obj = list_obj >> >> class X: >> blah >> >> x1 = X() >> x2 = X() >> x3 = X() >> >> container = Container( [ x1, x2, x3] ) >> ------------------------------------------------------------------------------ >> >> >> Me gustaria poder llamar a un metodo cualquiera en Container, y que >> internamente haga un loop de llamadas a ese mismo metodo para todos los >> objectos de la lista. >> Por ejemplo, si hago >> >> container.f() >> >> que internamente se haga un loop de llamadas: >> >> x1.f() >> x2.f() >> x3.f() >> >> >> Lo mas cerca que estoy de conseguirlo es con lo siguiente: >> >> >> ------------------------------------------------------------------------------ >> class Container: >> ... >> ... >> def __getattr__(self, atr): >> outs = [] >> for o in self.list_obj: >> out = getattr(o, atr)() >> outs.append(out) >> return outs >> ------------------------------------------------------------------------------ >> >> >> con ese codigo puedo hacer >> >> container.f >> container.g >> container.h >> >> pero no >> >> container.f() >> container.g() >> container.h() >> >> >> Sigo investigando, pero si mientras tanto alguien me ofrece una pista, sera >> mas que bienvenida. > > Tal vez algo como: > > class Cont: > ... > def __getattr__(self, atr): > def foo(): > outs = [] > for o in self.list_obj: > out = getattr(o, atr)() > outs.append(out) > return outs > return foo > > >> >> Saludos cordiales, >> Jose >> P.S. perdon por la ausencia de tildes. > > UTF existe, ya no es excusa ;-) > > Saludos > -- > A menudo unas pocas horas de "Prueba y error" podrán ahorrarte minutos > de leer manuales. > _______________________________________________ > Python-es mailing list > Python-es en python.org > http://mail.python.org/mailman/listinfo/python-es > FAQ: http://python-es-faq.wikidot.com/ From luis.salazar.ramirez en gmail.com Fri Oct 5 19:18:25 2012 From: luis.salazar.ramirez en gmail.com (luis-google) Date: Fri, 05 Oct 2012 12:18:25 -0500 Subject: [Python-es] Boa-constructor (no salen unas propiedades) Message-ID: <506F1661.2070509@gmail.com> Buen día para todos: resulta que estoy incursionando en python y comencé a trabajar con boa-constructor, estaba siguiendo el tutorial para crear la primera aplicación y cuando llegué a agregar el "status bar" me encontré con que en la guía dice que me debe aparecer '+++' en el campo fields, no me aparece nada y doy click y tampoco reacciona. Si alguien quisiera echarme una mano se lo agradecería mucho. att, luis jaime From jza en oooes.org Fri Oct 5 20:26:32 2012 From: jza en oooes.org (Alexandro Colorado) Date: Fri, 5 Oct 2012 13:26:32 -0500 Subject: [Python-es] =?utf-8?q?Aplicaci=C3=B3n_escritorio_para_windows?= In-Reply-To: <1349444009.16611.YahooMailNeo@web124901.mail.ne1.yahoo.com> References: <50445381.2080401@gmail.com> <505D52E3.6050201@gmail.com> <1349444009.16611.YahooMailNeo@web124901.mail.ne1.yahoo.com> Message-ID: Seria interesante un tutorial de esto, supongo que mucha gente nueva a python y que necesita trabajar en windows, le gustaria ver un ejemplo de algun gui siendo compilado para su plataforma sobretodo si viene de toolkits libres como GTK. On 10/5/12, Jean Carlos Rodrìguez wrote: > Yo utilizo GUI2EXE y va muy bien, te anexo el link > http://code.google.com/p/gui2exe/ > > > > ________________________________ > De: Francisco Moreno > Para: La lista de python en castellano > Enviado: Viernes, 5 de octubre, 2012 8:19 A.M. > Asunto: Re: [Python-es] Aplicación escritorio para windows > > > Yo he hecho algunas aplicaciones visuales en Windows con Python y he usado > Glade para generar las interfaces y py2exe para crear la aplicación windows > con sus dependencias y todo. No es complicado aunque tampoco es un > VisualBasic. > > Un saludo. > > > El 22 de septiembre de 2012 13:52, RadicalEd > escribió: > > Para wxPython existe uno bueno llamado VisualWX >>El 22/09/2012 1:42, "Alexandro Colorado" escribió: >>> >>> >>>Hola, >>>estas haciendo preguntas diferentes. Una cosa es hacer herramientas >>>visuales para windows. >>> >>>Otra cosa es tener un diseñador visual para estas herramientas. >>> >>>Las herramientas visuales para windows pueden ser hechas con Python y >>>algun toolkit que tenga compatibilidad en windows como lo es: >>>- Tk http://i1-win.softpedia-static.com/screenshots/Trebuchet-Tk_1.png >>>- Gtk  http://www.emcken.dk/screenshots/GTK%20i%20windows.jpg >>>- Qt4 http://sourceforge.net/projects/ncreport/screenshots/107225 >>>- WxWidgets http://www.wxwidgets.org/images/screens/gamedevelop_win.jpg >>> >>>Estos toolkits corren en windows como podras ver en las imagenes. Sin >>>embargo la programación es hecha dentro del codigo. >>> >>>El diseñador grafico es algo diferente el cual usualmente es un >>>programa dedicado a dibujar la interfaz. El diseñador para el toolkit >>>es algo que parece que es lo que estas buscando. No todos los toolkits >>>tienen un diseñado. La forma mas tradicional es que el diseñador >>>genera un XML copuesto de los componentes del toolkit y el codigo coje >>>estos elementos para hacer el rendering de instrucciones. >>> >>>http://doc.qt.digia.com/4.7-snapshot/designer-manual.html >>> >>>GTK usa Glade, TK y wxWidgets ignoro si tienen un diseñador o varios. >>>Tambien no se haya un diseñador universal que aplique una conversion a >>>todos los toolkits. >>> >>> >>>On 9/22/12, kausdiv wrote: >>>> Hola de nuevo. >>>> >>>> No consigo encontrar un sistema... >>>-- >>>Alexandro Colorado >>>PPMC Apache OpenOffice >>>http://es.openoffice.org >>> >>>_______________________________________________ >>>Python-es mailing list >>>Python-es en python.org >>>http://m... >>_______________________________________________ >>Python-es mailing list >>Python-es en python.org >>http://mail.python.org/mailman/listinfo/python-es >>FAQ: http://python-es-faq.wikidot.com/ >> >> > > > -- > Francisco Moreno > > _______________________________________________ > Python-es mailing list > Python-es en python.org > http://mail.python.org/mailman/listinfo/python-es > FAQ: http://python-es-faq.wikidot.com/ -- Alexandro Colorado PPMC Apache OpenOffice http://es.openoffice.org From carlos.zun en gmail.com Fri Oct 5 20:55:16 2012 From: carlos.zun en gmail.com (Carlos Zuniga) Date: Fri, 5 Oct 2012 13:55:16 -0500 Subject: [Python-es] =?utf-8?q?Aplicaci=C3=B3n_escritorio_para_windows?= In-Reply-To: References: <50445381.2080401@gmail.com> <505D52E3.6050201@gmail.com> <1349444009.16611.YahooMailNeo@web124901.mail.ne1.yahoo.com> Message-ID: 2012/10/5 Alexandro Colorado : > Seria interesante un tutorial de esto, supongo que mucha gente nueva a > python y que necesita trabajar en windows, le gustaria ver un ejemplo > de algun gui siendo compilado para su plataforma sobretodo si viene de > toolkits libres como GTK. > Tutoriales hay, el problema es que muchos se meten a preguntar antes de buscar... https://encrypted.google.com/search?q=py2exe%20tutorial https://encrypted.google.com/search?q=gui2exe%20tutorial Saludos -- A menudo unas pocas horas de "Prueba y error" podrán ahorrarte minutos de leer manuales. From jcaballero.hep en gmail.com Fri Oct 5 21:49:28 2012 From: jcaballero.hep en gmail.com (Jose Caballero) Date: Fri, 5 Oct 2012 15:49:28 -0400 Subject: [Python-es] encapsular llamadas a metodos arbitrarios desde una clase container In-Reply-To: References: Message-ID: > > Tal vez algo como: > > class Cont: > ... > def __getattr__(self, atr): > def foo(): > outs = [] > for o in self.list_obj: > out = getattr(o, atr)() > outs.append(out) > return outs > return foo > > > Funciona a la perfeccion. De esa forma se devuelve una funcion, y no el resultado de la misma, y asi puedo hacer luego cont.f() Muchas gracias ! > > UTF existe, ya no es excusa ;-) > Vale. Ahora me toca googlear para intentar entender lo que eso significa y, sobre todo, como me permite escribir con tildes en un teclado americano y sin .Xmodmap. ------------ próxima parte ------------ Se ha borrado un adjunto en formato HTML... URL: From carlos.zun en gmail.com Fri Oct 5 22:40:21 2012 From: carlos.zun en gmail.com (Carlos Zuniga) Date: Fri, 5 Oct 2012 15:40:21 -0500 Subject: [Python-es] encapsular llamadas a metodos arbitrarios desde una clase container In-Reply-To: References: Message-ID: 2012/10/5 Jose Caballero : > >> >> >> UTF existe, ya no es excusa ;-) > > > > Vale. Ahora me toca googlear para intentar entender lo que eso significa y, > sobre todo, como me permite escribir con tildes en un teclado americano y > sin .Xmodmap. > setxkbmap -layout "us(intl)" O en xorg.conf: Section "InputDevice" ... Option "XkbLayout" "us" Option "XkbVariant" "intl" EndSection Saludos -- A menudo unas pocas horas de "Prueba y error" podrán ahorrarte minutos de leer manuales. From jcaballero.hep en gmail.com Fri Oct 5 22:57:38 2012 From: jcaballero.hep en gmail.com (Jose Caballero) Date: Fri, 5 Oct 2012 16:57:38 -0400 Subject: [Python-es] encapsular llamadas a metodos arbitrarios desde una clase container In-Reply-To: References: Message-ID: On Oct 5, 2012, at 15:49, Jose Caballero wrote: > > > Tal vez algo como: > > class Cont: > ... > def __getattr__(self, atr): > def foo(): > outs = [] > for o in self.list_obj: > out = getattr(o, atr)() > outs.append(out) > return outs > return foo > > > > Funciona a la perfeccion. De esa forma se devuelve una funcion, y no el resultado de la misma, y asi puedo hacer luego cont.f() > Muchas gracias ! > > Hmmm. Ahora tengo que investigar qué hacer si quiero que alguno de los métodos acepte inputs. Por ej: container.f(1, 2, 3) Tengo que averiguar cómo pasar 1 2 3 a través de foo(). Imagino que haciendo algo tipo def foo(*k, **kw): ... out = getatr(o, atr)(k, kw) ... Jose ahora en una máquina con tildes ;) ------------ próxima parte ------------ Se ha borrado un adjunto en formato HTML... URL: From carlos.zun en gmail.com Fri Oct 5 23:59:59 2012 From: carlos.zun en gmail.com (Carlos Zuniga) Date: Fri, 5 Oct 2012 16:59:59 -0500 Subject: [Python-es] encapsular llamadas a metodos arbitrarios desde una clase container In-Reply-To: References: Message-ID: 2012/10/5 Jose Caballero : > Hmmm. > Ahora tengo que investigar qué hacer si quiero que alguno de los métodos > acepte inputs. > Por ej: > > container.f(1, 2, 3) > > Tengo que averiguar cómo pasar 1 2 3 a través de foo(). > Imagino que haciendo algo tipo > > def foo(*k, **kw): > ... > out = getatr(o, atr)(k, kw) Solo tienes que pasarle la expansión de argumentos a la función: out = getatr(o, atr)(*k, **kw) > > Jose > ahora en una máquina con tildes ;) :) Saludos -- A menudo unas pocas horas de "Prueba y error" podrán ahorrarte minutos de leer manuales. From txema en nabla.net Sat Oct 6 00:56:55 2012 From: txema en nabla.net (Txema Vicente) Date: Sat, 06 Oct 2012 00:56:55 +0200 Subject: [Python-es] Saber si un modulo existe aunque contenga un error ImportError. Message-ID: <506F65B7.2050600@nabla.net> Buenas. Acabo de topar con un detalle que no se me ocurre como resolver. En resumen, quiero poder distinguir entre estas dos situaciones: 1.- Excepcion ImportError porque un modulo no existe. 2.- Excepcion ImportError porque el modulo existe pero al importarlo tiene un error dentro, del tipo ImportError Ideas? Detalles: Estoy haciendo un toolkit en App Engine, y tengo un paquete "call" en el que voy metiendo modulos que se encargan de responder peticiones HTTP. Básicamente si se llama a http://...//call/hello/world, se mira a ver si existe un modulo call.hello.world, y si es asi se importa y la respuesta la da su método main(). Si hay un error en el modulo, la respuesta es el traceback. Si no existe el modulo, se va probando hacia arriba "call.hello" y finalmente call.main() dice que no tiene respuesta para esa ruta. Esto es un trozo de la clase que hace esto: @route(r'/pyojo/', "call") class Command(Request): def __module(self, command): code = None try: module = import_call(command) code = module.main(self) except ImportError: pass #<---- Problema si el modulo si que existe except Exception: code = Config.log_exception(command) return code Y esta es la funcion para importar, que funciona bien: def import_call(name, module=".call"): name = module+"."+name mod = __import__(name) components = name.split('.') for comp in components[1:]: mod = getattr(mod, comp) return mod Salud. From jza en oooes.org Sat Oct 6 15:52:21 2012 From: jza en oooes.org (Alexandro Colorado) Date: Sat, 6 Oct 2012 08:52:21 -0500 Subject: [Python-es] Tutoriales de Python para GUIs Message-ID: Se me hizo interesante esta coleccion de tutoriales de Python (y otros lenguajes) para desarrollar aplicaciones graficas en diferentes toolkits. http://zetcode.com/ -- Alexandro Colorado PPMC Apache OpenOffice http://es.openoffice.org From pych3m4 en gmail.com Sat Oct 6 18:47:22 2012 From: pych3m4 en gmail.com (Chema Cortes) Date: Sat, 6 Oct 2012 18:47:22 +0200 Subject: [Python-es] Saber si un modulo existe aunque contenga un error ImportError. In-Reply-To: <506F65B7.2050600@nabla.net> References: <506F65B7.2050600@nabla.net> Message-ID: El día 6 de octubre de 2012 00:56, Txema Vicente escribió: > Acabo de topar con un detalle que no se me ocurre como resolver. En resumen, > quiero poder distinguir entre estas dos situaciones: > > 1.- Excepcion ImportError porque un modulo no existe. > 2.- Excepcion ImportError porque el modulo existe pero al importarlo > tiene un error dentro, del tipo ImportError > > Ideas? Prueba a buscar el módulo con 'imp.find_module()' . Ten en cuenta que cuando se trata de buscar submódulos en un paquete, no te queda otro remedio que hacer la búsqueda de arriba a abajo, cargando cada uno de los módulos padre antes de hacer la búsqueda del submódulo (Viene explicado en la documentación). -- Hyperreals *R: http://ch3m4.org/blog Quarks, bits y otras criaturas infinitesimales From leandrodemarco en gmail.com Sat Oct 6 19:31:40 2012 From: leandrodemarco en gmail.com (Leandro Demarco Vedelago) Date: Sat, 6 Oct 2012 14:31:40 -0300 Subject: [Python-es] Ejecutar cliente/servidor SSL en la misma maquina Message-ID: Hola gente: estoy trabajando en una aplicacion que requiere un cliente/servidor. Como va a haber logueo y/o registro involucrado me decidi por usar SSL para encriptar la comunicacion y passlib para hacer hashing de las contrasenhas del lado del servidor. En fin, ayer lo termine y estuve testeandolo, ejecutando el servidor y el cliente en maquinas separadas y anduvo a la perfeccion. Pero hoy intente hacer algo de testeo ejecutando ambos en la misma maquina y me tira un error extranho. A continuacion el codigo relevante del servidor y del cliente, respectivamente: class Server(DirectObject): def __init__(self): # Socket where the server will be polling for incoming connections self.listener = socket.socket(socket.AF_INET, socket.SOCK_STREAM) self.listener.bind(('127.0.0.1', 9099)) self.activeSSLSockets = [] def updateConnections(self, task): r,w,e = select.select([self.listener], [], [], 0.01) if r != []: # We have a new connection! clientSock, addr = self.listener.accept() print "New connection from " + str(addr) # Wrap it under SSL so that all communication is encrypted encryptedSock = ssl.wrap_socket(clientSock, server_side=True, \ keyfile="privkey.pem", certfile="cacert.pem", \ do_handshake_on_connect=False,\ ssl_version=ssl.PROTOCOL_SSLv3) self.activeSSLSockets.append(encryptedSock) return task.cont def updateDatagrams(self, task): r = [] if self.activeSSLSockets != []: # Windows raises error if 3 arguments of select.select are empty r,w,e = select.select(self.activeSSLSockets, [], [], 0.01) for sslSock in r: data = sslSock.read(1024) self.process(data, sslSock) return task.cont El socket del cliente se crea en la siguiente funcion, connectAndThen: def connectAndThen(self, user, password, nTries=1, nextStep=None): """ Attempts to establish a connection through self.sckt with the server. It tries at most nTries, if in those nTries connection wasn't established, self.sckt remains None. @nextStep is the action to perform once connection is established It can be login(user, password) or register(user,password) """ i = 0 timeOut = 3000 while (not self.sckt and i References: Message-ID: ni idea del error pero pegue to códe en paste bin para que sea mas legible http://pastebin.com/9VKDYpeq 2012/10/6 Leandro Demarco Vedelago > ssl.SSLError: [Errno 1] _ssl.c:1354: error:1408F044:SSL > routines:SSL3_GET_RECORD:internal error > ------------ próxima parte ------------ Se ha borrado un adjunto en formato HTML... URL: From eltodi en gmail.com Sat Oct 6 20:16:53 2012 From: eltodi en gmail.com (Elias Torres) Date: Sat, 6 Oct 2012 20:16:53 +0200 Subject: [Python-es] Problema con fechas Message-ID: Buenas, Soy nuevo en python y ya me he encontrado con el primer problema que me hace llevar un día investigando, y nada... En mi modelo, guardo un campo fecha = models.Datetime,.... y luego quiero hacer una consulta que me devuelva solo las entradas donde fecha > hora actual ahora = datetime.datetime.now() entradas = Noticias.objects.filter(fecha > ahora) pero esto así no funciona, y por mas que busco, no encuentro. Pueden ayudarme? GRACIAS ------------ próxima parte ------------ Se ha borrado un adjunto en formato HTML... URL: From lgmerino en yahoo.com Sat Oct 6 21:11:07 2012 From: lgmerino en yahoo.com (Luis Garcia Merino) Date: Sat, 6 Oct 2012 20:11:07 +0100 (BST) Subject: [Python-es] Problema con fechas In-Reply-To: References: Message-ID: <1349550667.62840.YahooMailNeo@web171202.mail.ir2.yahoo.com> Buenas, prueba con: entradas = Noticias.objects.filter(fecha__gt=ahora) Si pones: fecha__gt -> mayor que fecha__gte -> mayor o igual fecha__lt -> menor que fecha__lte -> menor o igual Si no lo has leído, échale un vistazo a [https://docs.djangoproject.com/en/dev/topics/db/queries/] Un saludo. ________________________________ De: Elias Torres Para: Python-es en python.org Enviado: Sábado 6 de octubre de 2012 20:16 Asunto: [Python-es] Problema con fechas Buenas, Soy nuevo en python y ya me he encontrado con el primer problema que me hace llevar un día investigando, y nada... En mi modelo, guardo un campo fecha = models.Datetime,.... y luego quiero hacer una consulta que me devuelva solo las entradas  donde fecha > hora actual ahora = datetime.datetime.now() entradas = Noticias.objects.filter(fecha > ahora)  pero esto así no funciona, y por mas que busco, no encuentro. Pueden ayudarme? GRACIAS _______________________________________________ Python-es mailing list Python-es en python.org http://mail.python.org/mailman/listinfo/python-es FAQ: http://python-es-faq.wikidot.com/ ------------ próxima parte ------------ Se ha borrado un adjunto en formato HTML... URL: From eltodi en gmail.com Sat Oct 6 21:24:09 2012 From: eltodi en gmail.com (Elias Torres) Date: Sat, 6 Oct 2012 21:24:09 +0200 Subject: [Python-es] Problema con fechas In-Reply-To: <1349550667.62840.YahooMailNeo@web171202.mail.ir2.yahoo.com> References: <1349550667.62840.YahooMailNeo@web171202.mail.ir2.yahoo.com> Message-ID: Muchas gracias Luís, Me ha ido perfecto. Mil gracias El 6 de octubre de 2012 21:11, Luis Garcia Merino escribió: > > Buenas, > > prueba con: > > entradas = Noticias.objects.filter(fecha__gt=ahora) > > Si pones: > > fecha__gt -> mayor que > fecha__gte -> mayor o igual > fecha__lt -> menor que > fecha__lte -> menor o igual > > Si no lo has leído, échale un vistazo a [ > https://docs.djangoproject.com/en/dev/topics/db/queries/] > > Un saludo. > > ------------------------------ > *De:* Elias Torres > *Para:* Python-es en python.org > *Enviado:* Sábado 6 de octubre de 2012 20:16 > *Asunto:* [Python-es] Problema con fechas > > Buenas, > > Soy nuevo en python y ya me he encontrado con el primer problema que me > hace llevar un día investigando, y nada... > > En mi modelo, guardo un campo fecha = models.Datetime,.... > y luego quiero hacer una consulta que me devuelva solo las entradas donde > fecha > hora actual > > ahora = datetime.datetime.now() > > entradas = Noticias.objects.filter(fecha > ahora) > > pero esto así no funciona, y por mas que busco, no encuentro. > > Pueden ayudarme? > > GRACIAS > > _______________________________________________ > Python-es mailing list > Python-es en python.org > http://mail.python.org/mailman/listinfo/python-es > FAQ: http://python-es-faq.wikidot.com/ > > > > _______________________________________________ > Python-es mailing list > Python-es en python.org > http://mail.python.org/mailman/listinfo/python-es > FAQ: http://python-es-faq.wikidot.com/ > > ------------ próxima parte ------------ Se ha borrado un adjunto en formato HTML... URL: From gmourinopardo en yahoo.es Sat Oct 6 23:26:52 2012 From: gmourinopardo en yahoo.es (=?iso-8859-1?Q?Gonzalo_Mouri=FFfffffffffff1o_Pardo?=) Date: Sat, 6 Oct 2012 22:26:52 +0100 (BST) Subject: [Python-es] Variables wxpython In-Reply-To: References: <1349446358.90812.YahooMailNeo@web29601.mail.ird.yahoo.com> Message-ID: <1349558812.15799.YahooMailNeo@web29604.mail.ird.yahoo.com> Gracias tenias razon no iba me faltaba instanciar la aplicacion. ________________________________ De: Carlos Zuniga Para: La lista de python en castellano Enviado: Viernes 5 de octubre de 2012 18:53 Asunto: Re: [Python-es] Variables wxpython On Fri, Oct 5, 2012 at 9:12 AM, Gonzalo Mouriÿfffffffffff1o Pardo wrote: > Buenas llevo mas o menos 2 semanas aprendiendo a programar en python, y me > encuentro en una encrucijada. > Necesito sacar información de Ventana_New (tc1) y llevarlo al > Ventana.SetTitle, no se como podría hacerlo, gracias. > Ahí va mi código: > > import wx > > class Aplicacion(wx.App): > >    def OnInit(self): >        self.frame = Ventana(None, -1, title = "Control Horario") >        self.SetTopWindow(self.frame) >        self.frame.Show() >        return True > > class Ventana(wx.Frame): > >    def __init__(self, parent, id, title): >        super(Ventana, self).__init__(parent, id, title, size = (800, 275)) > panel = wx.Panel(self) > menubar = wx.MenuBar() > arcm = wx.Menu() >        anmi = wx.MenuItem(arcm, ID_NUEVO, "&Nuevo\tCtrl+N") >        arcm.AppendItem(anmi) >        menubar.Append(arcm, '&Archivo') >        self.SetMenuBar(menubar) >        self.Bind(wx.EVT_MENU, self.OnMenu) > >    def OnMenu(self, event): >  evt_id = event.GetId () >  if evt_id == ID_NUEVO: >      self.frame = Ventana_New(None, -1, title = "Nuevo Usuario") > > class Ventana_New(wx.Frame): > >    def __init__(self, parent, id, title): >        self.padre = parent >        wx.Frame.__init__(self, parent, id, title, size = (295, 357)) >        self.panel = Panel_New(self) >        self.Show(True) > > class Panel_New(wx.Panel): > >    def __init__(self, parent, *args, **kwargs): >        self.padre = parent >        wx.Panel.__init__(self, parent, *args, **kwargs) >        gs = wx.FlexGridSizer(3, 2, 9, 9) >        vbox = wx.BoxSizer(wx.VERTICAL) >        hbox = wx.BoxSizer(wx.HORIZONTAL) >        nombre = wx.StaticText(self, -1, "Nombre:") >        self.tc1 = wx.TextCtrl(self, -1, size = (150, -1)) > gs.AddMany([(nombre), (self.tc1, 1, wx.LEFT, 10)] >        self.btn = wx.Button(self, -1, 'Aceptar', size = (-1, 30)) >        hbox.Add(self.btn) >        vbox.Add(hbox, 0, wx.ALIGN_CENTER | wx.BOTTOM, 10) >        self.Bind(wx.EVT_BUTTON, self.OnAceptar, id = self.btn.GetId()) >        self.SetSizer(vbox) > >    def OnAceptar(self, event): >        padre = self.GetParent() >        padre.Close(True) > Hola, tu ejemplo no corre así que no lo puedo probar... pero lo que puedes hacer es añadir un evento close:     def OnMenu(self, event):         evt_id = event.GetId()         if evt_id == ID_NUEVO:             self.frame = Ventana_New(None, -1, title = "Nuevo Usuario")             self.frame.Bind(wx.EVT_CLOSE, self.OnCloseMenu)             self.frame.Show(True)     def OnCloseMenu(self, event):         print self.frame.panel.tc1.GetValue()         self.frame.Destroy() También tendrías que chequear si solo se ha cerrado la ventana o se ha presionado el boton Aceptar. Añade un atributo aceptado con valor False a Ventana_New y lo cambias a True en OnAceptar. Luego en OnCloseMenu chequeas `if self.frame.aceptado: ...` Saludos -- A menudo unas pocas horas de "Prueba y error" podrán ahorrarte minutos de leer manuales. _______________________________________________ Python-es mailing list Python-es en python.org http://mail.python.org/mailman/listinfo/python-es FAQ: http://python-es-faq.wikidot.com/ ------------ próxima parte ------------ Se ha borrado un adjunto en formato HTML... URL: From txema en nabla.net Sun Oct 7 14:16:37 2012 From: txema en nabla.net (Txema Vicente) Date: Sun, 07 Oct 2012 14:16:37 +0200 Subject: [Python-es] Saber si un modulo existe aunque contenga un error ImportError. In-Reply-To: References: <506F65B7.2050600@nabla.net> Message-ID: <507172A5.3090709@nabla.net> Gracias, no habia visto imp y me va a venir muy bien. Si no lo encuentra da ImportError, pero parece que no lo ejecuta, asi que valdria. Lo que no me acaba de gustar es tener que cargar modulos superiores para ir buscando, porque cuento con que pueden tener errores, pero al final va a terminar cargando todos, asi que intentare precargar todos de una vez al arrancar el servidor, y listar los errores. El 06/10/2012 18:47, Chema Cortes escribió: > El día 6 de octubre de 2012 00:56, Txema Vicente escribió: > >> Acabo de topar con un detalle que no se me ocurre como resolver. En resumen, >> quiero poder distinguir entre estas dos situaciones: >> >> 1.- Excepcion ImportError porque un modulo no existe. >> 2.- Excepcion ImportError porque el modulo existe pero al importarlo >> tiene un error dentro, del tipo ImportError >> >> Ideas? > Prueba a buscar el módulo con 'imp.find_module()' . Ten en cuenta que > cuando se trata de buscar submódulos en un paquete, no te queda otro > remedio que hacer la búsqueda de arriba a abajo, cargando cada uno de > los módulos padre antes de hacer la búsqueda del submódulo (Viene > explicado en la documentación). > > > > From pych3m4 en gmail.com Sun Oct 7 17:47:07 2012 From: pych3m4 en gmail.com (Chema Cortes) Date: Sun, 7 Oct 2012 17:47:07 +0200 Subject: [Python-es] Saber si un modulo existe aunque contenga un error ImportError. In-Reply-To: <507172A5.3090709@nabla.net> References: <506F65B7.2050600@nabla.net> <507172A5.3090709@nabla.net> Message-ID: El día 7 de octubre de 2012 14:16, Txema Vicente escribió: > Gracias, no habia visto imp y me va a venir muy bien. > Si no lo encuentra da ImportError, pero parece que no lo ejecuta, asi que > valdria. > > Lo que no me acaba de gustar es tener que cargar modulos superiores para ir > buscando, > porque cuento con que pueden tener errores, pero al final va a terminar > cargando todos, > asi que intentare precargar todos de una vez al arrancar el servidor, y > listar los errores. Lo de ir cargando los módulos es para incorporar el '__path__' de los paquetes al sys.path de búsqueda. Si no quieres que lo haga así, basta con indicar directamente los directorios de búsqueda en el 'imp.find_modules'. Puedes, incluso, llevar tu propia lista de directorios de búsqueda en paralelo, sin tener que cargar ningún módulo y sin afectar al sys.path -- Hyperreals *R: http://ch3m4.org/blog Quarks, bits y otras criaturas infinitesimales From txema en nabla.net Sun Oct 7 23:57:49 2012 From: txema en nabla.net (Txema Vicente) Date: Sun, 07 Oct 2012 23:57:49 +0200 Subject: [Python-es] Saber si un modulo existe aunque contenga un error ImportError. In-Reply-To: References: <506F65B7.2050600@nabla.net> <507172A5.3090709@nabla.net> Message-ID: <5071FADD.80404@nabla.net> Perfecto, porque en mi caso se exactamente en que directorio quiero buscar. Me lei la documentacion y pense que no le podia dar un directorio que puede no existir, pero si. Ya esta solucionado, muchas gracias. El 07/10/2012 17:47, Chema Cortes escribió: > El día 7 de octubre de 2012 14:16, Txema Vicente escribió: >> Gracias, no habia visto imp y me va a venir muy bien. >> Si no lo encuentra da ImportError, pero parece que no lo ejecuta, asi que >> valdria. >> >> Lo que no me acaba de gustar es tener que cargar modulos superiores para ir >> buscando, >> porque cuento con que pueden tener errores, pero al final va a terminar >> cargando todos, >> asi que intentare precargar todos de una vez al arrancar el servidor, y >> listar los errores. > Lo de ir cargando los módulos es para incorporar el '__path__' de los > paquetes al sys.path de búsqueda. Si no quieres que lo haga así, basta > con indicar directamente los directorios de búsqueda en el > 'imp.find_modules'. Puedes, incluso, llevar tu propia lista de > directorios de búsqueda en paralelo, sin tener que cargar ningún > módulo y sin afectar al sys.path > > > From txema en nabla.net Mon Oct 8 18:19:39 2012 From: txema en nabla.net (Txema Vicente) Date: Mon, 08 Oct 2012 18:19:39 +0200 Subject: [Python-es] webapp2 y REST - Guardar el estado del cliente con cookies. Message-ID: <5072FD1B.2060404@nabla.net> Buenas. Estoy intentando hacerme un toolkit con webapp2 en GAE, que me facilite producir aplicaciones Dojo desde python. La idea es que el cliente entra a la pagina principal (State: Ninguno), se le envia HTML y el Javascript básico (State: Iniciando), y despues de eso se crea el interfaz de usuario, y todas las peticiones a partir de aqui (Estado: Listo) son AJAX ó "Dojo-require" que le dan más javascript al cliente. No se responde nada si no viene de un cliente ya "Listo", y si la peticion requiere un State distinto, se le pasan las tareas al cliente que lo lleven a ese estado. Como intento que sea RESTfull, si lo entiendo bien no debo usar sesiones, sino que el propio cliente siempre debe comunicar su Estado en cada petición. Asi que lo hago con cookies, el problema es que si abre otra pestaña, las cookies siguen ahi pero el Estado no es correcto. De momento lo he arreglado así: - Al iniciar, javascript crea una cookie "state_hash" con un número único XXX que le ha pasado python. - Cada vez que hace una petición, el cliente javascript crea una cookie "state_check"=XXX - Cada vez que recibe una petición, el servidor python comprueba que state_hash==state_check: - Si: le elimina la cookie "state_check" y le responde lo que pida. - No: le responde 404 No cuela. Pero digo yo que para esto habra soluciones ya pensadas, no? Otra forma de hacerlo sin cookies? Agradezco enlaces o ideas sobre estos temas (REST, generar JS desde python). Saludos P.D: Se ve en http://www.vanstats.com. From pych3m4 en gmail.com Tue Oct 9 10:08:51 2012 From: pych3m4 en gmail.com (Chema Cortes) Date: Tue, 9 Oct 2012 10:08:51 +0200 Subject: [Python-es] webapp2 y REST - Guardar el estado del cliente con cookies. In-Reply-To: <5072FD1B.2060404@nabla.net> References: <5072FD1B.2060404@nabla.net> Message-ID: El día 8 de octubre de 2012 18:19, Txema Vicente escribió: > Estoy intentando hacerme un toolkit con webapp2 en GAE, que me facilite > producir aplicaciones Dojo desde python. > > La idea es que el cliente entra a la pagina principal (State: Ninguno), se > le envia HTML y el Javascript básico (State: Iniciando), y despues de eso se > crea el interfaz de usuario, y todas las peticiones a partir de aqui > (Estado: Listo) son AJAX ó "Dojo-require" que le dan más javascript al > cliente. No se responde nada si no viene de un cliente ya "Listo", y si la > peticion requiere un State distinto, se le pasan las tareas al cliente que > lo lleven a ese estado. > > Como intento que sea RESTfull, si lo entiendo bien no debo usar sesiones, > sino que el propio cliente siempre debe comunicar su Estado en cada > petición. > > Asi que lo hago con cookies, el problema es que si abre otra pestaña, las > cookies siguen ahi pero el Estado no es correcto. > > De momento lo he arreglado así: > > - Al iniciar, javascript crea una cookie "state_hash" con un número único > XXX que le ha pasado python. > - Cada vez que hace una petición, el cliente javascript crea una cookie > "state_check"=XXX > - Cada vez que recibe una petición, el servidor python comprueba que > state_hash==state_check: > - Si: le elimina la cookie "state_check" y le responde lo que pida. > - No: le responde 404 No cuela. > > Pero digo yo que para esto habra soluciones ya pensadas, no? > Otra forma de hacerlo sin cookies? > Agradezco enlaces o ideas sobre estos temas (REST, generar JS desde python). Aunque no conozco mucho estos sistemas, yo usaría exclusivamente las cookies para guardar estados "globales" tales como autenticaciones o parametrizaciones de la aplicación con los que construir más fácilmente la URL del servicio REST. Para lo que necesitas, veo más práctico crear código javascript "al vuelo" que incluya algún token con el que construir la URL según un formato aceptado por el servidor. Este token podría estar firmado por una clave pública para mayor seguridad, e incluir algún tipo de marca temporal para compararla con el reloj del cliente cuando sea necesario algún tipo de sincronismo o de timeout. -- Hyperreals *R: http://ch3m4.org/blog Quarks, bits y otras criaturas infinitesimales From lastvnm en gmail.com Tue Oct 9 19:18:52 2012 From: lastvnm en gmail.com (Hiko hitokiri) Date: Tue, 9 Oct 2012 11:18:52 -0600 Subject: [Python-es] webapp2 y REST - Guardar el estado del cliente con cookies. In-Reply-To: References: <5072FD1B.2060404@nabla.net> Message-ID: pues lo que puedes hacer es usar webstorage ya que te provee de mas almacenaje para tus configuraciones en el lado de el cliente que una cookie -- hiko_hitokiri no pidas que el mundo te trate de una manera cuando tu no le das el mismo trato a los demás ------------ próxima parte ------------ Se ha borrado un adjunto en formato HTML... URL: From olemis en gmail.com Tue Oct 9 20:55:54 2012 From: olemis en gmail.com (Olemis Lang) Date: Tue, 9 Oct 2012 13:55:54 -0500 Subject: [Python-es] Una pregunta sobre la conversion dot => xdot Message-ID: Hola a tod en s ! Recientemente necesito convertir unos diagramas en formato .dot para obtener la misma representación en formato .xdoc . Busqué una librería q permitiera hacer esto sin ejecutar los comandos de Graphviz en un proceso o hilo aparte . Finalmente encontré yapgvb [1]_ ... pero el siguiente ejemplo no me funciona . {{{ #!py >>> from yapgvb import Graph >>> from cStringIO import StringIO >>> gdot = StringIO("digraph G {Hello->World}") >>> g = Graph.read(gdot) Traceback (most recent call last): File "", line 1, in File "/usr/lib/python2.6/dist-packages/yapgvb/__init__.py", line 253, in read newgraph = cls(input_stream_or_filename) File "/usr/lib/python2.6/dist-packages/yapgvb/__init__.py", line 551, in __init__ GraphBase.__init__(self, arg) File "/usr/lib/python2.6/dist-packages/yapgvb/__init__.py", line 283, in __init__ CGraph.__init__(self,*args,**keywords) File "/usr/lib/python2.6/dist-packages/yapgvb/__init__.py", line 187, in cgraph_init_wrapper CGraph.__original_init__(self, *a,**b) Boost.Python.ArgumentError: Python argument types in CGraph.__init__(Graph, cStringIO.StringI) did not match C++ signature: __init__(_object*, _IO_FILE*) __init__(_object*, boost::python::str, agraph_type) __init__(_object*, boost::python::str) __init__(_object*) }}} Preguntas - ¿Conocen alguna forma de hacer funcionar este ejemplo y pasar el código .dot a graphviz utilizando un objeto StringIO o equivalente (i.e. sin utilizar el sistema de archivos ;) ? - ¿ Conocen alguna otra librería que permita hacer esto ? Gracias por adelantado . :) .. [1] yapgvb (http://code.google.com/p/yapgvb/) -- Regards, Olemis. Blog ES: http://simelo-es.blogspot.com/ Blog EN: http://simelo-en.blogspot.com/ Featured article: From aan.mcmxc en gmail.com Tue Oct 9 21:05:21 2012 From: aan.mcmxc en gmail.com (Axel Aguilar Nieto) Date: Tue, 9 Oct 2012 21:05:21 +0200 Subject: [Python-es] =?iso-8859-1?q?=BFDjangoeurope_de_dev_a_producci=F3n?= =?iso-8859-1?q?=3F?= Message-ID: Buenas, me he hecho mi primer hosting en djangoeurope, al ser la primera vez, cosas simples como subir por ssh archivos o crear dominios me ha costado un poco, pero bueno, está todo subido, y puedo ver mi web online en dev."dominio", pero solo si antes entro por ssh y hago un "sh RUN" del servidor. Esto no acabo de entenderlo... pero al ser dev, lo puedo llegar a comprender, pero la pregunta que no consigo contestar: ¿cómo puedo pasar esa web que me funciona en dev."dominio" al "dominio" simplemente? googleando encuentro más bien nada... espero haberme explicado. Gracias y saludos! ------------ próxima parte ------------ Se ha borrado un adjunto en formato HTML... URL: From carlos.zun en gmail.com Wed Oct 10 01:37:45 2012 From: carlos.zun en gmail.com (Carlos Zuniga) Date: Tue, 9 Oct 2012 18:37:45 -0500 Subject: [Python-es] Una pregunta sobre la conversion dot => xdot In-Reply-To: References: Message-ID: 2012/10/9 Olemis Lang : > Hola a tod en s ! > > Recientemente necesito convertir unos diagramas en formato .dot para > obtener la misma representación en formato .xdoc . Busqué una librería > q permitiera hacer esto sin ejecutar los comandos de Graphviz en un > proceso o hilo aparte . Finalmente encontré yapgvb [1]_ ... pero el > siguiente ejemplo no me funciona . > > {{{ > #!py > > >>> from yapgvb import Graph > >>> from cStringIO import StringIO > >>> gdot = StringIO("digraph G {Hello->World}") > >>> g = Graph.read(gdot) > Traceback (most recent call last): > File "", line 1, in > File "/usr/lib/python2.6/dist-packages/yapgvb/__init__.py", line 253, in read > newgraph = cls(input_stream_or_filename) > File "/usr/lib/python2.6/dist-packages/yapgvb/__init__.py", line > 551, in __init__ > GraphBase.__init__(self, arg) > File "/usr/lib/python2.6/dist-packages/yapgvb/__init__.py", line > 283, in __init__ > CGraph.__init__(self,*args,**keywords) > File "/usr/lib/python2.6/dist-packages/yapgvb/__init__.py", line > 187, in cgraph_init_wrapper > CGraph.__original_init__(self, *a,**b) > Boost.Python.ArgumentError: Python argument types in > CGraph.__init__(Graph, cStringIO.StringI) > did not match C++ signature: > __init__(_object*, _IO_FILE*) > __init__(_object*, boost::python::str, agraph_type) > __init__(_object*, boost::python::str) > __init__(_object*) > > }}} > > Preguntas > > - ¿Conocen alguna forma de hacer funcionar este ejemplo y > pasar el código .dot a graphviz utilizando un objeto StringIO o > equivalente (i.e. sin utilizar el sistema de archivos ;) ? Parece que su constructor también acepta cadenas de python, has intentado simplemente: >>> gdot = "digraph G {Hello->World}" >>> g = Graph.read(gdot) Saludos -- A menudo unas pocas horas de "Prueba y error" podrán ahorrarte minutos de leer manuales. From olemis en gmail.com Wed Oct 10 02:03:32 2012 From: olemis en gmail.com (Olemis Lang) Date: Tue, 9 Oct 2012 19:03:32 -0500 Subject: [Python-es] Una pregunta sobre la conversion dot => xdot In-Reply-To: References: Message-ID: On 10/9/12, Carlos Zuniga wrote: > 2012/10/9 Olemis Lang : >> Hola a tod en s ! >> >> Recientemente necesito convertir unos diagramas en formato .dot para >> obtener la misma representación en formato .xdoc . Busqué una librería >> q permitiera hacer esto sin ejecutar los comandos de Graphviz en un >> proceso o hilo aparte . Finalmente encontré yapgvb [1]_ ... pero el >> siguiente ejemplo no me funciona . >> >> {{{ >> #!py >> >> >>> from yapgvb import Graph >> >>> from cStringIO import StringIO >> >>> gdot = StringIO("digraph G {Hello->World}") >> >>> g = Graph.read(gdot) >> Traceback (most recent call last): >> File "", line 1, in >> File "/usr/lib/python2.6/dist-packages/yapgvb/__init__.py", line 253, >> in read >> newgraph = cls(input_stream_or_filename) >> File "/usr/lib/python2.6/dist-packages/yapgvb/__init__.py", line >> 551, in __init__ >> GraphBase.__init__(self, arg) >> File "/usr/lib/python2.6/dist-packages/yapgvb/__init__.py", line >> 283, in __init__ >> CGraph.__init__(self,*args,**keywords) >> File "/usr/lib/python2.6/dist-packages/yapgvb/__init__.py", line >> 187, in cgraph_init_wrapper >> CGraph.__original_init__(self, *a,**b) >> Boost.Python.ArgumentError: Python argument types in >> CGraph.__init__(Graph, cStringIO.StringI) >> did not match C++ signature: >> __init__(_object*, _IO_FILE*) >> __init__(_object*, boost::python::str, agraph_type) >> __init__(_object*, boost::python::str) >> __init__(_object*) >> >> }}} >> >> Preguntas >> >> - ¿Conocen alguna forma de hacer funcionar este ejemplo y >> pasar el código .dot a graphviz utilizando un objeto StringIO o >> equivalente (i.e. sin utilizar el sistema de archivos ;) ? > > Parece que su constructor también acepta cadenas de python, has > intentado simplemente: > >>>> gdot = "digraph G {Hello->World}" >>>> g = Graph.read(gdot) > rats ! Bue ... resulta ser q la definición creo q decía q si ese parámetro era de tipo str entonces representaba el camino a un fichero en el sistema de archivos . Sin embargo ... {{{ #!py >>> from yapgvb import Graph >>> gdot = "digraph G {Hello->World}" >>> g = Graph(gdot) >>> g World}"> >>> g.layout('dot') 0 >>> from StringIO import StringIO >>> gout = StringIO() >>> g.render(gout, 'xdot') Traceback (most recent call last): File "", line 1, in File "/usr/lib/python2.6/dist-packages/yapgvb/__init__.py", line 341, in render result = rendering_context.render(self, format, outstream) Boost.Python.ArgumentError: Python argument types in RenderingContext.render(RenderingContext, Graph, str, instance) did not match C++ signature: render(GVCWrapper {lvalue}, Graph*, boost::python::str, boost::python::str) render(GVCWrapper {lvalue}, Graph*, boost::python::str, _IO_FILE*) >>> from sys import stdout >>> g.render(stdout, 'xdot') graph "digraph G {Hello->World}" { graph [bb="0,0,0,0", _draw_="c 5 -white C 5 -white P 4 0 0 0 0 0 0 0 0 ", xdotversion="1.2"]; } 0 }}} ... o sea q si bien no tengo ese problema para leer el dot (50% a mi favor) continúo sufriendo las consecuencias para salvar el xdot en un stream in-memory . :-/ ... pero ... {{{ #!py >>> from cStringIO import StringIO >>> gout = StringIO() >>> g.render(stdout, 'xdot') graph "digraph G {Hello->World}" { graph [bb="0,0,0,0", _draw_="c 5 -white C 5 -white P 4 0 0 0 0 0 0 0 0 ", xdotversion="1.2"]; } 0 }}} al parecer como cStringIO está hecho en C resulta más amigable para «conectarse» con los bindings de Boost.Python ... y bue ... funciona 100% :) Gracias por su ayuda ! -- Regards, Olemis. Blog ES: http://simelo-es.blogspot.com/ Blog EN: http://simelo-en.blogspot.com/ Featured article: Resolviendo el problema 1 de la IMO 2012 con inversión - http://simelo-es.blogspot.com/2012/08/resolviendo-el-problema-1-de-la-imo.html From olemis en gmail.com Wed Oct 10 02:23:05 2012 From: olemis en gmail.com (Olemis Lang) Date: Tue, 9 Oct 2012 19:23:05 -0500 Subject: [Python-es] Una pregunta sobre la conversion dot => xdot In-Reply-To: References: Message-ID: On 10/9/12, Olemis Lang wrote: > On 10/9/12, Carlos Zuniga wrote: >> 2012/10/9 Olemis Lang : >>> Hola a tod en s ! >>> >>> Recientemente necesito convertir unos diagramas en formato .dot para >>> obtener la misma representación en formato .xdoc . Busqué una librería >>> q permitiera hacer esto sin ejecutar los comandos de Graphviz en un >>> proceso o hilo aparte . Finalmente encontré yapgvb [1]_ ... pero el >>> siguiente ejemplo no me funciona . >>> >>> {{{ >>> #!py >>> >>> >>> from yapgvb import Graph >>> >>> from cStringIO import StringIO >>> >>> gdot = StringIO("digraph G {Hello->World}") >>> >>> g = Graph.read(gdot) >>> Traceback (most recent call last): >>> File "", line 1, in >>> File "/usr/lib/python2.6/dist-packages/yapgvb/__init__.py", line 253, >>> in read >>> newgraph = cls(input_stream_or_filename) >>> File "/usr/lib/python2.6/dist-packages/yapgvb/__init__.py", line >>> 551, in __init__ >>> GraphBase.__init__(self, arg) >>> File "/usr/lib/python2.6/dist-packages/yapgvb/__init__.py", line >>> 283, in __init__ >>> CGraph.__init__(self,*args,**keywords) >>> File "/usr/lib/python2.6/dist-packages/yapgvb/__init__.py", line >>> 187, in cgraph_init_wrapper >>> CGraph.__original_init__(self, *a,**b) >>> Boost.Python.ArgumentError: Python argument types in >>> CGraph.__init__(Graph, cStringIO.StringI) >>> did not match C++ signature: >>> __init__(_object*, _IO_FILE*) >>> __init__(_object*, boost::python::str, agraph_type) >>> __init__(_object*, boost::python::str) >>> __init__(_object*) >>> >>> }}} >>> >>> Preguntas >>> >>> - ¿Conocen alguna forma de hacer funcionar este ejemplo y >>> pasar el código .dot a graphviz utilizando un objeto StringIO o >>> equivalente (i.e. sin utilizar el sistema de archivos ;) ? >> >> Parece que su constructor también acepta cadenas de python, has >> intentado simplemente: >> >>>>> gdot = "digraph G {Hello->World}" >>>>> g = Graph.read(gdot) >> > > rats ! > [...] A ver , rectifico todo lo q dije antes (y no repetí ...) pq está terriblemente mal . Probé esto con cierta prisa y está mal todo lo q dije ... a ver ... comentarios in-line en el código ... {{{ #!py # Imports >>> from yapgvb import Graph >>> from sys import stdout >>> import cStringIO >>> import StringIO # .dot graph >>> gdot = "digraph G {Hello->World}" # Passing .dot grap in to __init__ >>> g = Graph(gdot) >>> g.layout('dot') 0 # Same problem with cStringIO >>> gout = cStringIO.StringIO() >>> g.render(gout, 'xdot') Traceback (most recent call last): File "", line 1, in File "/usr/lib/python2.6/dist-packages/yapgvb/__init__.py", line 341, in render result = rendering_context.render(self, format, outstream) Boost.Python.ArgumentError: Python argument types in RenderingContext.render(RenderingContext, Graph, str, cStringIO.StringO) did not match C++ signature: render(GVCWrapper {lvalue}, Graph*, boost::python::str, boost::python::str) render(GVCWrapper {lvalue}, Graph*, boost::python::str, _IO_FILE*) # Same problem with StringIO >>> gout = StringIO.StringIO() >>> g.render(gout, 'xdot') Traceback (most recent call last): File "", line 1, in File "/usr/lib/python2.6/dist-packages/yapgvb/__init__.py", line 341, in render result = rendering_context.render(self, format, outstream) Boost.Python.ArgumentError: Python argument types in RenderingContext.render(RenderingContext, Graph, str, instance) did not match C++ signature: render(GVCWrapper {lvalue}, Graph*, boost::python::str, boost::python::str) render(GVCWrapper {lvalue}, Graph*, boost::python::str, _IO_FILE*) # Works with stdout but ... outputs empty graph :'( >>> g.render(stdout, 'xdot') graph "digraph G {Hello->World}" { graph [bb="0,0,0,0", _draw_="c 5 -white C 5 -white P 4 0 0 0 0 0 0 0 0 ", xdotversion="1.2"]; } 0 >>> g = Graph() # Reading from string ... rats ! >>> g.read(gdot) Traceback (most recent call last): File "", line 1, in File "/usr/lib/python2.6/dist-packages/yapgvb/__init__.py", line 249, in read input_stream_or_filename = open(input_stream_or_filename,'r') IOError: [Errno 2] No such file or directory: 'digraph G {Hello->World}' # Same problem reading from cStringIO >>> gin = cStringIO.StringIO(gdot) >>> g.read(gin) Traceback (most recent call last): File "", line 1, in File "/usr/lib/python2.6/dist-packages/yapgvb/__init__.py", line 253, in read newgraph = cls(input_stream_or_filename) File "/usr/lib/python2.6/dist-packages/yapgvb/__init__.py", line 551, in __init__ GraphBase.__init__(self, arg) File "/usr/lib/python2.6/dist-packages/yapgvb/__init__.py", line 283, in __init__ CGraph.__init__(self,*args,**keywords) File "/usr/lib/python2.6/dist-packages/yapgvb/__init__.py", line 187, in cgraph_init_wrapper CGraph.__original_init__(self, *a,**b) Boost.Python.ArgumentError: Python argument types in CGraph.__init__(Graph, cStringIO.StringI) did not match C++ signature: __init__(_object*, _IO_FILE*) __init__(_object*, boost::python::str, agraph_type) __init__(_object*, boost::python::str) __init__(_object*) # Same problem from StringIO >>> gin = StringIO.StringIO(gdot) >>> g.read(gin) Traceback (most recent call last): File "", line 1, in File "/usr/lib/python2.6/dist-packages/yapgvb/__init__.py", line 253, in read newgraph = cls(input_stream_or_filename) File "/usr/lib/python2.6/dist-packages/yapgvb/__init__.py", line 551, in __init__ GraphBase.__init__(self, arg) File "/usr/lib/python2.6/dist-packages/yapgvb/__init__.py", line 283, in __init__ CGraph.__init__(self,*args,**keywords) File "/usr/lib/python2.6/dist-packages/yapgvb/__init__.py", line 187, in cgraph_init_wrapper CGraph.__original_init__(self, *a,**b) Boost.Python.ArgumentError: Python argument types in CGraph.__init__(Graph, instance) did not match C++ signature: __init__(_object*, _IO_FILE*) __init__(_object*, boost::python::str, agraph_type) __init__(_object*, boost::python::str) __init__(_object*) }}} o sea ... q realmente no he avanzado nada . De todas formas gracias por su respuesta . -- Regards, Olemis. Blog ES: http://simelo-es.blogspot.com/ Blog EN: http://simelo-en.blogspot.com/ Featured article: From gmourinopardo en yahoo.es Wed Oct 10 14:43:50 2012 From: gmourinopardo en yahoo.es (=?iso-8859-1?Q?Gonzalo_Mouri=FFfffffffffff1o_Pardo?=) Date: Wed, 10 Oct 2012 13:43:50 +0100 (BST) Subject: [Python-es] Dudas de centrado de texto In-Reply-To: <5065cb9c.145b650a.6351.7a77@mx.google.com> References: <5065cb9c.145b650a.6351.7a77@mx.google.com> Message-ID: <1349873030.16521.YahooMailNeo@web133205.mail.ir2.yahoo.com> Buenas tardes, tengo otra duda: Necesito centrar un wx.Statictext (tc1) en la pantalla, pero al tenenr una fuente asociada no puedo como lo hago, gracias.   import wx   class Ventana(wx.Frame):     def __init__(self):         wx.Frame.__init__(self, None, -1, 'Static Text Example', size=(800, 300)                           )         panel = wx.Panel(self, -1)         font = wx.Font(16, wx.DECORATIVE, wx.ITALIC, wx.NORMAL)         tc1 = wx.StaticText(panel, -1, "Texto de prueba", (100, 70), (600, -1), wx.ALIGN_CENTER)         tc1.SetForegroundColour('white')         tc1.SetBackgroundColour('black')         tc1.SetFont(font)         tc2 = wx.StaticText(panel, -1, "Segunda linea", (100, 110), (600, -1), wx.ALIGN_CENTER)         tc2.SetForegroundColour('white')         tc2.SetBackgroundColour('black') if __name__ == '__main__':     app = wx.App()     frame = Ventana()     frame.Show()     app.MainLoop() ------------ próxima parte ------------ Se ha borrado un adjunto en formato HTML... URL: From mlacunza en gmail.com Wed Oct 10 14:48:27 2012 From: mlacunza en gmail.com (Mario Lacunza) Date: Wed, 10 Oct 2012 07:48:27 -0500 Subject: [Python-es] Dudas de centrado de texto In-Reply-To: <1349873030.16521.YahooMailNeo@web133205.mail.ir2.yahoo.com> References: <5065cb9c.145b650a.6351.7a77@mx.google.com> <1349873030.16521.YahooMailNeo@web133205.mail.ir2.yahoo.com> Message-ID: Utiliza sizers.... Mario Lacunza Enviado desde mi Motorola Atrix2 El oct 10, 2012 7:46 a.m., "Gonzalo Mouriÿfffffffffff1o Pardo" < gmourinopardo en yahoo.es> escribió: > Buenas tardes, tengo otra duda: > Necesito centrar un wx.Statictext (tc1) en la pantalla, pero al tenenr una > fuente asociada no puedo como lo hago, gracias. > > import wx > > class Ventana(wx.Frame): > def __init__(self): > wx.Frame.__init__(self, None, -1, 'Static Text Example', > size=(800, 300) > ) > panel = wx.Panel(self, -1) > font = wx.Font(16, wx.DECORATIVE, wx.ITALIC, wx.NORMAL) > tc1 = wx.StaticText(panel, -1, "Texto de prueba", (100, 70), (600, > -1), wx.ALIGN_CENTER) > tc1.SetForegroundColour('white') > tc1.SetBackgroundColour('black') > tc1.SetFont(font) > tc2 = wx.StaticText(panel, -1, "Segunda linea", (100, 110), (600, > -1), wx.ALIGN_CENTER) > tc2.SetForegroundColour('white') > tc2.SetBackgroundColour('black') > > if __name__ == '__main__': > app = wx.App() > frame = Ventana() > frame.Show() > app.MainLoop() > > _______________________________________________ > Python-es mailing list > Python-es en python.org > http://mail.python.org/mailman/listinfo/python-es > FAQ: http://python-es-faq.wikidot.com/ > > ------------ próxima parte ------------ Se ha borrado un adjunto en formato HTML... URL: From lesthack en gmail.com Thu Oct 11 23:30:09 2012 From: lesthack en gmail.com (lesthack) Date: Thu, 11 Oct 2012 16:30:09 -0500 Subject: [Python-es] =?iso-8859-1?q?Oportunidad_de_Empleo_Desarrollador_PH?= =?iso-8859-1?q?P/Python/C_M=E9xico?= Message-ID: Oportunidad Laboral en la Ciudad de México (Sur). Estamos buscando un desarrollador con experiencia C / Web que puede unirse a nuestro equipo de desarrollo de nuevos locales en la ciudad de México. Los candidatos deben ser cómodos trabajando en equipo y muy motivados por este viaje de aventura hacia el liderazgo en uno de los mercados más prósperos del mundo. Características: - Capacitación (de 3 a 4 meses) en España. - Trabajo de Planta. - Buen sueldo (determinado según aptitudes, capacidades y experiencia). - Prestaciones superiores a las de la ley. Las responsabilidades principales - Implementar módulos frontales integrados en los sistemas de back-end - Diseñar, codificar y probar los módulos de back-end que cumplen con las especificaciones de diseño - Crear, ejecutar y documentar las pruebas necesarias para garantizar que la solicitud cumple con los requisitos de desempeño (interfaz técnica, funcional y de usuario) - Participar en la identificación de cambios de configuración en las aplicaciones para que se reúnan los requisitos de los procesos de negocio - Ser capaz de interactuar y coordinar con los desarrolladores de diferentes países para trabajar en proyectos comunes - Ser capaz de escribir documentación técnica en Inglés Perfil - Formación o experiencia de un Blocket in situ - Formación académica en la programación - Buen conocimiento del lenguaje de programación PHP u otro lenguaje de programación (Python / Ruby) - Experiencia C (ANSI) programador (4 + años) - PL / SQL conocimiento (capacidad de escribir consultas SQL y funciones), idealmente con PostgreSQL - La comprensión de cómo funciona un motor de base de datos - Buen conocimiento de HTML / CSS / Javascript - Shell de Unix comandos (scripts, archivos crontab ...) - La familiaridad con al menos una herramienta de SCM (Git o Subversion, Git es un plus) - Buenas habilidades de Linux como usuario También valiosa - Bash / Perl / Python scripting - Software debuging y profiling - Software de diseño de sistemas y arquitectura - Configuración Apache servidor web Personalidad - El éxito de los candidatos debe ser de trabajo dinámico, duro, los jugadores del equipo ambicioso y bueno. - Buenas habilidades de comunicación en Inglés es esencial, el español es altamente deseable. Interesados mandar CV a lforastieri en enlacelo.com (Ingles/Español) -- ________________________________________________________ ISC. Jorge Luis Hernández C. Desarrollador de Software y Tecnologías Libres Colaborador GNU/Linux Debian México http://lesthack.com.mx http://blog.debian.mx/ @lesthack ________________________________________________________ ------------ próxima parte ------------ Se ha borrado un adjunto en formato HTML... URL: From alito81 en gmail.com Fri Oct 12 03:30:08 2012 From: alito81 en gmail.com (alito s) Date: Thu, 11 Oct 2012 20:30:08 -0500 Subject: [Python-es] =?iso-8859-1?q?Buscar_reiterativamente_un_patr=F3n?= Message-ID: Hola a tod en s: Estoy tratando de buscar un patrón (guión) repetidas veces en una línea para posteriormente restar lo que hay antes del guión y después del guión. Lo consigo, el punto es que hay varios guiones en la línea pero solo me cuenta el primero, así que solo me resta lo que hay antes y después del primer guión. Les dejo mi código para ver si ustedes me pueden ayudar. Código: mysearch = inp.readlines() for line in mysearch: y = re.search (r"(\d+(?=[-]))", line) z = re.search (r"((?<=[-])\d+)", line) if y: yz = int(z.group()) - int(y.group()) print yz Gracias. ------------ próxima parte ------------ Se ha borrado un adjunto en formato HTML... URL: From juande en jdmanjon.net Fri Oct 12 04:56:37 2012 From: juande en jdmanjon.net (Juande Manjon) Date: Thu, 11 Oct 2012 22:56:37 -0400 Subject: [Python-es] =?utf-8?q?Buscar_reiterativamente_un_patr=C3=B3n?= In-Reply-To: References: Message-ID: <3d9d56f990b1453fa1d850a68d6fc8f8@mail.webfaction.com> Tienes un ejemplo de la cadena de entrada y lo que esperas obtenter? -Juande El 2012-10-11 21:30, alito s escribió: > Hola a tod en s: > Estoy tratando de buscar un patrón (guión) repetidas veces en una > línea para posteriormente restar lo que hay antes del guión y después > del guión. Lo consigo, el punto es que hay varios guiones en la línea > pero solo me cuenta el primero, así que solo me resta lo que hay > antes > y después del primer guión. > Les dejo mi código para ver si ustedes me pueden ayudar. > > Código: > >         mysearch = inp.readlines() >         for line in mysearch: >             y = re.search (r"(d+(?=[-]))", line) >             z = re.search (r"((?<=[-])d+)", line) >             >             if y: >                 yz = int(z.group()) - int(y.group()) >                 print yz > > Gracias. > > _______________________________________________ > Python-es mailing list > Python-es en python.org > http://mail.python.org/mailman/listinfo/python-es > FAQ: http://python-es-faq.wikidot.com/ From willian.radicaled en gmail.com Fri Oct 12 05:10:16 2012 From: willian.radicaled en gmail.com (RadicalEd) Date: Thu, 11 Oct 2012 22:10:16 -0500 Subject: [Python-es] =?utf-8?q?Buscar_reiterativamente_un_patr=C3=B3n?= In-Reply-To: References: <3d9d56f990b1453fa1d850a68d6fc8f8@mail.webfaction.com> Message-ID: No sé si te entiendo, pero yo lo haría más fácil, hago un split de la cadena y hago lo que deba hacer con cada elemento de la tupla El 11/10/2012 21:57, "Juande Manjon" escribió: Tienes un ejemplo de la cadena de entrada y lo que esperas obtenter? -Juande El 2012-10-11 21:30, alito s escribió: > > > > Hola a tod en s: > > Estoy tratando de buscar un patrón (guión) repetidas veces en una > > línea para p... > y = re.search (r"(d+(?=[-]))", line) > z = re.search (r"((?<=[-])d+)", line) > > > > > > if y: > > yz = int(z.group()) - int(y.group()) > > ... > ______________________________**_________________ > Python-es mailing list > Python-es en python.org > http://mail.python.org/**mailman/listinfo/python-es > FAQ: http://python-es-faq.wikidot.**com/ > ______________________________**_________________ Python-es mailing list Python-es en python.org http://mail.python.org/**mailman/listinfo/python-es FAQ: http://python-es-faq.wikidot.**com/ ------------ próxima parte ------------ Se ha borrado un adjunto en formato HTML... URL: From jcaballero.hep en gmail.com Fri Oct 12 14:36:35 2012 From: jcaballero.hep en gmail.com (Jose Caballero) Date: Fri, 12 Oct 2012 08:36:35 -0400 Subject: [Python-es] =?utf-8?q?Buscar_reiterativamente_un_patr=C3=B3n?= In-Reply-To: References: <3d9d56f990b1453fa1d850a68d6fc8f8@mail.webfaction.com> Message-ID: <9123B0FA-8718-48EB-88E9-8B36AC2F6B66@gmail.com> Eso funciona o no dependiendo de si hay o no espacios en blanco. Necesitamos el formato exacto del input, como ha pedido Juande. On Oct 11, 2012, at 23:10, RadicalEd wrote: > No sé si te entiendo, pero yo lo haría más fácil, hago un split de la cadena y hago lo que deba hacer con cada elemento de la tupla > >> El 11/10/2012 21:57, "Juande Manjon" escribió: >> >> Tienes un ejemplo de la cadena de entrada y lo que esperas obtenter? >> >> -Juande >> >> El 2012-10-11 21:30, alito s escribió: >> > >> > Hola a tod en s: >> > Estoy tratando de buscar un patrón (guión) repetidas veces en una >> > línea para p... >> >> y = re.search (r"(d+(?=[-]))", line) >> z = re.search (r"((?<=[-])d+)", line) >> >> > >> > if y: >> > yz = int(z.group()) - int(y.group()) >> > ... >> >> _______________________________________________ >> Python-es mailing list >> Python-es en python.org >> http://mail.python.org/mailman/listinfo/python-es >> FAQ: http://python-es-faq.wikidot.com/ >> >> _______________________________________________ >> Python-es mailing list >> Python-es en python.org >> http://mail.python.org/mailman/listinfo/python-es >> FAQ: http://python-es-faq.wikidot.com/ > > _______________________________________________ > Python-es mailing list > Python-es en python.org > http://mail.python.org/mailman/listinfo/python-es > FAQ: http://python-es-faq.wikidot.com/ ------------ próxima parte ------------ Se ha borrado un adjunto en formato HTML... URL: From alito81 en gmail.com Fri Oct 12 17:15:50 2012 From: alito81 en gmail.com (alito s) Date: Fri, 12 Oct 2012 10:15:50 -0500 Subject: [Python-es] =?iso-8859-1?q?Buscar_reiterativamente_un_patr=F3n?= In-Reply-To: <9123B0FA-8718-48EB-88E9-8B36AC2F6B66@gmail.com> References: <3d9d56f990b1453fa1d850a68d6fc8f8@mail.webfaction.com> <9123B0FA-8718-48EB-88E9-8B36AC2F6B66@gmail.com> Message-ID: Si, un ejemplo es el siguiente: > YPR203W_COILS, 1-15, 49-74, 88-102 > YPR203W_HOTLOOPS, 1-18, 50-76 En cada línea cambia lo del principio. Lo que a mi me interesa es que se reste lo que hay antes y después del guión, para cada guión. 2012/10/12 Jose Caballero > Eso funciona o no dependiendo de si hay o no espacios en blanco. > Necesitamos el formato exacto del input, como ha pedido Juande. > > > > On Oct 11, 2012, at 23:10, RadicalEd wrote: > > No sé si te entiendo, pero yo lo haría más fácil, hago un split de la > cadena y hago lo que deba hacer con cada elemento de la tupla > > El 11/10/2012 21:57, "Juande Manjon" < > juande en jdmanjon.net> escribió: > > Tienes un ejemplo de la cadena de entrada y lo que esperas obtenter? > > -Juande > > El 2012-10-11 21:30, alito s escribió: > >> > >> > Hola a tod en s: >> > Estoy tratando de buscar un patrón (guión) repetidas veces en una >> > línea para p... >> y = re.search (r"(d+(?=[-]))", line) >> z = re.search (r"((?<=[-])d+)", line) >> >> >> > >> > if y: >> > yz = int(z.group()) - int(y.group()) >> > ... >> ______________________________**_________________ >> Python-es mailing list >> Python-es en python.org >> >> http://mail.python.org/**mailman/listinfo/python-es >> FAQ: http://python-es-faq.wikidot.** >> com/ >> > > ______________________________**_________________ > Python-es mailing list > Python-es en python.org > > http://mail.python.org/**mailman/listinfo/python-es > FAQ: http://python-es-faq.wikidot.** > com/ > > _______________________________________________ > Python-es mailing list > Python-es en python.org > http://mail.python.org/mailman/listinfo/python-es > FAQ: http://python-es-faq.wikidot.com/ > > > _______________________________________________ > Python-es mailing list > Python-es en python.org > http://mail.python.org/mailman/listinfo/python-es > FAQ: http://python-es-faq.wikidot.com/ > > ------------ próxima parte ------------ Se ha borrado un adjunto en formato HTML... URL: From jcaballero.hep en gmail.com Fri Oct 12 17:32:53 2012 From: jcaballero.hep en gmail.com (Jose Caballero) Date: Fri, 12 Oct 2012 11:32:53 -0400 Subject: [Python-es] =?iso-8859-1?q?Buscar_reiterativamente_un_patr=F3n?= In-Reply-To: References: <3d9d56f990b1453fa1d850a68d6fc8f8@mail.webfaction.com> <9123B0FA-8718-48EB-88E9-8B36AC2F6B66@gmail.com> Message-ID: es ese siempre el formato? .... Si la respuesta es afirmativa y no quieres usar regex, entonces puedes hacer primero split(',') y procesar [1:], con otro split('-') El 12 de octubre de 2012 11:15, alito s escribió: > Si, un ejemplo es el siguiente: > > > YPR203W_COILS, 1-15, 49-74, 88-102 > > YPR203W_HOTLOOPS, 1-18, 50-76 > > En cada línea cambia lo del principio. Lo que a mi me interesa es que se > reste lo que hay antes y después del guión, para cada guión. > > > > 2012/10/12 Jose Caballero > >> Eso funciona o no dependiendo de si hay o no espacios en blanco. >> Necesitamos el formato exacto del input, como ha pedido Juande. >> >> >> >> On Oct 11, 2012, at 23:10, RadicalEd wrote: >> >> No sé si te entiendo, pero yo lo haría más fácil, hago un split de la >> cadena y hago lo que deba hacer con cada elemento de la tupla >> >> El 11/10/2012 21:57, "Juande Manjon" < >> juande en jdmanjon.net> escribió: >> >> Tienes un ejemplo de la cadena de entrada y lo que esperas obtenter? >> >> -Juande >> >> El 2012-10-11 21:30, alito s escribió: >> >>> > >>> > Hola a tod en s: >>> > Estoy tratando de buscar un patrón (guión) repetidas veces en una >>> > línea para p... >>> y = re.search (r"(d+(?=[-]))", line) >>> z = re.search (r"((?<=[-])d+)", line) >>> >>> >>> > >>> > if y: >>> > yz = int(z.group()) - int(y.group()) >>> > ... >>> ______________________________**_________________ >>> Python-es mailing list >>> Python-es en python.org >>> >>> http://mail.python.org/**mailman/listinfo/python-es >>> FAQ: http://python-es-faq.wikidot.** >>> com/ >>> >> >> ______________________________**_________________ >> Python-es mailing list >> Python-es en python.org >> >> http://mail.python.org/**mailman/listinfo/python-es >> FAQ: http://python-es-faq.wikidot.** >> com/ >> >> _______________________________________________ >> Python-es mailing list >> Python-es en python.org >> http://mail.python.org/mailman/listinfo/python-es >> FAQ: http://python-es-faq.wikidot.com/ >> >> >> _______________________________________________ >> Python-es mailing list >> Python-es en python.org >> http://mail.python.org/mailman/listinfo/python-es >> FAQ: http://python-es-faq.wikidot.com/ >> >> > > _______________________________________________ > Python-es mailing list > Python-es en python.org > http://mail.python.org/mailman/listinfo/python-es > FAQ: http://python-es-faq.wikidot.com/ > > ------------ próxima parte ------------ Se ha borrado un adjunto en formato HTML... URL: From alito81 en gmail.com Fri Oct 12 17:50:15 2012 From: alito81 en gmail.com (alito s) Date: Fri, 12 Oct 2012 10:50:15 -0500 Subject: [Python-es] =?iso-8859-1?q?Buscar_reiterativamente_un_patr=F3n?= In-Reply-To: References: <3d9d56f990b1453fa1d850a68d6fc8f8@mail.webfaction.com> <9123B0FA-8718-48EB-88E9-8B36AC2F6B66@gmail.com> Message-ID: Si, ese es siempre el formato. Voy a intentar hacerlo como me sugieres 2012/10/12 Jose Caballero > es ese siempre el formato? > .... > Si la respuesta es afirmativa y no quieres usar regex, entonces puedes > hacer primero split(',') y procesar [1:], con otro split('-') > > > > El 12 de octubre de 2012 11:15, alito s escribió: > > Si, un ejemplo es el siguiente: >> >> > YPR203W_COILS, 1-15, 49-74, 88-102 >> > YPR203W_HOTLOOPS, 1-18, 50-76 >> >> En cada línea cambia lo del principio. Lo que a mi me interesa es que se >> reste lo que hay antes y después del guión, para cada guión. >> >> >> >> 2012/10/12 Jose Caballero >> >>> Eso funciona o no dependiendo de si hay o no espacios en blanco. >>> Necesitamos el formato exacto del input, como ha pedido Juande. >>> >>> >>> >>> On Oct 11, 2012, at 23:10, RadicalEd >>> wrote: >>> >>> No sé si te entiendo, pero yo lo haría más fácil, hago un split de la >>> cadena y hago lo que deba hacer con cada elemento de la tupla >>> >>> El 11/10/2012 21:57, "Juande Manjon" escribió: >>> >>> Tienes un ejemplo de la cadena de entrada y lo que esperas obtenter? >>> >>> -Juande >>> >>> El 2012-10-11 21:30, alito s escribió: >>> >>>> > >>>> > Hola a tod en s: >>>> > Estoy tratando de buscar un patrón (guión) repetidas veces en una >>>> > línea para p... >>>> y = re.search (r"(d+(?=[-]))", line) >>>> z = re.search (r"((?<=[-])d+)", line) >>>> >>>> >>>> > >>>> > if y: >>>> > yz = int(z.group()) - int(y.group()) >>>> > ... >>>> _______________________________________________ >>>> Python-es mailing list >>>> Python-es en python.org >>>> http://mail.python.org/mailman/listinfo/python-es >>>> FAQ: http://python-es-faq.wikidot.com/ >>>> >>> >>> _______________________________________________ >>> Python-es mailing list >>> Python-es en python.org >>> http://mail.python.org/mailman/listinfo/python-es >>> FAQ: http://python-es-faq.wikidot.com/ >>> >>> _______________________________________________ >>> Python-es mailing list >>> Python-es en python.org >>> http://mail.python.org/mailman/listinfo/python-es >>> FAQ: http://python-es-faq.wikidot.com/ >>> >>> >>> _______________________________________________ >>> Python-es mailing list >>> Python-es en python.org >>> http://mail.python.org/mailman/listinfo/python-es >>> FAQ: http://python-es-faq.wikidot.com/ >>> >>> >> >> _______________________________________________ >> Python-es mailing list >> Python-es en python.org >> http://mail.python.org/mailman/listinfo/python-es >> FAQ: http://python-es-faq.wikidot.com/ >> >> > > _______________________________________________ > Python-es mailing list > Python-es en python.org > http://mail.python.org/mailman/listinfo/python-es > FAQ: http://python-es-faq.wikidot.com/ > > ------------ próxima parte ------------ Se ha borrado un adjunto en formato HTML... URL: From mzmprog en gmail.com Sat Oct 13 00:00:55 2012 From: mzmprog en gmail.com (miguel zamora m.) Date: Fri, 12 Oct 2012 19:00:55 -0300 Subject: [Python-es] =?iso-8859-1?q?Buscar_reiterativamente_un_patr=F3n?= In-Reply-To: References: <3d9d56f990b1453fa1d850a68d6fc8f8@mail.webfaction.com> <9123B0FA-8718-48EB-88E9-8B36AC2F6B66@gmail.com> Message-ID: Mira esto es muy rustico pero funciona de seguro lo mejoras con un ciclo o algo mas refinado, lo puedes hacer con una expresión regular claro esta, pero a veces es mejor dejarlo extremadamente simple. >>> cosa = "YPR203W_COILS, 1-15, 49-74, 88-102" >>> cosa1 = cosa >>> var1 = cosa1.find("-") >>> print var1 ['YPR203W_COILS', ' 1-15', ' 49-74', ' 88-102'] >>> a=var1[1] >>> print a 1-15 >>> a=var2[2] >>> print a 49-74 >>> a=var2[3] >>> print a 88-102 >>> a=var2[1] >>> print a 1-15 >>> b = a.split("-") >>> print b [' 1', '15'] >>> c=int(b[0])-int(b[1]) >>> print c -14 El día 12 de octubre de 2012 12:50, alito s escribió: > Si, ese es siempre el formato. > Voy a intentar hacerlo como me sugieres > > > 2012/10/12 Jose Caballero >> >> es ese siempre el formato? >> .... >> Si la respuesta es afirmativa y no quieres usar regex, entonces puedes >> hacer primero split(',') y procesar [1:], con otro split('-') >> >> >> >> El 12 de octubre de 2012 11:15, alito s escribió: >> >>> Si, un ejemplo es el siguiente: >>> >>> > YPR203W_COILS, 1-15, 49-74, 88-102 >>> > YPR203W_HOTLOOPS, 1-18, 50-76 >>> >>> En cada línea cambia lo del principio. Lo que a mi me interesa es que se >>> reste lo que hay antes y después del guión, para cada guión. >>> >>> >>> >>> 2012/10/12 Jose Caballero >>>> >>>> Eso funciona o no dependiendo de si hay o no espacios en blanco. >>>> Necesitamos el formato exacto del input, como ha pedido Juande. >>>> >>>> >>>> >>>> On Oct 11, 2012, at 23:10, RadicalEd >>>> wrote: >>>> >>>> No sé si te entiendo, pero yo lo haría más fácil, hago un split de la >>>> cadena y hago lo que deba hacer con cada elemento de la tupla >>>> >>>> El 11/10/2012 21:57, "Juande Manjon" escribió: >>>> >>>> Tienes un ejemplo de la cadena de entrada y lo que esperas obtenter? >>>> >>>> -Juande >>>> >>>> El 2012-10-11 21:30, alito s escribió: >>>>> >>>>> > >>>>> > Hola a tod en s: >>>>> > Estoy tratando de buscar un patrón (guión) repetidas veces en una >>>>> > línea para p... >>>>> >>>>> y = re.search (r"(d+(?=[-]))", line) >>>>> z = re.search (r"((?<=[-])d+)", line) >>>>> >>>>> >>>>> > >>>>> > if y: >>>>> > yz = int(z.group()) - int(y.group()) >>>>> > ... >>>>> >>>>> _______________________________________________ >>>>> Python-es mailing list >>>>> Python-es en python.org >>>>> http://mail.python.org/mailman/listinfo/python-es >>>>> FAQ: http://python-es-faq.wikidot.com/ >>>> >>>> >>>> _______________________________________________ >>>> Python-es mailing list >>>> Python-es en python.org >>>> http://mail.python.org/mailman/listinfo/python-es >>>> FAQ: http://python-es-faq.wikidot.com/ >>>> >>>> _______________________________________________ >>>> Python-es mailing list >>>> Python-es en python.org >>>> http://mail.python.org/mailman/listinfo/python-es >>>> FAQ: http://python-es-faq.wikidot.com/ >>>> >>>> >>>> _______________________________________________ >>>> Python-es mailing list >>>> Python-es en python.org >>>> http://mail.python.org/mailman/listinfo/python-es >>>> FAQ: http://python-es-faq.wikidot.com/ >>>> >>> >>> >>> _______________________________________________ >>> Python-es mailing list >>> Python-es en python.org >>> http://mail.python.org/mailman/listinfo/python-es >>> FAQ: http://python-es-faq.wikidot.com/ >>> >> >> >> _______________________________________________ >> Python-es mailing list >> Python-es en python.org >> http://mail.python.org/mailman/listinfo/python-es >> FAQ: http://python-es-faq.wikidot.com/ >> > > > _______________________________________________ > Python-es mailing list > Python-es en python.org > http://mail.python.org/mailman/listinfo/python-es > FAQ: http://python-es-faq.wikidot.com/ > -- Miguel Zamora M. Programador en Computacion e Informatica 08-1351249 mzmprog en gmail.com Santiago-Chile From alito81 en gmail.com Sat Oct 13 00:35:46 2012 From: alito81 en gmail.com (alito s) Date: Fri, 12 Oct 2012 17:35:46 -0500 Subject: [Python-es] =?iso-8859-1?q?Buscar_reiterativamente_un_patr=F3n?= In-Reply-To: References: <3d9d56f990b1453fa1d850a68d6fc8f8@mail.webfaction.com> <9123B0FA-8718-48EB-88E9-8B36AC2F6B66@gmail.com> Message-ID: Oh, muchas gracias. Estoy en eso en estos momentos. 2012/10/12 miguel zamora m. > Mira esto es muy rustico pero funciona de seguro lo mejoras con un > ciclo o algo mas refinado, > > > lo puedes hacer con una expresión regular claro esta, > pero a veces es mejor dejarlo extremadamente simple. > > >>> cosa = "YPR203W_COILS, 1-15, 49-74, 88-102" > >>> cosa1 = cosa > >>> var1 = cosa1.find("-") > >>> print var1 > ['YPR203W_COILS', ' 1-15', ' 49-74', ' 88-102'] > >>> a=var1[1] > >>> print a > 1-15 > >>> a=var2[2] > >>> print a > 49-74 > >>> a=var2[3] > >>> print a > 88-102 > >>> a=var2[1] > >>> print a > 1-15 > >>> b = a.split("-") > >>> print b > [' 1', '15'] > >>> c=int(b[0])-int(b[1]) > >>> print c > -14 > > > > > > > > > El día 12 de octubre de 2012 12:50, alito s escribió: > > Si, ese es siempre el formato. > > Voy a intentar hacerlo como me sugieres > > > > > > 2012/10/12 Jose Caballero > >> > >> es ese siempre el formato? > >> .... > >> Si la respuesta es afirmativa y no quieres usar regex, entonces puedes > >> hacer primero split(',') y procesar [1:], con otro split('-') > >> > >> > >> > >> El 12 de octubre de 2012 11:15, alito s escribió: > >> > >>> Si, un ejemplo es el siguiente: > >>> > >>> > YPR203W_COILS, 1-15, 49-74, 88-102 > >>> > YPR203W_HOTLOOPS, 1-18, 50-76 > >>> > >>> En cada línea cambia lo del principio. Lo que a mi me interesa es que > se > >>> reste lo que hay antes y después del guión, para cada guión. > >>> > >>> > >>> > >>> 2012/10/12 Jose Caballero > >>>> > >>>> Eso funciona o no dependiendo de si hay o no espacios en blanco. > >>>> Necesitamos el formato exacto del input, como ha pedido Juande. > >>>> > >>>> > >>>> > >>>> On Oct 11, 2012, at 23:10, RadicalEd > >>>> wrote: > >>>> > >>>> No sé si te entiendo, pero yo lo haría más fácil, hago un split de la > >>>> cadena y hago lo que deba hacer con cada elemento de la tupla > >>>> > >>>> El 11/10/2012 21:57, "Juande Manjon" escribió: > >>>> > >>>> Tienes un ejemplo de la cadena de entrada y lo que esperas obtenter? > >>>> > >>>> -Juande > >>>> > >>>> El 2012-10-11 21:30, alito s escribió: > >>>>> > >>>>> > > >>>>> > Hola a tod en s: > >>>>> > Estoy tratando de buscar un patrón (guión) repetidas veces en una > >>>>> > línea para p... > >>>>> > >>>>> y = re.search (r"(d+(?=[-]))", line) > >>>>> z = re.search (r"((?<=[-])d+)", line) > >>>>> > >>>>> > >>>>> > > >>>>> > if y: > >>>>> > yz = int(z.group()) - int(y.group()) > >>>>> > ... > >>>>> > >>>>> _______________________________________________ > >>>>> Python-es mailing list > >>>>> Python-es en python.org > >>>>> http://mail.python.org/mailman/listinfo/python-es > >>>>> FAQ: http://python-es-faq.wikidot.com/ > >>>> > >>>> > >>>> _______________________________________________ > >>>> Python-es mailing list > >>>> Python-es en python.org > >>>> http://mail.python.org/mailman/listinfo/python-es > >>>> FAQ: http://python-es-faq.wikidot.com/ > >>>> > >>>> _______________________________________________ > >>>> Python-es mailing list > >>>> Python-es en python.org > >>>> http://mail.python.org/mailman/listinfo/python-es > >>>> FAQ: http://python-es-faq.wikidot.com/ > >>>> > >>>> > >>>> _______________________________________________ > >>>> Python-es mailing list > >>>> Python-es en python.org > >>>> http://mail.python.org/mailman/listinfo/python-es > >>>> FAQ: http://python-es-faq.wikidot.com/ > >>>> > >>> > >>> > >>> _______________________________________________ > >>> Python-es mailing list > >>> Python-es en python.org > >>> http://mail.python.org/mailman/listinfo/python-es > >>> FAQ: http://python-es-faq.wikidot.com/ > >>> > >> > >> > >> _______________________________________________ > >> Python-es mailing list > >> Python-es en python.org > >> http://mail.python.org/mailman/listinfo/python-es > >> FAQ: http://python-es-faq.wikidot.com/ > >> > > > > > > _______________________________________________ > > Python-es mailing list > > Python-es en python.org > > http://mail.python.org/mailman/listinfo/python-es > > FAQ: http://python-es-faq.wikidot.com/ > > > > > > -- > Miguel Zamora M. > Programador en Computacion e Informatica > 08-1351249 > mzmprog en gmail.com > Santiago-Chile > _______________________________________________ > Python-es mailing list > Python-es en python.org > http://mail.python.org/mailman/listinfo/python-es > FAQ: http://python-es-faq.wikidot.com/ > ------------ próxima parte ------------ Se ha borrado un adjunto en formato HTML... URL: From angel en angel-alvarez.com.ar Sat Oct 13 01:59:23 2012 From: angel en angel-alvarez.com.ar (Angel Claudio Alvarez) Date: Fri, 12 Oct 2012 20:59:23 -0300 Subject: [Python-es] =?iso-8859-1?q?Buscar_reiterativamente_un_patr=F3n?= In-Reply-To: References: <3d9d56f990b1453fa1d850a68d6fc8f8@mail.webfaction.com> <9123B0FA-8718-48EB-88E9-8B36AC2F6B66@gmail.com> Message-ID: <20121012205923.03a5dc9a4ad3f6b25713c262@angel-alvarez.com.ar> El Fri, 12 Oct 2012 10:15:50 -0500 alito s escribió: > Si, un ejemplo es el siguiente: > > > YPR203W_COILS, 1-15, 49-74, 88-102 > > YPR203W_HOTLOOPS, 1-18, 50-76 > > En cada línea cambia lo del principio. Lo que a mi me interesa es que se > reste lo que hay antes y después del guión, para cada guión. > Tratalo con un pila > > 2012/10/12 Jose Caballero > > > Eso funciona o no dependiendo de si hay o no espacios en blanco. > > Necesitamos el formato exacto del input, como ha pedido Juande. > > > > > > > > On Oct 11, 2012, at 23:10, RadicalEd wrote: > > > > No sé si te entiendo, pero yo lo haría más fácil, hago un split de la > > cadena y hago lo que deba hacer con cada elemento de la tupla > > > > El 11/10/2012 21:57, "Juande Manjon" < > > juande en jdmanjon.net> escribió: > > > > Tienes un ejemplo de la cadena de entrada y lo que esperas obtenter? > > > > -Juande > > > > El 2012-10-11 21:30, alito s escribió: > > > >> > > >> > Hola a tod en s: > >> > Estoy tratando de buscar un patrón (guión) repetidas veces en una > >> > línea para p... > >> y = re.search (r"(d+(?=[-]))", line) > >> z = re.search (r"((?<=[-])d+)", line) > >> > >> > >> > > >> > if y: > >> > yz = int(z.group()) - int(y.group()) > >> > ... > >> ______________________________**_________________ > >> Python-es mailing list > >> Python-es en python.org > >> > >> http://mail.python.org/**mailman/listinfo/python-es > >> FAQ: http://python-es-faq.wikidot.** > >> com/ > >> > > > > ______________________________**_________________ > > Python-es mailing list > > Python-es en python.org > > > > http://mail.python.org/**mailman/listinfo/python-es > > FAQ: http://python-es-faq.wikidot.** > > com/ > > > > _______________________________________________ > > Python-es mailing list > > Python-es en python.org > > http://mail.python.org/mailman/listinfo/python-es > > FAQ: http://python-es-faq.wikidot.com/ > > > > > > _______________________________________________ > > Python-es mailing list > > Python-es en python.org > > http://mail.python.org/mailman/listinfo/python-es > > FAQ: http://python-es-faq.wikidot.com/ > > > > -- Angel Claudio Alvarez From pych3m4 en gmail.com Sat Oct 13 02:52:48 2012 From: pych3m4 en gmail.com (Chema Cortes) Date: Sat, 13 Oct 2012 02:52:48 +0200 Subject: [Python-es] =?iso-8859-1?q?Buscar_reiterativamente_un_patr=F3n?= In-Reply-To: References: <3d9d56f990b1453fa1d850a68d6fc8f8@mail.webfaction.com> <9123B0FA-8718-48EB-88E9-8B36AC2F6B66@gmail.com> Message-ID: El día 12 de octubre de 2012 17:15, alito s escribió: > Si, un ejemplo es el siguiente: > >> YPR203W_COILS, 1-15, 49-74, 88-102 >> YPR203W_HOTLOOPS, 1-18, 50-76 > > En cada línea cambia lo del principio. Lo que a mi me interesa es que se > reste lo que hay antes y después del guión, para cada guión. Volviendo a tu código con regex, bastaría cambiar el 'search' por 'findall' para que localizar todas las ocurrencias: mysearch = inp.readlines() for line in mysearch: y = re.findall(r"(\d+(?=[-]))", line) z = re.findall(r"((?<=[-])\d+)", line) if y: print [int(b)-int(a) for (a,b) in zip(y,z)] Optimizando, una sóla expresión regular y sacando el patrón fuera del bucle, quedaría así: pat = re.compile(r"(\d+)[-](\d+)") for line in inp: print [ int(b)-int(a) for (a,b) in pat.findall(line)] -- Hyperreals *R: http://ch3m4.org/blog Quarks, bits y otras criaturas infinitesimales From mzmprog en gmail.com Sat Oct 13 10:06:00 2012 From: mzmprog en gmail.com (miguel zamora m.) Date: Sat, 13 Oct 2012 05:06:00 -0300 Subject: [Python-es] =?iso-8859-1?q?Buscar_reiterativamente_un_patr=F3n?= In-Reply-To: References: <3d9d56f990b1453fa1d850a68d6fc8f8@mail.webfaction.com> <9123B0FA-8718-48EB-88E9-8B36AC2F6B66@gmail.com> Message-ID: Haslo lo mas simple posible KISS El día 12 de octubre de 2012 21:52, Chema Cortes escribió: > El día 12 de octubre de 2012 17:15, alito s escribió: >> Si, un ejemplo es el siguiente: >> >>> YPR203W_COILS, 1-15, 49-74, 88-102 >>> YPR203W_HOTLOOPS, 1-18, 50-76 >> >> En cada línea cambia lo del principio. Lo que a mi me interesa es que se >> reste lo que hay antes y después del guión, para cada guión. > > Volviendo a tu código con regex, bastaría cambiar el 'search' por > 'findall' para que localizar todas las ocurrencias: > > mysearch = inp.readlines() > for line in mysearch: > y = re.findall(r"(\d+(?=[-]))", line) > z = re.findall(r"((?<=[-])\d+)", line) > > if y: > print [int(b)-int(a) for (a,b) in zip(y,z)] > > > Optimizando, una sóla expresión regular y sacando el patrón fuera del > bucle, quedaría así: > > pat = re.compile(r"(\d+)[-](\d+)") > > for line in inp: > > print [ int(b)-int(a) for (a,b) in pat.findall(line)] > > > -- > Hyperreals *R: http://ch3m4.org/blog > Quarks, bits y otras criaturas infinitesimales > _______________________________________________ > Python-es mailing list > Python-es en python.org > http://mail.python.org/mailman/listinfo/python-es > FAQ: http://python-es-faq.wikidot.com/ -- Miguel Zamora M. Programador en Computacion e Informatica 08-1351249 mzmprog en gmail.com Santiago-Chile From oggers en gmail.com Sat Oct 13 12:29:13 2012 From: oggers en gmail.com (=?ISO-8859-1?Q?Juan_Carlos_Coru=F1a?=) Date: Sat, 13 Oct 2012 12:29:13 +0200 Subject: [Python-es] =?iso-8859-1?q?=BFSugerencia_de_como_almacenar_c=F3di?= =?iso-8859-1?q?gos_postales_en_Plone=3F?= Message-ID: Hola gente, Se me plantea una duda que es la que menciono en el asunto de este correo. Lo explicaré mejor: En los usuarios de Plone he añadido los campos país, ciudad, código postal y provincia pero deseo que se pueda seleccionar desde un vocabulario para que solamente se puedan seleccionar los términos disponibles: 3 países, ~60 provincias, ~10.000 ciudades y ~30.000 códigos postales. La duda que tengo es como almacenar estos metadatos en Plone. He considerado estas alternativas: - collective.vdexvocabulary - Objetos dexterity creados al efecto con 4 niveles de jerarquía (los códigos postales serían un atributo de ciudad): códigos postales -> países -> provincias -> ciudades.codigosPostales - sqlalchemy: Implementar toda la infraestructura de sqlalchemy junto con un interface crud igual es demasiado. - plone.app.registry: Definiendo una estructura para almacenarlos, p.ej. pais|provincia|ciudad|codPostal1|codPostal2|.... Esto me parece engorroso de gestionar aunque hay que tener en cuenta que raramente se modifican estos datos. - eval(dict(...)): Almacenar la estructura de códigos postales en un dictionario que se encuentre en un archivo. Para gestionarlos se tendría que editar directamente el archivo. ¿Que otras alternativas se os ocurren? Un saludo. -- Juan Carlos Coruña ------------ próxima parte ------------ Se ha borrado un adjunto en formato HTML... URL: From kausdiv en gmail.com Sat Oct 13 12:44:44 2012 From: kausdiv en gmail.com (kausdiv) Date: Sat, 13 Oct 2012 12:44:44 +0200 Subject: [Python-es] Diccionarios Message-ID: <5079461C.8050605@gmail.com> Hola de nuevo. Quiero hacer un programita que me ayude con las clases de ingles. Se trata de meter vocabulario y que pueda consultar y que me pregunte aleatoriamente palabras. Había pensado en usar Diccionarios, pero me encuentro con 2 problemas: 1º: El diccionario tiene una clave y un valor (Clave "Cat" valor "Gato") por tanto va muy bien si siempre quier saber la traducción en un sentido: "Cat" -> "Gato". ¿ Pero como puedo hacer al revés ? 2º: Si yo quiero hacer un función para que me pregunte palabras aleatorias. Creo que no valdría hacer algo así: dic[2] ---- Viendo estas dificultades y el poco tiempo que llevo en python, me pregunto si no es mejor hacerlo con listas: lista=["Cat;Gato","Son;Hijo"] y obtener clave y valor con split(";") de forma que podría buscar tanto en ingles como en españo. ¿ que me aconsejáis ? From davidmenhur en gmail.com Sat Oct 13 13:19:32 2012 From: davidmenhur en gmail.com (=?UTF-8?B?RGHPgGlk?=) Date: Sat, 13 Oct 2012 13:19:32 +0200 Subject: [Python-es] Diccionarios In-Reply-To: <5079461C.8050605@gmail.com> References: <5079461C.8050605@gmail.com> Message-ID: 2012/10/13 kausdiv : > 1º: > El diccionario tiene una clave y un valor (Clave "Cat" valor "Gato") por > tanto va muy bien si siempre quier saber la traducción en un sentido: "Cat" > -> "Gato". > ¿ Pero como puedo hacer al revés ? Puedes construirte el diccionario inverso. No sé si existe una forma mejor, pero con algo de fuerza bruta puede salir en milisegundos. Necesitas los métodos .keys() y .values() > 2º: > Si yo quiero hacer un función para que me pregunte palabras aleatorias. Creo > que no valdría hacer algo así: dic[2] Puedes hacer dic[random.choice(dic.keys())]. > Viendo estas dificultades y el poco tiempo que llevo en python, me pregunto > si no es mejor hacerlo con listas: > lista=["Cat;Gato","Son;Hijo"] > y obtener clave y valor con split(";") de forma que podría buscar tanto en > ingles como en españo. Mucho más sencillo hacer una lista de listas. lista=[['cat', 'gato'],['son','hijo']] o de tuplas, si no las vas a modificar: lista_tupla=[('cat', 'gato'),('son','hijo')] Pero quizá sea más fácil hacer dos listas paralelas: esp=['gato', 'hijo'] eng=['cat', 'son'] Y llamarlas por el mismo número. Ambas son equivalentes, ya que: lista_tupla==zip(eng,esp) eng==[x[0] for x in lista_tupla] esp==[x[1] for x in lista_tupla] From willian.radicaled en gmail.com Sat Oct 13 13:35:29 2012 From: willian.radicaled en gmail.com (RadicalEd) Date: Sat, 13 Oct 2012 06:35:29 -0500 Subject: [Python-es] Diccionarios In-Reply-To: References: <5079461C.8050605@gmail.com> Message-ID: Y por qué no 2 diccionarios, uno inglés-español y otro español-inglés. Y consultas ambos, sino está el value en inglés es porque estás buscando en español y viceversa El 13/10/2012 6:20, "Da?id" escribió: 2012/10/13 kausdiv : > 1º: > El diccionario tiene una clave y un valor (Clave "Cat" valor "Gato") por > tanto va muy bien... Puedes construirte el diccionario inverso. No sé si existe una forma mejor, pero con algo de fuerza bruta puede salir en milisegundos. Necesitas los métodos .keys() y .values() > 2º: > Si yo quiero hacer un función para que me pregunte palabras aleatorias. Creo > que no valdr... Puedes hacer dic[random.choice(dic.keys())]. > Viendo estas dificultades y el poco tiempo que llevo en python, me pregunto > si no es mejor hac... Mucho más sencillo hacer una lista de listas. lista=[['cat', 'gato'],['son','hijo']] o de tuplas, si no las vas a modificar: lista_tupla=[('cat', 'gato'),('son','hijo')] Pero quizá sea más fácil hacer dos listas paralelas: esp=['gato', 'hijo'] eng=['cat', 'son'] Y llamarlas por el mismo número. Ambas son equivalentes, ya que: lista_tupla==zip(eng,esp) eng==[x[0] for x in lista_tupla] esp==[x[1] for x in lista_tupla] _______________________________________________ Python-es mailing list Python-es en python.org http://m... ------------ próxima parte ------------ Se ha borrado un adjunto en formato HTML... URL: From davidmenhur en gmail.com Sat Oct 13 13:49:28 2012 From: davidmenhur en gmail.com (=?UTF-8?B?RGHPgGlk?=) Date: Sat, 13 Oct 2012 13:49:28 +0200 Subject: [Python-es] Diccionarios In-Reply-To: References: <5079461C.8050605@gmail.com> Message-ID: Añado: si con esto quieres aprender Python, estupendo, es un proyecto divertido. Pero para aprender un idioma, hay métodos más optimizados. Aquí dos ejemplos gratuitos, escritos en Python, además: http://en.wikipedia.org/wiki/Anki http://en.wikipedia.org/wiki/Mnemosyne_(software) Quizá, con tiempo, puedas implementar el algoritmo en tu programa. Yo he pensado en adaptarme estos para teatro. 2012/10/13 RadicalEd : > Y por qué no 2 diccionarios, uno inglés-español y otro español-inglés. > Y consultas ambos, sino está el value en inglés es porque estás buscando en > español y viceversa > > El 13/10/2012 6:20, "Da?id" escribió: > > 2012/10/13 kausdiv : > >> 1º: >> El diccionario tiene una clave y un valor (Clave "Cat" valor "Gato") por >> tanto va muy bien... > > Puedes construirte el diccionario inverso. No sé si existe una forma > mejor, pero con algo de fuerza bruta puede salir en milisegundos. > Necesitas los métodos .keys() y .values() > > >> 2º: >> Si yo quiero hacer un función para que me pregunte palabras aleatorias. >> Creo >> que no valdr... > > Puedes hacer dic[random.choice(dic.keys())]. > > > >> Viendo estas dificultades y el poco tiempo que llevo en python, me >> pregunto >> si no es mejor hac... > > Mucho más sencillo hacer una lista de listas. > > lista=[['cat', 'gato'],['son','hijo']] > > o de tuplas, si no las vas a modificar: > > lista_tupla=[('cat', 'gato'),('son','hijo')] > > Pero quizá sea más fácil hacer dos listas paralelas: > > esp=['gato', 'hijo'] > eng=['cat', 'son'] > > Y llamarlas por el mismo número. Ambas son equivalentes, ya que: > > lista_tupla==zip(eng,esp) > > eng==[x[0] for x in lista_tupla] > esp==[x[1] for x in lista_tupla] > > _______________________________________________ > Python-es mailing list > Python-es en python.org > http://m... > > > _______________________________________________ > Python-es mailing list > Python-es en python.org > http://mail.python.org/mailman/listinfo/python-es > FAQ: http://python-es-faq.wikidot.com/ > From kausdiv en gmail.com Sun Oct 14 09:40:25 2012 From: kausdiv en gmail.com (kausdiv) Date: Sun, 14 Oct 2012 09:40:25 +0200 Subject: [Python-es] Diccionarios In-Reply-To: References: <5079461C.8050605@gmail.com> Message-ID: <507A6C69.4050400@gmail.com> Se ha borrado un adjunto en formato HTML... URL: From py en ch3m4.org Mon Oct 15 03:36:14 2012 From: py en ch3m4.org (Chema Cortes) Date: Mon, 15 Oct 2012 03:36:14 +0200 Subject: [Python-es] Diccionarios In-Reply-To: <507A6C69.4050400@gmail.com> References: <5079461C.8050605@gmail.com> <507A6C69.4050400@gmail.com> Message-ID: <507B688E.8030303@ch3m4.org> El 14/10/12 09:40, kausdiv escribió: > Efectivamente he realizado el programa para aprender Python y de paso > dotarme de una utilidad para memorizar vocabulario ingles. > > Dejo el código que he creado para que hagais las mejoras que veais > oportunas y así pueda aprender más: OK. Pues te voy a dar algunos consejos: Lo primero es que sigas el modo "pythónico" de hacer las cosas. Intenta usar compresiones de listas e iteradores donde sea más oportuno, como es, por ejemplo, cuando leas un fichero secuencialmente. Expresiones como: f=open(self.nfich,"r") for l in f: self.dic.append(l[:-1]) f.close() se pueden expresar mejor si consideramos que el objeto 'file' es un iterador: for l in open(self.nfich,"r"): self.dic.append(l[:-1]) En el caso de fichero de salida (método guardar), se puede gestionar mejor si usas "contextos": with open(self.nfich,"w") as f: for l in self.dic: f.write(l) f.write("\n") El contexto se encarga de cerrar automáticamente el fichero. Así mismo, existe un modo más directo de escribir en un fichero de texto usando la sentencia 'print', con la cual se añade automáticamente el retorno de línea '\n': with open(self.nfich,"w") as f: for l in self.dic: print >>f, l Existe un modo de hacerlo todavía más compacto: open(self.nfich,"w").writelines(self.dic) ...pero no funciona bien ya que no añade los saltos de línea. Se puede apañar con compresiones de lista; pero mejor lo dejo para más adelante. En cuanto al estilo, se nota bastante que vienes de la programación imperativa. Tu clase es un saco de funciones que realizan acciones, evidente en que son "verbos" de una oración linguística. Deberías intentar expresar la clase como "mensajes" que el objeto debe responder. Desacoplando mejor los objetos de tu aplicación podrás reutilizarlos mejor en otras aplicaciones. Al fin de cuentas, la reusabilidad es uno de los grandes fines de la programación orientada a objeto. Por ejemplo: separa el código en dos clases, la clase Diccionario, encargada de manejar la lista de palabras, y la clase Menu, encargada del menú de opciones. El diccionario podría ser así: class Diccionario(): def __init__(self): self.eng=[] self.spa=[] def agregar(self, eng, spa): self.eng.append(eng) self.span.append(spa) return len(self.dic) #idx def obtener(self, idx): if 0<=idx ") Para elegir una acción: fmenu[op][1]() Espero que te sirva de ayuda. Saludos, Chema Cortés -- Hyperreals *R: http://ch3m4.org/blog Quarks, bits y otras criaturas infinitesimales From py en ch3m4.org Mon Oct 15 04:21:19 2012 From: py en ch3m4.org (Chema Cortes) Date: Mon, 15 Oct 2012 04:21:19 +0200 Subject: [Python-es] Compilar a javascript (pyjs/pyjaco) Message-ID: <507B731F.1000400@ch3m4.org> -----BEGIN PGP SIGNED MESSAGE----- Hash: SHA1 Parece que se están poniendo de moda los lenguajes que compilan a javascript. Por un lado, el CoffeeScript de microsoft, por otro el Dart de Google. No parece que vayan a desbancar a javascript de los navegadores, pero da idea del esfuerzo que se está empleando en buscar alternativas que mejoren la productividad. Como no quisiera volver a aprender otro lenguaje más, me preguntaba qué tal son los sistemas que compilan desde python, como pyjs (de pyjamas) o pyjaco. Me interesaría comentarios sobre: - - qué limitaciones en la sintáxis python imponen estas herramientas - - calidad del código javascript generado (velocidad de compilación y ejecución) - - posibilidad de que el compilado se haga en cliente (como permite coffeescript) - - otros compiladores a javascript recomendables (desde python o desde otros lenguajes) Muchas Gracias, Chema Cortés - --- Hyperreals *R: http://ch3m4.org/blog Quarks, bits y otras criaturas infinitesimales -----BEGIN PGP SIGNATURE----- Version: GnuPG v1.4.11 (GNU/Linux) Comment: Using GnuPG with Mozilla - http://www.enigmail.net/ iQEcBAEBAgAGBQJQe3MHAAoJEFdWyBWwhL4FQI0H/iQhhZi++L9LyZNkaj/OVcep d9ent8o59fRBXx/i0DjA77BEdkrOw9vwt0dfP6FObzjCYkUwdt6o4jXj42tHVx72 S4jFMeyC33szRkFUeKWvWwovNwWIuqKnRATyXBofzA8NBZt56LbMksGNM5ijtmrk iMA6psGI5zHyvLxTQhXeJTnGvYEVITqEW5VsNDR8S62KiRhhLO0S4TO8moePH7ok dobTgl51eV/fdhhJ+8WCPdWK4DTT5TfxSdWcO+C5r7UzY0NMh/DEdRN/SNcDQU82 SW0Me25nH9OWhYY19EX0ZRLkRaMFJOaKB1pIGXIY9hnES+D+xYdOqGhrBdYc/m4= =7LrO -----END PGP SIGNATURE----- From kikocorreoso en gmail.com Mon Oct 15 08:01:31 2012 From: kikocorreoso en gmail.com (Kiko) Date: Mon, 15 Oct 2012 08:01:31 +0200 Subject: [Python-es] Diccionarios In-Reply-To: <507B688E.8030303@ch3m4.org> References: <5079461C.8050605@gmail.com> <507A6C69.4050400@gmail.com> <507B688E.8030303@ch3m4.org> Message-ID: El 15 de octubre de 2012 03:36, Chema Cortes escribió: > El 14/10/12 09:40, kausdiv escribió: > > > Efectivamente he realizado el programa para aprender Python y de paso > > dotarme de una utilidad para memorizar vocabulario ingles. > > > > Dejo el código que he creado para que hagais las mejoras que veais > > oportunas y así pueda aprender más: > > OK. Pues te voy a dar algunos consejos: > > Plas, plas, plas!!!! Siempre se aprende algo de Chema. ------------ próxima parte ------------ Se ha borrado un adjunto en formato HTML... URL: From euribates en gmail.com Mon Oct 15 11:21:27 2012 From: euribates en gmail.com (Juan Ignacio) Date: Mon, 15 Oct 2012 10:21:27 +0100 Subject: [Python-es] Compilar a javascript (pyjs/pyjaco) In-Reply-To: <507B731F.1000400@ch3m4.org> References: <507B731F.1000400@ch3m4.org> Message-ID: Creo que el de microsoft se llama TypeScript, eso si, parece bastabte "inspirado" en CoffeeScript. From pych3m4 en gmail.com Mon Oct 15 13:32:52 2012 From: pych3m4 en gmail.com (Chema Cortes) Date: Mon, 15 Oct 2012 13:32:52 +0200 Subject: [Python-es] Compilar a javascript (pyjs/pyjaco) In-Reply-To: References: <507B731F.1000400@ch3m4.org> Message-ID: El 2012/10/15 Juan Ignacio : > Creo que el de microsoft se llama TypeScript, eso si, parece bastabte > "inspirado" en CoffeeScript. Pues sí, he debido confundir el anuncio del plugin de coffeescript para visual studio con la salida del typescript. Otro lenguaje más para mirar. -- Hyperreals *R: http://ch3m4.org/blog Quarks, bits y otras criaturas infinitesimales From kausdiv en gmail.com Mon Oct 15 14:17:37 2012 From: kausdiv en gmail.com (kausdiv) Date: Mon, 15 Oct 2012 14:17:37 +0200 Subject: [Python-es] Diccionarios In-Reply-To: <507B688E.8030303@ch3m4.org> References: <5079461C.8050605@gmail.com> <507A6C69.4050400@gmail.com> <507B688E.8030303@ch3m4.org> Message-ID: <507BFEE1.1020807@gmail.com> Se ha borrado un adjunto en formato HTML... URL: From nadaird en gmail.com Mon Oct 15 16:02:39 2012 From: nadaird en gmail.com (G V) Date: Mon, 15 Oct 2012 14:02:39 +0000 Subject: [Python-es] webapp2 y REST - Guardar el estado del cliente con cookies. In-Reply-To: References: <5072FD1B.2060404@nabla.net> Message-ID: Creo que si quieres que sea restfull, no deberias usar ni sesiones ni cookies. Ahora bien, si no eres muy tiquismiquis con esas cosas, pues nadie te va a saltar al cuello. Por si te sirve: http://blog.mikepearce.net/2010/08/24/cookies-and-the-restful-api/ 2012/10/9 Hiko hitokiri : > > pues lo que puedes hacer es usar webstorage ya que te provee de mas > almacenaje para tus configuraciones en el lado de el cliente que una cookie > -- > hiko_hitokiri no pidas que el mundo te trate de una manera cuando tu no le > das el mismo trato a los demás > > > _______________________________________________ > Python-es mailing list > Python-es en python.org > http://mail.python.org/mailman/listinfo/python-es > FAQ: http://python-es-faq.wikidot.com/ > From kausdiv en gmail.com Tue Oct 16 12:56:06 2012 From: kausdiv en gmail.com (kausdiv) Date: Tue, 16 Oct 2012 12:56:06 +0200 Subject: [Python-es] Salto de linea Message-ID: <507D3D46.50201@gmail.com> Hola. os comento algo que me pasa casi siempre que uso ficheros. :-( Tengo un archivo CSV donde hay muchas lineas repetidas. He preparado el siguiente programa que busca los registros repetidos. La idea es que se crea un nuevo archivo "bb.css" donde van todos los registros menos los repetidos. El problema es que se agrega un salto de linea que no debería ("\n") de forma que el resultado es algo as: --------- linea ok linea ok --------- Entre cada linea ok hay una linea en blanco. Este es el programa: ----------------------------------------- import csv cn=0 # para contar reptediso farchi="aa.csv" # archivo con todos los registros cm=[] # Para poder conocer los repetidos. f=open("bb.csv","w") # Archivo destino con los registros sin repetir. swriter = csv.writer(f, delimiter=',') with open(farchi, 'rb') as csvfile: spamreader = csv.reader(csvfile, delimiter=',') for row in spamreader: if len(row)>5: s = row[2]+" "+row[5] s = s.lower() if s in cm: cn += 1 else: cm.append(s) swriter.writerow(row) f.close() print "Registros duplicados ",cn From lgmerino en yahoo.com Tue Oct 16 13:22:34 2012 From: lgmerino en yahoo.com (Luis Garcia Merino) Date: Tue, 16 Oct 2012 12:22:34 +0100 (BST) Subject: [Python-es] Salto de linea In-Reply-To: <507D3D46.50201@gmail.com> References: <507D3D46.50201@gmail.com> Message-ID: <1350386554.98975.YahooMailNeo@web171203.mail.ir2.yahoo.com> Buenas, supongo que es porque el archivo CSV, al final de cada fila, tiene un salto de línea. Imagino también que row[5] es la última columna del CSV y, por lo tanto, tiene ese salto de línea. Podrías solucionarlo haciendo: s = row[2]+" "+row[5][:-1] en vez de s = row[2]+" "+row[5] Yo uso esto para limpiar las celdas de un CSV (y pasarlas a utf-8): def unicode_csv_reader(unicode_csv_data, dialect=csv.excel, **kwargs):         csv_reader = csv.reader(unicode_csv_data,dialect=dialect, **kwargs)     for row in csv_reader:         # en el yield puedes hacer lo que quieras con la celda... por ejemplo .strip() o el lower() que haces tu         yield [unicode(cell.replace("\n",""), 'utf-8') for cell in row] def haz_lo_que_sea_con_el_csv( csv_file ):     f =open(csv_file, 'r')     csv_reader = unicode_csv_reader( f )     # con esto lees una línea     row = csv_reader.next() No sé si es la mejor manera de hacerlo, pero funciona :) Un saludo. ________________________________ De: kausdiv Para: La lista de python en castellano Enviado: Martes 16 de octubre de 2012 12:56 Asunto: [Python-es] Salto de linea Hola. os comento algo que me pasa casi siempre que uso ficheros. :-( Tengo un archivo CSV donde hay muchas lineas repetidas. He preparado el siguiente programa que busca los registros repetidos. La idea es que se crea un nuevo archivo "bb.css" donde van todos los registros menos los repetidos. El problema es que se agrega un salto de linea que no debería ("\n") de forma que el resultado es algo as: --------- linea ok linea ok --------- Entre cada linea ok hay una linea en blanco. Este es el programa: ----------------------------------------- import csv cn=0 # para contar reptediso farchi="aa.csv"  # archivo con todos los registros cm=[]  # Para poder conocer los repetidos. f=open("bb.csv","w")  # Archivo destino con los registros sin repetir. swriter = csv.writer(f, delimiter=',') with open(farchi, 'rb') as csvfile:     spamreader = csv.reader(csvfile, delimiter=',')     for row in spamreader:         if len(row)>5:             s = row[2]+" "+row[5]             s = s.lower()             if s in cm:                 cn += 1             else:                 cm.append(s)                 swriter.writerow(row) f.close() print "Registros duplicados ",cn _______________________________________________ Python-es mailing list Python-es en python.org http://mail.python.org/mailman/listinfo/python-es FAQ: http://python-es-faq.wikidot.com/ ------------ próxima parte ------------ Se ha borrado un adjunto en formato HTML... URL: From kausdiv en gmail.com Tue Oct 16 13:38:57 2012 From: kausdiv en gmail.com (kausdiv) Date: Tue, 16 Oct 2012 13:38:57 +0200 Subject: [Python-es] Salto de linea In-Reply-To: <1350386554.98975.YahooMailNeo@web171203.mail.ir2.yahoo.com> References: <507D3D46.50201@gmail.com> <1350386554.98975.YahooMailNeo@web171203.mail.ir2.yahoo.com> Message-ID: <507D4751.1000109@gmail.com> Se ha borrado un adjunto en formato HTML... URL: From pych3m4 en gmail.com Tue Oct 16 14:13:42 2012 From: pych3m4 en gmail.com (Chema Cortes) Date: Tue, 16 Oct 2012 14:13:42 +0200 Subject: [Python-es] Salto de linea In-Reply-To: <507D4751.1000109@gmail.com> References: <507D3D46.50201@gmail.com> <1350386554.98975.YahooMailNeo@web171203.mail.ir2.yahoo.com> <507D4751.1000109@gmail.com> Message-ID: El día 16 de octubre de 2012 13:38, kausdiv escribió: > Hola Luis. Gracias por contestarme. > > Verás row[2] y 5 se guardan en una lista para ir comprobando si existe la > linea. Pero no se guarda. > > Lo que se guarda es row tal como se lee: > > if s in cm: > cn += 1 > else: > cm.append(s) > swriter.writerow(row) > > Es decir si s (row[2] y row[5]) no esta en la lista cm (donde van estos dos > registros unidos.) Entonces garda s en la lista cm para comprobar más > adelante y guarda row tal como se leyó en el archivo de salida. > > Pero si ves, row no ha sufrido modificación alguna. Eso es lo que me > desconcierta. Por cierto probé swriter.writerow(row[:-1]) y no ha > funcionado. > > ¿ que podrá ser ? Emplea el parámetro 'lineterminator'. Según la documentación, csv.reader detecta siempre el fin de línea, pero csv.writer emplea por defecto '\r\n'. Supongo que usas linux: spamreader = csv.reader(csvfile, delimiter=',', lineterminator='\r') -- Hyperreals *R: http://ch3m4.org/blog Quarks, bits y otras criaturas infinitesimales From lgmerino en yahoo.com Tue Oct 16 14:51:20 2012 From: lgmerino en yahoo.com (Luis Garcia Merino) Date: Tue, 16 Oct 2012 13:51:20 +0100 (BST) Subject: [Python-es] Salto de linea In-Reply-To: References: <507D3D46.50201@gmail.com> <1350386554.98975.YahooMailNeo@web171203.mail.ir2.yahoo.com> <507D4751.1000109@gmail.com> Message-ID: <1350391880.6459.YahooMailNeo@web171202.mail.ir2.yahoo.com> Upssss... perdón, no leí bien tu código... Si sirve de algo, he ejecutado tu programa y me genera correctamente el csv de salida, sin líneas en blanco...  Uso GNU/Linux y mi versión de Python es la 2.7.2. ________________________________ De: Chema Cortes Para: La lista de python en castellano CC: Luis Garcia Merino Enviado: Martes 16 de octubre de 2012 14:13 Asunto: Re: [Python-es] Salto de linea El día 16 de octubre de 2012 13:38, kausdiv escribió: > Hola Luis. Gracias por contestarme. > > Verás row[2] y 5 se guardan en una lista para ir comprobando si existe la > linea. Pero no se guarda. > > Lo que se guarda es row tal como se lee: > > if s in cm: >                cn += 1 >    else: >          cm.append(s) >          swriter.writerow(row) > > Es decir si s (row[2] y row[5]) no esta en la lista cm (donde van estos dos > registros unidos.) Entonces garda s en la lista cm para comprobar más > adelante y guarda row tal como se leyó en el archivo de salida. > > Pero si ves, row no ha sufrido modificación alguna. Eso es lo que me > desconcierta. Por cierto probé swriter.writerow(row[:-1]) y no ha > funcionado. > > ¿ que podrá ser ? Emplea el parámetro 'lineterminator'. Según la documentación, csv.reader detecta siempre el fin de línea, pero csv.writer emplea por defecto '\r\n'. Supongo que usas linux:   spamreader = csv.reader(csvfile, delimiter=',', lineterminator='\r') -- Hyperreals *R: http://ch3m4.org/blog Quarks, bits y otras criaturas infinitesimales ------------ próxima parte ------------ Se ha borrado un adjunto en formato HTML... URL: From kausdiv en gmail.com Tue Oct 16 14:55:52 2012 From: kausdiv en gmail.com (kausdiv) Date: Tue, 16 Oct 2012 14:55:52 +0200 Subject: [Python-es] Salto de linea In-Reply-To: References: <507D3D46.50201@gmail.com> <1350386554.98975.YahooMailNeo@web171203.mail.ir2.yahoo.com> <507D4751.1000109@gmail.com> Message-ID: <507D5958.7060800@gmail.com> Hola Chema. Muchas gracias. No ha funcionado. Uso windows. :-( he probado también antes de guardar: row.pop() swriter.writerow(row) para que elimine la ultima posición de la lista, pero aún asi nova. hemirado con notepad++ y las lineas tienen: la lina ok termina con el caracer (cr) las lineas en blanco: (cr)(lf) también he probado: swriter.writerow(row, lineterminator='\r ) Tampoco va. :-( que podrá ser ? > Emplea el parámetro 'lineterminator'. Según la documentación, > csv.reader detecta siempre el fin de línea, pero csv.writer emplea por > defecto '\r\n'. > > Supongo que usas linux: > > spamreader = csv.reader(csvfile, delimiter=',', lineterminator='\r') > > > From kausdiv en gmail.com Tue Oct 16 15:12:10 2012 From: kausdiv en gmail.com (kausdiv) Date: Tue, 16 Oct 2012 15:12:10 +0200 Subject: [Python-es] Salto de linea In-Reply-To: <1350391880.6459.YahooMailNeo@web171202.mail.ir2.yahoo.com> References: <507D3D46.50201@gmail.com> <1350386554.98975.YahooMailNeo@web171203.mail.ir2.yahoo.com> <507D4751.1000109@gmail.com> <1350391880.6459.YahooMailNeo@web171202.mail.ir2.yahoo.com> Message-ID: <507D5D2A.2020306@gmail.com> Se ha borrado un adjunto en formato HTML... URL: From clatorreg en dphuesca.es Tue Oct 16 15:12:39 2012 From: clatorreg en dphuesca.es (Carlos Latorre) Date: Tue, 16 Oct 2012 15:12:39 +0200 Subject: [Python-es] Salto de linea In-Reply-To: <507D4751.1000109@gmail.com> References: <507D3D46.50201@gmail.com> <1350386554.98975.YahooMailNeo@web171203.mail.ir2.yahoo.com> <507D4751.1000109@gmail.com> Message-ID: El 16 de octubre de 2012 13:38, kausdiv escribió: > Hola Luis. Gracias por contestarme. > > Verás row[2] y 5 se guardan en una lista para ir comprobando si existe la > linea. Pero no se guarda. > > Lo que se guarda es row tal como se lee: > > if s in cm: > cn += 1 > else: > cm.append(s) > swriter.writerow(row) > > Es decir si s (row[2] y row[5]) no esta en la lista cm (donde van estos > dos registros unidos.) Entonces garda s en la lista cm para comprobar más > adelante y guarda row tal como se leyó en el archivo de salida. > > Pero si ves, row no ha sufrido modificación alguna. Eso es lo que me > desconcierta. Por cierto probé swriter.writerow(row[:-1]) y no ha > funcionado. > prueba swriter.writerow(row[:-1]+[row[-1][:-1]]) a ver que pasa Saludos, Carlos L. > > ¿ que podrá ser ? > > Gracias. > > > El 16/10/2012 13:22, Luis Garcia Merino escribió: > > Buenas, > > supongo que es porque el archivo CSV, al final de cada fila, tiene un > salto de línea. Imagino también que row[5] es la última columna del CSV y, > por lo tanto, tiene ese salto de línea. Podrías solucionarlo haciendo: > > s = row[2]+" "+row[5][:-1] > > en vez de > > s = row[2]+" "+row[5] > > > > Yo uso esto para limpiar las celdas de un CSV (y pasarlas a utf-8): > > def unicode_csv_reader(unicode_csv_data, dialect=csv.excel, **kwargs): > > csv_reader = csv.reader(unicode_csv_data,dialect=dialect, **kwargs) > > for row in csv_reader: > # en el yield puedes hacer lo que quieras con la celda... por > ejemplo .strip() o el lower() que haces tu > yield [unicode(cell.replace("\n",""), 'utf-8') for cell in row] > > > def haz_lo_que_sea_con_el_csv( csv_file ): > > f = open(csv_file, 'r') > > csv_reader = unicode_csv_reader( f ) > > # con esto lees una línea > row = csv_reader.next() > > > No sé si es la mejor manera de hacerlo, pero funciona :) > > Un saludo. > > ------------------------------ > *De:* kausdiv > *Para:* La lista de python en castellano > *Enviado:* Martes 16 de octubre de 2012 12:56 > *Asunto:* [Python-es] Salto de linea > > Hola. > > os comento algo que me pasa casi siempre que uso ficheros. :-( > > Tengo un archivo CSV donde hay muchas lineas repetidas. He preparado el > siguiente programa que busca los registros repetidos. > La idea es que se crea un nuevo archivo "bb.css" donde van todos los > registros menos los repetidos. > > El problema es que se agrega un salto de linea que no debería ("\n") de > forma que el resultado es algo as: > --------- > linea ok > > linea ok > --------- > Entre cada linea ok hay una linea en blanco. > > Este es el programa: > ----------------------------------------- > import csv > > cn=0 # para contar reptediso > farchi="aa.csv" # archivo con todos los registros > cm=[] # Para poder conocer los repetidos. > > f=open("bb.csv","w") # Archivo destino con los registros sin repetir. > swriter = csv.writer(f, delimiter=',') > > with open(farchi, 'rb') as csvfile: > spamreader = csv.reader(csvfile, delimiter=',') > for row in spamreader: > if len(row)>5: > s = row[2]+" "+row[5] > s = s.lower() > if s in cm: > cn += 1 > else: > cm.append(s) > swriter.writerow(row) > > f.close() > > print "Registros duplicados ",cn > _______________________________________________ > Python-es mailing list > Python-es en python.org > http://mail.python.org/mailman/listinfo/python-es > FAQ: http://python-es-faq.wikidot.com/ > > > > > _______________________________________________ > Python-es mailing listPython-es en python.orghttp://mail.python.org/mailman/listinfo/python-es > FAQ: http://python-es-faq.wikidot.com/ > > > > > > _______________________________________________ > Python-es mailing list > Python-es en python.org > http://mail.python.org/mailman/listinfo/python-es > FAQ: http://python-es-faq.wikidot.com/ > > ------------ próxima parte ------------ Se ha borrado un adjunto en formato HTML... URL: From lgmerino en yahoo.com Tue Oct 16 15:23:32 2012 From: lgmerino en yahoo.com (Luis Garcia Merino) Date: Tue, 16 Oct 2012 14:23:32 +0100 (BST) Subject: [Python-es] Salto de linea In-Reply-To: <507D5D2A.2020306@gmail.com> References: <507D3D46.50201@gmail.com> <1350386554.98975.YahooMailNeo@web171203.mail.ir2.yahoo.com> <507D4751.1000109@gmail.com> <1350391880.6459.YahooMailNeo@web171202.mail.ir2.yahoo.com> <507D5D2A.2020306@gmail.com> Message-ID: <1350393812.75611.YahooMailNeo@web171201.mail.ir2.yahoo.com> Bueno, el \r\n lo puedes quitar también en el código... y quizás por eso tampoco te funcionaba lo del row[:-1], porque \r\n son dos caracteres y con eso sólo quitas el último, si quieres, prueba con row[:-2]. ________________________________ De: kausdiv Para: Luis Garcia Merino ; La lista de python en castellano Enviado: Martes 16 de octubre de 2012 15:12 Asunto: Re: [Python-es] Salto de linea Gracias de nuevo Luis. No se qué puede ser. Si veo el archivo de entrada (aa.csv) no hay lineas en blanco. Cosas de Windows, quizá. :-) como no lo he podido solucionar he optado por buscar y reemplazar con notepad++ los calaractere \r\n por nada y así se han eliminado las lineas en blanco. Al menos salgo del paso. Gracias a todos. :-) Seguiré viendo como resolverlo para próximas aplicaciones. El 16/10/2012 14:51, Luis Garcia Merino escribió: Upssss... perdón, no leí bien tu código... > >Si sirve de algo, he ejecutado tu programa y me genera correctamente el csv de salida, sin líneas en blanco...  > >Uso GNU/Linux y mi versión de Python es la 2.7.2. > > > >________________________________ > De: Chema Cortes >Para: La lista de python en castellano >CC: Luis Garcia Merino >Enviado: Martes 16 de octubre de 2012 14:13 >Asunto: Re: [Python-es] Salto de linea > >El día 16 de octubre de 2012 13:38, kausdiv escribió: >> Hola Luis. Gracias por contestarme. >> >> Verás row[2] y 5 se guardan en una lista para ir comprobando si existe la >> linea. Pero no se guarda. >> >> Lo que se guarda es row tal como se lee: >> >> if s in cm: >>                cn += 1 >>    else: >>          cm.append(s) >>          swriter.writerow(row) >> >> Es decir si s (row[2] y row[5]) no esta en la lista cm (donde van estos dos >> registros unidos.) Entonces garda s en la lista cm para comprobar más >> adelante y guarda row tal como se leyó en el archivo de salida. >> >> Pero si ves, row no ha sufrido modificación alguna. Eso es lo que me >> desconcierta. Por cierto probé swriter.writerow(row[:-1]) y no ha >> funcionado. >> >> ¿ que podrá ser ? > >Emplea el parámetro 'lineterminator'. Según la documentación, >csv.reader detecta siempre el fin de línea, pero csv.writer emplea por >defecto '\r\n'. > >Supongo que usas linux: > >  spamreader = csv.reader(csvfile, delimiter=',', lineterminator='\r') > > > >-- >Hyperreals *R: http://ch3m4.org/blog >Quarks, bits y otras criaturas infinitesimales > > > > > >_______________________________________________ Python-es mailing list Python-es en python.org http://mail.python.org/mailman/listinfo/python-es FAQ: http://python-es-faq.wikidot.com/ ------------ próxima parte ------------ Se ha borrado un adjunto en formato HTML... URL: From pych3m4 en gmail.com Tue Oct 16 17:34:03 2012 From: pych3m4 en gmail.com (Chema Cortes) Date: Tue, 16 Oct 2012 17:34:03 +0200 Subject: [Python-es] Salto de linea In-Reply-To: <507D5958.7060800@gmail.com> References: <507D3D46.50201@gmail.com> <1350386554.98975.YahooMailNeo@web171203.mail.ir2.yahoo.com> <507D4751.1000109@gmail.com> <507D5958.7060800@gmail.com> Message-ID: El día 16 de octubre de 2012 14:55, kausdiv escribió: > No ha funcionado. > Uso windows. :-( Revisando tu código original, veo que no has abierto el fichero de escritura como binario. Mira a ver si es éso. -- Hyperreals *R: http://ch3m4.org/blog Quarks, bits y otras criaturas infinitesimales From gerardojuarez en buyteknet.info Wed Oct 17 17:49:31 2012 From: gerardojuarez en buyteknet.info (gerardo Juarez) Date: Wed, 17 Oct 2012 10:49:31 -0500 Subject: [Python-es] Salto de linea In-Reply-To: <507D5958.7060800@gmail.com> References: <507D3D46.50201@gmail.com> <1350386554.98975.YahooMailNeo@web171203.mail.ir2.yahoo.com> <507D4751.1000109@gmail.com> <507D5958.7060800@gmail.com> Message-ID: <507ED38B.6090601@buyteknet.info> kausdiv wrote: > Hola Chema. Muchas gracias. > > No ha funcionado. > Uso windows. :-( > > he probado también antes de guardar: > row.pop() > swriter.writerow(row) > > para que elimine la ultima posición de la lista, pero aún asi nova. > > hemirado con notepad++ y las lineas tienen: > la lina ok termina con el caracer (cr) > las lineas en blanco: (cr)(lf) > > también he probado: > swriter.writerow(row, lineterminator='\r ) > > Tampoco va. :-( > > que podrá ser ? > >> Emplea el parámetro 'lineterminator'. Según la documentación, >> csv.reader detecta siempre el fin de línea, pero csv.writer emplea por >> defecto '\r\n'. >> >> Supongo que usas linux: >> >> spamreader = csv.reader(csvfile, delimiter=',', lineterminator='\r') >> >> >> > > _______________________________________________ > Python-es mailing list > Python-es en python.org > http://mail.python.org/mailman/listinfo/python-es > FAQ: http://python-es-faq.wikidot.com/ > El terminador de las líneas de tu archivo parece inconsistente. A veces es '\n', otras es '\r\n'. No te sirve strip()?: while 1: line = input.readline() if not line: break line = line.strip() Saludos Gerardo From gmourinopardo en yahoo.es Thu Oct 18 09:58:09 2012 From: gmourinopardo en yahoo.es (=?iso-8859-1?Q?Gonzalo_Mouri=FFfffffffffff1o_Pardo?=) Date: Thu, 18 Oct 2012 08:58:09 +0100 (BST) Subject: [Python-es] GridBagSizer no responde In-Reply-To: <507ED38B.6090601@buyteknet.info> References: <507D3D46.50201@gmail.com> <1350386554.98975.YahooMailNeo@web171203.mail.ir2.yahoo.com> <507D4751.1000109@gmail.com> <507D5958.7060800@gmail.com> <507ED38B.6090601@buyteknet.info> Message-ID: <1350547089.86460.YahooMailNeo@web171706.mail.ir2.yahoo.com> Buenos días: Tengo una duda el código anexo no me genera bien el GriBagSizer, pero si la función la realizo en la función __init__ si, alguna solución: Gracias. #!/usr/bin/env python # -*- coding: cp1252 -*- import os import wx import new_user import about import data # News Identifiers ID_NUEVO = wx.NewId() ID_ABRIR = wx.NewId() ID_CERRAR = wx.NewId() ID_GUARDAR = wx.NewId() ID_GUARDAR_COMO = wx.NewId() ID_IMPRIMIR = wx.NewId() ID_SALIR = wx.NewId() ID_DESHACER = wx.NewId() ID_REHACER = wx.NewId() ID_CORTAR = wx.NewId() ID_COPIAR = wx.NewId() ID_PEGAR = wx.NewId() ID_MESES = wx.NewId() ID_ENERO = wx.NewId() ID_AYUDA = wx.NewId() ID_ACERCA_DE = wx.NewId() class Aplicacion(wx.App):     def OnInit(self):         self.frame = Ventana(None, -1, title = "Control Horario")         self.SetTopWindow(self.frame)         self.frame.Show()         return True class Pagina(wx.Panel):     def __init__(self, parent):         wx.Panel.__init__(self, parent)         self.carga = data.TimeData()         self.month = self.carga.month     def InitUI(self):         font = wx.Font(15, family = wx.FONTFAMILY_DEFAULT, style =                         wx.FONTSTYLE_NORMAL, weight = wx.FONTWEIGHT_BOLD,                         encoding = wx.FONTENCODING_CP1252)         # Layout         self.sizer = wx.BoxSizer(wx.HORIZONTAL)         self.gbsizer = wx.GridBagSizer(0, 0)         self.sizer.Add(self.gbsizer, 1, wx.ALL, border = 5)         self.SetSizer(self.sizer)         # Column's Labels         sem = {0 : ("Lunes", 169), 1 : ("Martes", 266), 2 : ("Miercoles" , 359),                3 : ("Jueves",  478), 4 : ("Viernes", 578), 5: ("Sabado", 686),                6 : ("Domingo", 783)}         for num in range(0,7):             self.cont = sem[num]             dia = self.cont[0]             y = num + 1             self.collab = wx.StaticText(self, -1, label = dia,  size = wx.Size                                         (105, 25), style = wx.ALIGN_CENTER |                                         wx.SIMPLE_BORDER)             self.collab.SetFont(font)             if y < 6:                 self.collab.SetBackgroundColour(wx.Colour(255, 255, 255))             else:                 self.collab.SetBackgroundColour(wx.Colour(255, 0, 0))             self.gbsizer.Add(self.collab, pos = wx.GBPosition(0, y), flag =                         wx.ALIGN_CENTER_VERTICAL, border = 5)         # Month's Label         self.mes = self.carga.month_dict[self.month]         self.monlab = wx.StaticText(self, -1, label = self.mes, size = wx.Size(140,                                                                           25),                                     style = wx.ALIGN_CENTER)         self.monlab.SetFont(font)         self.monlab.SetForegroundColour(wx.Colour(255, 0, 255))         self.gbsizer.Add(self.monlab, pos = wx.GBPosition(0, 0), flag =                     wx.ALIGN_CENTER_VERTICAL,  border = 5)         # Row's Labels         self.month_name = self.carga.month_tup[self.month - 1]         num_month = self.carga.nweek_dict[self.month_name]         numero = str(num_month)         for num in range(0, 5):             x = num + 1             self.rowlab = wx.StaticText(self, -1, label = numero, size = wx.Size                                         (140, 25), style = wx.ALIGN_CENTER |                                         wx.SIMPLE_BORDER)             self.rowlab.SetFont(font)             self.rowlab.SetForegroundColour(wx.Colour(0, 0, 255))             self.rowlab.SetBackgroundColour(wx.Colour(255, 255, 255))             self.rowlab.Wrap(-1)             self.gbsizer.Add(self.rowlab, pos = wx.GBPosition(x, 0), flag =                         wx.ALIGN_CENTER_VERTICAL, border = 5)             number = int(numero) + 1             numero = str(number)         # Day's Labels         day = self.carga.week_dict[self.month_name]         end_dict = self.carga.num_dict[self.month_name]         self.daylab_dict = {}         if day == "Mon":             y = 1             self.coor_x = 145         elif day == "Tue":             y = 2         elif day == "Wed":             y = 3         elif day == "Thu":             y = 4         elif day == "Fri":             y = 5         elif day == "Sat":             y = 6         else:             y = 7         x = 1         for cont in range (0, end_dict):             contador = str(cont+1)             if y > 7:                 x = x + 1                 y = 1             self.daylab = wx.StaticText(self, label = contador, size = wx.Size                                         (105, 25), style = wx.ALIGN_CENTRE |                                         wx.SIMPLE_BORDER)             self.daylab.SetFont(font)             self.daylab.SetBackgroundColour(wx.Colour(255, 255, 255))             if y > 5:                 self.daylab.SetBackgroundColour(wx.Colour(255, 0, 0))             self.daylab_dict [(x, y)] = contador             self.gbsizer.Add(self.daylab, pos = wx.GBPosition(x, y), flag = wx.ALIGN_CENTER_VERTICAL,                         border = 5)             self.daylab.Bind(wx.EVT_LEFT_DOWN, self.OnLeftDown)             y = y + 1         # Blank's Labels         resto = 35 - end_dict         for cont in range (0, resto):             self.blalab = wx.StaticText(self, label = "", size = wx.Size(105, 25                                                                          ),                                         style = wx.ALIGN_CENTER |                                         wx.SIMPLE_BORDER)             self.blalab.SetFont(font)             self.blalab.SetBackgroundColour(wx.Colour(255, 255, 255))             if y > 5:                 self.blalab.SetBackgroundColour(wx.Colour(255, 0, 0))             self.gbsizer.Add(self.blalab, pos = wx.GBPosition(x, y), flag = wx.ALIGN_CENTER_VERTICAL,                        border = 5)             y = y + 1     def OnLeftDown(self, event):         mou_pos = wx.GetMousePosition()         scr_pos = self.GetScreenPosition()         abs_x = mou_pos.x - scr_pos.x         abs_y = mou_pos.y - scr_pos.y         if abs_x < 250:             coor_x = 1         elif abs_x >= 250 and abs_x < 355:             coor_x = 2         elif abs_x >= 355 and abs_x < 460:             coor_x = 3         elif abs_x >= 460 and abs_x < 565:             coor_x = 4         elif abs_x >= 565 and abs_x < 670:             coor_x = 5         elif abs_x >= 670 and abs_x < 775:             coor_x = 6         else:             coor_x = 7         if abs_y < 55:             coor_y = 1         elif abs_y >= 55 and abs_y < 80:             coor_y = 2         elif abs_y >= 80 and abs_y < 105:             coor_y = 3         elif abs_y >= 105 and abs_y < 130:             coor_y = 4         else:             coor_y = 5         data_day = self.daylab_dict[(coor_y, coor_x)]         event.Skip() class Ventana(wx.Frame):          """Constructor"""     def __init__(self, parent, id, title):         super(Ventana, self).__init__(parent, id, title, size = (895, 275))         # Attributes         self.panel = Pagina(self)         # Setup Menu         menubar = wx.MenuBar()         # File's Menu         arcm = wx.Menu()         aami = wx.MenuItem(arcm, ID_ABRIR, "&Abrir...\tCtrl+A")         arcm.AppendItem(aami)         menubar.Append(arcm, '&Archivo')         self.SetMenuBar(menubar)                 # Event Handlers         self.Bind(wx.EVT_MENU, self.OnMenu)     """Do menu events"""     def OnMenu(self, event):         self.evt_id = event.GetId ()         if self.evt_id == ID_ABRIR:             self.month = 2         event.Skip()          if __name__ == "__main__":     app = Aplicacion(False)     app.MainLoop() ------------ próxima parte ------------ Se ha borrado un adjunto en formato HTML... URL: From pych3m4 en gmail.com Thu Oct 18 13:14:01 2012 From: pych3m4 en gmail.com (Chema Cortes) Date: Thu, 18 Oct 2012 13:14:01 +0200 Subject: [Python-es] GridBagSizer no responde In-Reply-To: <1350547089.86460.YahooMailNeo@web171706.mail.ir2.yahoo.com> References: <507D3D46.50201@gmail.com> <1350386554.98975.YahooMailNeo@web171203.mail.ir2.yahoo.com> <507D4751.1000109@gmail.com> <507D5958.7060800@gmail.com> <507ED38B.6090601@buyteknet.info> <1350547089.86460.YahooMailNeo@web171706.mail.ir2.yahoo.com> Message-ID: El 2012/10/18 Gonzalo Mouriÿfffffffffff1o Pardo : > Tengo una duda el código anexo no me genera bien el GriBagSizer, pero si la > función la realizo en la función __init__ si, alguna solución: No sé si te entiendo bien. Debes llamar a self.InitUI() en __init__ para que se inicialice el GUI -- Hyperreals *R: http://ch3m4.org/blog Quarks, bits y otras criaturas infinitesimales From emperor.cu en gmail.com Thu Oct 18 15:18:24 2012 From: emperor.cu en gmail.com (=?UTF-8?Q?Tony_Pe=C3=B1a?=) Date: Thu, 18 Oct 2012 09:18:24 -0400 Subject: [Python-es] 3d globe Message-ID: buenas, como o conque se puede hacer una presentacion de la tierra, y poner una posicion por ejemplo 0,0 un imagen pequeña cualquiera, he probado usando openstreetmap, y googlemaps, pero asi solo alcanzo a verlo en un plano, y lo mas bonito quedaria viendolo como en googleearth.. que se pueda dar vueltas y esas cosas y se vea ese objeto en esa posicion. tambien probe marble pero funciona mas para la gente que sabe C/C++, los bindings para python no estan bien definidos aun y dan mucho problemas porque lo quiero usar con PyQt4 y marble es de la gente de PyKDE. algo como python y opengl o cualquier otra via que uds conozcan que hayan utilizado, no se algun que otra persona que sea deutilidad en su carrera de geografo o cosas asi que le guste python puede que le haya pensado hacer cosas parecida usando esto,.... pero me gustaria poder tener un ejemplo donde se vea claro, lo mas simple posible, alguien tiene por ahi guardado algo como esto?. gracias. -- Antonio Peña Secure email with PGP 0x8B021001 available at http://pgp.mit.edu Fingerprint: 74E6 2974 B090 366D CE71 7BB2 6476 FA09 8B02 1001 ------------ próxima parte ------------ Se ha borrado un adjunto en formato HTML... URL: From kikocorreoso en gmail.com Thu Oct 18 16:02:39 2012 From: kikocorreoso en gmail.com (Kiko) Date: Thu, 18 Oct 2012 16:02:39 +0200 Subject: [Python-es] 3d globe In-Reply-To: References: Message-ID: El 18 de octubre de 2012 15:18, Tony Peña escribió: > buenas, > > como o conque se puede hacer una presentacion de la tierra, y poner una > posicion por ejemplo 0,0 un imagen pequeña cualquiera, > he probado usando openstreetmap, y googlemaps, pero asi solo alcanzo a > verlo en un plano, y lo mas bonito quedaria viendolo como en googleearth.. > que se pueda dar vueltas y esas cosas y se vea ese objeto en esa posicion. > > tambien probe marble pero funciona mas para la gente que sabe C/C++, los > bindings para python no estan bien definidos aun y dan mucho problemas > porque lo quiero usar con PyQt4 > y marble es de la gente de PyKDE. > > algo como python y opengl o cualquier otra via que uds conozcan que hayan > utilizado, no se algun que otra persona que sea deutilidad en su carrera de > geografo o cosas asi que le guste python puede que le haya pensado hacer > cosas parecida usando esto,.... pero me gustaria poder tener un ejemplo > donde se vea claro, lo mas simple posible, > alguien tiene por ahi guardado algo como esto?. > > gracias. > Hola, con pyqt puedes usar el QtWebKit e intentar meter ahí el plugin de google earth ahí e intentar controlarlo desde pyqt: http://lists.qt.nokia.com/pipermail/qt-interest/2011-October/036188.html O meter javascript en la vista web de qt ( http://www.webglearth.com/#ll=18.92011,-2.56421;alt=10000000;h=0.000;t=0.000), básicamente un navegador hecho con pyqt.... Otra opción es controlar directamente google earth (la aplicación de escritorio) desde python con https://github.com/posborne/pygoogleearth(creo que es solo windows...). Una última opción es hacer algo más estático 'incrustando' matplotlib+basemap en Qt. Si consigues alguna cosa u otra aproximación lo podrías compartir por aquí puesto que es algo que me interesa. Saludos. ------------ próxima parte ------------ Se ha borrado un adjunto en formato HTML... URL: From emperor.cu en gmail.com Thu Oct 18 16:11:49 2012 From: emperor.cu en gmail.com (=?UTF-8?Q?Tony_Pe=C3=B1a?=) Date: Thu, 18 Oct 2012 10:11:49 -0400 Subject: [Python-es] 3d globe In-Reply-To: References: Message-ID: ah vale estare al tanto de las propuestas, es algo que quiero utilizar para mejorar mi hobby https://github.com/emper0r/IVAO-status :-) saludos El 18 de octubre de 2012 10:02, Kiko escribió: > > > El 18 de octubre de 2012 15:18, Tony Peña escribió: > > buenas, >> >> como o conque se puede hacer una presentacion de la tierra, y poner una >> posicion por ejemplo 0,0 un imagen pequeña cualquiera, >> he probado usando openstreetmap, y googlemaps, pero asi solo alcanzo a >> verlo en un plano, y lo mas bonito quedaria viendolo como en googleearth.. >> que se pueda dar vueltas y esas cosas y se vea ese objeto en esa posicion. >> >> tambien probe marble pero funciona mas para la gente que sabe C/C++, los >> bindings para python no estan bien definidos aun y dan mucho problemas >> porque lo quiero usar con PyQt4 >> y marble es de la gente de PyKDE. >> >> algo como python y opengl o cualquier otra via que uds conozcan que hayan >> utilizado, no se algun que otra persona que sea deutilidad en su carrera de >> geografo o cosas asi que le guste python puede que le haya pensado hacer >> cosas parecida usando esto,.... pero me gustaria poder tener un ejemplo >> donde se vea claro, lo mas simple posible, >> alguien tiene por ahi guardado algo como esto?. >> >> gracias. >> > > Hola, con pyqt puedes usar el QtWebKit e intentar meter ahí el plugin de > google earth ahí e intentar controlarlo desde pyqt: > http://lists.qt.nokia.com/pipermail/qt-interest/2011-October/036188.html > > O meter javascript en la vista web de qt ( > http://www.webglearth.com/#ll=18.92011,-2.56421;alt=10000000;h=0.000;t=0.000), > básicamente un navegador hecho con pyqt.... > > Otra opción es controlar directamente google earth (la aplicación de > escritorio) desde python con https://github.com/posborne/pygoogleearth(creo que es solo windows...). > > Una última opción es hacer algo más estático 'incrustando' > matplotlib+basemap en Qt. > > Si consigues alguna cosa u otra aproximación lo podrías compartir por aquí > puesto que es algo que me interesa. > > Saludos. > > _______________________________________________ > Python-es mailing list > Python-es en python.org > http://mail.python.org/mailman/listinfo/python-es > FAQ: http://python-es-faq.wikidot.com/ > > -- Antonio Peña Secure email with PGP 0x8B021001 available at http://pgp.mit.edu Fingerprint: 74E6 2974 B090 366D CE71 7BB2 6476 FA09 8B02 1001 ------------ próxima parte ------------ Se ha borrado un adjunto en formato HTML... URL: From davidmenhur en gmail.com Thu Oct 18 16:28:21 2012 From: davidmenhur en gmail.com (=?UTF-8?B?RGHPgGlk?=) Date: Thu, 18 Oct 2012 16:28:21 +0200 Subject: [Python-es] 3d globe In-Reply-To: References: Message-ID: Creo que con Visual Python [1] y un poco de curro se puede hacer. Una de las texturas que vienen incluídas es una imagen satélite de la Tierra, y en el código fuente lo único que han hecho ha sido añadido la imagen plana, en una cierta proyección. [1] http://www.vpython.org/ 2012/10/18 Tony Peña : > buenas, > > como o conque se puede hacer una presentacion de la tierra, y poner una > posicion por ejemplo 0,0 un imagen pequeña cualquiera, > he probado usando openstreetmap, y googlemaps, pero asi solo alcanzo a verlo > en un plano, y lo mas bonito quedaria viendolo como en googleearth.. > que se pueda dar vueltas y esas cosas y se vea ese objeto en esa posicion. > > tambien probe marble pero funciona mas para la gente que sabe C/C++, los > bindings para python no estan bien definidos aun y dan mucho problemas > porque lo quiero usar con PyQt4 > y marble es de la gente de PyKDE. > > algo como python y opengl o cualquier otra via que uds conozcan que hayan > utilizado, no se algun que otra persona que sea deutilidad en su carrera de > geografo o cosas asi que le guste python puede que le haya pensado hacer > cosas parecida usando esto,.... pero me gustaria poder tener un ejemplo > donde se vea claro, lo mas simple posible, > alguien tiene por ahi guardado algo como esto?. > > gracias. > > -- > Antonio Peña > Secure email with PGP 0x8B021001 available at http://pgp.mit.edu > Fingerprint: 74E6 2974 B090 366D CE71 7BB2 6476 FA09 8B02 1001 > > _______________________________________________ > Python-es mailing list > Python-es en python.org > http://mail.python.org/mailman/listinfo/python-es > FAQ: http://python-es-faq.wikidot.com/ > From soft.sirius en gmail.com Thu Oct 18 17:57:33 2012 From: soft.sirius en gmail.com (Juan M. Puertas) Date: Thu, 18 Oct 2012 17:57:33 +0200 Subject: [Python-es] Compilar a javascript (pyjs/pyjaco) In-Reply-To: <507B731F.1000400@ch3m4.org> References: <507B731F.1000400@ch3m4.org> Message-ID: Hola Chema. En ello estoy también. En mi primera prueba con pyjamas en Ubuntu 12.04, sólo ha funcionado la parte cliente, no la del servidor, hay que tener en cuenta que aún se encuentra en versión alfa. Por lo que he podido leer, Luke; el autor de pyjamas, deja su desarrollo debido a los "secuestradores" de pyjs (?), https://groups.google.com/forum/#!topic/gwt-coffee/pEnCfUDTqTQ y Christian, de pyjaco está continuando la labor. Lo que más me gusta del concepto de pyjamas es que puedes desarrollar en python para el escritorio y para la web sin cambiar el código, lo cual, según mi visión, resuelve el "eterno" dilema sobre qué GUI utilizar, ya que pyjamas está/estaba (?) desarrollando unas bibliotecas gráficas que podrían acordarse como estándares. Aquí Christian muestra un ejemplo con pyjaco y jquery: http://archlinux.me/dusty/2011/12/27/pyjaco-and-jquery/ Ya lo tengo claro; ni TK, ni WX, ni GTK, ni QT, sino *HTML, *así puedes estar "casi" seguro de que no vas a tener que construir los formularios de tu aplicación por enésima vez... ;-))* *. Desde ahora, gracias a Websocket de HTML5, puedes programar con el lenguaje que más te guste y comunicarlo con el formulario en HTML5 de una manera facilísima. Saludos afectuosos desde Granada. :-) Juanma.- El 15 de octubre de 2012 04:21, Chema Cortes escribió: > > -----BEGIN PGP SIGNED MESSAGE----- > Hash: SHA1 > > Parece que se están poniendo de moda los lenguajes que compilan a > javascript. Por un lado, el CoffeeScript de microsoft, por otro el Dart > de Google. No parece que vayan a desbancar a javascript de los > navegadores, pero da idea del esfuerzo que se está empleando en buscar > alternativas que mejoren la productividad. > > Como no quisiera volver a aprender otro lenguaje más, me preguntaba qué > tal son los sistemas que compilan desde python, como pyjs (de pyjamas) o > pyjaco. Me interesaría comentarios sobre: > > - - qué limitaciones en la sintáxis python imponen estas herramientas > - - calidad del código javascript generado (velocidad de compilación y > ejecución) > - - posibilidad de que el compilado se haga en cliente (como permite > coffeescript) > - - otros compiladores a javascript recomendables (desde python o desde > otros lenguajes) > > > Muchas Gracias, > Chema Cortés > > - --- > Hyperreals *R: http://ch3m4.org/blog > Quarks, bits y otras criaturas infinitesimales > -----BEGIN PGP SIGNATURE----- > Version: GnuPG v1.4.11 (GNU/Linux) > Comment: Using GnuPG with Mozilla - http://www.enigmail.net/ > > iQEcBAEBAgAGBQJQe3MHAAoJEFdWyBWwhL4FQI0H/iQhhZi++L9LyZNkaj/OVcep > d9ent8o59fRBXx/i0DjA77BEdkrOw9vwt0dfP6FObzjCYkUwdt6o4jXj42tHVx72 > S4jFMeyC33szRkFUeKWvWwovNwWIuqKnRATyXBofzA8NBZt56LbMksGNM5ijtmrk > iMA6psGI5zHyvLxTQhXeJTnGvYEVITqEW5VsNDR8S62KiRhhLO0S4TO8moePH7ok > dobTgl51eV/fdhhJ+8WCPdWK4DTT5TfxSdWcO+C5r7UzY0NMh/DEdRN/SNcDQU82 > SW0Me25nH9OWhYY19EX0ZRLkRaMFJOaKB1pIGXIY9hnES+D+xYdOqGhrBdYc/m4= > =7LrO > -----END PGP SIGNATURE----- > > _______________________________________________ > Python-es mailing list > Python-es en python.org > http://mail.python.org/mailman/listinfo/python-es > FAQ: http://python-es-faq.wikidot.com/ > -- *Nota: Por problemas con el e-mail. Puede que recibas de nuevo este mensaje en un tiempo.* ------------ próxima parte ------------ Se ha borrado un adjunto en formato HTML... URL: From maengora en gmail.com Thu Oct 18 18:59:52 2012 From: maengora en gmail.com (=?ISO-8859-1?Q?Manuel_Enrique_Gonz=E1lez_Ram=EDrez?=) Date: Thu, 18 Oct 2012 11:59:52 -0500 Subject: [Python-es] Compilar a javascript (pyjs/pyjaco) In-Reply-To: References: <507B731F.1000400@ch3m4.org> Message-ID: UUUy super interesante. Quizás esto es lo que andaba buscando. Leeré y seguramente aprenderé sino... preguntaré Gracias por información. El 18 de octubre de 2012 10:57, Juan M. Puertas escribió: > Hola Chema. > En ello estoy también. En mi primera prueba con pyjamas en Ubuntu 12.04, > sólo ha funcionado la parte cliente, no la del servidor, hay que tener en > cuenta que aún se encuentra en versión alfa. > Por lo que he podido leer, Luke; el autor de pyjamas, deja su desarrollo > debido a los "secuestradores" de pyjs (?), > https://groups.google.com/forum/#!topic/gwt-coffee/pEnCfUDTqTQ y > Christian, de pyjaco está continuando la labor. > Lo que más me gusta del concepto de pyjamas es que puedes desarrollar en > python para el escritorio y para la web sin cambiar el código, lo cual, > según mi visión, resuelve el "eterno" dilema sobre qué GUI utilizar, ya que > pyjamas está/estaba (?) desarrollando unas bibliotecas gráficas que podrían > acordarse como estándares. > Aquí Christian muestra un ejemplo con pyjaco y jquery: > http://archlinux.me/dusty/2011/12/27/pyjaco-and-jquery/ > Ya lo tengo claro; ni TK, ni WX, ni GTK, ni QT, sino *HTML, *así puedes > estar "casi" seguro de que no vas a tener que construir los formularios de > tu aplicación por enésima vez... ;-))* *. Desde ahora, gracias a > Websocket de HTML5, puedes programar con el lenguaje que más te guste y > comunicarlo con el formulario en HTML5 de una manera facilísima. > > Saludos afectuosos desde Granada. :-) > Juanma.- > > > El 15 de octubre de 2012 04:21, Chema Cortes escribió: > >> >> -----BEGIN PGP SIGNED MESSAGE----- >> Hash: SHA1 >> >> Parece que se están poniendo de moda los lenguajes que compilan a >> javascript. Por un lado, el CoffeeScript de microsoft, por otro el Dart >> de Google. No parece que vayan a desbancar a javascript de los >> navegadores, pero da idea del esfuerzo que se está empleando en buscar >> alternativas que mejoren la productividad. >> >> Como no quisiera volver a aprender otro lenguaje más, me preguntaba qué >> tal son los sistemas que compilan desde python, como pyjs (de pyjamas) o >> pyjaco. Me interesaría comentarios sobre: >> >> - - qué limitaciones en la sintáxis python imponen estas herramientas >> - - calidad del código javascript generado (velocidad de compilación y >> ejecución) >> - - posibilidad de que el compilado se haga en cliente (como permite >> coffeescript) >> - - otros compiladores a javascript recomendables (desde python o desde >> otros lenguajes) >> >> >> Muchas Gracias, >> Chema Cortés >> >> - --- >> Hyperreals *R: http://ch3m4.org/blog >> Quarks, bits y otras criaturas infinitesimales >> -----BEGIN PGP SIGNATURE----- >> Version: GnuPG v1.4.11 (GNU/Linux) >> Comment: Using GnuPG with Mozilla - http://www.enigmail.net/ >> >> iQEcBAEBAgAGBQJQe3MHAAoJEFdWyBWwhL4FQI0H/iQhhZi++L9LyZNkaj/OVcep >> d9ent8o59fRBXx/i0DjA77BEdkrOw9vwt0dfP6FObzjCYkUwdt6o4jXj42tHVx72 >> S4jFMeyC33szRkFUeKWvWwovNwWIuqKnRATyXBofzA8NBZt56LbMksGNM5ijtmrk >> iMA6psGI5zHyvLxTQhXeJTnGvYEVITqEW5VsNDR8S62KiRhhLO0S4TO8moePH7ok >> dobTgl51eV/fdhhJ+8WCPdWK4DTT5TfxSdWcO+C5r7UzY0NMh/DEdRN/SNcDQU82 >> SW0Me25nH9OWhYY19EX0ZRLkRaMFJOaKB1pIGXIY9hnES+D+xYdOqGhrBdYc/m4= >> =7LrO >> -----END PGP SIGNATURE----- >> >> _______________________________________________ >> Python-es mailing list >> Python-es en python.org >> http://mail.python.org/mailman/listinfo/python-es >> FAQ: http://python-es-faq.wikidot.com/ >> > > > > -- > *Nota: Por problemas con el e-mail. Puede que recibas de nuevo este > mensaje en un tiempo.* > > _______________________________________________ > Python-es mailing list > Python-es en python.org > http://mail.python.org/mailman/listinfo/python-es > FAQ: http://python-es-faq.wikidot.com/ > > -- Manuel Enrique González Ramírez http://maengora.blogspot.com ------------ próxima parte ------------ Se ha borrado un adjunto en formato HTML... URL: From kikocorreoso en gmail.com Thu Oct 18 19:43:16 2012 From: kikocorreoso en gmail.com (Kiko) Date: Thu, 18 Oct 2012 19:43:16 +0200 Subject: [Python-es] Compilar a javascript (pyjs/pyjaco) In-Reply-To: References: <507B731F.1000400@ch3m4.org> Message-ID: > >> El 15 de octubre de 2012 04:21, Chema Cortes escribió: >> >>> >>> -----BEGIN PGP SIGNED MESSAGE----- >>> Hash: SHA1 >>> >>> Parece que se están poniendo de moda los lenguajes que compilan a >>> javascript. Por un lado, el CoffeeScript de microsoft, por otro el Dart >>> de Google. No parece que vayan a desbancar a javascript de los >>> navegadores, pero da idea del esfuerzo que se está empleando en buscar >>> alternativas que mejoren la productividad. >>> >>> Como no quisiera volver a aprender otro lenguaje más, me preguntaba qué >>> tal son los sistemas que compilan desde python, como pyjs (de pyjamas) o >>> pyjaco. Me interesaría comentarios sobre: >>> >>> - - qué limitaciones en la sintáxis python imponen estas herramientas >>> - - calidad del código javascript generado (velocidad de compilación y >>> ejecución) >>> - - posibilidad de que el compilado se haga en cliente (como permite >>> coffeescript) >>> - - otros compiladores a javascript recomendables (desde python o desde >>> otros lenguajes) >>> >>> >>> Muchas Gracias, >>> Chema Cortés >>> >>> - --- >>> >> Una pregunta desde el total desconocimiento y la más completa ignorancia,... ¿Por qué dart, typescript, coffeescript, con sintaxis parecida a python o ruby? Con empresas tan grandes detrás, ¿no sería más fácil 'reescribir' un javascript más bonito o usar un lenguaje más mainstream (python, por ejemplo :-P) para crear un compilador a javascript en lugar de hacer la guerra cada uno por su cuenta? Como sé que no soy el más listo de la clase, supongo que habrá imposibilidades técnicas, ¿alguien las puede explicar para un eterno profano como yo? Gracias. P.D.: Por otro lado, también existe esto: http://www.skulpt.org/ (python 'metido' en js). ------------ próxima parte ------------ Se ha borrado un adjunto en formato HTML... URL: From jjgomera en gmail.com Thu Oct 18 20:26:37 2012 From: jjgomera en gmail.com (=?ISO-8859-1?Q?Juan_Jos=E9_G=F3mez_Romera?=) Date: Thu, 18 Oct 2012 20:26:37 +0200 Subject: [Python-es] 3d globe In-Reply-To: References: Message-ID: y con matplotlib, aunque no entiendo exactamente lo que quieres hacer: http://www.scipy.org/Cookbook/Matplotlib/Maps http://matplotlib.org/basemap/users/examples.html Podrías integrar la imagen de matplotlib en pyqt y hacerlo todo usando python El 18 de octubre de 2012 16:28, Da?id escribió: > Creo que con Visual Python [1] y un poco de curro se puede hacer. Una > de las texturas que vienen incluídas es una imagen satélite de la > Tierra, y en el código fuente lo único que han hecho ha sido añadido > la imagen plana, en una cierta proyección. > > > [1] http://www.vpython.org/ > > 2012/10/18 Tony Peña : > > buenas, > > > > como o conque se puede hacer una presentacion de la tierra, y poner una > > posicion por ejemplo 0,0 un imagen pequeña cualquiera, > > he probado usando openstreetmap, y googlemaps, pero asi solo alcanzo a > verlo > > en un plano, y lo mas bonito quedaria viendolo como en googleearth.. > > que se pueda dar vueltas y esas cosas y se vea ese objeto en esa > posicion. > > > > tambien probe marble pero funciona mas para la gente que sabe C/C++, los > > bindings para python no estan bien definidos aun y dan mucho problemas > > porque lo quiero usar con PyQt4 > > y marble es de la gente de PyKDE. > > > > algo como python y opengl o cualquier otra via que uds conozcan que hayan > > utilizado, no se algun que otra persona que sea deutilidad en su carrera > de > > geografo o cosas asi que le guste python puede que le haya pensado hacer > > cosas parecida usando esto,.... pero me gustaria poder tener un ejemplo > > donde se vea claro, lo mas simple posible, > > alguien tiene por ahi guardado algo como esto?. > > > > gracias. > > > > -- > > Antonio Peña > > Secure email with PGP 0x8B021001 available at http://pgp.mit.edu > > Fingerprint: 74E6 2974 B090 366D CE71 7BB2 6476 FA09 8B02 1001 > > > > _______________________________________________ > > Python-es mailing list > > Python-es en python.org > > http://mail.python.org/mailman/listinfo/python-es > > FAQ: http://python-es-faq.wikidot.com/ > > > _______________________________________________ > Python-es mailing list > Python-es en python.org > http://mail.python.org/mailman/listinfo/python-es > FAQ: http://python-es-faq.wikidot.com/ > ------------ próxima parte ------------ Se ha borrado un adjunto en formato HTML... URL: From emperor.cu en gmail.com Thu Oct 18 20:29:23 2012 From: emperor.cu en gmail.com (=?UTF-8?Q?Tony_Pe=C3=B1a?=) Date: Thu, 18 Oct 2012 14:29:23 -0400 Subject: [Python-es] 3d globe In-Reply-To: References: Message-ID: lo que quiero hacer es simple.. en los screenshots de mi hobbiy se ve perfectamente el googlemaps para mostrar lo que quiero... lo que no pasa se veria al menos a mi gusto, mas curioso hacerlo dentro del QTabWidget donde esta el mapa sea en 3d el globo que la imagen plana. El 18 de octubre de 2012 14:26, Juan José Gómez Romera escribió: > y con matplotlib, aunque no entiendo exactamente lo que quieres hacer: > > http://www.scipy.org/Cookbook/Matplotlib/Maps > http://matplotlib.org/basemap/users/examples.html > > Podrías integrar la imagen de matplotlib en pyqt y hacerlo todo usando > python > > El 18 de octubre de 2012 16:28, Da?id escribió: > > Creo que con Visual Python [1] y un poco de curro se puede hacer. Una >> de las texturas que vienen incluídas es una imagen satélite de la >> Tierra, y en el código fuente lo único que han hecho ha sido añadido >> la imagen plana, en una cierta proyección. >> >> >> [1] http://www.vpython.org/ >> >> 2012/10/18 Tony Peña : >> > buenas, >> > >> > como o conque se puede hacer una presentacion de la tierra, y poner una >> > posicion por ejemplo 0,0 un imagen pequeña cualquiera, >> > he probado usando openstreetmap, y googlemaps, pero asi solo alcanzo a >> verlo >> > en un plano, y lo mas bonito quedaria viendolo como en googleearth.. >> > que se pueda dar vueltas y esas cosas y se vea ese objeto en esa >> posicion. >> > >> > tambien probe marble pero funciona mas para la gente que sabe C/C++, los >> > bindings para python no estan bien definidos aun y dan mucho problemas >> > porque lo quiero usar con PyQt4 >> > y marble es de la gente de PyKDE. >> > >> > algo como python y opengl o cualquier otra via que uds conozcan que >> hayan >> > utilizado, no se algun que otra persona que sea deutilidad en su >> carrera de >> > geografo o cosas asi que le guste python puede que le haya pensado hacer >> > cosas parecida usando esto,.... pero me gustaria poder tener un ejemplo >> > donde se vea claro, lo mas simple posible, >> > alguien tiene por ahi guardado algo como esto?. >> > >> > gracias. >> > >> > -- >> > Antonio Peña >> > Secure email with PGP 0x8B021001 available at http://pgp.mit.edu >> > Fingerprint: 74E6 2974 B090 366D CE71 7BB2 6476 FA09 8B02 1001 >> > >> > _______________________________________________ >> > Python-es mailing list >> > Python-es en python.org >> > http://mail.python.org/mailman/listinfo/python-es >> > FAQ: http://python-es-faq.wikidot.com/ >> > >> _______________________________________________ >> Python-es mailing list >> Python-es en python.org >> http://mail.python.org/mailman/listinfo/python-es >> FAQ: http://python-es-faq.wikidot.com/ >> > > > _______________________________________________ > Python-es mailing list > Python-es en python.org > http://mail.python.org/mailman/listinfo/python-es > FAQ: http://python-es-faq.wikidot.com/ > > -- Antonio Peña Secure email with PGP 0x8B021001 available at http://pgp.mit.edu Fingerprint: 74E6 2974 B090 366D CE71 7BB2 6476 FA09 8B02 1001 ------------ próxima parte ------------ Se ha borrado un adjunto en formato HTML... URL: From gmourinopardo en yahoo.es Fri Oct 19 10:29:00 2012 From: gmourinopardo en yahoo.es (=?iso-8859-1?Q?Gonzalo_Mouri=FFfffffffffff1o_Pardo?=) Date: Fri, 19 Oct 2012 09:29:00 +0100 (BST) Subject: [Python-es] GribBagSizer No Responde In-Reply-To: References: Message-ID: <1350635340.1300.YahooMailNeo@web171702.mail.ir2.yahoo.com> Buenos dias, lo que intento hacer es poder regenerar los calendarios, para que pueda elegir un mes, pero antes de elegir que no me muestre nada, de hay que llame al InitUI desde el evento de menu. De momento he conseguido se me actualize el mes y los dias pero no puedo regenerarlo pues si empezamos en el mes de enero al hacer febrero, me sigue poniendo la ultima fila de enero.   #!/usr/bin/env python # -*- coding: cp1252 -*- import os import wx import new_user import about import data # News Identifiers ID_NUEVO = wx.NewId() ID_ABRIR = wx.NewId() ID_CERRAR = wx.NewId() ID_GUARDAR = wx.NewId() ID_GUARDAR_COMO = wx.NewId() ID_IMPRIMIR = wx.NewId() ID_SALIR = wx.NewId() ID_DESHACER = wx.NewId() ID_REHACER = wx.NewId() ID_CORTAR = wx.NewId() ID_COPIAR = wx.NewId() ID_PEGAR = wx.NewId() ID_MESES = wx.NewId() ID_ENERO = wx.NewId() ID_FEBRERO = wx.NewId() ID_AYUDA = wx.NewId() ID_ACERCA_DE = wx.NewId() class Aplicacion(wx.App):     def OnInit(self):         self.frame = Ventana(None, -1, title = "Control Horario")         self.SetTopWindow(self.frame)         self.frame.Show()         return True class Pagina(wx.Panel):     def __init__(self, parent):         wx.Panel.__init__(self, parent, size = wx.Size(895, 275), style =                           wx.CLIP_CHILDREN)         self.carga = data.TimeData()         self.month = self.carga.month     def InitUI(self, month):         font = wx.Font(15, family = wx.FONTFAMILY_DEFAULT, style =                         wx.FONTSTYLE_NORMAL, weight = wx.FONTWEIGHT_BOLD,                         encoding = wx.FONTENCODING_CP1252)         self.month = month         # Layout         self.sizer = wx.BoxSizer(wx.HORIZONTAL)         self.gbsizer = wx.GridBagSizer(0, 0)         self.sizer.Add(self.gbsizer, 1, wx.ALL, border = 5)         self.SetSizer(self.sizer)         # Column's Labels         sem = {0 : ("Lunes", 169), 1 : ("Martes", 266), 2 : ("Miercoles" , 359),                3 : ("Jueves",  478), 4 : ("Viernes", 578), 5: ("Sabado", 686),                6 : ("Domingo", 783)}         for num in range(0,7):             self.cont = sem[num]             dia = self.cont[0]             y = num + 1             self.collab = wx.StaticText(self, -1, label = dia,  size = wx.Size                                         (105, 25), style = wx.ALIGN_CENTER |                                         wx.SIMPLE_BORDER)             self.collab.SetFont(font)             if y < 6:                 self.collab.SetBackgroundColour(wx.Colour(255, 255, 255))             else:                 self.collab.SetBackgroundColour(wx.Colour(255, 0, 0))             self.gbsizer.Add(self.collab, pos = wx.GBPosition(0, y), flag =                         wx.ALIGN_CENTER_VERTICAL, border = 5)         # Month's Label         self.mes = self.carga.month_dict[self.month]         self.monlab = wx.StaticText(self, -1, label = self.mes, size = wx.Size(140,                                                                           25),                                     style = wx.ALIGN_CENTER)         self.monlab.SetFont(font)         self.monlab.SetForegroundColour(wx.Colour(255, 0, 255))         self.gbsizer.Add(self.monlab, pos = wx.GBPosition(0, 0), flag =                     wx.ALIGN_CENTER_VERTICAL,  border = 5)         # Row's Labels         self.month_name = self.carga.month_tup[12 - self.month]         num_month = self.carga.fweek_dict[self.month_name]         numero = str(num_month)         week_range = self.carga.nweek_dict[self.month_name]         for num in range(0, week_range):             x = num + 1             self.rowlab = wx.StaticText(self, -1, label = numero, size = wx.Size                                         (140, 25), style = wx.ALIGN_CENTER |                                         wx.SIMPLE_BORDER)             self.rowlab.SetFont(font)             self.rowlab.SetForegroundColour(wx.Colour(0, 0, 255))             self.rowlab.SetBackgroundColour(wx.Colour(255, 255, 255))             self.rowlab.Wrap(-1)             self.gbsizer.Add(self.rowlab, pos = wx.GBPosition(x, 0), flag =                         wx.ALIGN_CENTER_VERTICAL, border = 5)             number = int(numero) + 1             numero = str(number)         # Day's Labels         day = self.carga.week_dict[self.month_name]         end_dict = self.carga.nday_dict[self.month_name]         self.daylab_dict = {}         if day == "Mon":             y = 1         elif day == "Tue":             y = 2         elif day == "Wed":             y = 3         elif day == "Thu":             y = 4         elif day == "Fri":             y = 5         elif day == "Sat":             y = 6         else:             y = 7         frest = y - 1         x = 1         for cont in range (0, end_dict):             contador = str(cont+1)             if y > 7:                 x = x + 1                 y = 1             self.daylab = wx.StaticText(self, label = contador, size = wx.Size                                         (105, 25), style = wx.ALIGN_CENTRE |                                         wx.SIMPLE_BORDER)             self.daylab.SetFont(font)             self.daylab.SetBackgroundColour(wx.Colour(255, 255, 255))             if y > 5:                 self.daylab.SetBackgroundColour(wx.Colour(255, 0, 0))             self.daylab_dict [(x, y)] = contador             self.gbsizer.Add(self.daylab, pos = wx.GBPosition(x, y), flag = wx.ALIGN_CENTER_VERTICAL,                         border = 5)             self.daylab.Bind(wx.EVT_LEFT_DOWN, self.OnLeftDown)             y = y + 1         # Blank's Labels         for cont in range (0, frest):             self.blalab = wx.StaticText(self, label = "", size = wx.Size(105, 25                                                                          ),                                         style = wx.ALIGN_CENTER |                                         wx.SIMPLE_BORDER)             self.blalab.SetFont(font)             self.blalab.SetBackgroundColour(wx.Colour(255, 255, 255))             if cont > 4:                 self.blalab.SetBackgroundColour(wx.Colour(255, 0, 0))             self.gbsizer.Add(self.blalab, pos = wx.GBPosition(1, (cont + 1)), flag = wx.ALIGN_CENTER_VERTICAL,                        border = 5)         erest = (week_range * 7) - end_dict - frest         print("week_range: " + str(week_range))         print("end_dict: " + str(end_dict))         print("frest: " + str(frest))         print ("erest: " + str(erest))         for cont in range (0, erest):             self.blalab = wx.StaticText(self, label = "", size = wx.Size(105, 25                                                                          ),                                         style = wx.ALIGN_CENTER |                                         wx.SIMPLE_BORDER)             self.blalab.SetFont(font)             self.blalab.SetBackgroundColour(wx.Colour(255, 255, 255))             if y > 5:                 self.blalab.SetBackgroundColour(wx.Colour(255, 0, 0))             self.gbsizer.Add(self.blalab, pos = wx.GBPosition(x, y), flag = wx.ALIGN_CENTER_VERTICAL,                        border = 5)             y = y + 1     def OnLeftDown(self, event):         mou_pos = wx.GetMousePosition()         scr_pos = self.GetScreenPosition()         abs_x = mou_pos.x - scr_pos.x         abs_y = mou_pos.y - scr_pos.y         if abs_x < 250:             coor_x = 1         elif abs_x >= 250 and abs_x < 355:             coor_x = 2         elif abs_x >= 355 and abs_x < 460:             coor_x = 3         elif abs_x >= 460 and abs_x < 565:             coor_x = 4         elif abs_x >= 565 and abs_x < 670:             coor_x = 5         elif abs_x >= 670 and abs_x < 775:             coor_x = 6         else:             coor_x = 7         if abs_y < 55:             coor_y = 1         elif abs_y >= 55 and abs_y < 80:             coor_y = 2         elif abs_y >= 80 and abs_y < 105:             coor_y = 3         elif abs_y >= 105 and abs_y < 130:             coor_y = 4         else:             coor_y = 5         data_day = self.daylab_dict[(coor_y, coor_x)]         event.Skip() class Ventana(wx.Frame):         """Constructor"""     def __init__(self, parent, id, title):         super(Ventana, self).__init__(parent, id, title, size = wx.Size(895, 275                                                                         ))         # Attributes         self.panel = Pagina(self)         # Setup Menu         menubar = wx.MenuBar()         # File's Menu         mesm = wx.Menu()         emmi = wx.MenuItem(mesm, ID_ENERO, "&Enero")         mesm.AppendItem(emmi)         fmmi = wx.MenuItem(mesm, ID_FEBRERO, "&Febrero")         mesm.AppendItem(fmmi)                menubar.Append(mesm, '&Meses')         self.SetMenuBar(menubar)                 # Event Handlers         self.Bind(wx.EVT_MENU, self.OnMenu)     """Do menu events"""     def OnMenu(self, event):         self.evt_id = event.GetId ()         if self.evt_id == ID_ENERO:             self.month = 1         if self.evt_id == ID_FEBRERO:             self.month = 2         self.DataMonth()         event.Skip()     def DataMonth(self):         self.panel.Fit()         gui = self.panel.InitUI(month = self.month)         self.panel.Layout()         if __name__ == "__main__":     app = Aplicacion(False)     app.MainLoop() ------------ próxima parte ------------ Se ha borrado un adjunto en formato HTML... URL: From pych3m4 en gmail.com Fri Oct 19 13:54:22 2012 From: pych3m4 en gmail.com (Chema Cortes) Date: Fri, 19 Oct 2012 13:54:22 +0200 Subject: [Python-es] Compilar a javascript (pyjs/pyjaco) In-Reply-To: References: <507B731F.1000400@ch3m4.org> Message-ID: El día 18 de octubre de 2012 17:57, Juan M. Puertas escribió: > En ello estoy también. En mi primera prueba con pyjamas en Ubuntu 12.04, > sólo ha funcionado la parte cliente, no la del servidor, hay que tener en > cuenta que aún se encuentra en versión alfa. > Por lo que he podido leer, Luke; el autor de pyjamas, deja su desarrollo > debido a los "secuestradores" de pyjs (?), > https://groups.google.com/forum/#!topic/gwt-coffee/pEnCfUDTqTQ y Christian, > de pyjaco está continuando la labor. > Lo que más me gusta del concepto de pyjamas es que puedes desarrollar en > python para el escritorio y para la web sin cambiar el código, lo cual, > según mi visión, resuelve el "eterno" dilema sobre qué GUI utilizar, ya que > pyjamas está/estaba (?) desarrollando unas bibliotecas gráficas que podrían > acordarse como estándares. > Aquí Christian muestra un ejemplo con pyjaco y jquery: > http://archlinux.me/dusty/2011/12/27/pyjaco-and-jquery/ > Ya lo tengo claro; ni TK, ni WX, ni GTK, ni QT, sino HTML, así puedes estar > "casi" seguro de que no vas a tener que construir los formularios de tu > aplicación por enésima vez... ;-)) . Desde ahora, gracias a Websocket de > HTML5, puedes programar con el lenguaje que más te guste y comunicarlo con > el formulario en HTML5 de una manera facilísima. Muchas gracias, Juanma, ha sido bastante esclarecedor. Desarrollar interfaces gráficos en HTML5 supongo que será la siguiente oleada de modernidades que vamos a tener con las que justificar las CPUs multinúcleo de los modernos dispositivos móviles (que serán los nuevo "servidores" de internet en lugar de alquilar servicios de hosting). -- Hyperreals *R: http://ch3m4.org/blog Quarks, bits y otras criaturas infinitesimales From jza en oooes.org Fri Oct 19 17:50:56 2012 From: jza en oooes.org (Alexandro Colorado) Date: Fri, 19 Oct 2012 10:50:56 -0500 Subject: [Python-es] Compilar a javascript (pyjs/pyjaco) In-Reply-To: <507B731F.1000400@ch3m4.org> References: <507B731F.1000400@ch3m4.org> Message-ID: Pyjamas es un framework muy viejo y robusto, tiene implementaciones para una infinidad de frameworks incluyendo Google Web Toolkit, RaphaelJS, etc. Te recomiendo que lo descarges, lo uses y hagas tutoriales, y tu mismo compruebes esto, hay mucho codigo de ejemplo para observar. Si aun no te convence algunos aspectos, vuelve a preguntar. On 10/14/12, Chema Cortes wrote: > > -----BEGIN PGP SIGNED MESSAGE----- > Hash: SHA1 > > Parece que se están poniendo de moda los lenguajes que compilan a > javascript. Por un lado, el CoffeeScript de microsoft, por otro el Dart > de Google. No parece que vayan a desbancar a javascript de los > navegadores, pero da idea del esfuerzo que se está empleando en buscar > alternativas que mejoren la productividad. > > Como no quisiera volver a aprender otro lenguaje más, me preguntaba qué > tal son los sistemas que compilan desde python, como pyjs (de pyjamas) o > pyjaco. Me interesaría comentarios sobre: > > - - qué limitaciones en la sintáxis python imponen estas herramientas > - - calidad del código javascript generado (velocidad de compilación y > ejecución) > - - posibilidad de que el compilado se haga en cliente (como permite > coffeescript) > - - otros compiladores a javascript recomendables (desde python o desde > otros lenguajes) > > > Muchas Gracias, > Chema Cortés > > - --- > Hyperreals *R: http://ch3m4.org/blog > Quarks, bits y otras criaturas infinitesimales > -----BEGIN PGP SIGNATURE----- > Version: GnuPG v1.4.11 (GNU/Linux) > Comment: Using GnuPG with Mozilla - http://www.enigmail.net/ > > iQEcBAEBAgAGBQJQe3MHAAoJEFdWyBWwhL4FQI0H/iQhhZi++L9LyZNkaj/OVcep > d9ent8o59fRBXx/i0DjA77BEdkrOw9vwt0dfP6FObzjCYkUwdt6o4jXj42tHVx72 > S4jFMeyC33szRkFUeKWvWwovNwWIuqKnRATyXBofzA8NBZt56LbMksGNM5ijtmrk > iMA6psGI5zHyvLxTQhXeJTnGvYEVITqEW5VsNDR8S62KiRhhLO0S4TO8moePH7ok > dobTgl51eV/fdhhJ+8WCPdWK4DTT5TfxSdWcO+C5r7UzY0NMh/DEdRN/SNcDQU82 > SW0Me25nH9OWhYY19EX0ZRLkRaMFJOaKB1pIGXIY9hnES+D+xYdOqGhrBdYc/m4= > =7LrO > -----END PGP SIGNATURE----- > > _______________________________________________ > Python-es mailing list > Python-es en python.org > http://mail.python.org/mailman/listinfo/python-es > FAQ: http://python-es-faq.wikidot.com/ > -- Alexandro Colorado PPMC Apache OpenOffice http://es.openoffice.org From yohangra en gmail.com Fri Oct 19 18:09:32 2012 From: yohangra en gmail.com (Yohan Graterol) Date: Fri, 19 Oct 2012 11:39:32 -0430 Subject: [Python-es] Compilar a javascript (pyjs/pyjaco) In-Reply-To: References: <507B731F.1000400@ch3m4.org> Message-ID: He seguido de cerca este correo, pero mi pregunta donde esta disponible pyjamas? porque al ingresar a su sitio aparece un error 404 en github, entonces me interesa mucho ese tema. El 19 de octubre de 2012 11:20, Alexandro Colorado escribió: > Pyjamas es un framework muy viejo y robusto, tiene implementaciones > para una infinidad de frameworks incluyendo Google Web Toolkit, > RaphaelJS, etc. Te recomiendo que lo descarges, lo uses y hagas > tutoriales, y tu mismo compruebes esto, hay mucho codigo de ejemplo > para observar. Si aun no te convence algunos aspectos, vuelve a > preguntar. > > On 10/14/12, Chema Cortes wrote: > > > > -----BEGIN PGP SIGNED MESSAGE----- > > Hash: SHA1 > > > > Parece que se están poniendo de moda los lenguajes que compilan a > > javascript. Por un lado, el CoffeeScript de microsoft, por otro el Dart > > de Google. No parece que vayan a desbancar a javascript de los > > navegadores, pero da idea del esfuerzo que se está empleando en buscar > > alternativas que mejoren la productividad. > > > > Como no quisiera volver a aprender otro lenguaje más, me preguntaba qué > > tal son los sistemas que compilan desde python, como pyjs (de pyjamas) o > > pyjaco. Me interesaría comentarios sobre: > > > > - - qué limitaciones en la sintáxis python imponen estas herramientas > > - - calidad del código javascript generado (velocidad de compilación y > > ejecución) > > - - posibilidad de que el compilado se haga en cliente (como permite > > coffeescript) > > - - otros compiladores a javascript recomendables (desde python o desde > > otros lenguajes) > > > > > > Muchas Gracias, > > Chema Cortés > > > > - --- > > Hyperreals *R: http://ch3m4.org/blog > > Quarks, bits y otras criaturas infinitesimales > > -----BEGIN PGP SIGNATURE----- > > Version: GnuPG v1.4.11 (GNU/Linux) > > Comment: Using GnuPG with Mozilla - http://www.enigmail.net/ > > > > iQEcBAEBAgAGBQJQe3MHAAoJEFdWyBWwhL4FQI0H/iQhhZi++L9LyZNkaj/OVcep > > d9ent8o59fRBXx/i0DjA77BEdkrOw9vwt0dfP6FObzjCYkUwdt6o4jXj42tHVx72 > > S4jFMeyC33szRkFUeKWvWwovNwWIuqKnRATyXBofzA8NBZt56LbMksGNM5ijtmrk > > iMA6psGI5zHyvLxTQhXeJTnGvYEVITqEW5VsNDR8S62KiRhhLO0S4TO8moePH7ok > > dobTgl51eV/fdhhJ+8WCPdWK4DTT5TfxSdWcO+C5r7UzY0NMh/DEdRN/SNcDQU82 > > SW0Me25nH9OWhYY19EX0ZRLkRaMFJOaKB1pIGXIY9hnES+D+xYdOqGhrBdYc/m4= > > =7LrO > > -----END PGP SIGNATURE----- > > > > _______________________________________________ > > Python-es mailing list > > Python-es en python.org > > http://mail.python.org/mailman/listinfo/python-es > > FAQ: http://python-es-faq.wikidot.com/ > > > > > -- > Alexandro Colorado > PPMC Apache OpenOffice > http://es.openoffice.org > _______________________________________________ > Python-es mailing list > Python-es en python.org > http://mail.python.org/mailman/listinfo/python-es > FAQ: http://python-es-faq.wikidot.com/ > ------------ próxima parte ------------ Se ha borrado un adjunto en formato HTML... URL: From jza en oooes.org Fri Oct 19 18:19:30 2012 From: jza en oooes.org (Alexandro Colorado) Date: Fri, 19 Oct 2012 11:19:30 -0500 Subject: [Python-es] Compilar a javascript (pyjs/pyjaco) In-Reply-To: References: <507B731F.1000400@ch3m4.org> Message-ID: 2012/10/19 Yohan Graterol > He seguido de cerca este correo, pero mi pregunta donde esta disponible > pyjamas? porque al ingresar a su sitio aparece un error 404 en github, > entonces me interesa mucho ese tema. > Pues usa el siguiente resultado que te avienta el senior google https://github.com/pyjs/ > > El 19 de octubre de 2012 11:20, Alexandro Colorado escribió: > > Pyjamas es un framework muy viejo y robusto, tiene implementaciones >> para una infinidad de frameworks incluyendo Google Web Toolkit, >> RaphaelJS, etc. Te recomiendo que lo descarges, lo uses y hagas >> tutoriales, y tu mismo compruebes esto, hay mucho codigo de ejemplo >> para observar. Si aun no te convence algunos aspectos, vuelve a >> preguntar. >> >> On 10/14/12, Chema Cortes wrote: >> > >> > >> > >> > _______________________________________________ >> > Python-es mailing list >> > Python-es en python.org >> > http://mail.python.org/mailman/listinfo/python-es >> > FAQ: http://python-es-faq.wikidot.com/ >> > >> >> >> -- >> Alexandro Colorado >> PPMC Apache OpenOffice >> http://es.openoffice.org >> _______________________________________________ >> Python-es mailing list >> Python-es en python.org >> http://mail.python.org/mailman/listinfo/python-es >> FAQ: http://python-es-faq.wikidot.com/ >> > > > _______________________________________________ > Python-es mailing list > Python-es en python.org > http://mail.python.org/mailman/listinfo/python-es > FAQ: http://python-es-faq.wikidot.com/ > > -- Alexandro Colorado PPMC Apache OpenOffice http://es.openoffice.org ------------ próxima parte ------------ Se ha borrado un adjunto en formato HTML... URL: From pych3m4 en gmail.com Fri Oct 19 20:08:11 2012 From: pych3m4 en gmail.com (Chema Cortes) Date: Fri, 19 Oct 2012 20:08:11 +0200 Subject: [Python-es] Compilar a javascript (pyjs/pyjaco) In-Reply-To: References: <507B731F.1000400@ch3m4.org> Message-ID: El día 18 de octubre de 2012 19:43, Kiko escribió: > Una pregunta desde el total desconocimiento y la más completa ignorancia,... > ¿Por qué dart, typescript, coffeescript, con sintaxis parecida a python o > ruby? Con empresas tan grandes detrás, ¿no sería más fácil 'reescribir' un > javascript más bonito o usar un lenguaje más mainstream (python, por ejemplo > :-P) para crear un compilador a javascript en lugar de hacer la guerra cada > uno por su cuenta? Hasta ahora la lógica de la aplicación iba en el servidor al que se conectaba el cliente javascript, que no era más que un interface que enviaba y traía cosas. Con el tiempo, la parte cliente empieza a tener más peso en la aplicación web, controlando la presentación e integración de datos. Incluso ya hay aplicaciones "offline" completas que solo usan internet si está disponible (eg: gmail offline) Con todo, el javascript es horrible para crear grandes ficheros de código. Con estos lenguajes específicos (dart, typescript, coffeescript) se busca mejorar la productividad aproximando la generación de código javascript al lenguaje usado en la programación de páginas webs. Lo de crear javascript a partir de python era precisamente mi pregunta desde el principio :D > P.D.: Por otro lado, también existe esto: http://www.skulpt.org/ (python > 'metido' en js). Lo interesante de usar javascript es poder manipular el DOM del documento HTML y controlar eventos. Ejecutar python en javascript por sí sólo sirve para hacerlo todo más pesado. También hay que tener en cuenta que todos estos intérpretes hechos en javascript limitan el código python que se puede usar y que no suelen permitir importar cualquier módulo python. Voy a añadir una lista de compiladores de python a javascript: pyjs: http://pyjs.org https://github.com/pyjs/ pyjaco: http://pyjaco.org pycow (parece abandonado): https://github.com/p2k/PyCow También he encontrado un par de lenguaje similares a python que compilan a javascript: pyvascript: http://www.allbuttonspressed.com/projects/pyvascript rapyscriprt (aka pyvascript++): https://bitbucket.org/pyjeon/rapydscript Creo que entre todos éstos voy a darle una oportunidad a rapyscript. Espero contar lo que descubra por mi blog. -- Hyperreals *R: http://ch3m4.org/blog Quarks, bits y otras criaturas infinitesimales From leonardocaballero en gmail.com Tue Oct 23 03:30:33 2012 From: leonardocaballero en gmail.com (Leonardo Caballero) Date: Mon, 22 Oct 2012 21:00:33 -0430 Subject: [Python-es] =?iso-8859-1?q?Articulo_sobre_Creaci=F3n_de_entornos_?= =?iso-8859-1?q?virtuales_Python?= Message-ID: Hola Pythonistas Espero que se encuentren bien ;) Quería compartir con ustedes un articulo que escribí hace un tiempo sobre Creación de entornos virtuales Python (0) espero les sea de utilidad cualquier comentario o sugerencia es bienvenida :D (0) http://lcaballero.wordpress.com/2012/10/22/creacion-de-entornos-virtuales-python/ -- Atentamente T.S.U. Leonardo Caballero Linux Counter ID = https://linuxcounter.net/user/369081.html Key fingerprint = 9FD2 DC71 38E7 A1D1 57F5 1D29 04DE 43BC 8A27 424A /me Corriendo Debian Wheezy y Canaina GNU/Linux 3 /me Cree "El Conocimiento Humano le Pertenece al Mundo" ------------ próxima parte ------------ Se ha borrado un adjunto en formato HTML... URL: From asdrubal.ivan.suarez.rivera en gmail.com Tue Oct 23 03:56:42 2012 From: asdrubal.ivan.suarez.rivera en gmail.com (=?ISO-8859-1?Q?Asdr=FAbal_Iv=E1n_Su=E1rez_Rivera?=) Date: Mon, 22 Oct 2012 21:26:42 -0430 Subject: [Python-es] =?iso-8859-1?q?Articulo_sobre_Creaci=F3n_de_entornos_?= =?iso-8859-1?q?virtuales_Python?= In-Reply-To: References: Message-ID: El día 22 de octubre de 2012 21:00, Leonardo Caballero escribió: > Hola Pythonistas > > Espero que se encuentren bien ;) > > Quería compartir con ustedes un articulo que escribí hace un tiempo sobre > Creación de entornos virtuales Python (0) espero les sea de utilidad > cualquier comentario o sugerencia es bienvenida :D > > (0) > http://lcaballero.wordpress.com/2012/10/22/creacion-de-entornos-virtuales-python/ > Excelente Leonardo... Quizás sería buena idea en mi caso hacer "fork" de tu artículo para adaptarlo a Arch Linux. Obviamente dándote crédito por el original :) Saludos. > -- > Atentamente > > T.S.U. Leonardo Caballero > Linux Counter ID = https://linuxcounter.net/user/369081.html > Key fingerprint = 9FD2 DC71 38E7 A1D1 57F5 1D29 04DE 43BC 8A27 424A > > /me Corriendo Debian Wheezy y Canaina GNU/Linux 3 > /me Cree "El Conocimiento Humano le Pertenece al Mundo" > > _______________________________________________ > Python-es mailing list > Python-es en python.org > http://mail.python.org/mailman/listinfo/python-es > FAQ: http://python-es-faq.wikidot.com/ > -- Asdrúbal Iván Suárez Rivera Si quieres aprender, enseña. Marco Tulio Cicerón. ¿Tienes twitter? Sígueme: @asdrubalivan From pych3m4 en gmail.com Tue Oct 23 10:12:36 2012 From: pych3m4 en gmail.com (Chema Cortes) Date: Tue, 23 Oct 2012 10:12:36 +0200 Subject: [Python-es] =?iso-8859-1?q?Articulo_sobre_Creaci=F3n_de_entornos_?= =?iso-8859-1?q?virtuales_Python?= In-Reply-To: References: Message-ID: El día 23 de octubre de 2012 03:30, Leonardo Caballero escribió: > Quería compartir con ustedes un articulo que escribí hace un tiempo sobre > Creación de entornos virtuales Python (0) espero les sea de utilidad > cualquier comentario o sugerencia es bienvenida :D > > (0) > http://lcaballero.wordpress.com/2012/10/22/creacion-de-entornos-virtuales-python/ Hace poco que también escribí algo similar relacionado con la instalación de paquetes: http://ch3m4.org/blog/2012/07/24/instalacion-de-modulos/ Con python 3.3 se ha introducido en la librería estándar los entornos virtuales (módulo 'venv' - PEP-405). Precisamente pensaba publicar esta semana un artículo sobre venv que complementaría el anterior. -- Hyperreals *R: http://ch3m4.org/blog Quarks, bits y otras criaturas infinitesimales From kikocorreoso en gmail.com Tue Oct 23 10:21:02 2012 From: kikocorreoso en gmail.com (Kiko) Date: Tue, 23 Oct 2012 10:21:02 +0200 Subject: [Python-es] =?iso-8859-1?q?Articulo_sobre_Creaci=F3n_de_entornos_?= =?iso-8859-1?q?virtuales_Python?= In-Reply-To: References: Message-ID: El 23 de octubre de 2012 10:12, Chema Cortes escribió: > El día 23 de octubre de 2012 03:30, Leonardo Caballero > escribió: > > > Quería compartir con ustedes un articulo que escribí hace un tiempo sobre > > Creación de entornos virtuales Python (0) espero les sea de utilidad > > cualquier comentario o sugerencia es bienvenida :D > > > > (0) > > > http://lcaballero.wordpress.com/2012/10/22/creacion-de-entornos-virtuales-python/ > > Hace poco que también escribí algo similar relacionado con la > instalación de paquetes: > > http://ch3m4.org/blog/2012/07/24/instalacion-de-modulos/ > > Con python 3.3 se ha introducido en la librería estándar los entornos > virtuales (módulo 'venv' - PEP-405). Precisamente pensaba publicar > esta semana un artículo sobre venv que complementaría el anterior. > Aprovechando la coyuntura, si tengo, por ejemplo, instalado python 2.7, ¿puedo crear un venv con python 3.x o con pypy (sin tenerlos instalados) o previamente tengo que instalarme python 3.x o pypy y que el venv apunte al python que quiero usar, 2.7, 3.x o pypy? Entiendo que es lo segundo, que tengo que instalar la versión de python que quiero usar previamente, pero sin leer la documentación me preguntaba si se podría hacer y aprovechando que hay dos expertos por aquí... Gracias. saludos. ------------ próxima parte ------------ Se ha borrado un adjunto en formato HTML... URL: From pych3m4 en gmail.com Tue Oct 23 13:36:03 2012 From: pych3m4 en gmail.com (Chema Cortes) Date: Tue, 23 Oct 2012 13:36:03 +0200 Subject: [Python-es] =?iso-8859-1?q?Articulo_sobre_Creaci=F3n_de_entornos_?= =?iso-8859-1?q?virtuales_Python?= In-Reply-To: References: Message-ID: El día 23 de octubre de 2012 10:21, Kiko escribió: > Aprovechando la coyuntura, si tengo, por ejemplo, instalado python 2.7, > ¿puedo crear un venv con python 3.x o con pypy (sin tenerlos instalados) o > previamente tengo que instalarme python 3.x o pypy y que el venv apunte al > python que quiero usar, 2.7, 3.x o pypy? > > Entiendo que es lo segundo, que tengo que instalar la versión de python que > quiero usar previamente, pero sin leer la documentación me preguntaba si se > podría hacer y aprovechando que hay dos expertos por aquí... De momento el módulo 'venv' está sólo en python 3.3, así que es requisito tenerlo instalado. Si hablamos de virtualenv, con python 2.7 puedes crear cualquier entorno para cualquier otra versión de python, pero necesita que esté instalada esta versión puesto que la necesita para copiarla en sus propios directorios. Aplicando las opciones adecuadas, la copia que hace virtualenv puede ser independiente de la del sistema e incluso se puede mover a otro disco. Una vez creado el entorno virtual, se puede desinstalar la versión de python del sistema sin problemas. Si tu pregunta iba sobre poder instalar python3.3 en un entorno virtual sin tener que instalarlo necesariamente en el sistema, supongo que se puede. En la última fase de la compilación de python3.3 a partir de los fuentes, en lugar de instalarlo se podría probar a crear el entorno virtual a partir de los ficheros compilados. Tendría que probar, aunque veo preferible dejar que termine y se instalale en el sistema (/usr/local). Si no quieres que cambie la versión de python por defecto, siempre puedes instalar como 'make altinstall'. -- Hyperreals *R: http://ch3m4.org/blog Quarks, bits y otras criaturas infinitesimales From leonardocaballero en gmail.com Tue Oct 23 14:11:28 2012 From: leonardocaballero en gmail.com (Leonardo Caballero) Date: Tue, 23 Oct 2012 07:41:28 -0430 Subject: [Python-es] =?iso-8859-1?q?Articulo_sobre_Creaci=F3n_de_entornos_?= =?iso-8859-1?q?virtuales_Python?= In-Reply-To: References: Message-ID: 2012/10/22 Asdrúbal Iván Suárez Rivera < asdrubal.ivan.suarez.rivera en gmail.com> > El día 22 de octubre de 2012 21:00, Leonardo Caballero > escribió: > > Hola Pythonistas > > > > Espero que se encuentren bien ;) > > > > Quería compartir con ustedes un articulo que escribí hace un tiempo sobre > > Creación de entornos virtuales Python (0) espero les sea de utilidad > > cualquier comentario o sugerencia es bienvenida :D > > > > (0) > > > http://lcaballero.wordpress.com/2012/10/22/creacion-de-entornos-virtuales-python/ > > > > Excelente Leonardo... Quizás sería buena idea en mi caso hacer "fork" > de tu artículo para adaptarlo a Arch Linux. Obviamente dándote crédito > por el original :) > > Hey que bueno seguro me parece bien cualquier cosa lo vinculo desde acá ;-) https://github.com/collective/collective.spanishdocumentation/blob/master/source/python/creacion_entornos_virtuales.rst Saludos > Saludos. > > > -- > > Atentamente > > > > T.S.U. Leonardo Caballero > > Linux Counter ID = https://linuxcounter.net/user/369081.html > > Key fingerprint = 9FD2 DC71 38E7 A1D1 57F5 1D29 04DE 43BC 8A27 424A > > > > /me Corriendo Debian Wheezy y Canaina GNU/Linux 3 > > /me Cree "El Conocimiento Humano le Pertenece al Mundo" > > > > _______________________________________________ > > Python-es mailing list > > Python-es en python.org > > http://mail.python.org/mailman/listinfo/python-es > > FAQ: http://python-es-faq.wikidot.com/ > > > > > > -- > Asdrúbal Iván Suárez Rivera > > Si quieres aprender, enseña. Marco Tulio Cicerón. > > ¿Tienes twitter? > Sígueme: @asdrubalivan > _______________________________________________ > Python-es mailing list > Python-es en python.org > http://mail.python.org/mailman/listinfo/python-es > FAQ: http://python-es-faq.wikidot.com/ > -- Atentamente T.S.U. Leonardo Caballero Linux Counter ID = https://linuxcounter.net/user/369081.html Key fingerprint = 9FD2 DC71 38E7 A1D1 57F5 1D29 04DE 43BC 8A27 424A /me Corriendo Debian Wheezy y Canaina GNU/Linux 3 /me Cree "El Conocimiento Humano le Pertenece al Mundo" ------------ próxima parte ------------ Se ha borrado un adjunto en formato HTML... URL: From dhannier en gmail.com Thu Oct 25 16:29:11 2012 From: dhannier en gmail.com (Dhannier Molina) Date: Thu, 25 Oct 2012 09:59:11 -0430 Subject: [Python-es] Accesoria sobre Seguridad Informatica en Python Message-ID: Hola!! Necesito accesoria en relación a la seguridad Informática en Python. He escuchado que los lenguajes interpretados son pocos seguros y pues necesito información respecto a eso. Estoy trabajando en un sistema hecho en python y necesito desarrollar un modulo de seguridad (Usuario y Perfiles), donde se pueda administrar los permisos para cada usuario y garantizar la protección de los datos. Entonces necesito saber que tan seguros son los sistemas desarrollados en python, contra posibles ataques externos? Cual seria la mejor forma o método para encriptar datos? Los principales ataques al sistemas son a los archivos con exenciones .PYC? Gracias de ante mano, soy nuevo en el área de desarrollo de sistemas computacionales... Saludos!! ------------ próxima parte ------------ Se ha borrado un adjunto en formato HTML... URL: From jza en oooes.org Thu Oct 25 16:47:57 2012 From: jza en oooes.org (Alexandro Colorado) Date: Thu, 25 Oct 2012 09:47:57 -0500 Subject: [Python-es] Accesoria sobre Seguridad Informatica en Python In-Reply-To: References: Message-ID: Creo que primero debes investigar un poco sobre la seguridad aplicacada a lenguajes ya que no es la misma a la seguridad del usuario comun de escritorio, los sistemas web, los sistemas en web, o las redes. La seguridad en los lenguajes debe de ver mas con el manejo de memoria, y cosas similares. Un buen sitio es el de CERN, este explica como se manejan los lenguajes y las formas seguras de programar y de paradigma. https://security.web.cern.ch/security/recommendations/en/index.shtml Contestando tu pregunta de herramientas para auditar el codigo o binarios, pues por lo menos en python aqui listan uno. https://security.web.cern.ch/security/recommendations/en/code_tools.shtml On 10/25/12, Dhannier Molina wrote: > Hola!! Necesito accesoria en relación a la seguridad Informática en Python. > > He escuchado que los lenguajes interpretados son pocos seguros y pues > necesito información respecto a eso. Estoy trabajando en un sistema hecho > en python y necesito desarrollar un modulo de seguridad (Usuario y > Perfiles), donde se pueda administrar los permisos para cada usuario y > garantizar la protección de los datos. > > Entonces necesito saber que tan seguros son los sistemas desarrollados en > python, contra posibles ataques externos? > Cual seria la mejor forma o método para encriptar datos? > Los principales ataques al sistemas son a los archivos con exenciones .PYC? > > Gracias de ante mano, soy nuevo en el área de desarrollo de sistemas > computacionales... > > Saludos!! > -- Alexandro Colorado PPMC Apache OpenOffice http://es.openoffice.org From alito81 en gmail.com Thu Oct 25 20:13:06 2012 From: alito81 en gmail.com (alito s) Date: Thu, 25 Oct 2012 13:13:06 -0500 Subject: [Python-es] TypeError Message-ID: Hola a tod en s: Molestandolos de nuevo. Estoy buscando unos números al final de una línea. Cada línea tiene diferentes especificaciones y solo me interesa el número al final de cada línea. Lo hice con expresiones regulares y todo bien, como debe de ser, la única restricción es que los números deben ser ">=20" así que se me ocurrió poner un "if" como condicionante. Y todo bien, me lo imprime en pantalla, sin embargo, a la hora que quiero que me guarde solo los números me manda el "TypeError: expected a character buffer object" y no se por qué. Seguramente es una tontería pero no le encuentro como solucionarlo. Me interesa gurdarlo. Les dejo mi código. outp = open("errores.txt", "w") for line2 in a: a2 = re.search(r"\d+$", line2) a3 = int(a2.group()) if a3 >=20: print (a3) outp.write(a3) outp.close() De antemano, gracias. ------------ próxima parte ------------ Se ha borrado un adjunto en formato HTML... URL: From jcaballero.hep en gmail.com Thu Oct 25 20:21:55 2012 From: jcaballero.hep en gmail.com (Jose Caballero) Date: Thu, 25 Oct 2012 14:21:55 -0400 Subject: [Python-es] TypeError In-Reply-To: References: Message-ID: El 25 de octubre de 2012 14:13, alito s escribió: > Hola a tod en s: > Molestandolos de nuevo. Estoy buscando unos números al final de una línea. > Cada línea tiene diferentes especificaciones y solo me interesa el número > al final de cada línea. Lo hice con expresiones regulares y todo bien, como > debe de ser, la única restricción es que los números deben ser ">=20" así > que se me ocurrió poner un "if" como condicionante. Y todo bien, me lo > imprime en pantalla, sin embargo, a la hora que quiero que me guarde solo > los números me manda el "TypeError: expected a character buffer object" y > no se por qué. Seguramente es una tontería pero no le encuentro como > solucionarlo. Me interesa gurdarlo. > Les dejo mi código. > > > outp = open("errores.txt", "w") > > for line2 in a: > a2 = re.search(r"\d+$", line2) > a3 = int(a2.group()) > if a3 >=20: > print (a3) > outp.write(a3) > > outp.close() > > De antemano, gracias. > > el propio mensaje de error lo dice: " expected a character buffer object" >>> o = open('/tmp/kk', 'w') >>> o.write(3) Traceback (most recent call last): File "", line 1, in TypeError: expected a character buffer object >>> o.write('3') >>> >>> help(o.write) write(...) write(str) -> None. Write string str to file. Note that due to buffering, flush() or close() may be needed before the file on disk reflects the data written. Jose ------------ próxima parte ------------ Se ha borrado un adjunto en formato HTML... URL: From kikocorreoso en gmail.com Thu Oct 25 20:23:42 2012 From: kikocorreoso en gmail.com (Kiko) Date: Thu, 25 Oct 2012 20:23:42 +0200 Subject: [Python-es] TypeError In-Reply-To: References: Message-ID: El 25 de octubre de 2012 20:21, Jose Caballero escribió: > > > El 25 de octubre de 2012 14:13, alito s escribió: > > Hola a tod en s: >> Molestandolos de nuevo. Estoy buscando unos números al final de una >> línea. Cada línea tiene diferentes especificaciones y solo me interesa el >> número al final de cada línea. Lo hice con expresiones regulares y todo >> bien, como debe de ser, la única restricción es que los números deben ser >> ">=20" así que se me ocurrió poner un "if" como condicionante. Y todo bien, >> me lo imprime en pantalla, sin embargo, a la hora que quiero que me guarde >> solo los números me manda el "TypeError: expected a character buffer >> object" y no se por qué. Seguramente es una tontería pero no le encuentro >> como solucionarlo. Me interesa gurdarlo. >> Les dejo mi código. >> >> >> outp = open("errores.txt", "w") >> >> for line2 in a: >> a2 = re.search(r"\d+$", line2) >> a3 = int(a2.group()) >> if a3 >=20: >> print (a3) >> outp.write(a3) >> >> outp.close() >> >> De antemano, gracias. >> >> > > el propio mensaje de error lo dice: " expected a character buffer object" > > > >>> o = open('/tmp/kk', 'w') > >>> o.write(3) > Traceback (most recent call last): > File "", line 1, in > > TypeError: expected a character buffer object > > >>> o.write('3') > >>> > > >>> help(o.write) > > write(...) > write(str) -> None. Write string str to file. > > Note that due to buffering, flush() or close() may be needed before > the file on disk reflects the data written. > > > > Jose > > La doc oficial: http://docs.python.org/tutorial/inputoutput.html#methods-of-file-objects;-) ------------ próxima parte ------------ Se ha borrado un adjunto en formato HTML... URL: From alito81 en gmail.com Thu Oct 25 20:48:44 2012 From: alito81 en gmail.com (alito s) Date: Thu, 25 Oct 2012 13:48:44 -0500 Subject: [Python-es] TypeError In-Reply-To: References: Message-ID: Cierto, lo único que tuve que hacer fue agregarle una línea en donde a3 se convirtiera en "str". Gracias 2012/10/25 Kiko > > > El 25 de octubre de 2012 20:21, Jose Caballero escribió: > > >> >> El 25 de octubre de 2012 14:13, alito s escribió: >> >> Hola a tod en s: >>> Molestandolos de nuevo. Estoy buscando unos números al final de una >>> línea. Cada línea tiene diferentes especificaciones y solo me interesa el >>> número al final de cada línea. Lo hice con expresiones regulares y todo >>> bien, como debe de ser, la única restricción es que los números deben ser >>> ">=20" así que se me ocurrió poner un "if" como condicionante. Y todo bien, >>> me lo imprime en pantalla, sin embargo, a la hora que quiero que me guarde >>> solo los números me manda el "TypeError: expected a character buffer >>> object" y no se por qué. Seguramente es una tontería pero no le encuentro >>> como solucionarlo. Me interesa gurdarlo. >>> Les dejo mi código. >>> >>> >>> outp = open("errores.txt", "w") >>> >>> for line2 in a: >>> a2 = re.search(r"\d+$", line2) >>> a3 = int(a2.group()) >>> if a3 >=20: >>> print (a3) >>> outp.write(a3) >>> >>> outp.close() >>> >>> De antemano, gracias. >>> >>> >> >> el propio mensaje de error lo dice: " expected a character buffer object" >> >> >> >>> o = open('/tmp/kk', 'w') >> >>> o.write(3) >> Traceback (most recent call last): >> File "", line 1, in >> >> TypeError: expected a character buffer object >> >> >>> o.write('3') >> >>> >> >> >>> help(o.write) >> >> write(...) >> write(str) -> None. Write string str to file. >> >> Note that due to buffering, flush() or close() may be needed before >> the file on disk reflects the data written. >> >> >> >> Jose >> >> La doc oficial: > http://docs.python.org/tutorial/inputoutput.html#methods-of-file-objects;-) > > _______________________________________________ > Python-es mailing list > Python-es en python.org > http://mail.python.org/mailman/listinfo/python-es > FAQ: http://python-es-faq.wikidot.com/ > > ------------ próxima parte ------------ Se ha borrado un adjunto en formato HTML... URL: From francisco.palm en gmail.com Fri Oct 26 00:48:51 2012 From: francisco.palm en gmail.com (Francisco Palm) Date: Thu, 25 Oct 2012 18:18:51 -0430 Subject: [Python-es] Video Promocional PyConVE Message-ID: Muchas gracias a David Rodríguez http://www.youtube.com/user/effetagroove por realizar este vídeo promocional. http://youtu.be/musKJXDtCqk descargar las funtes del video: http://dl.dropbox.com/u/4781508/pyconve.rar blends, audio y demas Creative commons 0 download source of the video: http://dl.dropbox.com/u/4781508/pyconve.rar Vale la pena anotar que el video está hecho en Blender y la música también la realizó con herramientas libres: Lmms, Hydrogen y Audacity http://t.co/mUT2z5sd Saludos y a preparar las pilas para el PyConVE 2012 que ya lo tenemos a la vuelta de la esquina. F. Palm -- -------------------------------------- fpalm en mapologo.org.ve francisco.palm en gmail.com cel: +58 +424 7228252 tel: +58 +274 6352001 ---- Debemos ser libres, no para hacer lo que nos plazca, sino libres para comprender muy profundamente nuestros propios instintos e impulsos. K ------------ próxima parte ------------ Se ha borrado un adjunto en formato HTML... URL: From jza en oooes.org Fri Oct 26 01:25:08 2012 From: jza en oooes.org (Alexandro Colorado) Date: Thu, 25 Oct 2012 18:25:08 -0500 Subject: [Python-es] Video Promocional PyConVE In-Reply-To: References: Message-ID: Caray la gente de Venezuela siempre es mucho de comunidad, me da mucho gusto que se tomen iniciativas como estas en todo latinoamerica. Sin embargo tambien tengo ganas que se rompa esa barrera de adoptar tecnologias como usuarios (usuarios de lenguajes en este caso), y pasemos al frente como generadores de estas tecnologias. Me pregunto cuanto de este evento habra gente trabajando en el desarrollo del lenguaje Python, que haya implementado PEPs o vea el futuro de alguno de las implementaciones de Python (Jython, Pypy), etc. O inclusive que haya creado lenguajes nuevos, como Vala, o Genie o BaCon. Usualmente los developers son Alemanes o Escandinavos. Y me da risa que su razon por la cual no hay gente en latam haciendo esto es, aya tienen sol y tienen mejores cosas que hacer que perder el tiempo encerrado en sus casas. :) Perdon por secuestrar el topico pero es una pequeña reflexión sobre algunas cosas que veo en el SL a nivel global. On 10/25/12, Francisco Palm wrote: > Muchas gracias a David Rodríguez http://www.youtube.com/user/effetagroove > por > realizar este vídeo promocional. > > http://youtu.be/musKJXDtCqk > > descargar las funtes del video: http://dl.dropbox.com/u/4781508/pyconve.rar > blends, audio y demas Creative commons 0 > download source of the video: http://dl.dropbox.com/u/4781508/pyconve.rar > > Vale la pena anotar que el video está hecho en Blender y la música también > la realizó con herramientas libres: Lmms, Hydrogen y Audacity > http://t.co/mUT2z5sd > > Saludos y a preparar las pilas para el PyConVE 2012 que ya lo tenemos a la > vuelta de la esquina. > > F. Palm > > -- > -------------------------------------- > fpalm en mapologo.org.ve > francisco.palm en gmail.com > > cel: +58 +424 7228252 > tel: +58 +274 6352001 > > ---- > Debemos ser libres, no para hacer lo que nos plazca, sino libres para > comprender muy profundamente nuestros propios instintos e impulsos. K > -- Alexandro Colorado PPMC Apache OpenOffice http://es.openoffice.org From francisco.palm en gmail.com Fri Oct 26 02:43:07 2012 From: francisco.palm en gmail.com (Francisco Palm) Date: Thu, 25 Oct 2012 20:13:07 -0430 Subject: [Python-es] Video Promocional PyConVE In-Reply-To: References: Message-ID: 2012/10/25 Alexandro Colorado > Caray la gente de Venezuela siempre es mucho de comunidad, me da mucho > gusto que se tomen iniciativas como estas en todo latinoamerica. > > Sin embargo tambien tengo ganas que se rompa esa barrera de adoptar > tecnologias como usuarios (usuarios de lenguajes en este caso), y > pasemos al frente como generadores de estas tecnologias. Amigo, la reflexión es válida y bienvenida. Sin embargo, no es un tema trivial, es difícil para mi encontrar mejor referencia al respecto que el libro "Invention: The Care and Feeding of Ideas" de Norbert Wiener. "Generar tecnología", parte de entender profundamente el hecho tecnológico y crear las condiciones o "climas" para que esto sea posible, lo cuál no es de ningún modo un tema de esfuerzo individual, ni siquiera el trabajo de pequeños colectivos, se trata de cocinar un denso y rico caldo de cultivo. Basta analizar antropológica y sociológicamente desde que entornos aparecieron personas como Linus Torvalds o Guido Van Rossum. Una de las principales trabas para lograr esto es la insistencia en los grupos técnicos de no hablar de filosofía ni política ni ideologías, como si el mantener a la tecnología impermeabilizada a todo esto no fuese otra forma de ideología muy particular y limitada que la castra y la condiciona con severidad. Pienso que Internet y la Sociedad Red pueden hacer mucho en este sentido, pero mientras que los problemas sociales-políticos entre unos y otros permanezcan invisibilizados por causa del discurso tecnicista predominante es realmente muy difícil que en virtud de las restricciones propias de nuestras sociedades haya un avance significativo. Disculpen lo que para muchos será un OFF TOPIC, si acaso alguien quiere hablar del tema puede escribirme en privado, por mi parte el hilo se acaba acá. Saludos F. Palm -- -------------------------------------- fpalm en mapologo.org.ve francisco.palm en gmail.com cel: +58 +424 7228252 tel: +58 +274 6352001 ---- Debemos ser libres, no para hacer lo que nos plazca, sino libres para comprender muy profundamente nuestros propios instintos e impulsos. K ------------ próxima parte ------------ Se ha borrado un adjunto en formato HTML... URL: From alvarezlucas en gmail.com Fri Oct 26 02:58:03 2012 From: alvarezlucas en gmail.com (alvarezlucas en gmail.com) Date: Fri, 26 Oct 2012 00:58:03 +0000 Subject: [Python-es] Obtener tecla pulsada.. Message-ID: <424129677-1351213161-cardhu_decombobulator_blackberry.rim.net-1193286072-@b26.c24.bise6.blackberry> Hola gente.. Soy nuevo en la lista y en python.. Así mis disculpas si es básica la pregunta... Pero quiero obtener la tecla pulsada... Si alguien me puede orientar por donde mirar .. Agradecido seré... Saludos.. From asdrubal.ivan.suarez.rivera en gmail.com Fri Oct 26 03:11:46 2012 From: asdrubal.ivan.suarez.rivera en gmail.com (=?ISO-8859-1?Q?Asdr=FAbal_Iv=E1n_Su=E1rez_Rivera?=) Date: Thu, 25 Oct 2012 20:41:46 -0430 Subject: [Python-es] Obtener tecla pulsada.. In-Reply-To: <424129677-1351213161-cardhu_decombobulator_blackberry.rim.net-1193286072-@b26.c24.bise6.blackberry> References: <424129677-1351213161-cardhu_decombobulator_blackberry.rim.net-1193286072-@b26.c24.bise6.blackberry> Message-ID: El día 25 de octubre de 2012 20:28, escribió: > Hola gente.. Soy nuevo en la lista y en python.. Así mis disculpas si es básica la pregunta... Pero quiero obtener la tecla pulsada... > Si alguien me puede orientar por donde mirar .. Agradecido seré... > Hola Lucas, quizás esto te ayude http://stackoverflow.com/questions/983354/how-do-i-make-python-to-wait-for-a-pressed-key Saludos > Saludos.. > > _______________________________________________ > Python-es mailing list > Python-es en python.org > http://mail.python.org/mailman/listinfo/python-es > FAQ: http://python-es-faq.wikidot.com/ -- Asdrúbal Iván Suárez Rivera Si quieres aprender, enseña. Marco Tulio Cicerón. ¿Tienes twitter? Sígueme: @asdrubalivan From pych3m4 en gmail.com Fri Oct 26 09:35:58 2012 From: pych3m4 en gmail.com (Chema Cortes) Date: Fri, 26 Oct 2012 09:35:58 +0200 Subject: [Python-es] TypeError In-Reply-To: References: Message-ID: El día 25 de octubre de 2012 20:48, alito s escribió: > Cierto, lo único que tuve que hacer fue agregarle una línea en donde a3 se > convirtiera en "str". Para estas cosas, a veces resulta más cómodo usar la función print() print( a,b,c,d, file=outp) En python 2.7, tienes una sintáxis alternativa (que habría que dejar de usar): print >>outp, a,b,c,d -- Hyperreals *R: http://ch3m4.org/blog Quarks, bits y otras criaturas infinitesimales From jcaballero.hep en gmail.com Fri Oct 26 18:21:58 2012 From: jcaballero.hep en gmail.com (Jose Caballero) Date: Fri, 26 Oct 2012 12:21:58 -0400 Subject: [Python-es] recomendacion para comunicacion cliente-servidor, ambos en la misma maquina Message-ID: Hola, tengo un proceso 'daemon' escrito en python. Esta corriendo el 100% del tiempo. Necesitaria que algunos scripts que se ejecutan desde la linea de comandos, tambien escritos en python, sean capaces de enviarle mensajes a ese daemon. No hay problemas de seguridad ni de autenticacion. Ambos procesos se ejecutan en la misma maquina, y se presupone que las etapas de autenticacion/autorizacion ya se han hecho antes. Por otro lado, al estar en la misma maquina, y por tratarse de mensajes muy cortos, no hay problemas de eficiencia. Cual es la forma mas sencilla que me recomiendan para implementar la comunicacion? Una busqueda en google me da dos posibles alternativas (o quizas son la misma y a mi me parecen diferentes): usar un httpd (que ------------ próxima parte ------------ Se ha borrado un adjunto en formato HTML... URL: From jcaballero.hep en gmail.com Fri Oct 26 18:23:37 2012 From: jcaballero.hep en gmail.com (Jose Caballero) Date: Fri, 26 Oct 2012 12:23:37 -0400 Subject: [Python-es] recomendacion para comunicacion cliente-servidor, ambos en la misma maquina In-Reply-To: References: Message-ID: (perdon, he enviado el mensaje a medias por error) El 26 de octubre de 2012 12:21, Jose Caballero escribió: > Hola, > > > tengo un proceso 'daemon' escrito en python. Esta corriendo el 100% del > tiempo. > Necesitaria que algunos scripts que se ejecutan desde la linea de > comandos, tambien escritos en python, sean capaces de enviarle mensajes a > ese daemon. > > No hay problemas de seguridad ni de autenticacion. Ambos procesos se > ejecutan en la misma maquina, y se presupone que las etapas de > autenticacion/autorizacion ya se han hecho antes. > Por otro lado, al estar en la misma maquina, y por tratarse de mensajes > muy cortos, no hay problemas de eficiencia. > > Cual es la forma mas sencilla que me recomiendan para implementar la > comunicacion? > Una busqueda en google me da dos posibles alternativas (o quizas son la > misma y a mi me parecen diferentes): > > - usar un servidor http (puede el ser el que trae python) y escuchar llamadas hechas por ejemplo con libcurl - sockets. Ando algo perdido. Cualquier sugerencia (o link donde pueda aprender) es mas que bienvenida. Gracias por adelantado. Jose (de nuevo sin tildes) ------------ próxima parte ------------ Se ha borrado un adjunto en formato HTML... URL: From jespinog en gmail.com Fri Oct 26 18:41:37 2012 From: jespinog en gmail.com (=?ISO-8859-1?Q?Jes=FAs_Espino?=) Date: Fri, 26 Oct 2012 18:41:37 +0200 Subject: [Python-es] recomendacion para comunicacion cliente-servidor, ambos en la misma maquina In-Reply-To: References: Message-ID: Depende del tipo de comunicaciones que quieras hacer igual te podría ir bien zeromq o rabbitmq. Un saludo El 26/10/2012 18:24, "Jose Caballero" escribió: > (perdon, he enviado el mensaje a medias por error) > > > > El 26 de octubre de 2012 12:21, Jose Caballero escribió: > >> Hola, >> >> >> tengo un proceso 'daemon' escrito en python. Esta corriendo el 100% del >> tiempo. >> Necesitaria que algunos scripts que se ejecutan desde la linea de >> comandos, tambien escritos en python, sean capaces de enviarle mensajes a >> ese daemon. >> >> No hay problemas de seguridad ni de autenticacion. Ambos procesos se >> ejecutan en la misma maquina, y se presupone que las etapas de >> autenticacion/autorizacion ya se han hecho antes. >> Por otro lado, al estar en la misma maquina, y por tratarse de mensajes >> muy cortos, no hay problemas de eficiencia. >> >> Cual es la forma mas sencilla que me recomiendan para implementar la >> comunicacion? >> Una busqueda en google me da dos posibles alternativas (o quizas son la >> misma y a mi me parecen diferentes): >> >> > - usar un servidor http (puede el ser el que trae python) y escuchar > llamadas hechas por ejemplo con libcurl > > - sockets. > > Ando algo perdido. Cualquier sugerencia (o link donde pueda aprender) es > mas que bienvenida. > > > > > Gracias por adelantado. > Jose (de nuevo sin tildes) > > > _______________________________________________ > Python-es mailing list > Python-es en python.org > http://mail.python.org/mailman/listinfo/python-es > FAQ: http://python-es-faq.wikidot.com/ > > ------------ próxima parte ------------ Se ha borrado un adjunto en formato HTML... URL: From carlos.zun en gmail.com Fri Oct 26 18:46:00 2012 From: carlos.zun en gmail.com (Carlos Zuniga) Date: Fri, 26 Oct 2012 11:46:00 -0500 Subject: [Python-es] recomendacion para comunicacion cliente-servidor, ambos en la misma maquina In-Reply-To: References: Message-ID: 2012/10/26 Jose Caballero : > (perdon, he enviado el mensaje a medias por error) > > > > El 26 de octubre de 2012 12:21, Jose Caballero > escribió: > >> Hola, >> >> >> tengo un proceso 'daemon' escrito en python. Esta corriendo el 100% del >> tiempo. >> Necesitaria que algunos scripts que se ejecutan desde la linea de >> comandos, tambien escritos en python, sean capaces de enviarle mensajes a >> ese daemon. >> >> No hay problemas de seguridad ni de autenticacion. Ambos procesos se >> ejecutan en la misma maquina, y se presupone que las etapas de >> autenticacion/autorizacion ya se han hecho antes. >> Por otro lado, al estar en la misma maquina, y por tratarse de mensajes >> muy cortos, no hay problemas de eficiencia. >> >> Cual es la forma mas sencilla que me recomiendan para implementar la >> comunicacion? >> Una busqueda en google me da dos posibles alternativas (o quizas son la >> misma y a mi me parecen diferentes): >> > > - usar un servidor http (puede el ser el que trae python) y escuchar > llamadas hechas por ejemplo con libcurl > > - sockets. > > Ando algo perdido. Cualquier sugerencia (o link donde pueda aprender) es mas > que bienvenida. > Una opción es utilizar twisted: http://twistedmatrix.com/trac/ Y una más para que invesgues es usar dbus. Saludos -- A menudo unas pocas horas de "Prueba y error" podrán ahorrarte minutos de leer manuales. From antonio.cascales en gmail.com Fri Oct 26 03:33:59 2012 From: antonio.cascales en gmail.com (Antonio Cascales) Date: Fri, 26 Oct 2012 03:33:59 +0200 Subject: [Python-es] =?iso-8859-1?q?Presentaci=F3n?= Message-ID: Hola, buenas tardes: Me llamo Antonio Cascales, y soy miembro de la lista desde hace apenas un par de días. me he apuntado a ella para poder aprender lo que buenamente pueda acerca de Python. He empezado hace nada a estudiar Python, o a leer acerca de él en distintos blogs, webs, o cualquier otro medio que voy encontrando por la red. Pese que es mi primer correo a la lista, me gustaría haceros una pregunta. Como ya he dicho, la información que por el momento sé de Python, la he obtenido de distintas vías, pero me da la impresión de que ninguna es del todo completa. ¿Vosotros podríais recomendarme un buen libro para aprender Python? alguno que sea lo más completo posible. O bien, en caso de no haber uno solo, pues los que consideréis. Tened en cuenta que soy bastante novato en el mundillo, pese que me interesa bastante el mundo de la programación. Es por eso que creo que el libro en cuestión debe empezar por cosas básicas, y tener una progresión moderada. he intentado aprender otros lenguajes de programación hasta el momento sin resultados, pues todos se adentran, a mi modo de ver, en temas demasiado complejos muy pronto. Es muy probable que esté pidiendo un imposible, pero puesto que imagino que algunos de vosotros habéis pasado por lo mismo, y ahora tenéis bastantes conocimientos de Python, podréis aconsejarme el más idóneo. Muchas gracias por vuestras respuestas. Un saludo, Antonio Cascales ------------ próxima parte ------------ Se ha borrado un adjunto en formato HTML... URL: From diego.uribe.gamez en gmail.com Fri Oct 26 19:15:51 2012 From: diego.uribe.gamez en gmail.com (Diego Uribe Gamez) Date: Fri, 26 Oct 2012 12:15:51 -0500 Subject: [Python-es] =?iso-8859-1?q?Presentaci=F3n?= In-Reply-To: References: Message-ID: python para todos http://mundogeek.net/tutorial-python/ Por aquí inicie yo. El 25 de octubre de 2012 20:33, Antonio Cascales escribió: > ** > Hola, buenas tardes: > > Me llamo Antonio Cascales, y soy miembro de la lista desde hace apenas > un par de días. > > me he apuntado a ella para poder aprender lo que buenamente pueda > acerca de Python. He empezado hace nada a estudiar Python, o a leer acerca > de él en distintos blogs, webs, o cualquier otro medio que voy encontrando > por la red. > > Pese que es mi primer correo a la lista, me gustaría haceros una > pregunta. Como ya he dicho, la información que por el momento sé de Python, > la he obtenido de distintas vías, pero me da la impresión de que ninguna es > del todo completa. > > ¿Vosotros podríais recomendarme un buen libro para aprender Python? > alguno que sea lo más completo posible. O bien, en caso de no haber uno > solo, pues los que consideréis. Tened en cuenta que soy bastante novato en > el mundillo, pese que me interesa bastante el mundo de la programación. Es > por eso que creo que el libro en cuestión debe empezar por cosas básicas, y > tener una progresión moderada. > > he intentado aprender otros lenguajes de programación hasta el momento > sin resultados, pues todos se adentran, a mi modo de ver, en temas > demasiado complejos muy pronto. > > Es muy probable que esté pidiendo un imposible, pero puesto que imagino > que algunos de vosotros habéis pasado por lo mismo, y ahora tenéis > bastantes conocimientos de Python, podréis aconsejarme el más idóneo. > > Muchas gracias por vuestras respuestas. Un saludo, > > Antonio Cascales > > > _______________________________________________ > Python-es mailing list > Python-es en python.org > http://mail.python.org/mailman/listinfo/python-es > FAQ: http://python-es-faq.wikidot.com/ > > -- *Diego Alonso Uribe Gamez* ------------------------------ *Desarrollador web* Twitter: @DiegoUG Google+: http://gplus.to/diegoug ------------------------------ ------------ próxima parte ------------ Se ha borrado un adjunto en formato HTML... URL: From miguelglafuente en gmail.com Fri Oct 26 19:22:15 2012 From: miguelglafuente en gmail.com (Rock Neurotiko) Date: Fri, 26 Oct 2012 19:22:15 +0200 Subject: [Python-es] =?iso-8859-1?q?Presentaci=F3n?= In-Reply-To: References: Message-ID: Para mi lo mejor para empezar es "learning python", un libro de o'reilly, que tambien estará en español (yo lo lei en ingles), luego es muy bueno thinking in python (pensando en python, este si que esta en español), pero si no recuerdo mal era un poco más avanzado. Si prefieres ver videos y practicar el curso 101 de udacity es muy bueno :-) un saludo!!!! El 26 de octubre de 2012 19:15, Diego Uribe Gamez < diego.uribe.gamez en gmail.com> escribió: > python para todos > http://mundogeek.net/tutorial-python/ > > Por aquí inicie yo. > > El 25 de octubre de 2012 20:33, Antonio Cascales < > antonio.cascales en gmail.com> escribió: > >> ** >> Hola, buenas tardes: >> >> Me llamo Antonio Cascales, y soy miembro de la lista desde hace apenas >> un par de días. >> >> me he apuntado a ella para poder aprender lo que buenamente pueda >> acerca de Python. He empezado hace nada a estudiar Python, o a leer acerca >> de él en distintos blogs, webs, o cualquier otro medio que voy encontrando >> por la red. >> >> Pese que es mi primer correo a la lista, me gustaría haceros una >> pregunta. Como ya he dicho, la información que por el momento sé de Python, >> la he obtenido de distintas vías, pero me da la impresión de que ninguna es >> del todo completa. >> >> ¿Vosotros podríais recomendarme un buen libro para aprender Python? >> alguno que sea lo más completo posible. O bien, en caso de no haber uno >> solo, pues los que consideréis. Tened en cuenta que soy bastante novato en >> el mundillo, pese que me interesa bastante el mundo de la programación. Es >> por eso que creo que el libro en cuestión debe empezar por cosas básicas, y >> tener una progresión moderada. >> >> he intentado aprender otros lenguajes de programación hasta el momento >> sin resultados, pues todos se adentran, a mi modo de ver, en temas >> demasiado complejos muy pronto. >> >> Es muy probable que esté pidiendo un imposible, pero puesto que >> imagino que algunos de vosotros habéis pasado por lo mismo, y ahora tenéis >> bastantes conocimientos de Python, podréis aconsejarme el más idóneo. >> >> Muchas gracias por vuestras respuestas. Un saludo, >> >> Antonio Cascales >> >> >> _______________________________________________ >> Python-es mailing list >> Python-es en python.org >> http://mail.python.org/mailman/listinfo/python-es >> FAQ: http://python-es-faq.wikidot.com/ >> >> > > > -- > *Diego Alonso Uribe Gamez* > ------------------------------ > > *Desarrollador web* > > Twitter: @DiegoUG > > Google+: http://gplus.to/diegoug > ------------------------------ > > > > _______________________________________________ > Python-es mailing list > Python-es en python.org > http://mail.python.org/mailman/listinfo/python-es > FAQ: http://python-es-faq.wikidot.com/ > > -- Miguel García Lafuente - Rock Neurotiko The quieter you become, the more you are able to hear. "Libertad en lugar de miedo." - "Información libre, sociedad libre." El contenido de este e-mail es privado, no se permite la revelacion del contenido de este e-mail a gente ajena a él. ------------ próxima parte ------------ Se ha borrado un adjunto en formato HTML... URL: From marcelo.martinovic en gmail.com Fri Oct 26 23:32:40 2012 From: marcelo.martinovic en gmail.com (Marcelo Martinovic) Date: Fri, 26 Oct 2012 18:32:40 -0300 Subject: [Python-es] =?iso-8859-1?q?Presentaci=F3n?= In-Reply-To: References: Message-ID: On Oct 26, 2012 2:12 PM, "Antonio Cascales" wrote: > > Hola, buenas tardes: > > Me llamo Antonio Cascales, y soy miembro de la lista desde hace apenas un par de días. > > me he apuntado a ella para poder aprender lo que buenamente pueda acerca de Python. He empezado hace nada a estudiar Python, o a leer acerca de él en distintos blogs, webs, o cualquier otro medio que voy encontrando por la red. > > Pese que es mi primer correo a la lista, me gustaría haceros una pregunta. Como ya he dicho, la información que por el momento sé de Python, la he obtenido de distintas vías, pero me da la impresión de que ninguna es del todo completa. > > ¿Vosotros podríais recomendarme un buen libro para aprender Python? alguno que sea lo más completo posible. O bien, en caso de no haber uno solo, pues los que consideréis. Tened en cuenta que soy bastante novato en el mundillo, pese que me interesa bastante el mundo de la programación. Es por eso que creo que el libro en cuestión debe empezar por cosas básicas, y tener una progresión moderada. > > he intentado aprender otros lenguajes de programación hasta el momento sin resultados, pues todos se adentran, a mi modo de ver, en temas demasiado complejos muy pronto. > > Es muy probable que esté pidiendo un imposible, pero puesto que imagino que algunos de vosotros habéis pasado por lo mismo, y ahora tenéis bastantes conocimientos de Python, podréis aconsejarme el más idóneo. > > Muchas gracias por vuestras respuestas. Un saludo, > > Antonio Cascales > > > _______________________________________________ > Python-es mailing list > Python-es en python.org > http://mail.python.org/mailman/listinfo/python-es > FAQ: http://python-es-faq.wikidot.com/ > Te recomiendo uno. Autor: Roberto Alsina. Titulo: python no muerde, yo si. Saludos ------------ próxima parte ------------ Se ha borrado un adjunto en formato HTML... URL: From zahory en gmail.com Fri Oct 26 23:51:52 2012 From: zahory en gmail.com (Hector R. De los Santos ) Date: Fri, 26 Oct 2012 17:51:52 -0400 Subject: [Python-es] =?iso-8859-1?q?Presentaci=F3n?= In-Reply-To: References: Message-ID: > > Te recomiendo uno. > Autor: Roberto Alsina. > Titulo: python no muerde, yo si. Muy buen libro, si no aprendes al menos te diviertes un rato ;) :: HDS Consultores TI Servidores | Redes | Programacion | GNU/Linux | PostgreSQL Web: http://hdsconsultores.net Blog: http://codigohds.com Linux User #:320363 El 26 de octubre de 2012 17:32, Marcelo Martinovic < marcelo.martinovic en gmail.com> escribió: > > On Oct 26, 2012 2:12 PM, "Antonio Cascales" > wrote: > > > > Hola, buenas tardes: > > > > Me llamo Antonio Cascales, y soy miembro de la lista desde hace > apenas un par de días. > > > > me he apuntado a ella para poder aprender lo que buenamente pueda > acerca de Python. He empezado hace nada a estudiar Python, o a leer acerca > de él en distintos blogs, webs, o cualquier otro medio que voy encontrando > por la red. > > > > Pese que es mi primer correo a la lista, me gustaría haceros una > pregunta. Como ya he dicho, la información que por el momento sé de Python, > la he obtenido de distintas vías, pero me da la impresión de que ninguna es > del todo completa. > > > > ¿Vosotros podríais recomendarme un buen libro para aprender Python? > alguno que sea lo más completo posible. O bien, en caso de no haber uno > solo, pues los que consideréis. Tened en cuenta que soy bastante novato en > el mundillo, pese que me interesa bastante el mundo de la programación. Es > por eso que creo que el libro en cuestión debe empezar por cosas básicas, y > tener una progresión moderada. > > > > he intentado aprender otros lenguajes de programación hasta el > momento sin resultados, pues todos se adentran, a mi modo de ver, en temas > demasiado complejos muy pronto. > > > > Es muy probable que esté pidiendo un imposible, pero puesto que > imagino que algunos de vosotros habéis pasado por lo mismo, y ahora tenéis > bastantes conocimientos de Python, podréis aconsejarme el más idóneo. > > > > Muchas gracias por vuestras respuestas. Un saludo, > > > > Antonio Cascales > > > > > > _______________________________________________ > > Python-es mailing list > > Python-es en python.org > > http://mail.python.org/mailman/listinfo/python-es > > FAQ: http://python-es-faq.wikidot.com/ > > > Te recomiendo uno. > Autor: Roberto Alsina. > Titulo: python no muerde, yo si. > > Saludos > > _______________________________________________ > Python-es mailing list > Python-es en python.org > http://mail.python.org/mailman/listinfo/python-es > FAQ: http://python-es-faq.wikidot.com/ > > ------------ próxima parte ------------ Se ha borrado un adjunto en formato HTML... URL: From davidmenhur en gmail.com Sat Oct 27 02:14:25 2012 From: davidmenhur en gmail.com (=?UTF-8?B?RGHPgGlk?=) Date: Sat, 27 Oct 2012 02:14:25 +0200 Subject: [Python-es] =?iso-8859-1?q?Presentaci=F3n?= In-Reply-To: References: Message-ID: 2012/10/26 Hector R. De los Santos : >> Te recomiendo uno. >> Autor: Roberto Alsina. >> Titulo: python no muerde, yo si. > Muy buen libro, si no aprendes al menos te diviertes un rato ;) A mí me pareció muy bueno si ya sabes algo de programación, ideal para gente que venga a Python de otros lenguages, aunque no requiera mucho nivel. Sin embargo, la parte de programar una GUI es excelente, con un ejemplo desde la priera versión a todas las correcciones y virguerías que se le ocurren. From alvarezlucas en gmail.com Sat Oct 27 02:44:33 2012 From: alvarezlucas en gmail.com (alvarezlucas en gmail.com) Date: Sat, 27 Oct 2012 00:44:33 +0000 Subject: [Python-es] =?windows-1252?q?Presentaci=F3n?= In-Reply-To: References: Message-ID: <1640901444-1351298750-cardhu_decombobulator_blackberry.rim.net-60902886-@b26.c24.bise6.blackberry> Tengo algunos de esos libros... Y te puedo pasar los apuntes de la facultad mas los practicos... Yo hace poco que estoy también arrancando con este. Lenguaje... Si queres lo mando... Saludos.. -----Original Message----- From: Daid Sender: "Python-es" Date: Sat, 27 Oct 2012 02:14:25 To: La lista de python en castellano Reply-To: La lista de python en castellano Subject: Re: [Python-es] Presentación 2012/10/26 Hector R. De los Santos : >> Te recomiendo uno. >> Autor: Roberto Alsina. >> Titulo: python no muerde, yo si. > Muy buen libro, si no aprendes al menos te diviertes un rato ;) A mí me pareció muy bueno si ya sabes algo de programación, ideal para gente que venga a Python de otros lenguages, aunque no requiera mucho nivel. Sin embargo, la parte de programar una GUI es excelente, con un ejemplo desde la priera versión a todas las correcciones y virguerías que se le ocurren. _______________________________________________ Python-es mailing list Python-es en python.org http://mail.python.org/mailman/listinfo/python-es FAQ: http://python-es-faq.wikidot.com/ From kurokysan en gmail.com Sat Oct 27 05:51:14 2012 From: kurokysan en gmail.com (Kennedy Sanchez) Date: Fri, 26 Oct 2012 23:51:14 -0400 Subject: [Python-es] =?iso-8859-1?q?Presentaci=F3n?= Message-ID: Colocalo en un lugar publico. Ya que habemos varias personas necesitando los mismos materiales. Agradeciendo tu buena disposicion d antemano Ing. Kennedy Sanchez - MGP - Comptia Security + (829-754-3492) alvarezlucas en gmail.com wrote: >Tengo algunos de esos libros... Y te puedo pasar los apuntes de la facultad mas los practicos... Yo hace poco que estoy también arrancando con este. Lenguaje... Si queres lo mando... > >Saludos.. >-----Original Message----- >From: Daid >Sender: "Python-es" Date: Sat, 27 Oct 2012 02:14:25 >To: La lista de python en castellano >Reply-To: La lista de python en castellano >Subject: Re: [Python-es] Presentación > >2012/10/26 Hector R. De los Santos : >>> Te recomiendo uno. >>> Autor: Roberto Alsina. >>> Titulo: python no muerde, yo si. >> Muy buen libro, si no aprendes al menos te diviertes un rato ;) > >A mí me pareció muy bueno si ya sabes algo de programación, ideal para >gente que venga a Python de otros lenguages, aunque no requiera mucho >nivel. > >Sin embargo, la parte de programar una GUI es excelente, con un >ejemplo desde la priera versión a todas las correcciones y virguerías >que se le ocurren. >_______________________________________________ >Python-es mailing list >Python-es en python.org >http://mail.python.org/mailman/listinfo/python-es >FAQ: http://python-es-faq.wikidot.com/ >_______________________________________________ >Python-es mailing list >Python-es en python.org >http://mail.python.org/mailman/listinfo/python-es >FAQ: http://python-es-faq.wikidot.com/ From sergio en wikier.org Sat Oct 27 09:21:26 2012 From: sergio en wikier.org (=?UTF-8?Q?Sergio_Fern=C3=A1ndez?=) Date: Sat, 27 Oct 2012 09:21:26 +0200 Subject: [Python-es] =?utf-8?q?Presentaci=C3=B3n?= In-Reply-To: References: Message-ID: http://www.diveintopython.net/ 2012/10/26 Antonio Cascales : > Hola, buenas tardes: > > Me llamo Antonio Cascales, y soy miembro de la lista desde hace apenas un > par de días. > > me he apuntado a ella para poder aprender lo que buenamente pueda acerca > de Python. He empezado hace nada a estudiar Python, o a leer acerca de él en > distintos blogs, webs, o cualquier otro medio que voy encontrando por la > red. > > Pese que es mi primer correo a la lista, me gustaría haceros una > pregunta. Como ya he dicho, la información que por el momento sé de Python, > la he obtenido de distintas vías, pero me da la impresión de que ninguna es > del todo completa. > > ¿Vosotros podríais recomendarme un buen libro para aprender Python? > alguno que sea lo más completo posible. O bien, en caso de no haber uno > solo, pues los que consideréis. Tened en cuenta que soy bastante novato en > el mundillo, pese que me interesa bastante el mundo de la programación. Es > por eso que creo que el libro en cuestión debe empezar por cosas básicas, y > tener una progresión moderada. > > he intentado aprender otros lenguajes de programación hasta el momento > sin resultados, pues todos se adentran, a mi modo de ver, en temas demasiado > complejos muy pronto. > > Es muy probable que esté pidiendo un imposible, pero puesto que imagino > que algunos de vosotros habéis pasado por lo mismo, y ahora tenéis bastantes > conocimientos de Python, podréis aconsejarme el más idóneo. > > Muchas gracias por vuestras respuestas. Un saludo, > > Antonio Cascales > > > _______________________________________________ > Python-es mailing list > Python-es en python.org > http://mail.python.org/mailman/listinfo/python-es > FAQ: http://python-es-faq.wikidot.com/ > -- Sergio Fernández From antonio.cascales en gmail.com Fri Oct 26 09:47:57 2012 From: antonio.cascales en gmail.com (Antonio Cascales) Date: Fri, 26 Oct 2012 09:47:57 +0200 Subject: [Python-es] =?iso-8859-1?q?Presentaci=F3n?= References: Message-ID: <1ADB109D2FBB498F9CE4CE605FAADCB5@antoniopc> Hola: Muchas gracias a todos por las distintas recomendaciones. Al chico que comentaba lo de pasar los apuntes de Python o los libros comentarle que si puede, todo material es bien recibido cuando se está empezando. :) Muchas gracias, de verdad. Ya os iré bombardeando a preguntas según me vayan surgiendo. Un saludo, Antonio Cascales ----- Original Message ----- From: "Sergio Fernández" To: "La lista de python en castellano" Sent: Saturday, October 27, 2012 9:21 AM Subject: Re: [Python-es] Presentación http://www.diveintopython.net/ 2012/10/26 Antonio Cascales : > Hola, buenas tardes: > > Me llamo Antonio Cascales, y soy miembro de la lista desde hace apenas > un > par de días. > > me he apuntado a ella para poder aprender lo que buenamente pueda > acerca > de Python. He empezado hace nada a estudiar Python, o a leer acerca de él > en > distintos blogs, webs, o cualquier otro medio que voy encontrando por la > red. > > Pese que es mi primer correo a la lista, me gustaría haceros una > pregunta. Como ya he dicho, la información que por el momento sé de > Python, > la he obtenido de distintas vías, pero me da la impresión de que ninguna > es > del todo completa. > > ¿Vosotros podríais recomendarme un buen libro para aprender Python? > alguno que sea lo más completo posible. O bien, en caso de no haber uno > solo, pues los que consideréis. Tened en cuenta que soy bastante novato en > el mundillo, pese que me interesa bastante el mundo de la programación. Es > por eso que creo que el libro en cuestión debe empezar por cosas básicas, > y > tener una progresión moderada. > > he intentado aprender otros lenguajes de programación hasta el momento > sin resultados, pues todos se adentran, a mi modo de ver, en temas > demasiado > complejos muy pronto. > > Es muy probable que esté pidiendo un imposible, pero puesto que imagino > que algunos de vosotros habéis pasado por lo mismo, y ahora tenéis > bastantes > conocimientos de Python, podréis aconsejarme el más idóneo. > > Muchas gracias por vuestras respuestas. Un saludo, > > Antonio Cascales > > > _______________________________________________ > Python-es mailing list > Python-es en python.org > http://mail.python.org/mailman/listinfo/python-es > FAQ: http://python-es-faq.wikidot.com/ > -- Sergio Fernández _______________________________________________ Python-es mailing list Python-es en python.org http://mail.python.org/mailman/listinfo/python-es FAQ: http://python-es-faq.wikidot.com/ From antonio.cascales en gmail.com Sun Oct 28 00:32:27 2012 From: antonio.cascales en gmail.com (Antonio Cascales) Date: Sun, 28 Oct 2012 00:32:27 +0200 Subject: [Python-es] Estructuras de datos Message-ID: Hola a todos: He empezado a leer los distintos recursos acerca de Python que me recomendásteis, y al leer los distintos tipos de datos que existen en Python, me surge una duda. Veo que los tipos básicos son los mismos prácticamente, pero sin embargo, las colecciones de Python como los diccionarios, listas, etc, me suenan más a los Arrays, pero ninguna a las estructuras propiamente dichas de C/C++. Las listas en python son conjuntos de datos ordenados, pero no son lo mismo. Y los diccionarios o tuplas, menos. Me gustaría saber si existe alguna manera en Python, de almacenar en una estructura, por ejemplo, los datos de una persona, para crear una agenda de contactos. ¿Cómo podría conseguirse esto? Gracias a quien pueda responder. Un saludo, Antonio ------------ próxima parte ------------ Se ha borrado un adjunto en formato HTML... URL: From miguelglafuente en gmail.com Sun Oct 28 16:11:02 2012 From: miguelglafuente en gmail.com (Rock Neurotiko) Date: Sun, 28 Oct 2012 16:11:02 +0100 Subject: [Python-es] Estructuras de datos In-Reply-To: References: Message-ID: Creas una clase persona, y en una lista, diccionario, o lo que se tercie, como datos estarian objetos de esa clase. El 28/10/2012 15:10, "Antonio Cascales" escribió: > ** > Hola a todos: > > He empezado a leer los distintos recursos acerca de Python que me > recomendásteis, y al leer los distintos tipos de datos que existen en > Python, me surge una duda. > > Veo que los tipos básicos son los mismos prácticamente, pero sin > embargo, las colecciones de Python como los diccionarios, listas, etc, me > suenan más a los Arrays, pero ninguna a las estructuras propiamente dichas > de C/C++. Las listas en python son conjuntos de datos ordenados, pero no > son lo mismo. Y los diccionarios o tuplas, menos. > > Me gustaría saber si existe alguna manera en Python, de almacenar en > una estructura, por ejemplo, los datos de una persona, para crear una > agenda de contactos. ¿Cómo podría conseguirse esto? > > Gracias a quien pueda responder. Un saludo, > > Antonio > > _______________________________________________ > Python-es mailing list > Python-es en python.org > http://mail.python.org/mailman/listinfo/python-es > FAQ: http://python-es-faq.wikidot.com/ > > ------------ próxima parte ------------ Se ha borrado un adjunto en formato HTML... URL: From pych3m4 en gmail.com Mon Oct 29 02:00:25 2012 From: pych3m4 en gmail.com (Chema Cortes) Date: Mon, 29 Oct 2012 02:00:25 +0100 Subject: [Python-es] Estructuras de datos In-Reply-To: References: Message-ID: El día 28 de octubre de 2012 00:32, Antonio Cascales escribió: > He empezado a leer los distintos recursos acerca de Python que me > recomendásteis, y al leer los distintos tipos de datos que existen en > Python, me surge una duda. > > Veo que los tipos básicos son los mismos prácticamente, pero sin embargo, > las colecciones de Python como los diccionarios, listas, etc, me suenan más > a los Arrays, pero ninguna a las estructuras propiamente dichas de C/C++. > Las listas en python son conjuntos de datos ordenados, pero no son lo mismo. > Y los diccionarios o tuplas, menos. > > Me gustaría saber si existe alguna manera en Python, de almacenar en una > estructura, por ejemplo, los datos de una persona, para crear una agenda de > contactos. ¿Cómo podría conseguirse esto? Es una pregunta frecuente que puedes ver en la FAQ: http://python-es-faq.wikidot.com/#toc37 En resumidas cuentas, las estructuras de C/C++ tienen poco sentido en python sin poder especificar el tipo de cada campo. O empleas listas o empleas clases. Si prefieres probar otra cosa, tienes "namedtuples": from collections import namedtuple Empleado = namedtuple('Empleado', 'nombre, edad, departamento, categoria') emp1=Empleado("Martinez", 38, "Informática", "Programador Python") print emp1.categoria print emp1 El problema de namedtuple es que es "inmutable". Para mutabilidad, puedes mirar la siguiente receta: http://code.activestate.com/recipes/578041-namedlist/ -- Hyperreals *R: http://ch3m4.org/blog Quarks, bits y otras criaturas infinitesimales From antonio.beamud en gmail.com Mon Oct 29 10:25:10 2012 From: antonio.beamud en gmail.com (Antonio Beamud Montero) Date: Mon, 29 Oct 2012 10:25:10 +0100 Subject: [Python-es] recomendacion para comunicacion cliente-servidor, ambos en la misma maquina In-Reply-To: References: Message-ID: <508E4B76.3090703@gmail.com> El 26/10/12 18:46, Carlos Zuniga escribió: > 2012/10/26 Jose Caballero : >> (perdon, he enviado el mensaje a medias por error) >> >> >> >> El 26 de octubre de 2012 12:21, Jose Caballero >> escribió: >> >>> Hola, >>> >>> >>> tengo un proceso 'daemon' escrito en python. Esta corriendo el 100% del >>> tiempo. >>> Necesitaria que algunos scripts que se ejecutan desde la linea de >>> comandos, tambien escritos en python, sean capaces de enviarle mensajes a >>> ese daemon. >>> >>> No hay problemas de seguridad ni de autenticacion. Ambos procesos se >>> ejecutan en la misma maquina, y se presupone que las etapas de >>> autenticacion/autorizacion ya se han hecho antes. >>> Por otro lado, al estar en la misma maquina, y por tratarse de mensajes >>> muy cortos, no hay problemas de eficiencia. >>> >>> Cual es la forma mas sencilla que me recomiendan para implementar la >>> comunicacion? >>> Una busqueda en google me da dos posibles alternativas (o quizas son la >>> misma y a mi me parecen diferentes): >>> >> - usar un servidor http (puede el ser el que trae python) y escuchar >> llamadas hechas por ejemplo con libcurl >> >> - sockets. >> >> Ando algo perdido. Cualquier sugerencia (o link donde pueda aprender) es mas >> que bienvenida. >> > Una opción es utilizar twisted: > http://twistedmatrix.com/trac/ > > Y una más para que invesgues es usar dbus. > > Saludos Otras opciones son: Puedes echarle un vistazo a Pyro, si ambas partes están implementadas en python: http://pypi.python.org/pypi/Pyro4 Más flexible sería usar CORBA, por ejemplo omniORB, lo que te permite usar diferentes lenguajes y es muy, muy rápido. Un saludo. From dhannier en gmail.com Mon Oct 29 20:29:43 2012 From: dhannier en gmail.com (Dhannier Molina) Date: Mon, 29 Oct 2012 14:59:43 -0430 Subject: [Python-es] Seguridad en Python Message-ID: Hola!! Alguien me puede ayudar, tengo dudas en relacion a la seguridad en python. Hay alguna forma de impedir que el Bytecode generado al correr un programa en python sea decompilado? Es decir, que un intruso pueda generar el archivo .PY a partir del .PYC. Necesito blindar lo mejor posible mi sistema, que herramientas podría usar?. Saludos!! ------------ próxima parte ------------ Se ha borrado un adjunto en formato HTML... URL: From ricardo.cardenes en gmail.com Tue Oct 30 01:41:12 2012 From: ricardo.cardenes en gmail.com (=?ISO-8859-1?Q?Ricardo_C=E1rdenes?=) Date: Tue, 30 Oct 2012 00:41:12 +0000 Subject: [Python-es] Seguridad en Python In-Reply-To: References: Message-ID: El mejor consejo en este caso sería... no uses Python. Si el usuario tiene acceso al bytecode, se puede descompilar. 2012/10/29 Dhannier Molina : > Hola!! Alguien me puede ayudar, tengo dudas en relacion a la seguridad en > python. > > Hay alguna forma de impedir que el Bytecode generado al correr un programa > en python sea decompilado? Es decir, que un intruso pueda generar el archivo > .PY a partir del .PYC. > > Necesito blindar lo mejor posible mi sistema, que herramientas podría usar?. > Saludos!! > > _______________________________________________ > Python-es mailing list > Python-es en python.org > http://mail.python.org/mailman/listinfo/python-es > FAQ: http://python-es-faq.wikidot.com/ > From asdrubal.ivan.suarez.rivera en gmail.com Tue Oct 30 01:46:52 2012 From: asdrubal.ivan.suarez.rivera en gmail.com (=?ISO-8859-1?Q?Asdr=FAbal_Iv=E1n_Su=E1rez_Rivera?=) Date: Mon, 29 Oct 2012 20:16:52 -0430 Subject: [Python-es] Seguridad en Python In-Reply-To: References: Message-ID: El día 29 de octubre de 2012 20:11, Ricardo Cárdenes < ricardo.cardenes en gmail.com> escribió: > El mejor consejo en este caso sería... no uses Python. Si el usuario > tiene acceso al bytecode, se puede descompilar. > Quizás esto sirva http://stackoverflow.com/questions/3694413/distributing-closed-source-python-program De todas maneras, en mi opinión personal, python *NO ES* el mejor lenguaje para aplicaciones que no sean open source. > 2012/10/29 Dhannier Molina : >> Hola!! Alguien me puede ayudar, tengo dudas en relacion a la seguridad en >> python. >> >> Hay alguna forma de impedir que el Bytecode generado al correr un programa >> en python sea decompilado? Es decir, que un intruso pueda generar el archivo >> .PY a partir del .PYC. >> >> Necesito blindar lo mejor posible mi sistema, que herramientas podría usar?. >> Saludos!! >> >> _______________________________________________ >> Python-es mailing list >> Python-es en python.org >> http://mail.python.org/mailman/listinfo/python-es >> FAQ: http://python-es-faq.wikidot.com/ >> > _______________________________________________ > Python-es mailing list > Python-es en python.org > http://mail.python.org/mailman/listinfo/python-es > FAQ: http://python-es-faq.wikidot.com/ -- Asdrúbal Iván Suárez Rivera Si quieres aprender, enseña. Marco Tulio Cicerón. ¿Tienes twitter? Sígueme: @asdrubalivan ------------ próxima parte ------------ Se ha borrado un adjunto en formato HTML... URL: From ricardo.cardenes en gmail.com Tue Oct 30 01:53:30 2012 From: ricardo.cardenes en gmail.com (=?ISO-8859-1?Q?Ricardo_C=E1rdenes?=) Date: Tue, 30 Oct 2012 00:53:30 +0000 Subject: [Python-es] Seguridad en Python In-Reply-To: References: Message-ID: Como le dicen, lo único que se puede conseguir con eso es dificultar el acceso al bytecode. Pero nada más. Una vez con acceso al código ya se puede descompilar. Su única opción realista es usar ofuscadores, pero eso dificulta la depuración de código en producción, entre otras cosas. 2012/10/30 Asdrúbal Iván Suárez Rivera : > > > El día 29 de octubre de 2012 20:11, Ricardo Cárdenes > escribió: > >> El mejor consejo en este caso sería... no uses Python. Si el usuario >> tiene acceso al bytecode, se puede descompilar. >> > > Quizás esto sirva > > http://stackoverflow.com/questions/3694413/distributing-closed-source-python-program > > De todas maneras, en mi opinión personal, python NO ES el mejor lenguaje > para aplicaciones que no sean open source. > > >> 2012/10/29 Dhannier Molina : >>> Hola!! Alguien me puede ayudar, tengo dudas en relacion a la seguridad en >>> python. >>> >>> Hay alguna forma de impedir que el Bytecode generado al correr un >>> programa >>> en python sea decompilado? Es decir, que un intruso pueda generar el >>> archivo >>> .PY a partir del .PYC. >>> >>> Necesito blindar lo mejor posible mi sistema, que herramientas podría >>> usar?. >>> Saludos!! >>> >>> _______________________________________________ >>> Python-es mailing list >>> Python-es en python.org >>> http://mail.python.org/mailman/listinfo/python-es >>> FAQ: http://python-es-faq.wikidot.com/ >>> >> _______________________________________________ >> Python-es mailing list >> Python-es en python.org >> http://mail.python.org/mailman/listinfo/python-es >> FAQ: http://python-es-faq.wikidot.com/ > > > > -- > Asdrúbal Iván Suárez Rivera > > Si quieres aprender, enseña. Marco Tulio Cicerón. > > ¿Tienes twitter? > Sígueme: @asdrubalivan > _______________________________________________ > Python-es mailing list > Python-es en python.org > http://mail.python.org/mailman/listinfo/python-es > FAQ: http://python-es-faq.wikidot.com/ > From jza en oooes.org Tue Oct 30 01:56:14 2012 From: jza en oooes.org (Alexandro Colorado) Date: Mon, 29 Oct 2012 18:56:14 -0600 Subject: [Python-es] Seguridad en Python In-Reply-To: References: Message-ID: 2012/10/29 Asdrúbal Iván Suárez Rivera < asdrubal.ivan.suarez.rivera en gmail.com> > > > El día 29 de octubre de 2012 20:11, Ricardo Cárdenes < > ricardo.cardenes en gmail.com> escribió: > > > El mejor consejo en este caso sería... no uses Python. Si el usuario > > tiene acceso al bytecode, se puede descompilar. > > > > Quizás esto sirva > > > http://stackoverflow.com/questions/3694413/distributing-closed-source-python-program > > De todas maneras, en mi opinión personal, python *NO ES* el mejor > lenguaje para aplicaciones que no sean open source. Se me hace completamente desvirtuado lo que esta persona entiende por seguridad. El acceso al codigo fuente no es una falla de seguridad. Muchos productos son abiertos como MoinMoin donde tu version de MoinMoin tendra el mismo codigo que el del repositorio de moinmoin, esto NO lo vuelve vulnerable. Primero debes entender cual es el estado de la inseguidad para aplicaciones web, como ataques DoS, Scripting de sitios cruzados etc. Y a partir de ahi generar capaz de seguridad. > > > > 2012/10/29 Dhannier Molina : > >> Hola!! Alguien me puede ayudar, tengo dudas en relacion a la seguridad > en > >> python. > >> > >> Hay alguna forma de impedir que el Bytecode generado al correr un > programa > >> en python sea decompilado? Es decir, que un intruso pueda generar el > archivo > >> .PY a partir del .PYC. > >> > >> Necesito blindar lo mejor posible mi sistema, que herramientas podría > usar?. > >> Saludos!! > >> > >> _______________________________________________ > >> Python-es mailing list > >> Python-es en python.org > >> http://mail.python.org/mailman/listinfo/python-es > >> FAQ: http://python-es-faq.wikidot.com/ > >> > > _______________________________________________ > > Python-es mailing list > > Python-es en python.org > > http://mail.python.org/mailman/listinfo/python-es > > FAQ: http://python-es-faq.wikidot.com/ > > > > -- > Asdrúbal Iván Suárez Rivera > > Si quieres aprender, enseña. Marco Tulio Cicerón. > > ¿Tienes twitter? > Sígueme: @asdrubalivan > _______________________________________________ > Python-es mailing list > Python-es en python.org > http://mail.python.org/mailman/listinfo/python-es > FAQ: http://python-es-faq.wikidot.com/ > > -- Alexandro Colorado PPMC Apache OpenOffice http://es.openoffice.org ------------ próxima parte ------------ Se ha borrado un adjunto en formato HTML... URL: From alfonsodg en gmail.com Tue Oct 30 01:52:28 2012 From: alfonsodg en gmail.com (Alfonso de la Guarda) Date: Mon, 29 Oct 2012 19:52:28 -0500 Subject: [Python-es] Seguridad en Python In-Reply-To: References: Message-ID: Hola, Lo que sí podrías hacer es combinar Python con C (cython) y las partes no críticas ofuscarlas...... O llevar Python a C++ con Nuitka http://www.nuitka.net/ o shedskin http://code.google.com/p/shedskin/ Saludos, -------------------------------- Alfonso de la Guarda Twitter: @alfonsodg Redes sociales: alfonsodg Telef. 991935157 1024D/B23B24A4 5469 ED92 75A3 BBDB FD6B 58A5 54A1 851D B23B 24A4 2012/10/29 Dhannier Molina : > Hola!! Alguien me puede ayudar, tengo dudas en relacion a la seguridad en > python. > > Hay alguna forma de impedir que el Bytecode generado al correr un programa > en python sea decompilado? Es decir, que un intruso pueda generar el archivo > .PY a partir del .PYC. > > Necesito blindar lo mejor posible mi sistema, que herramientas podría usar?. > Saludos!! > > _______________________________________________ > Python-es mailing list > Python-es en python.org > http://mail.python.org/mailman/listinfo/python-es > FAQ: http://python-es-faq.wikidot.com/ > From jza en oooes.org Tue Oct 30 01:59:23 2012 From: jza en oooes.org (Alexandro Colorado) Date: Mon, 29 Oct 2012 18:59:23 -0600 Subject: [Python-es] Seguridad en Python In-Reply-To: References: Message-ID: 2012/10/29 Dhannier Molina > Hola!! Alguien me puede ayudar, tengo dudas en relacion a la seguridad en > python. > > Hay alguna forma de impedir que el Bytecode generado al correr un programa > en python sea decompilado? Es decir, que un intruso pueda generar el > archivo .PY a partir del .PYC. > > Necesito blindar lo mejor posible mi sistema, que > herramientas podría usar?. Saludos!! > Creo que tienes conceptos errados de la seguridad. Te recomiendo que te heches un clavado en lo que se conoce como seguridad o quizas mejor dicho inseguridad. Cuando entiendas el riesgo (o falta de este) que se tiene por generar un Py a partir de un Pyc, podras entender como mejor proceder para asegurar un sistema. > > _______________________________________________ > Python-es mailing list > Python-es en python.org > http://mail.python.org/mailman/listinfo/python-es > FAQ: http://python-es-faq.wikidot.com/ > > -- Alexandro Colorado PPMC Apache OpenOffice http://es.openoffice.org ------------ próxima parte ------------ Se ha borrado un adjunto en formato HTML... URL: From mlacunza en gmail.com Tue Oct 30 02:01:36 2012 From: mlacunza en gmail.com (Mario Lacunza) Date: Mon, 29 Oct 2012 20:01:36 -0500 Subject: [Python-es] Seguridad en Python In-Reply-To: References: Message-ID: <508F26F0.8080404@gmail.com> Hola, uhm... puedes utilizar tecnicas de ofuscacion principalmente.... es decir aun si leen tu codigo no se entenderia nada.. algo como Source code: def ObtenerImpuesto(valor1): val = valor1 * 0,18 return val es diferente a Source code ofuscado: def sGhyshe4ksu(sh2f): dsy = sh2f * 0,18 return dsy como veras aqui solo se q estas multiplicando y si eres mosca sabras q puede ser porcentaje, si a esto le agregas la creacion de funciones falsas, llamadas repetitivas, loops, etc etc etc la hace aun mas dificil.... Ahora la idea es: si vas a crear un soft con 1000 lineas de codigo mejor create un EXE o similar para Linux/Mac, si tu soft va a atener 20,000 lineas de codigo les va a costar mas crakearlo, leerlas y entenderlas q pagarte :) depende mas del modelo de negocio q escojas. Saludos / Best regards Mario Lacunza Email:: mlacunza en gmail.com Personal Website:: http://www.lacunza.biz/ OpenOffice.org Perú:: http://openoffice-peru.com/ Hosting:: http://mlv-host.com/ Mascotas Perdidas:: http://mascotas-perdidas.com/ Google Talk / Y! messenger / Skype: mlacunzav MSN: mlacunzav en hotmail.com El 29/10/12 19:46, Asdrúbal Iván Suárez Rivera escribió: > > > El día 29 de octubre de 2012 20:11, Ricardo Cárdenes > > escribió: > > El mejor consejo en este caso sería... no uses Python. Si el usuario > > tiene acceso al bytecode, se puede descompilar. > > > > Quizás esto sirva > > http://stackoverflow.com/questions/3694413/distributing-closed-source-python-program > > De todas maneras, en mi opinión personal, python *_NO ES_* el mejor > lenguaje para aplicaciones que no sean open source. > > > 2012/10/29 Dhannier Molina >: > >> Hola!! Alguien me puede ayudar, tengo dudas en relacion a la > seguridad en > >> python. > >> > >> Hay alguna forma de impedir que el Bytecode generado al correr un > programa > >> en python sea decompilado? Es decir, que un intruso pueda generar > el archivo > >> .PY a partir del .PYC. > >> > >> Necesito blindar lo mejor posible mi sistema, que herramientas > podría usar?. > >> Saludos!! > >> > >> _______________________________________________ > >> Python-es mailing list > >> Python-es en python.org > >> http://mail.python.org/mailman/listinfo/python-es > >> FAQ: http://python-es-faq.wikidot.com/ > >> > > _______________________________________________ > > Python-es mailing list > > Python-es en python.org > > http://mail.python.org/mailman/listinfo/python-es > > FAQ: http://python-es-faq.wikidot.com/ > > > > -- > Asdrúbal Iván Suárez Rivera > > Si quieres aprender, enseña. Marco Tulio Cicerón. > > ¿Tienes twitter? > Sígueme: @asdrubalivan > > > _______________________________________________ > Python-es mailing list > Python-es en python.org > http://mail.python.org/mailman/listinfo/python-es > FAQ: http://python-es-faq.wikidot.com/ ------------ próxima parte ------------ Se ha borrado un adjunto en formato HTML... URL: From jza en oooes.org Tue Oct 30 02:09:39 2012 From: jza en oooes.org (Alexandro Colorado) Date: Mon, 29 Oct 2012 19:09:39 -0600 Subject: [Python-es] Estructuras de datos In-Reply-To: References: Message-ID: 2012/10/28 Chema Cortes > El día 28 de octubre de 2012 00:32, Antonio Cascales > escribió: > > > He empezado a leer los distintos recursos acerca de Python que me > > recomendásteis, y al leer los distintos tipos de datos que existen en > > Python, me surge una duda. > > > > Veo que los tipos básicos son los mismos prácticamente, pero sin > embargo, > > las colecciones de Python como los diccionarios, listas, etc, me suenan > más > > a los Arrays, pero ninguna a las estructuras propiamente dichas de C/C++. > > Las listas en python son conjuntos de datos ordenados, pero no son lo > mismo. > > Y los diccionarios o tuplas, menos. > > > > Me gustaría saber si existe alguna manera en Python, de almacenar en > una > > estructura, por ejemplo, los datos de una persona, para crear una agenda > de > > contactos. ¿Cómo podría conseguirse esto? > > Es una pregunta frecuente que puedes ver en la FAQ: > > http://python-es-faq.wikidot.com/#toc37 > > En resumidas cuentas, las estructuras de C/C++ tienen poco sentido en > python sin poder especificar el tipo de cada campo. O empleas listas o > empleas clases. > > Si prefieres probar otra cosa, tienes "namedtuples": > > from collections import namedtuple > > Empleado = namedtuple('Empleado', 'nombre, edad, departamento, categoria') > > emp1=Empleado("Martinez", 38, "Informática", "Programador Python") > > print emp1.categoria > Que diferencia hay con collection.abc http://docs.python.org/dev/library/collections.abc.html > print emp1 > > El problema de namedtuple es que es "inmutable". Para mutabilidad, > puedes mirar la siguiente receta: > > http://code.activestate.com/recipes/578041-namedlist/ > > > > -- > Hyperreals *R: http://ch3m4.org/blog > Quarks, bits y otras criaturas infinitesimales > _______________________________________________ > Python-es mailing list > Python-es en python.org > http://mail.python.org/mailman/listinfo/python-es > FAQ: http://python-es-faq.wikidot.com/ > -- Alexandro Colorado PPMC Apache OpenOffice http://es.openoffice.org ------------ próxima parte ------------ Se ha borrado un adjunto en formato HTML... URL: From jza en oooes.org Tue Oct 30 02:16:49 2012 From: jza en oooes.org (Alexandro Colorado) Date: Mon, 29 Oct 2012 19:16:49 -0600 Subject: [Python-es] Video Promocional PyConVE In-Reply-To: References: Message-ID: 2012/10/25 Francisco Palm > > 2012/10/25 Alexandro Colorado > >> Caray la gente de Venezuela siempre es mucho de comunidad, me da mucho >> gusto que se tomen iniciativas como estas en todo latinoamerica. >> >> Sin embargo tambien tengo ganas que se rompa esa barrera de adoptar >> tecnologias como usuarios (usuarios de lenguajes en este caso), y >> pasemos al frente como generadores de estas tecnologias. > > > Amigo, la reflexión es válida y bienvenida. Sin embargo, no es un tema > trivial, es difícil para mi encontrar mejor referencia al respecto que el > libro "Invention: The Care and Feeding of Ideas" de Norbert Wiener. > "Generar tecnología", parte de entender profundamente el hecho tecnológico > y crear las condiciones o "climas" para que esto sea posible, lo cuál no es > de ningún modo un tema de esfuerzo individual, ni siquiera el trabajo de > pequeños colectivos, se trata de cocinar un denso y rico caldo de cultivo. > > Basta analizar antropológica y sociológicamente desde que entornos > aparecieron personas como Linus Torvalds o Guido Van Rossum. > > Una de las principales trabas para lograr esto es la insistencia en los > grupos técnicos de no hablar de filosofía ni política ni ideologías, como > si el mantener a la tecnología impermeabilizada a todo esto no fuese otra > forma de ideología muy particular y limitada que la castra y la condiciona > con severidad. > > Pienso que Internet y la Sociedad Red pueden hacer mucho en este sentido, > pero mientras que los problemas sociales-políticos entre unos y otros > permanezcan invisibilizados por causa del discurso tecnicista predominante > es realmente muy difícil que en virtud de las restricciones propias de > nuestras sociedades haya un avance significativo. > No entendi ni una palabra de lo que dijiste. Pero creo que el simple proceso de tener una idea, y traducirla a codigo duro es suficieinte para crear un proyecto. Si la idea es muy buena, el proyecto cobrara vida, aunque, no por si solo, falta que se promocione en los grupos adecuados y de la forrma adecuada. > > Disculpen lo que para muchos será un OFF TOPIC, si acaso alguien quiere > hablar del tema puede escribirme en privado, por mi parte el hilo se acaba > acá. > > Saludos > > F. Palm > > -- > -------------------------------------- > fpalm en mapologo.org.ve > francisco.palm en gmail.com > > cel: +58 +424 7228252 > tel: +58 +274 6352001 > > ---- > Debemos ser libres, no para hacer lo que nos plazca, sino libres para > comprender muy profundamente nuestros propios instintos e impulsos. K > > _______________________________________________ > Python-es mailing list > Python-es en python.org > http://mail.python.org/mailman/listinfo/python-es > FAQ: http://python-es-faq.wikidot.com/ > > -- Alexandro Colorado PPMC Apache OpenOffice http://es.openoffice.org ------------ próxima parte ------------ Se ha borrado un adjunto en formato HTML... URL: From davidmenhur en gmail.com Tue Oct 30 08:39:38 2012 From: davidmenhur en gmail.com (=?UTF-8?B?RGHPgGlk?=) Date: Tue, 30 Oct 2012 08:39:38 +0100 Subject: [Python-es] Estructuras de datos In-Reply-To: References: Message-ID: 2012/10/28 Rock Neurotiko : > Creas una clase persona, y en una lista, diccionario, o lo que se tercie, > como datos estarian objetos de esa clase. Y si lo que quieres es un contenedor de datos uniforme (donde el tipo venga predefinido), más parecido a C++, puedes usar los struct arrays de NumPy. Añades una dependencia, pero las operaciones son rapidísimas. From pych3m4 en gmail.com Tue Oct 30 09:22:27 2012 From: pych3m4 en gmail.com (Chema Cortes) Date: Tue, 30 Oct 2012 09:22:27 +0100 Subject: [Python-es] Estructuras de datos In-Reply-To: References: Message-ID: El día 30 de octubre de 2012 02:09, Alexandro Colorado escribió: > Que diferencia hay con collection.abc > http://docs.python.org/dev/library/collections.abc.html Las ABC son "clases abstractas" que conforman el "modelo de datos" de python o, tal como se entiende en otros lenguajes no-dinámicos, el sistema de tipado de datos. No se pueden usar directamente para crear instancias, tan sólo para derivar nuevas clases. Su principal uso es en el chequeo de tipos de datos ("introspección de tipos"[1]). Lo que no sé es qué ves de similar. Tal vez no entiendo tu pregunta. [1]: http://en.wikipedia.org/wiki/Type_introspection -- Hyperreals *R: http://ch3m4.org/blog Quarks, bits y otras criaturas infinitesimales From lasizoillo en gmail.com Tue Oct 30 09:26:22 2012 From: lasizoillo en gmail.com (lasizoillo) Date: Tue, 30 Oct 2012 09:26:22 +0100 Subject: [Python-es] Seguridad en Python In-Reply-To: References: Message-ID: El día 29 de octubre de 2012 20:29, Dhannier Molina escribió: > Hola!! Alguien me puede ayudar, tengo dudas en relacion a la seguridad en > python. > > Hay alguna forma de impedir que el Bytecode generado al correr un programa > en python sea decompilado? Es decir, que un intruso pueda generar el archivo > .PY a partir del .PYC. > > Necesito blindar lo mejor posible mi sistema, que herramientas podría usar?. > Saludos!! > En mis tiempos de crackercillo solia usar algún que otro disassembler[1] para obtener el código fuente ensamblador y cackear los programas. En una primera pasada anulaba las protecciones anti-debugging y luego con el debugger era más fácil encontrar las partes del código "interesante". Puede que el programador que comprimia los ejecutables, ponía anti-debuggers en el código, ... creyese que hacía más seguro el código. Pero hasta un adolescente con granos como yo sabía que si lo puede comprender una máquina, también lo puede comprender un humano. La única manera de "asegurar" tu código es no compartirlo. Y esta es una condición necesaria, pero no suficiente, tienes que asegurar que nadie pueda acceder a el a pesar de que no lo compartas. Solo se me ocurren dos opciones aquí: - O dejas de asociar seguridad a la disponibilidad del código fuente. - O implementas la lógica que quieras "asegurar" en servidores que ejecutan el código mediante procedimientos RPC y securizas los servidores. Para el resto de cosas, lo único que vas a hacer es perder más tiempo tu del que vas a hacer perder a quien realmente quiera hacer ingeniería inversa de tu código. Un código limpio es más fácil de auditar, securizar y depurar que un código con herramientas de ofuscación, compresión, anti-debuging que dificultan la depuración y en ocasiones introducen bugs en la ejecución. [1] http://en.wikipedia.org/wiki/Disassembler Un saludo, Javi From eventgrafic en gmail.com Tue Oct 30 11:54:35 2012 From: eventgrafic en gmail.com (Eduard Diaz) Date: Tue, 30 Oct 2012 11:54:35 +0100 Subject: [Python-es] Seguridad en Python In-Reply-To: References: Message-ID: Hola Creo que el problema está en el concepto de seguridad, tal y como te han contestado anteriormente. El bitecode y python no es una buena elección para ese tipo de requisitos (primera vulnerabilidad) Como mucho podrias utilizar pirex[1] o directamente C para proteger según que partes Los egipcios hicieron tumbas cubriéndolas con toneladas de piedras en formas de pirámides, en desiertos casi inaccesibles y enterrando a todos los que conocían la existencia de las tumbas y aún así fueron saqueadas! Ese modelo de seguridad hace miles de años que se ha demostrado que no funciona. [1] http://www.cosc.canterbury.ac.nz/greg.ewing/python/Pyrex/ un saludo From antonio.cascales en gmail.com Tue Oct 30 12:08:22 2012 From: antonio.cascales en gmail.com (Antonio Cascales) Date: Tue, 30 Oct 2012 12:08:22 +0100 Subject: [Python-es] Estructuras de datos In-Reply-To: References: Message-ID: Hola a todos: Gracias por las distintas respuestas. me he guardado todos los mails, para futuras consultas, pero actualmente, con lo verde que estoy todavía, cualquiera de ellas me parecen soluciones imposibles para mí. he leído el tema de las clases en python, y aunque llego a entender algunos conceptos y ayudándome de los ejemplos he podido copiar el funcionamiento, creo que todavía estoy lejos de conseguir entender las clases en sí, y clases sencillas, como para meterme ya en cosas tan complicadas como las que comentáis. De hecho, todavía hay conceptos que se me escapan de las clases, o de cosas más básicas como los diccionarios, las listas o las funciones, pese que estas últimas se entienden mucho mejor que en C. La pena es que en ninguno de los recursos que he visto por internet, hay propuestas de ejercicios para ponernos a prueba con los conocimientos que se van adquiriendo. O al menos yo no he llegado a ver ninguno. Os agradezco de nuevo la ayuda. Voy a pegarle algún repaso más a los recursos que tengo, y ya os iré preguntando cositas más sencillas para afianzar conocimientos. Un saludo, El 30/10/12, Chema Cortes escribió: > El día 30 de octubre de 2012 02:09, Alexandro Colorado > escribió: > >> Que diferencia hay con collection.abc >> http://docs.python.org/dev/library/collections.abc.html > > Las ABC son "clases abstractas" que conforman el "modelo de datos" de > python o, tal como se entiende en otros lenguajes no-dinámicos, el > sistema de tipado de datos. No se pueden usar directamente para crear > instancias, tan sólo para derivar nuevas clases. Su principal uso es > en el chequeo de tipos de datos ("introspección de tipos"[1]). > > Lo que no sé es qué ves de similar. Tal vez no entiendo tu pregunta. > > > [1]: http://en.wikipedia.org/wiki/Type_introspection > > -- > Hyperreals *R: http://ch3m4.org/blog > Quarks, bits y otras criaturas infinitesimales > _______________________________________________ > Python-es mailing list > Python-es en python.org > http://mail.python.org/mailman/listinfo/python-es > FAQ: http://python-es-faq.wikidot.com/ > -- Antonio Cascales Sánchez From kikocorreoso en gmail.com Tue Oct 30 12:22:26 2012 From: kikocorreoso en gmail.com (Kiko) Date: Tue, 30 Oct 2012 12:22:26 +0100 Subject: [Python-es] Estructuras de datos In-Reply-To: References: Message-ID: El 30 de octubre de 2012 12:08, Antonio Cascales escribió: > Hola a todos: > > Gracias por las distintas respuestas. me he guardado todos los > mails, para futuras consultas, pero actualmente, con lo verde que > estoy todavía, cualquiera de ellas me parecen soluciones imposibles > para mí. he leído el tema de las clases en python, y aunque llego a > entender algunos conceptos y ayudándome de los ejemplos he podido > copiar el funcionamiento, creo que todavía estoy lejos de conseguir > entender las clases en sí, y clases sencillas, como para meterme ya en > cosas tan complicadas como las que comentáis. De hecho, todavía hay > conceptos que se me escapan de las clases, o de cosas más básicas como > los diccionarios, las listas o las funciones, pese que estas últimas > se entienden mucho mejor que en C. > > La pena es que en ninguno de los recursos que he visto por > internet, hay propuestas de ejercicios para ponernos a prueba con los > conocimientos que se van adquiriendo. O al menos yo no he llegado a > ver ninguno. > En este libro, por ejemplo, sí que tienes ejercicios: http://eu.wiley.com/WileyCDA/WileyTitle/productCd-0470414634.html En codeacademy tienes cursos de python (básicos): http://www.codecademy.com/es/tracks/python Udacity, coursera,... Hoy en día hay muchos recursos. Coge uno y sigue uno. Escribe mucho código y mira el código de los demás (github, bitbucket,...). No te vuelvas loco a leer. Por otra parte, tienes esta charla de la pycon US de este año donde se comenta que se abusa del uso de las clases: http://pyvideo.org/video/880/stop-writing-classes :-) ------------ próxima parte ------------ Se ha borrado un adjunto en formato HTML... URL: From gmourinopardo en yahoo.es Tue Oct 30 14:40:15 2012 From: gmourinopardo en yahoo.es (=?iso-8859-1?Q?Gonzalo_Mouri=FFfffffffffff1o_Pardo?=) Date: Tue, 30 Oct 2012 13:40:15 +0000 (GMT) Subject: [Python-es] Estructuras de datos In-Reply-To: References: Message-ID: <1351604415.91591.YahooMailNeo@web171701.mail.ir2.yahoo.com> Buenos días, llevo más de una semana intentando cerrar la ventana del código que adjunto, no se si es porque los eventos no se paran o porque hay algún comando que no me se, pido ayuda a algún alm a caritativa. Garcias. > #!/usr/bin/env python ># -*- coding: cp1252 -*- > >> >import os >import wx > >> >import day_data > >> ># News Identifiers >ID_SALIR = wx.NewId() > >> >class Aplicacion(wx.App): > >> >    def OnInit(self): >        self.frame = Ventana(None, -1, title = "Control Horario") >        self.SetTopWindow(self.frame) >        self.frame.Show() >        return True > >> >"""Main Panel""" >class Panel(wx.Panel): >    def __init__(self, parent): >        wx.Panel.__init__(self, parent) >        self.month = "Octubre" >        self.InitUI() > >> >    """Horario's GUI""" >    def InitUI(self, month = None): >        font = wx.Font(15, family = wx.FONTFAMILY_DEFAULT, style = >                        wx.FONTSTYLE_NORMAL, weight = wx.FONTWEIGHT_BOLD, >                        encoding = wx.FONTENCODING_CP1252) >        self.month = month > >> >        # Layout >        self.sizer = wx.BoxSizer(wx.HORIZONTAL) >        self.gbsizer = wx.GridBagSizer(2, 2) >        self.sizer.Add(self.gbsizer, 1, wx.ALL | wx.EXPAND, border = 6) >        self.SetSizer(self.sizer) > >> >        # Column's Labels >        sem = {0 : ("Lunes", 169), 1 : ("Martes", 266), 2 : ("Miercoles" , 359), >               3 : ("Jueves",  478), 4 : ("Viernes", 578), 5: ("Sabado", 686), >               6 : ("Domingo", 783)} >        for num in range(0,7): >            self.cont = sem[num] >            dia = self.cont[0] >            y = num + 1 >            self.collab = wx.StaticText(self, -1, label = dia,  size = wx.Size >                                        (103, 23), style = wx.ALIGN_CENTER) >            self.collab.SetFont(font) >            if y < 6: >                self.collab.SetBackgroundColour(wx.Colour(255, 255, 255)) >            else: >                self.collab.SetBackgroundColour(wx.Colour(255, 0, 0)) > >> >            self.gbsizer.Add(self.collab, pos = wx.GBPosition(0, y), flag = >                        wx.ALIGN_CENTER_VERTICAL, border = 6) > >> >        # Month's Label >        self.mes = "Octubre" >        self.monlab = wx.StaticText(self, -1, label = self.mes, size = wx.Size >                                    (138, 23), style = wx.ALIGN_CENTER) >        self.monlab.SetFont(font) >        self.monlab.SetForegroundColour(wx.Colour(255, 0, 255)) >        self.gbsizer.Add(self.monlab, pos = wx.GBPosition(0, 0), flag = >                    wx.ALIGN_CENTER_VERTICAL,  border = 6) > >> >        # Row's Labels >        self.month_name = "Oct" >        num_month = 41 >        numero = str(num_month) >        self.week_range = 5 >        for num in range(6): >            x = num + 1 >            if x > self.week_range: >                numero = "" >                number = 7 >            else: >                number = int(numero) + 1 >            self.rowlab = wx.StaticText(self, -1, label = numero, size = wx.Size >                                        (138, 23), style = wx.ALIGN_CENTER) >            self.rowlab.SetFont(font) >            self.rowlab.SetForegroundColour(wx.Colour(0, 0, 255)) >            self.rowlab.SetBackgroundColour(wx.Colour(255, 255, 255)) >            if x > self.week_range: >                self.rowlab.SetBackgroundColour(wx.NullColor) >            self.gbsizer.Add(self.rowlab, pos = wx.GBPosition(x, 0), flag = >                        wx.ALIGN_CENTER_VERTICAL, border = 6) >            numero = str(number) > >> >        # Day's Labels >        day = "Mon" >        end_dict = 31 >        self.daylab_dict = {} >        if day == "Mon": >            frest = 0 >        elif day == "Tue": >            frest = 1 >        elif day == "Wed": >            frest = 2 >        elif day == "Thu": >            frest = 3 >        elif day == "Fri": >            frest = 4 >        elif day == "Sat": >            frest = 5 >        else: >            frest = 6 >        x = 1 >        y = 1 >        rest = frest >        mday = 42 >        erest = (self.week_range * 7) - end_dict - rest >        for cont in range ((mday + 1)): >            if frest > 0: >                contador = "" >                frest = frest - 1 >            elif cont >= (rest + end_dict): >                contador = "" >                erest = erest - 1 >            else: >                contador = str(cont - rest + 1) >            if y > 7: >                x = x + 1 >                y = 1 >            self.daylab = wx.StaticText(self, label = contador, size = wx.Size >                                        (103, 23), style = wx.ALIGN_CENTRE) >            self.daylab.SetFont(font) >            self.daylab.SetBackgroundColour(wx.Colour(255, 255, 255)) >            if y > 5: >                self.daylab.SetBackgroundColour(wx.Colour(255, 0, 0)) >            if x > self.week_range: >                self.daylab.SetBackgroundColour(wx.NullColor) >            self.daylab_dict [(y, x)] = contador >            self.gbsizer.Add(self.daylab, pos = wx.GBPosition(x, y), flag = >                             wx.ALIGN_CENTER_VERTICAL, border = 6) >            y = y + 1 >            self.daylab.Bind(wx.EVT_LEFT_DOWN, self.OnLeftDown) > >> >        self.Bind(wx.EVT_PAINT,self.OnPaint) > >> >    """Do mouse events""" >    def OnLeftDown(self, event): >        mou_pos = wx.GetMousePosition() >        scr_pos = self.GetScreenPosition() >        abs_x = mou_pos.x - scr_pos.x >        abs_y = mou_pos.y - scr_pos.y >        if abs_x < 250: >            coor_x = 1 >        elif abs_x >= 250 and abs_x < 355: >            coor_x = 2 >        elif abs_x >= 355 and abs_x < 460: >            coor_x = 3 >        elif abs_x >= 460 and abs_x < 565: >            coor_x = 4 >        elif abs_x >= 565 and abs_x < 670: >            coor_x = 5 >        elif abs_x >= 670 and abs_x < 775: >            coor_x = 6 >        else: >            coor_x = 7 >        if abs_y < 55: >            coor_y = 1 >        elif abs_y >= 55 and abs_y < 80: >            coor_y = 2 >        elif abs_y >= 80 and abs_y < 105: >            coor_y = 3 >        elif abs_y >= 105 and abs_y < 130: >            coor_y = 4 >        elif abs_y >= 130 and abs_y < 155: >            coor_y = 5 >        else: >            coor_y = 6 >        self.dia = self.daylab_dict[coor_x, coor_y] >         >        if str(self.dia) != "": >            self.day = ("Día " + str(self.dia) + " - " + self.mes + " - " + >                        "2012") >            print(self.day) >            #self.frame = day_data.Ventana(None, title = self.day) >            #self.frame.Bind(wx.EVT_CLOSE,self.OnCloseLeftDown) >            #self.frame.Show(True) > >> > >> >    """Do paint events""" >    def OnPaint(self, event): >        self.dc = wx.PaintDC(self) >        for row in range((self.week_range + 1)): >            rect_y = 5 + (25 * row) >            for colu in range (8): >                if colu == 0: >                    rect_x = 5 >                    width = 140 >                else: >                    rect_x = 145 + (105 * (colu - 1)) >                    width = 105 >                if rect_x == 5 and rect_y == 5: >                    a = 3 >                else: >                    self.dc.DrawRectangle(rect_x, rect_y, width, 25) > >> >    """Close's event""" >    def OnCloseLeftDown(self, event): >        self.ihour = self.frame.panel.tc1a.GetValue() >        self.ohour = self.frame.panel.tc2a.GetValue() >        print(self.ihour) >        print(self.ohour) >        self.frame.Destroy() >        self.Refresh() > >> >"""Main Frame""" >class Ventana(wx.Frame): >     >    """Constructor""" >    def __init__(self, parent, id, title): >        super(Ventana, self).__init__(parent, id, title, size = (900, 275)) > >> >        # Attributes >        self.panel = Panel(self) > >> >        # Setup >        dir_pa = os.getcwd() >        dir_hi_1 = os.path.split(dir_pa) >        dir_hi_2 = dir_hi_1[0] >        path = os.path.join(dir_hi_2, "Iconos/ICO.png") >        icon = wx.Icon(path, wx.BITMAP_TYPE_PNG) >        self.SetIcon(icon) > >> >        # Setup Menu >        menubar = wx.MenuBar() > >> >        # File's Menu >        arcm = wx.Menu() >        asmi = wx.MenuItem(arcm, ID_SALIR, "&Salir\tCtrl+S") >        arcm.AppendItem(asmi) >        menubar.Append(arcm, '&Archivo') >        self.SetMenuBar(menubar) > >> >        # Event Handlers >        self.Bind(wx.EVT_MENU, self.OnMenu) > >> >    """Do menu events""" >    def OnMenu(self, event): >        self.evt_id = event.GetId () >        if self.evt_id == ID_SALIR: >            self.Close() >             >class BaseDatos(): >    def __init__ (self): >        print ("Base Datos (En curso)") > >> >if __name__ == "__main__": >    app = Aplicacion(False) >    app.MainLoop() > > ------------ próxima parte ------------ Se ha borrado un adjunto en formato HTML... URL: From carlos.zun en gmail.com Tue Oct 30 15:30:25 2012 From: carlos.zun en gmail.com (Carlos Zuniga) Date: Tue, 30 Oct 2012 09:30:25 -0500 Subject: [Python-es] Seguridad en Python In-Reply-To: References: Message-ID: 2012/10/29 Dhannier Molina : > Hola!! Alguien me puede ayudar, tengo dudas en relacion a la seguridad en > python. > > Hay alguna forma de impedir que el Bytecode generado al correr un programa > en python sea decompilado? Es decir, que un intruso pueda generar el archivo > .PY a partir del .PYC. > > Necesito blindar lo mejor posible mi sistema, que herramientas podría usar?. > Saludos!! > Creo que es mucho más seguro escribir un programa con la expectativa de que cualquiera podría tener acceso al código fuente en cualquier momento. La seguridad por obscuridad solo es aparente. Si lo que quieres es evitar que puedan modificar el código, eso es un simple problema de permisos. Saludos -- A menudo unas pocas horas de "Prueba y error" podrán ahorrarte minutos de leer manuales. From dhannier en gmail.com Tue Oct 30 15:42:06 2012 From: dhannier en gmail.com (Dhannier Molina) Date: Tue, 30 Oct 2012 10:12:06 -0430 Subject: [Python-es] Seguridad en Python In-Reply-To: References: Message-ID: Hola!! A que te refieres con un simple problema de permisos? Estoy evaluando varias alternativas como Cython, PyPy, Pyrex y Ofuscación para proteger lo mejor posible mi sistema hecho en Python. Y evaluar en que lenguaje de programación hacer un modulo de seguridad a nivel de usuario y perfiles. (Python u otro lenguaje que pueda ligarlo a Python). Saludos!! El 30 de octubre de 2012 10:00, Carlos Zuniga escribió: > 2012/10/29 Dhannier Molina : > > Hola!! Alguien me puede ayudar, tengo dudas en relacion a la seguridad en > > python. > > > > Hay alguna forma de impedir que el Bytecode generado al correr un > programa > > en python sea decompilado? Es decir, que un intruso pueda generar el > archivo > > .PY a partir del .PYC. > > > > Necesito blindar lo mejor posible mi sistema, que herramientas podría > usar?. > > Saludos!! > > > > Creo que es mucho más seguro escribir un programa con la expectativa > de que cualquiera podría tener acceso al código fuente en cualquier > momento. La seguridad por obscuridad solo es aparente. > > Si lo que quieres es evitar que puedan modificar el código, eso es un > simple problema de permisos. > > Saludos > -- > A menudo unas pocas horas de "Prueba y error" podrán ahorrarte minutos > de leer manuales. > _______________________________________________ > Python-es mailing list > Python-es en python.org > http://mail.python.org/mailman/listinfo/python-es > FAQ: http://python-es-faq.wikidot.com/ > ------------ próxima parte ------------ Se ha borrado un adjunto en formato HTML... URL: From jza en oooes.org Tue Oct 30 15:45:42 2012 From: jza en oooes.org (Alexandro Colorado) Date: Tue, 30 Oct 2012 08:45:42 -0600 Subject: [Python-es] Seguridad en Python In-Reply-To: References: Message-ID: On 10/30/12, Dhannier Molina wrote: > Hola!! A que te refieres con un simple problema de permisos? Estoy > evaluando varias alternativas como Cython, PyPy, Pyrex y Ofuscación para > proteger lo mejor posible mi sistema hecho en Python. > > Y evaluar en que lenguaje de programación hacer un modulo de seguridad a > nivel de usuario y perfiles. (Python u otro lenguaje que pueda ligarlo a > Python). A que estas perdiendo el tiempo tratando de ofuscar el codigo. Esa tecnica NO funciona. La seguridad por obscuridad (que nadie lo vea) esta comprobadisimo que no sirve. Windows puede tener las fuentes altamente protegidas, sin embargo es lo mas inseguro que existe. > > Saludos!! > > El 30 de octubre de 2012 10:00, Carlos Zuniga > escribió: > >> 2012/10/29 Dhannier Molina : >> > Hola!! Alguien me puede ayudar, tengo dudas en relacion a la seguridad >> > en >> > python. >> > >> > Hay alguna forma de impedir que el Bytecode generado al correr un >> programa >> > en python sea decompilado? Es decir, que un intruso pueda generar el >> archivo >> > .PY a partir del .PYC. >> > >> > Necesito blindar lo mejor posible mi sistema, que herramientas podría >> usar?. >> > Saludos!! >> > >> >> Creo que es mucho más seguro escribir un programa con la expectativa >> de que cualquiera podría tener acceso al código fuente en cualquier >> momento. La seguridad por obscuridad solo es aparente. >> >> Si lo que quieres es evitar que puedan modificar el código, eso es un >> simple problema de permisos. >> >> Saludos >> -- >> A menudo unas pocas horas de "Prueba y error" podrán ahorrarte minutos >> de leer manuales. >> _______________________________________________ >> Python-es mailing list >> Python-es en python.org >> http://mail.python.org/mailman/listinfo/python-es >> FAQ: http://python-es-faq.wikidot.com/ >> > -- Alexandro Colorado PPMC Apache OpenOffice http://es.openoffice.org From carlos.zun en gmail.com Tue Oct 30 16:10:10 2012 From: carlos.zun en gmail.com (Carlos Zuniga) Date: Tue, 30 Oct 2012 10:10:10 -0500 Subject: [Python-es] Seguridad en Python In-Reply-To: References: Message-ID: 2012/10/30 Dhannier Molina : > Hola!! A que te refieres con un simple problema de permisos? A que si no quieres que el usuario modifique el programa, quítale los permisos de escritura: chmod -R -w directorio/del/programa El usuario solo debería ser capaz de leer y ejecutar el programa, nada más. > Estoy evaluando > varias alternativas como Cython, PyPy, Pyrex y Ofuscación para proteger lo > mejor posible mi sistema hecho en Python. Creo que la pregunta es, protegerlo de qué? si es un sistema web y quieres protegerlo de ataques de inyección de sql, xss, etc; ofuscarlo no te ayudará, mejor mira herramientas de tests de penetración como w3af. Si es un sistema de escritorio y quieres evitar que el usuario modifique el programa, los permisos son más que suficiente. Si quieres evitar que un intruso pueda modificar tu programa en un servidor, entonces tienes que proteger al servidor. > Y evaluar en que lenguaje de programación hacer un modulo de seguridad a > nivel de usuario y perfiles. (Python u otro lenguaje que pueda ligarlo a > Python). > Eso ya queda dentro de tu aplicación, es mejor escribir los permisos de manera que aún asumiendo que el usuario pueda leer el código fuente, esto no represente un problema de seguridad en tu aplicación. Saludos -- A menudo unas pocas horas de "Prueba y error" podrán ahorrarte minutos de leer manuales. From jza en oooes.org Tue Oct 30 16:20:56 2012 From: jza en oooes.org (Alexandro Colorado) Date: Tue, 30 Oct 2012 09:20:56 -0600 Subject: [Python-es] Seguridad en Python In-Reply-To: References: Message-ID: On 10/30/12, Carlos Zuniga wrote: > 2012/10/30 Dhannier Molina : >> Hola!! A que te refieres con un simple problema de permisos? > > A que si no quieres que el usuario modifique el programa, quítale los > permisos de escritura: > > chmod -R -w directorio/del/programa > > El usuario solo debería ser capaz de leer y ejecutar el programa, nada más. Asi es, es importante saber que es lo que el quiere hacer. Si es una respuesta tan vaga como "no quiero que me hackeen". Entonces creo que debera hacer mas trabajo de investigacion. Otra cosa es que tenga inteligencia "hardcoded" o programada en duro dentro del codigo. > >> Estoy evaluando >> varias alternativas como Cython, PyPy, Pyrex y Ofuscación para proteger >> lo >> mejor posible mi sistema hecho en Python. La implementacion de Python, no es muy relevante, aunque si, en Jython podrias tener el codigo dentro de un war o jar y hacerlo 'portable' desde la JVM. El entendido es que el codigo en si sea seguro, no ofuscarlo. > > Creo que la pregunta es, protegerlo de qué? si es un sistema web y > quieres protegerlo de ataques de inyección de sql, xss, etc; ofuscarlo > no te ayudará, mejor mira herramientas de tests de penetración como > w3af. > > Si es un sistema de escritorio y quieres evitar que el usuario > modifique el programa, los permisos son más que suficiente. > > Si quieres evitar que un intruso pueda modificar tu programa en un > servidor, entonces tienes que proteger al servidor. > >> Y evaluar en que lenguaje de programación hacer un modulo de seguridad a >> nivel de usuario y perfiles. (Python u otro lenguaje que pueda ligarlo a >> Python). >> > > Eso ya queda dentro de tu aplicación, es mejor escribir los permisos > de manera que aún asumiendo que el usuario pueda leer el código > fuente, esto no represente un problema de seguridad en tu aplicación. > > Saludos > -- > A menudo unas pocas horas de "Prueba y error" podrán ahorrarte minutos > de leer manuales. > _______________________________________________ > Python-es mailing list > Python-es en python.org > http://mail.python.org/mailman/listinfo/python-es > FAQ: http://python-es-faq.wikidot.com/ > -- Alexandro Colorado PPMC Apache OpenOffice http://es.openoffice.org From diego.uribe.gamez en gmail.com Tue Oct 30 16:41:17 2012 From: diego.uribe.gamez en gmail.com (Diego Uribe Gamez) Date: Tue, 30 Oct 2012 10:41:17 -0500 Subject: [Python-es] Seguridad en Python In-Reply-To: References: Message-ID: Super interesante el tema, yo creo que la mejor seguridad que puedes hacer es a nivel de maquina, permisos de usuario y de mas, podrías también usar multimaquinas para hacer mas profundo la entrada al programa, como un servidor centos con maquinas virtuales redhat que entre si tengan toda tu app por separado con claves separadas y todo lo demás en seguridad de redes, esto es lo mas complicado que he logrado para que no entren al codigo. pero si el programa se ejecuta en local y el usuario puede leerlo, lo que podrías hacer es hacer que el programa consulte un servidor externo con el código esencial y que nadie mas que tu puedan entrar a verlo o leerlo. esa seria mi solución. El 30 de octubre de 2012 10:20, Alexandro Colorado escribió: > On 10/30/12, Carlos Zuniga wrote: > > 2012/10/30 Dhannier Molina : > >> Hola!! A que te refieres con un simple problema de permisos? > > > > A que si no quieres que el usuario modifique el programa, quítale los > > permisos de escritura: > > > > chmod -R -w directorio/del/programa > > > > El usuario solo debería ser capaz de leer y ejecutar el programa, nada > más. > > Asi es, es importante saber que es lo que el quiere hacer. Si es una > respuesta tan vaga como "no quiero que me hackeen". Entonces creo que > debera hacer mas trabajo de investigacion. Otra cosa es que tenga > inteligencia "hardcoded" o programada en duro dentro del codigo. > > > > >> Estoy evaluando > >> varias alternativas como Cython, PyPy, Pyrex y Ofuscación para proteger > >> lo > >> mejor posible mi sistema hecho en Python. > > La implementacion de Python, no es muy relevante, aunque si, en Jython > podrias tener el codigo dentro de un war o jar y hacerlo 'portable' > desde la JVM. El entendido es que el codigo en si sea seguro, no > ofuscarlo. > > > > > Creo que la pregunta es, protegerlo de qué? si es un sistema web y > > quieres protegerlo de ataques de inyección de sql, xss, etc; ofuscarlo > > no te ayudará, mejor mira herramientas de tests de penetración como > > w3af. > > > > Si es un sistema de escritorio y quieres evitar que el usuario > > modifique el programa, los permisos son más que suficiente. > > > > Si quieres evitar que un intruso pueda modificar tu programa en un > > servidor, entonces tienes que proteger al servidor. > > > >> Y evaluar en que lenguaje de programación hacer un modulo de seguridad a > >> nivel de usuario y perfiles. (Python u otro lenguaje que pueda ligarlo a > >> Python). > >> > > > > Eso ya queda dentro de tu aplicación, es mejor escribir los permisos > > de manera que aún asumiendo que el usuario pueda leer el código > > fuente, esto no represente un problema de seguridad en tu aplicación. > > > > Saludos > > -- > > A menudo unas pocas horas de "Prueba y error" podrán ahorrarte minutos > > de leer manuales. > > _______________________________________________ > > Python-es mailing list > > Python-es en python.org > > http://mail.python.org/mailman/listinfo/python-es > > FAQ: http://python-es-faq.wikidot.com/ > > > > > -- > Alexandro Colorado > PPMC Apache OpenOffice > http://es.openoffice.org > _______________________________________________ > Python-es mailing list > Python-es en python.org > http://mail.python.org/mailman/listinfo/python-es > FAQ: http://python-es-faq.wikidot.com/ > -- *Diego Alonso Uribe Gamez* ------------------------------ *Desarrollador web* Twitter: @DiegoUG Google+: http://gplus.to/diegoug ------------------------------ ------------ próxima parte ------------ Se ha borrado un adjunto en formato HTML... URL: From dhannier en gmail.com Tue Oct 30 17:13:34 2012 From: dhannier en gmail.com (Dhannier Molina) Date: Tue, 30 Oct 2012 11:43:34 -0430 Subject: [Python-es] Seguridad en Python In-Reply-To: References: Message-ID: Ok. Pero seria buena opción hacer el modulo de seguridad en un lenguaje compilado, por ejemplo como c++? Es decir, se quiere usar este modulo en diferentes partes del sistema para validar los accesos a los datos dependiendo de los perfiles de usuario. Evitar que el sistema reciba ataques al modulo de seguridad y que puedan modificar los permisos de accesos y demás... Seria buena opción combinar Python con C++... Disculpen las molestias... Saludos!! ------------ próxima parte ------------ Se ha borrado un adjunto en formato HTML... URL: From pych3m4 en gmail.com Tue Oct 30 17:51:49 2012 From: pych3m4 en gmail.com (Chema Cortes) Date: Tue, 30 Oct 2012 17:51:49 +0100 Subject: [Python-es] WXPYTHON no se cierra la ventana (Re: Estructuras de datos) Message-ID: El 2012/10/30 Gonzalo Mouriÿfffffffffff1o Pardo : > Buenos días, llevo más de una semana intentando cerrar la ventana del código > que adjunto, no se si es porque los eventos no se paran o porque hay algún > comando que no me se, pido ayuda a algún alm a caritativa. Antes de preguntar en esta lista, deberías leer lo que dice la FAQ (que aparece al final de todos los mensajes) los apartados relativos a "cómo preguntar", "cómo enviar código a la lista" y, de paso, "porqué somos tan antipáticos". Respondiendo a tu pregunta, el evento EVT_PAINT debe "escalar" a la ventana para que ésta también pueda actualizarse. Añade esta línea al final del OnPaint del panel: event.Skip() -- Hyperreals *R: http://ch3m4.org/blog Quarks, bits y otras criaturas infinitesimales From davidmenhur en gmail.com Tue Oct 30 18:10:19 2012 From: davidmenhur en gmail.com (=?UTF-8?B?RGHPgGlk?=) Date: Tue, 30 Oct 2012 18:10:19 +0100 Subject: [Python-es] Seguridad en Python In-Reply-To: References: Message-ID: ¿Qué es lo que quieres proteger, exactamente? Si es el acceso a unos datos con una estructura cifrada usando una lógica secreta (en un archivo C++ compilado), y Python para lo demás, no es muy efectivo. Aunque no tenga ni idea de cómo narices funciona tu código, puedo ver cómo lo llamas desde Python, y hacer yo lo mismo sobre archivos vacíos y viendo la salida, así hasta que aprenda cómo funciona. Y esto, considerando tu código como una caja negra que no intento abrir. Si además le acoplo un debugger u otras cosas, puedo saber mucho más de lo que está pasando ahí. 2012/10/30 Dhannier Molina : > Ok. Pero seria buena opción hacer el modulo de seguridad en un lenguaje > compilado, por ejemplo como c++? Es decir, se quiere usar este modulo en > diferentes partes del sistema para validar los accesos a los datos > dependiendo de los perfiles de usuario. Evitar que el sistema reciba ataques > al modulo de seguridad y que puedan modificar los permisos de accesos y > demás... Seria buena opción combinar Python con C++... > > Disculpen las molestias... > > Saludos!! > > _______________________________________________ > Python-es mailing list > Python-es en python.org > http://mail.python.org/mailman/listinfo/python-es > FAQ: http://python-es-faq.wikidot.com/ > From jza en oooes.org Tue Oct 30 18:17:31 2012 From: jza en oooes.org (Alexandro Colorado) Date: Tue, 30 Oct 2012 11:17:31 -0600 Subject: [Python-es] Mision de esta lista Message-ID: Hola, hoy estube buscando informacion de python en español y veo que mucha de la que se ha traducido esta desactualizada. Me pregunto cual es la mismion de esta lista y que rol juega. Trabajo en varios proyectos de SL donde las listas en español usualmente tienen una tarea de localizar el software del proyecto. Python es un lenguaje, sin embargo me pregunto quienes hacen la localizacion de su documentacion, archivos man, y otras cosas similares (modulos externos o STL). Tambien si hay coordinacion con proyectos externos (digamos un framework o un libro libre). Si alguien tiene experiencia sobre este topico y cual es el 'roadmap' a seguir. -- Alexandro Colorado PPMC Apache OpenOffice http://es.openoffice.org From juanlu001 en gmail.com Tue Oct 30 18:57:10 2012 From: juanlu001 en gmail.com (=?ISO-8859-1?Q?Juan_Luis_Cano_Rodr=EDguez?=) Date: Tue, 30 Oct 2012 18:57:10 +0100 Subject: [Python-es] Mision de esta lista In-Reply-To: References: Message-ID: Efectivamente hay muchas traducciones de documentos obsoletos, pero ¿podrías señalar los que te has encontrado y/o los que consideras más importantes? 2012/10/30 Alexandro Colorado > Hola, hoy estube buscando informacion de python en español y veo que > mucha de la que se ha traducido esta desactualizada. Me pregunto cual > es la mismion de esta lista y que rol juega. > > Trabajo en varios proyectos de SL donde las listas en español > usualmente tienen una tarea de localizar el software del proyecto. > Python es un lenguaje, sin embargo me pregunto quienes hacen la > localizacion de su documentacion, archivos man, y otras cosas > similares (modulos externos o STL). > > Tambien si hay coordinacion con proyectos externos (digamos un > framework o un libro libre). > > Si alguien tiene experiencia sobre este topico y cual es el 'roadmap' a > seguir. > > -- > Alexandro Colorado > PPMC Apache OpenOffice > http://es.openoffice.org > _______________________________________________ > Python-es mailing list > Python-es en python.org > http://mail.python.org/mailman/listinfo/python-es > FAQ: http://python-es-faq.wikidot.com/ > ------------ próxima parte ------------ Se ha borrado un adjunto en formato HTML... URL: From jza en oooes.org Tue Oct 30 19:02:38 2012 From: jza en oooes.org (Alexandro Colorado) Date: Tue, 30 Oct 2012 12:02:38 -0600 Subject: [Python-es] Mision de esta lista In-Reply-To: References: Message-ID: On 10/30/12, Juan Luis Cano Rodríguez wrote: > Efectivamente hay muchas traducciones de documentos obsoletos, pero > ¿podrías señalar los que te has encontrado y/o los que consideras más > importantes? Hay un puñado que son los mas mencionaos como: "How to think like a computer scientist" "A Byte of Python" Hay contenido en formato de recetario, que existe en ingles pero no he visto aun uno en español. Material mas academico sobre python en español, pyDocs, etc. Lo mismo video clases en youtube con narraciones. etc. En fin, solo quiero saber si fuera de documentacion hay algun rol donde esta lista este trabajando o por lo menos en planes de formular. Entiendo que hay muchas comunidades nacionales independientes, PyVer, PyAr, PythomMexico, etc. Pero tambien su funcion principal es el soporte tecnico pero no veo mucho aparte de esto. > > 2012/10/30 Alexandro Colorado > >> Hola, hoy estube buscando informacion de python en español y veo que >> mucha de la que se ha traducido esta desactualizada. Me pregunto cual >> es la mismion de esta lista y que rol juega. >> >> Trabajo en varios proyectos de SL donde las listas en español >> usualmente tienen una tarea de localizar el software del proyecto. >> Python es un lenguaje, sin embargo me pregunto quienes hacen la >> localizacion de su documentacion, archivos man, y otras cosas >> similares (modulos externos o STL). >> >> Tambien si hay coordinacion con proyectos externos (digamos un >> framework o un libro libre). >> >> Si alguien tiene experiencia sobre este topico y cual es el 'roadmap' a >> seguir. >> >> -- >> Alexandro Colorado >> PPMC Apache OpenOffice >> http://es.openoffice.org >> _______________________________________________ >> Python-es mailing list >> Python-es en python.org >> http://mail.python.org/mailman/listinfo/python-es >> FAQ: http://python-es-faq.wikidot.com/ >> > -- Alexandro Colorado PPMC Apache OpenOffice http://es.openoffice.org From dhannier en gmail.com Tue Oct 30 19:18:40 2012 From: dhannier en gmail.com (Dhannier Molina) Date: Tue, 30 Oct 2012 13:48:40 -0430 Subject: [Python-es] Seguridad en Python In-Reply-To: References: Message-ID: Proteger el sistema (permisos de accesos y datos) haciendo un modulo de seguridad (Ademas de la seguridad del servidor). El sistema esta desarrollado en python pero veo como mejor opción desarrollar el modulo en un lenguaje de programación compilado como C++... Saludos!! ------------ próxima parte ------------ Se ha borrado un adjunto en formato HTML... URL: From reingart en gmail.com Tue Oct 30 19:30:39 2012 From: reingart en gmail.com (Mariano Reingart) Date: Tue, 30 Oct 2012 15:30:39 -0300 Subject: [Python-es] Mision de esta lista In-Reply-To: References: Message-ID: Hola Alexandro: Estoy trabajando en una propuesta para internacionalizar y localizar python, comenzando por los mensajes de exepción y similares: http://python.org.ar/pyar/TracebackInternationalizationProposal http://bugs.python.org/issue16344 Si hay interesados avísenme y vemos, Sds Mariano Reingart http://www.sistemasagiles.com.ar http://reingart.blogspot.com 2012/10/30 Alexandro Colorado : > On 10/30/12, Juan Luis Cano Rodríguez wrote: >> Efectivamente hay muchas traducciones de documentos obsoletos, pero >> ¿podrías señalar los que te has encontrado y/o los que consideras más >> importantes? > > Hay un puñado que son los mas mencionaos como: > "How to think like a computer scientist" > "A Byte of Python" > > Hay contenido en formato de recetario, que existe en ingles pero no he > visto aun uno en español. > > Material mas academico sobre python en español, pyDocs, etc. > > Lo mismo video clases en youtube con narraciones. etc. > > En fin, solo quiero saber si fuera de documentacion hay algun rol > donde esta lista este trabajando o por lo menos en planes de formular. > Entiendo que hay muchas comunidades nacionales independientes, PyVer, > PyAr, PythomMexico, etc. Pero tambien su funcion principal es el > soporte tecnico pero no veo mucho aparte de esto. > >> >> 2012/10/30 Alexandro Colorado >> >>> Hola, hoy estube buscando informacion de python en español y veo que >>> mucha de la que se ha traducido esta desactualizada. Me pregunto cual >>> es la mismion de esta lista y que rol juega. >>> >>> Trabajo en varios proyectos de SL donde las listas en español >>> usualmente tienen una tarea de localizar el software del proyecto. >>> Python es un lenguaje, sin embargo me pregunto quienes hacen la >>> localizacion de su documentacion, archivos man, y otras cosas >>> similares (modulos externos o STL). >>> >>> Tambien si hay coordinacion con proyectos externos (digamos un >>> framework o un libro libre). >>> >>> Si alguien tiene experiencia sobre este topico y cual es el 'roadmap' a >>> seguir. >>> >>> -- >>> Alexandro Colorado >>> PPMC Apache OpenOffice >>> http://es.openoffice.org >>> _______________________________________________ >>> Python-es mailing list >>> Python-es en python.org >>> http://mail.python.org/mailman/listinfo/python-es >>> FAQ: http://python-es-faq.wikidot.com/ >>> >> > > > -- > Alexandro Colorado > PPMC Apache OpenOffice > http://es.openoffice.org > _______________________________________________ > Python-es mailing list > Python-es en python.org > http://mail.python.org/mailman/listinfo/python-es > FAQ: http://python-es-faq.wikidot.com/ From jza en oooes.org Tue Oct 30 20:51:24 2012 From: jza en oooes.org (Alexandro Colorado) Date: Tue, 30 Oct 2012 13:51:24 -0600 Subject: [Python-es] Mision de esta lista In-Reply-To: References: Message-ID: On 10/30/12, Mariano Reingart wrote: > Hola Alexandro: > > Estoy trabajando en una propuesta para internacionalizar y localizar > python, comenzando por los mensajes de exepción y similares: > > http://python.org.ar/pyar/TracebackInternationalizationProposal Excelente, y gracias por responder. Esto lo estan manejando desde PyAr o de Python.org/es? > > http://bugs.python.org/issue16344 > > Si hay interesados avísenme y vemos, > > Sds > > Mariano Reingart > http://www.sistemasagiles.com.ar > http://reingart.blogspot.com > > > 2012/10/30 Alexandro Colorado : >> On 10/30/12, Juan Luis Cano Rodríguez wrote: >>> Efectivamente hay muchas traducciones de documentos obsoletos, pero >>> ¿podrías señalar los que te has encontrado y/o los que consideras más >>> importantes? >> >> Hay un puñado que son los mas mencionaos como: >> "How to think like a computer scientist" >> "A Byte of Python" >> >> Hay contenido en formato de recetario, que existe en ingles pero no he >> visto aun uno en español. >> >> Material mas academico sobre python en español, pyDocs, etc. >> >> Lo mismo video clases en youtube con narraciones. etc. >> >> En fin, solo quiero saber si fuera de documentacion hay algun rol >> donde esta lista este trabajando o por lo menos en planes de formular. >> Entiendo que hay muchas comunidades nacionales independientes, PyVer, >> PyAr, PythomMexico, etc. Pero tambien su funcion principal es el >> soporte tecnico pero no veo mucho aparte de esto. >> >>> >>> 2012/10/30 Alexandro Colorado >>> >>>> Hola, hoy estube buscando informacion de python en español y veo que >>>> mucha de la que se ha traducido esta desactualizada. Me pregunto cual >>>> es la mismion de esta lista y que rol juega. >>>> >>>> Trabajo en varios proyectos de SL donde las listas en español >>>> usualmente tienen una tarea de localizar el software del proyecto. >>>> Python es un lenguaje, sin embargo me pregunto quienes hacen la >>>> localizacion de su documentacion, archivos man, y otras cosas >>>> similares (modulos externos o STL). >>>> >>>> Tambien si hay coordinacion con proyectos externos (digamos un >>>> framework o un libro libre). >>>> >>>> Si alguien tiene experiencia sobre este topico y cual es el 'roadmap' a >>>> seguir. >>>> >>>> -- >>>> Alexandro Colorado >>>> PPMC Apache OpenOffice >>>> http://es.openoffice.org >>>> _______________________________________________ >>>> Python-es mailing list >>>> Python-es en python.org >>>> http://mail.python.org/mailman/listinfo/python-es >>>> FAQ: http://python-es-faq.wikidot.com/ >>>> >>> >> >> >> -- >> Alexandro Colorado >> PPMC Apache OpenOffice >> http://es.openoffice.org >> _______________________________________________ >> Python-es mailing list >> Python-es en python.org >> http://mail.python.org/mailman/listinfo/python-es >> FAQ: http://python-es-faq.wikidot.com/ > _______________________________________________ > Python-es mailing list > Python-es en python.org > http://mail.python.org/mailman/listinfo/python-es > FAQ: http://python-es-faq.wikidot.com/ > -- Alexandro Colorado PPMC Apache OpenOffice http://es.openoffice.org From reingart en gmail.com Wed Oct 31 03:03:03 2012 From: reingart en gmail.com (Mariano Reingart) Date: Tue, 30 Oct 2012 23:03:03 -0300 Subject: [Python-es] Mision de esta lista In-Reply-To: References: Message-ID: 2012/10/30 Alexandro Colorado : > On 10/30/12, Mariano Reingart wrote: >> Hola Alexandro: >> >> Estoy trabajando en una propuesta para internacionalizar y localizar >> python, comenzando por los mensajes de exepción y similares: >> >> http://python.org.ar/pyar/TracebackInternationalizationProposal > > Excelente, y gracias por responder. Esto lo estan manejando desde PyAr > o de Python.org/es? > Si bien nació de algunas discuciones en PyAr (y python-ideas), por ahora es un borrador y se estaría manejando de forma independiente directamente en http://bugs.python.org/16344, pero sería muy conveniente tener el apoyo de grupos de usuarios, y desde ya toda colaboración es bienvenida. Sds Mariano Reingart http://www.sistemasagiles.com.ar http://reingart.blogspot.com From joe en joedicastro.com Wed Oct 31 10:21:37 2012 From: joe en joedicastro.com (joe di castro) Date: Wed, 31 Oct 2012 10:21:37 +0100 Subject: [Python-es] Mision de esta lista In-Reply-To: References: Message-ID: <5090EDA1.7040008@joedicastro.com> Hola Mariano, ¿Qué sentido tiene traducir el traceback de Python? No acabo de entenderlo. Las palabras claves seguirán estando en inglés, y lo que es peor, aunque llegaseis a traducir las excepciones y mensajes mostradas en toda la librería estándar, todas aquellas que sean propias de un montón de paquetes externos seguirán estando en Inglés. Y los comentarios del código... ¿Te imaginas un traceback en Español con una excepción en Inglés? Un desastre... además el interprete seguirá estando en Inglés (los interpretes, mejor dicho, e.g. ipython, bpython). Además las excepciones están orientadas a los desarrolladores, y los programadores DEBEN saber Inglés, esa idea de hacer una huida hacía delante, rehuyendo el aprenderlo correctamente todo el tiempo que sea posible es un error GRAVISIMO que acabará lastrando la carrera de cualquiera. Aunque consiguiéramos traducir toda la documentación, tracebacks, etc.. al Español, habría que mantenerlo y eso lleva tiempo y esfuerzo. Y en cuanto tuviéramos que emplear otro lenguaje (algo inevitable tarde o temprano) nos encontraríamos con el mismo problema. Entiendo la necesidad de traducir interfaces y documentación especifica para el usuario final, lo entiendo y lo defiendo. Pero el programador tiene que entender que el Inglés es y será durante mucho tiempo la LENGUA FRANCA en la programación, y eludir el aprendizaje del idioma es un error imperdonable. Si hay dos cosas que cualquier programador debería dominar, en cualquier lugar del mundo, son las siguientes: * Mecanografía, teclear con diez dedos a una velocidad razonable (60WPM por lo menos). * Idioma Inglés, con al menos una buena compresión al leerlo (lo mínimo razonable) aunque debería entenderlo con fluidez y poder escribirlo y hablarlo con unas mínimas garantías. Puedo entender que haya tutoriales, manuales de iniciación, libros de introducción, artículos, webs, ... en idiomas distintos al Inglés, pero duplicar la documentación y el lenguaje es un esfuerzo inútil que no beneficia a nadie, desde luego no a largo plazo. Lo veo como pan para hoy y hambre para mañana. Es mi opinión personal y seguro que choca con la de muchos otros, pero entiendo que uno debe comprender y aceptar las reglas del entorno en el que se mueve. Saludos. On 30/10/12 19:30, Mariano Reingart wrote: > Hola Alexandro: > > Estoy trabajando en una propuesta para internacionalizar y localizar > python, comenzando por los mensajes de exepción y similares: > > http://python.org.ar/pyar/TracebackInternationalizationProposal > > http://bugs.python.org/issue16344 > > Si hay interesados avísenme y vemos, > > Sds > > Mariano Reingart > http://www.sistemasagiles.com.ar > http://reingart.blogspot.com > > > 2012/10/30 Alexandro Colorado : >> On 10/30/12, Juan Luis Cano Rodríguez wrote: >>> Efectivamente hay muchas traducciones de documentos obsoletos, pero >>> ¿podrías señalar los que te has encontrado y/o los que consideras más >>> importantes? >> >> Hay un puñado que son los mas mencionaos como: >> "How to think like a computer scientist" >> "A Byte of Python" >> >> Hay contenido en formato de recetario, que existe en ingles pero no he >> visto aun uno en español. >> >> Material mas academico sobre python en español, pyDocs, etc. >> >> Lo mismo video clases en youtube con narraciones. etc. >> >> En fin, solo quiero saber si fuera de documentacion hay algun rol >> donde esta lista este trabajando o por lo menos en planes de formular. >> Entiendo que hay muchas comunidades nacionales independientes, PyVer, >> PyAr, PythomMexico, etc. Pero tambien su funcion principal es el >> soporte tecnico pero no veo mucho aparte de esto. >> >>> >>> 2012/10/30 Alexandro Colorado >>> >>>> Hola, hoy estube buscando informacion de python en español y veo que >>>> mucha de la que se ha traducido esta desactualizada. Me pregunto cual >>>> es la mismion de esta lista y que rol juega. >>>> >>>> Trabajo en varios proyectos de SL donde las listas en español >>>> usualmente tienen una tarea de localizar el software del proyecto. >>>> Python es un lenguaje, sin embargo me pregunto quienes hacen la >>>> localizacion de su documentacion, archivos man, y otras cosas >>>> similares (modulos externos o STL). >>>> >>>> Tambien si hay coordinacion con proyectos externos (digamos un >>>> framework o un libro libre). >>>> >>>> Si alguien tiene experiencia sobre este topico y cual es el 'roadmap' a >>>> seguir. >>>> >>>> -- >>>> Alexandro Colorado >>>> PPMC Apache OpenOffice >>>> http://es.openoffice.org >>>> _______________________________________________ >>>> Python-es mailing list >>>> Python-es en python.org >>>> http://mail.python.org/mailman/listinfo/python-es >>>> FAQ: http://python-es-faq.wikidot.com/ >>>> >>> >> >> >> -- >> Alexandro Colorado >> PPMC Apache OpenOffice >> http://es.openoffice.org >> _______________________________________________ >> Python-es mailing list >> Python-es en python.org >> http://mail.python.org/mailman/listinfo/python-es >> FAQ: http://python-es-faq.wikidot.com/ > _______________________________________________ > Python-es mailing list > Python-es en python.org > http://mail.python.org/mailman/listinfo/python-es > FAQ: http://python-es-faq.wikidot.com/ > From jza en oooes.org Wed Oct 31 11:07:10 2012 From: jza en oooes.org (Alexandro Colorado) Date: Wed, 31 Oct 2012 04:07:10 -0600 Subject: [Python-es] Mision de esta lista In-Reply-To: <5090EDA1.7040008@joedicastro.com> References: <5090EDA1.7040008@joedicastro.com> Message-ID: Hola Joe, muy buen punto y coincido con el constante "pan de hoy, hambre de maniana". Regresando al topico original, va mas a si esta lista hace o ha hecho algo mas que solo dar soporte a usuarios. Creo que aun no me responden esa pregunta, quizas alguien con mas experiencia en esta lista me pueda resumir lo que se ha manejado desde que participa en esta? On 10/31/12, joe di castro wrote: > Hola Mariano, > > ¿Qué sentido tiene traducir el traceback de Python? > > No acabo de entenderlo. Las palabras claves seguirán estando en inglés, > y lo que es peor, aunque llegaseis a traducir las excepciones y mensajes > mostradas en toda la librería estándar, todas aquellas que sean propias > de un montón de paquetes externos seguirán estando en Inglés. Y los > comentarios del código... > > ¿Te imaginas un traceback en Español con una excepción en Inglés? Un > desastre... además el interprete seguirá estando en Inglés (los > interpretes, mejor dicho, e.g. ipython, bpython). > > Además las excepciones están orientadas a los desarrolladores, y los > programadores DEBEN saber Inglés, esa idea de hacer una huida hacía > delante, rehuyendo el aprenderlo correctamente todo el tiempo que sea > posible es un error GRAVISIMO que acabará lastrando la carrera de > cualquiera. Aunque consiguiéramos traducir toda la documentación, > tracebacks, etc.. al Español, habría que mantenerlo y eso lleva tiempo y > esfuerzo. Y en cuanto tuviéramos que emplear otro lenguaje (algo > inevitable tarde o temprano) nos encontraríamos con el mismo problema. > > Entiendo la necesidad de traducir interfaces y documentación especifica > para el usuario final, lo entiendo y lo defiendo. > > Pero el programador tiene que entender que el Inglés es y será durante > mucho tiempo la LENGUA FRANCA en la programación, y eludir el > aprendizaje del idioma es un error imperdonable. Si hay dos cosas que > cualquier programador debería dominar, en cualquier lugar del mundo, son > las siguientes: > > * Mecanografía, teclear con diez dedos a una velocidad razonable (60WPM > por lo menos). > > * Idioma Inglés, con al menos una buena compresión al leerlo (lo mínimo > razonable) aunque debería entenderlo con fluidez y poder escribirlo y > hablarlo con unas mínimas garantías. > > Puedo entender que haya tutoriales, manuales de iniciación, libros de > introducción, artículos, webs, ... en idiomas distintos al Inglés, pero > duplicar la documentación y el lenguaje es un esfuerzo inútil que no > beneficia a nadie, desde luego no a largo plazo. Lo veo como pan para > hoy y hambre para mañana. > > Es mi opinión personal y seguro que choca con la de muchos otros, pero > entiendo que uno debe comprender y aceptar las reglas del entorno en el > que se mueve. > > Saludos. > > On 30/10/12 19:30, Mariano Reingart wrote: >> Hola Alexandro: >> >> Estoy trabajando en una propuesta para internacionalizar y localizar >> python, comenzando por los mensajes de exepción y similares: >> >> http://python.org.ar/pyar/TracebackInternationalizationProposal >> >> http://bugs.python.org/issue16344 >> >> Si hay interesados avísenme y vemos, >> >> Sds >> >> Mariano Reingart >> http://www.sistemasagiles.com.ar >> http://reingart.blogspot.com >> >> >> 2012/10/30 Alexandro Colorado : >>> On 10/30/12, Juan Luis Cano Rodríguez wrote: >>>> Efectivamente hay muchas traducciones de documentos obsoletos, pero >>>> ¿podrías señalar los que te has encontrado y/o los que consideras más >>>> importantes? >>> >>> Hay un puñado que son los mas mencionaos como: >>> "How to think like a computer scientist" >>> "A Byte of Python" >>> >>> Hay contenido en formato de recetario, que existe en ingles pero no he >>> visto aun uno en español. >>> >>> Material mas academico sobre python en español, pyDocs, etc. >>> >>> Lo mismo video clases en youtube con narraciones. etc. >>> >>> En fin, solo quiero saber si fuera de documentacion hay algun rol >>> donde esta lista este trabajando o por lo menos en planes de formular. >>> Entiendo que hay muchas comunidades nacionales independientes, PyVer, >>> PyAr, PythomMexico, etc. Pero tambien su funcion principal es el >>> soporte tecnico pero no veo mucho aparte de esto. >>> >>>> >>>> 2012/10/30 Alexandro Colorado >>>> >>>>> Hola, hoy estube buscando informacion de python en español y veo que >>>>> mucha de la que se ha traducido esta desactualizada. Me pregunto cual >>>>> es la mismion de esta lista y que rol juega. >>>>> >>>>> Trabajo en varios proyectos de SL donde las listas en español >>>>> usualmente tienen una tarea de localizar el software del proyecto. >>>>> Python es un lenguaje, sin embargo me pregunto quienes hacen la >>>>> localizacion de su documentacion, archivos man, y otras cosas >>>>> similares (modulos externos o STL). >>>>> >>>>> Tambien si hay coordinacion con proyectos externos (digamos un >>>>> framework o un libro libre). >>>>> >>>>> Si alguien tiene experiencia sobre este topico y cual es el 'roadmap' >>>>> a >>>>> seguir. >>>>> >>>>> -- >>>>> Alexandro Colorado >>>>> PPMC Apache OpenOffice >>>>> http://es.openoffice.org >>>>> _______________________________________________ >>>>> Python-es mailing list >>>>> Python-es en python.org >>>>> http://mail.python.org/mailman/listinfo/python-es >>>>> FAQ: http://python-es-faq.wikidot.com/ >>>>> >>>> >>> >>> >>> -- >>> Alexandro Colorado >>> PPMC Apache OpenOffice >>> http://es.openoffice.org >>> _______________________________________________ >>> Python-es mailing list >>> Python-es en python.org >>> http://mail.python.org/mailman/listinfo/python-es >>> FAQ: http://python-es-faq.wikidot.com/ >> _______________________________________________ >> Python-es mailing list >> Python-es en python.org >> http://mail.python.org/mailman/listinfo/python-es >> FAQ: http://python-es-faq.wikidot.com/ >> > > _______________________________________________ > Python-es mailing list > Python-es en python.org > http://mail.python.org/mailman/listinfo/python-es > FAQ: http://python-es-faq.wikidot.com/ > -- Alexandro Colorado PPMC Apache OpenOffice http://es.openoffice.org From kikocorreoso en gmail.com Wed Oct 31 11:20:28 2012 From: kikocorreoso en gmail.com (Kiko) Date: Wed, 31 Oct 2012 11:20:28 +0100 Subject: [Python-es] Mision de esta lista In-Reply-To: References: <5090EDA1.7040008@joedicastro.com> Message-ID: Regresando al topico original, va mas a si esta > lista hace o ha hecho algo mas que solo dar soporte a usuarios. > Creo que esa ha sido la funcionalidad habitual de esta lista. Se intentó hace un tiempo integrar las cosas comunes entre las distintas comunidades hispanas de python en http://python-hispano.org pero debido a diferentes causas eso está un poco parado pero se consiguieron pequeñas cositas como un planet y diferentes recursos. Se propuso en diferentes comunidades hispanas y la respuesta de algunas fue ni fú ni fá. ------------ próxima parte ------------ Se ha borrado un adjunto en formato HTML... URL: From gmourinopardo en yahoo.es Wed Oct 31 11:26:42 2012 From: gmourinopardo en yahoo.es (=?iso-8859-1?Q?Gonzalo_Mouri=FFfffffffffff1o_Pardo?=) Date: Wed, 31 Oct 2012 10:26:42 +0000 (GMT) Subject: [Python-es] Rv: WXPYTHON no se cierra la ventana In-Reply-To: References: Message-ID: <1351679202.83210.YahooMailNeo@web171701.mail.ir2.yahoo.com> Muchisimas gracias, pero al hacer esto no se me regeneran los dc.rectangle, ya que en teoria el OnLeftDown, llama a un módulo aparte y al cerrarlo, ya que dicho módulo, es una pantalla de entrada de datos. Ya he visto la FAQ, pero no se que es eso de los snippets. y por lo tanto no se subir el código, te agradeceria que me dieras alguna indicacioón sobre esto otro también. Gracias. ----- Mensaje reenviado ----- De: Chema Cortes Para: Gonzalo Mouriÿfffffffffff1o Pardo ; La lista de python en castellano Enviado: Martes 30 de octubre de 2012 17:51 Asunto: WXPYTHON no se cierra la ventana (Re: [Python-es] Estructuras de datos) El 2012/10/30 Gonzalo Mouriÿfffffffffff1o Pardo : > Buenos días, llevo más de una semana intentando cerrar la ventana del código > que adjunto, no se si es porque los eventos no se paran o porque hay algún > comando que no me se, pido ayuda a algún alm a caritativa. Antes de preguntar en esta lista, deberías leer lo que dice la FAQ (que aparece al final de todos los mensajes) los apartados relativos a "cómo preguntar", "cómo enviar código a la lista" y, de paso, "porqué somos tan antipáticos". Respondiendo a tu pregunta, el evento EVT_PAINT debe "escalar" a la ventana para que ésta también pueda actualizarse. Añade esta línea al final del OnPaint del panel:   event.Skip() -- Hyperreals *R: http://ch3m4.org/blog Quarks, bits y otras criaturas infinitesimales ------------ próxima parte ------------ Se ha borrado un adjunto en formato HTML... URL: From jdpd en gmx.es Wed Oct 31 11:39:07 2012 From: jdpd en gmx.es (Jaime Perea) Date: Wed, 31 Oct 2012 11:39:07 +0100 Subject: [Python-es] Mision de esta lista In-Reply-To: <5090EDA1.7040008@joedicastro.com> References: <5090EDA1.7040008@joedicastro.com> Message-ID: <2163020.3H3bVHb2Kx@cristina> Está bien que los mensajes se puedan entender. Mira lo siguiente, intento usar el gcc sobre un fichero que no existe. cristina: gcc lll.c gcc: error: lll.c: No existe el archivo o el directorio gcc: error fatal: no hay ficheros de entrada compilación terminada. cristina es mi laptop, va sobre kubuntu. Aquí se dice muy claro que tipo de error he cometido y eso es útil para cualquier persona que quiera compilar algo, no necesariamente un programador experto. O sea, si la gente que trabaja en algo tan básico y bajo nivel como el compilador de C, se han molestado en posibilitar la traducción, es que será útil, digo yo. De hecho gettext y similares existe desde hace mucho tiempo y no sólo sirve para los mensajes, Una de las razones más básicas del éxito de python, es que es cómodo y fácil de programar. Facilitarle a la gente el acceso a la programación es bueno. Saluditos -- Jaime D. Perea Duarte. Linux registered user #10472 Dep. Astrofisica Extragalactica. Instituto de Astrofisica de Andalucia (CSIC) Apdo. 3004, 18080 Granada, Spain. On Miércoles, 31 de octubre de 2012 10:21:37 joe di castro escribió: > Hola Mariano, > > ¿Qué sentido tiene traducir el traceback de Python? > > No acabo de entenderlo. Las palabras claves seguirán estando en inglés, > y lo que es peor, aunque llegaseis a traducir las excepciones y mensajes > mostradas en toda la librería estándar, todas aquellas que sean propias > de un montón de paquetes externos seguirán estando en Inglés. Y los > comentarios del código... > > ¿Te imaginas un traceback en Español con una excepción en Inglés? Un > desastre... además el interprete seguirá estando en Inglés (los > interpretes, mejor dicho, e.g. ipython, bpython). > > Además las excepciones están orientadas a los desarrolladores, y los > programadores DEBEN saber Inglés, esa idea de hacer una huida hacía > delante, rehuyendo el aprenderlo correctamente todo el tiempo que sea > posible es un error GRAVISIMO que acabará lastrando la carrera de > cualquiera. Aunque consiguiéramos traducir toda la documentación, > tracebacks, etc.. al Español, habría que mantenerlo y eso lleva tiempo y > esfuerzo. Y en cuanto tuviéramos que emplear otro lenguaje (algo > inevitable tarde o temprano) nos encontraríamos con el mismo problema. > > Entiendo la necesidad de traducir interfaces y documentación especifica > para el usuario final, lo entiendo y lo defiendo. > > Pero el programador tiene que entender que el Inglés es y será durante > mucho tiempo la LENGUA FRANCA en la programación, y eludir el > aprendizaje del idioma es un error imperdonable. Si hay dos cosas que > cualquier programador debería dominar, en cualquier lugar del mundo, son > las siguientes: > > * Mecanografía, teclear con diez dedos a una velocidad razonable (60WPM > por lo menos). > > * Idioma Inglés, con al menos una buena compresión al leerlo (lo mínimo > razonable) aunque debería entenderlo con fluidez y poder escribirlo y > hablarlo con unas mínimas garantías. > > Puedo entender que haya tutoriales, manuales de iniciación, libros de > introducción, artículos, webs, ... en idiomas distintos al Inglés, pero > duplicar la documentación y el lenguaje es un esfuerzo inútil que no > beneficia a nadie, desde luego no a largo plazo. Lo veo como pan para > hoy y hambre para mañana. > > Es mi opinión personal y seguro que choca con la de muchos otros, pero > entiendo que uno debe comprender y aceptar las reglas del entorno en el > que se mueve. > > Saludos. > > On 30/10/12 19:30, Mariano Reingart wrote: > > Hola Alexandro: > > > > Estoy trabajando en una propuesta para internacionalizar y localizar > > python, comenzando por los mensajes de exepción y similares: > > > > http://python.org.ar/pyar/TracebackInternationalizationProposal > > > > http://bugs.python.org/issue16344 > > > > Si hay interesados avísenme y vemos, > > > > Sds > > > > Mariano Reingart > > http://www.sistemasagiles.com.ar > > http://reingart.blogspot.com > > > > 2012/10/30 Alexandro Colorado : > >> On 10/30/12, Juan Luis Cano Rodríguez wrote: > >>> Efectivamente hay muchas traducciones de documentos obsoletos, pero > >>> ¿podrías señalar los que te has encontrado y/o los que consideras más > >>> importantes? > >> > >> Hay un puñado que son los mas mencionaos como: > >> "How to think like a computer scientist" > >> "A Byte of Python" > >> > >> Hay contenido en formato de recetario, que existe en ingles pero no he > >> visto aun uno en español. > >> > >> Material mas academico sobre python en español, pyDocs, etc. > >> > >> Lo mismo video clases en youtube con narraciones. etc. > >> > >> En fin, solo quiero saber si fuera de documentacion hay algun rol > >> donde esta lista este trabajando o por lo menos en planes de formular. > >> Entiendo que hay muchas comunidades nacionales independientes, PyVer, > >> PyAr, PythomMexico, etc. Pero tambien su funcion principal es el > >> soporte tecnico pero no veo mucho aparte de esto. > >> > >>> 2012/10/30 Alexandro Colorado > >>> > >>>> Hola, hoy estube buscando informacion de python en español y veo que > >>>> mucha de la que se ha traducido esta desactualizada. Me pregunto cual > >>>> es la mismion de esta lista y que rol juega. > >>>> > >>>> Trabajo en varios proyectos de SL donde las listas en español > >>>> usualmente tienen una tarea de localizar el software del proyecto. > >>>> Python es un lenguaje, sin embargo me pregunto quienes hacen la > >>>> localizacion de su documentacion, archivos man, y otras cosas > >>>> similares (modulos externos o STL). > >>>> > >>>> Tambien si hay coordinacion con proyectos externos (digamos un > >>>> framework o un libro libre). > >>>> > >>>> Si alguien tiene experiencia sobre este topico y cual es el 'roadmap' a > >>>> seguir. > >>>> > >>>> -- > >>>> Alexandro Colorado > >>>> PPMC Apache OpenOffice > >>>> http://es.openoffice.org > >>>> _______________________________________________ > >>>> Python-es mailing list > >>>> Python-es en python.org > >>>> http://mail.python.org/mailman/listinfo/python-es > >>>> FAQ: http://python-es-faq.wikidot.com/ > >> > >> -- > >> Alexandro Colorado > >> PPMC Apache OpenOffice > >> http://es.openoffice.org > >> _______________________________________________ > >> Python-es mailing list > >> Python-es en python.org > >> http://mail.python.org/mailman/listinfo/python-es > >> FAQ: http://python-es-faq.wikidot.com/ > > > > _______________________________________________ > > Python-es mailing list > > Python-es en python.org > > http://mail.python.org/mailman/listinfo/python-es > > FAQ: http://python-es-faq.wikidot.com/ > > _______________________________________________ > Python-es mailing list > Python-es en python.org > http://mail.python.org/mailman/listinfo/python-es > FAQ: http://python-es-faq.wikidot.com/ From lasizoillo en gmail.com Wed Oct 31 12:07:03 2012 From: lasizoillo en gmail.com (lasizoillo) Date: Wed, 31 Oct 2012 12:07:03 +0100 Subject: [Python-es] Mision de esta lista In-Reply-To: <2163020.3H3bVHb2Kx@cristina> References: <5090EDA1.7040008@joedicastro.com> <2163020.3H3bVHb2Kx@cristina> Message-ID: El día 31 de octubre de 2012 11:39, Jaime Perea escribió: > Está bien que los mensajes se puedan entender. Mira lo siguiente, intento > usar el gcc sobre un fichero que no existe. > > cristina: gcc lll.c > gcc: error: lll.c: No existe el archivo o el directorio > gcc: error fatal: no hay ficheros de entrada > compilación terminada. > > cristina es mi laptop, va sobre kubuntu. Aquí se dice muy claro que tipo de > error he cometido y eso es útil para cualquier persona que quiera compilar > algo, no necesariamente un programador experto. > > O sea, si la gente que trabaja en algo tan básico y bajo nivel como el > compilador de C, se han molestado en posibilitar la traducción, es que será > útil, digo yo. De hecho gettext y similares existe desde hace mucho tiempo y > no sólo sirve para los mensajes, > > Una de las razones más básicas del éxito de python, es que es cómodo y fácil > de programar. Facilitarle a la gente el acceso a la programación es bueno. > Solo hay un problema con tu propuesta. ¿Cómo se traducen las siguientes excepciones? http://hg.python.org/cpython/file/9f464eff218a/Lib/gettext.py No se me ocurre ninguna forma limpia de romper las dependencias circulares entre gettext y excepciones con tu planteamiento. Aunque también es verdad que no le he dedicado tiempo a pensarlo, prefiero dedicarle tiempo a aprender inglés ;-) Un saludo, Javi From kikocorreoso en gmail.com Wed Oct 31 12:13:43 2012 From: kikocorreoso en gmail.com (Kiko) Date: Wed, 31 Oct 2012 12:13:43 +0100 Subject: [Python-es] Mision de esta lista In-Reply-To: <2163020.3H3bVHb2Kx@cristina> References: <5090EDA1.7040008@joedicastro.com> <2163020.3H3bVHb2Kx@cristina> Message-ID: El 31 de octubre de 2012 11:39, Jaime Perea escribió: > Está bien que los mensajes se puedan entender. Mira lo siguiente, intento > usar el gcc sobre un fichero que no existe. > > cristina: gcc lll.c > gcc: error: lll.c: No existe el archivo o el directorio > gcc: error fatal: no hay ficheros de entrada > compilación terminada. > > cristina es mi laptop, va sobre kubuntu. Aquí se dice muy claro que tipo de > error he cometido y eso es útil para cualquier persona que quiera compilar > algo, no necesariamente un programador experto. > > O sea, si la gente que trabaja en algo tan básico y bajo nivel como el > compilador de C, se han molestado en posibilitar la traducción, es que será > útil, digo yo. De hecho gettext y similares existe desde hace mucho > tiempo y > no sólo sirve para los mensajes, > > Una de las razones más básicas del éxito de python, es que es cómodo y > fácil > de programar. Facilitarle a la gente el acceso a la programación es bueno. > Por esta misma razón (llevándolo al absurdo), porqué no traducir for, while, if, class,... :-) El tema es que el que se mete a programar debería de aprender inglés sí o sí para poder desenvolverse correctamente (al igual que el que se mete en ciencia sabe que el lenguaje estándar de facto es el inglés). Dicho esto, debería establecerse una relación esfuerzo/rentabilidad de la traducción y si se considera que la relación es aceptable, perfecto. Esto me parece aceptable para, por ejemplo, el tutorial de python, algunos libros,... Para lo que comenta Mariano, por ejemplo, no lo veo tan claro. Dicho esto, estoy totalmente de acuerdo con el comentario de Joe. Saludos. ------------ próxima parte ------------ Se ha borrado un adjunto en formato HTML... URL: From joe en joedicastro.com Wed Oct 31 12:14:45 2012 From: joe en joedicastro.com (joe di castro) Date: Wed, 31 Oct 2012 12:14:45 +0100 Subject: [Python-es] Mision de esta lista In-Reply-To: <2163020.3H3bVHb2Kx@cristina> References: <5090EDA1.7040008@joedicastro.com> <2163020.3H3bVHb2Kx@cristina> Message-ID: <50910825.9050600@joedicastro.com> Como decía, un compilador está destinado a programadores, no al usuario final. Por otro lado, ¿cuando compilas un paquete externo, en que idioma crees que salen los errores particulares del mismo (los que captura a propósito) el 90% de las veces? Por ejemplo, imagina algo así: cristina: gcc foo.c gcc: error: foo.c: Error 3245 The input is no a valid file compilación terminada. es algo inventado, solo a propósito de ejemplo... Pues anda que no estoy cansado de compilar paquetes (soy sysadmin)... y el español lo he visto, pero que muy poquito... y no hay nada que me fastidie más que encontrarme las cosas a medias... como páginas man en español mezcladas con otras en inglés... Y hay muchos tutoriales, manuales, etc.. en español... si un futuro programador no sabe leer una excepción en Inglés... el buscador o traductor de Google o lo que sea hasta que aprenda... ponérselo facilito con el español, en realidad es hacerle una putada de cara al futuro, pronto se encontrará en un callejón sin salida. Y a un usuario, aunque se lo pongas en español, muchos van a acabar igual tirando de Google... Saludos On 31/10/12 11:39, Jaime Perea wrote: > Está bien que los mensajes se puedan entender. Mira lo siguiente, intento > usar el gcc sobre un fichero que no existe. > > cristina: gcc lll.c > gcc: error: lll.c: No existe el archivo o el directorio > gcc: error fatal: no hay ficheros de entrada > compilación terminada. > > cristina es mi laptop, va sobre kubuntu. Aquí se dice muy claro que tipo de > error he cometido y eso es útil para cualquier persona que quiera compilar > algo, no necesariamente un programador experto. > > O sea, si la gente que trabaja en algo tan básico y bajo nivel como el > compilador de C, se han molestado en posibilitar la traducción, es que será > útil, digo yo. De hecho gettext y similares existe desde hace mucho tiempo y > no sólo sirve para los mensajes, > > Una de las razones más básicas del éxito de python, es que es cómodo y fácil > de programar. Facilitarle a la gente el acceso a la programación es bueno. > > Saluditos > -- > > Jaime D. Perea Duarte. > Linux registered user #10472 > > Dep. Astrofisica Extragalactica. > Instituto de Astrofisica de Andalucia (CSIC) > Apdo. 3004, 18080 Granada, Spain. > > > On Miércoles, 31 de octubre de 2012 10:21:37 joe di castro escribió: >> Hola Mariano, >> >> ¿Qué sentido tiene traducir el traceback de Python? >> >> No acabo de entenderlo. Las palabras claves seguirán estando en inglés, >> y lo que es peor, aunque llegaseis a traducir las excepciones y mensajes >> mostradas en toda la librería estándar, todas aquellas que sean propias >> de un montón de paquetes externos seguirán estando en Inglés. Y los >> comentarios del código... >> >> ¿Te imaginas un traceback en Español con una excepción en Inglés? Un >> desastre... además el interprete seguirá estando en Inglés (los >> interpretes, mejor dicho, e.g. ipython, bpython). >> >> Además las excepciones están orientadas a los desarrolladores, y los >> programadores DEBEN saber Inglés, esa idea de hacer una huida hacía >> delante, rehuyendo el aprenderlo correctamente todo el tiempo que sea >> posible es un error GRAVISIMO que acabará lastrando la carrera de >> cualquiera. Aunque consiguiéramos traducir toda la documentación, >> tracebacks, etc.. al Español, habría que mantenerlo y eso lleva tiempo y >> esfuerzo. Y en cuanto tuviéramos que emplear otro lenguaje (algo >> inevitable tarde o temprano) nos encontraríamos con el mismo problema. >> >> Entiendo la necesidad de traducir interfaces y documentación especifica >> para el usuario final, lo entiendo y lo defiendo. >> >> Pero el programador tiene que entender que el Inglés es y será durante >> mucho tiempo la LENGUA FRANCA en la programación, y eludir el >> aprendizaje del idioma es un error imperdonable. Si hay dos cosas que >> cualquier programador debería dominar, en cualquier lugar del mundo, son >> las siguientes: >> >> * Mecanografía, teclear con diez dedos a una velocidad razonable (60WPM >> por lo menos). >> >> * Idioma Inglés, con al menos una buena compresión al leerlo (lo mínimo >> razonable) aunque debería entenderlo con fluidez y poder escribirlo y >> hablarlo con unas mínimas garantías. >> >> Puedo entender que haya tutoriales, manuales de iniciación, libros de >> introducción, artículos, webs, ... en idiomas distintos al Inglés, pero >> duplicar la documentación y el lenguaje es un esfuerzo inútil que no >> beneficia a nadie, desde luego no a largo plazo. Lo veo como pan para >> hoy y hambre para mañana. >> >> Es mi opinión personal y seguro que choca con la de muchos otros, pero >> entiendo que uno debe comprender y aceptar las reglas del entorno en el >> que se mueve. >> >> Saludos. >> >> On 30/10/12 19:30, Mariano Reingart wrote: >>> Hola Alexandro: >>> >>> Estoy trabajando en una propuesta para internacionalizar y localizar >>> python, comenzando por los mensajes de exepción y similares: >>> >>> http://python.org.ar/pyar/TracebackInternationalizationProposal >>> >>> http://bugs.python.org/issue16344 >>> >>> Si hay interesados avísenme y vemos, >>> >>> Sds >>> >>> Mariano Reingart >>> http://www.sistemasagiles.com.ar >>> http://reingart.blogspot.com >>> >>> 2012/10/30 Alexandro Colorado : >>>> On 10/30/12, Juan Luis Cano Rodríguez wrote: >>>>> Efectivamente hay muchas traducciones de documentos obsoletos, pero >>>>> ¿podrías señalar los que te has encontrado y/o los que consideras más >>>>> importantes? >>>> >>>> Hay un puñado que son los mas mencionaos como: >>>> "How to think like a computer scientist" >>>> "A Byte of Python" >>>> >>>> Hay contenido en formato de recetario, que existe en ingles pero no he >>>> visto aun uno en español. >>>> >>>> Material mas academico sobre python en español, pyDocs, etc. >>>> >>>> Lo mismo video clases en youtube con narraciones. etc. >>>> >>>> En fin, solo quiero saber si fuera de documentacion hay algun rol >>>> donde esta lista este trabajando o por lo menos en planes de formular. >>>> Entiendo que hay muchas comunidades nacionales independientes, PyVer, >>>> PyAr, PythomMexico, etc. Pero tambien su funcion principal es el >>>> soporte tecnico pero no veo mucho aparte de esto. >>>> >>>>> 2012/10/30 Alexandro Colorado >>>>> >>>>>> Hola, hoy estube buscando informacion de python en español y veo que >>>>>> mucha de la que se ha traducido esta desactualizada. Me pregunto cual >>>>>> es la mismion de esta lista y que rol juega. >>>>>> >>>>>> Trabajo en varios proyectos de SL donde las listas en español >>>>>> usualmente tienen una tarea de localizar el software del proyecto. >>>>>> Python es un lenguaje, sin embargo me pregunto quienes hacen la >>>>>> localizacion de su documentacion, archivos man, y otras cosas >>>>>> similares (modulos externos o STL). >>>>>> >>>>>> Tambien si hay coordinacion con proyectos externos (digamos un >>>>>> framework o un libro libre). >>>>>> >>>>>> Si alguien tiene experiencia sobre este topico y cual es el 'roadmap' a >>>>>> seguir. >>>>>> >>>>>> -- >>>>>> Alexandro Colorado >>>>>> PPMC Apache OpenOffice >>>>>> http://es.openoffice.org >>>>>> _______________________________________________ >>>>>> Python-es mailing list >>>>>> Python-es en python.org >>>>>> http://mail.python.org/mailman/listinfo/python-es >>>>>> FAQ: http://python-es-faq.wikidot.com/ >>>> >>>> -- >>>> Alexandro Colorado >>>> PPMC Apache OpenOffice >>>> http://es.openoffice.org >>>> _______________________________________________ >>>> Python-es mailing list >>>> Python-es en python.org >>>> http://mail.python.org/mailman/listinfo/python-es >>>> FAQ: http://python-es-faq.wikidot.com/ >>> >>> _______________________________________________ >>> Python-es mailing list >>> Python-es en python.org >>> http://mail.python.org/mailman/listinfo/python-es >>> FAQ: http://python-es-faq.wikidot.com/ >> >> _______________________________________________ >> Python-es mailing list >> Python-es en python.org >> http://mail.python.org/mailman/listinfo/python-es >> FAQ: http://python-es-faq.wikidot.com/ > _______________________________________________ > Python-es mailing list > Python-es en python.org > http://mail.python.org/mailman/listinfo/python-es > FAQ: http://python-es-faq.wikidot.com/ > From santicamps en gmail.com Wed Oct 31 12:19:11 2012 From: santicamps en gmail.com (Santi Camps) Date: Wed, 31 Oct 2012 12:19:11 +0100 Subject: [Python-es] Buscamos un programador python Message-ID: Hola, me llamo Santi Camps y soy el director técnico de KMKey. Estamos buscando un programador/a para incorporar a nuestro equipo. Pedimos titulación mínima de FP, familiarizado con el uso de GNU/Linux y con conocimientos de python y desarrollo web. Buen nivel de inglés escrito. El candidato seleccionado se incorporará a un proyecto en expansión basado en KMKey. Se ofrece posibilidad de teletrabajo total o parcial según características del candidato. Por favor, si hay alguien interesado que me mande su CV a santicamps en gmail.com Gracias -- Santi Camps KMKey (http://www.kmkey.com) Earcon S.L. (http://www.earcon.com) ------------ próxima parte ------------ Se ha borrado un adjunto en formato HTML... URL: From pych3m4 en gmail.com Wed Oct 31 12:41:22 2012 From: pych3m4 en gmail.com (Chema Cortes) Date: Wed, 31 Oct 2012 12:41:22 +0100 Subject: [Python-es] Estructuras de datos In-Reply-To: References: Message-ID: El día 30 de octubre de 2012 12:22, Kiko escribió: > > En este libro, por ejemplo, sí que tienes ejercicios: > http://eu.wiley.com/WileyCDA/WileyTitle/productCd-0470414634.html > En codeacademy tienes cursos de python (básicos): > http://www.codecademy.com/es/tracks/python > > Udacity, coursera,... Hoy en día hay muchos recursos. Coge uno y sigue uno. > Escribe mucho código y mira el código de los demás (github, bitbucket,...). > No te vuelvas loco a leer. > > Por otra parte, tienes esta charla de la pycon US de este año donde se > comenta que se abusa del uso de las clases: > http://pyvideo.org/video/880/stop-writing-classes > Un libro de reciente publicación que intenta ayudar a entender mejor la programación orientada a objetos: "Teoría sintáctico-gramatical de objetos (Python y PHP)" de Eugenia Bahit http://www.bubok.es/libros/219288/Teoria-sintacticogramatical-de-objetos -- Hyperreals *R: http://ch3m4.org/blog Quarks, bits y otras criaturas infinitesimales From jdpd en gmx.es Wed Oct 31 12:44:24 2012 From: jdpd en gmx.es (Jaime Perea) Date: Wed, 31 Oct 2012 12:44:24 +0100 Subject: [Python-es] Mision de esta lista In-Reply-To: References: <2163020.3H3bVHb2Kx@cristina> Message-ID: <2516210.ASNfJpeoxN@cristina> On Miércoles, 31 de octubre de 2012 12:07:03 lasizoillo escribió: > El día 31 de octubre de 2012 11:39, Jaime Perea escribió: > > Está bien que los mensajes se puedan entender. Mira lo siguiente, intento > > usar el gcc sobre un fichero que no existe. > > > > cristina: gcc lll.c > > gcc: error: lll.c: No existe el archivo o el directorio > > gcc: error fatal: no hay ficheros de entrada > > compilación terminada. > > > > cristina es mi laptop, va sobre kubuntu. Aquí se dice muy claro que tipo > > de > > error he cometido y eso es útil para cualquier persona que quiera compilar > > algo, no necesariamente un programador experto. > > > > O sea, si la gente que trabaja en algo tan básico y bajo nivel como el > > compilador de C, se han molestado en posibilitar la traducción, es que > > será > > útil, digo yo. De hecho gettext y similares existe desde hace mucho > > tiempo y no sólo sirve para los mensajes, > > > > Una de las razones más básicas del éxito de python, es que es cómodo y > > fácil de programar. Facilitarle a la gente el acceso a la programación > > es bueno. > Solo hay un problema con tu propuesta. ¿Cómo se traducen las > siguientes excepciones? > http://hg.python.org/cpython/file/9f464eff218a/Lib/gettext.py > > No se me ocurre ninguna forma limpia de romper las dependencias > circulares entre gettext y excepciones con tu planteamiento. Aunque > también es verdad que no le he dedicado tiempo a pensarlo, prefiero > dedicarle tiempo a aprender inglés ;-) > > Un saludo, > > Javi > _______________________________________________ > Python-es mailing list > Python-es en python.org > http://mail.python.org/mailman/listinfo/python-es > FAQ: http://python-es-faq.wikidot.com/ No hay que equivocarse, estamos hablando de cosas distintas. Yo lo que creo es que hay varios niveles en lo que te puedes involucrar en programación python. El más básico es el de gente que quiere hacer un script corto para resolver un problema. Hay varios ejemplos, en particular todo aquello que se refiere a gente que usa python como herramienta de cálculo o análisis, algo así como al nivel de matlab octave. Para este usuario, python (con scipy/numpy/matplotlib) es una bendición (incluso para el bolsillo) y por lo tanto no veo ninguna razón que impida facilitarles el trabajo. Hay más casos posibles, pero tienen que ver siempre con el uso de python como un lenguaje de programación fácil de aprender, fácil de utilizar y productivo desde el primer momento. Tanto, que incluso le perdonamos que no se pueda compilar. Más saludos -- Jaime D. Perea Duarte. Linux registered user #10472 Dep. Astrofisica Extragalactica. Instituto de Astrofisica de Andalucia (CSIC) Apdo. 3004, 18080 Granada, Spain. ------------ próxima parte ------------ Se ha borrado un adjunto en formato HTML... URL: From pych3m4 en gmail.com Wed Oct 31 13:20:55 2012 From: pych3m4 en gmail.com (Chema Cortes) Date: Wed, 31 Oct 2012 13:20:55 +0100 Subject: [Python-es] Rv: WXPYTHON no se cierra la ventana In-Reply-To: <1351679202.83210.YahooMailNeo@web171701.mail.ir2.yahoo.com> References: <1351679202.83210.YahooMailNeo@web171701.mail.ir2.yahoo.com> Message-ID: El día 31 de octubre de 2012 11:26, Gonzalo Mouriÿfffffffffff1o Pardo escribió: > Muchisimas gracias, pero al hacer esto no se me regeneran los dc.rectangle, > ya que en teoria el OnLeftDown, llama a un módulo aparte y al cerrarlo, ya > que dicho módulo, es una pantalla de entrada de datos. Tendría que ver el código. Acostumbra a que tus manejadores de eventos no filtren los eventos que necesitan los demás widgets. Por lo que cuentas, es casi seguro que el formulario que abres no deja pasar el evento EVT_PAINT a la ventana que estaba debajo. Hay quien se asegura generando a mano el evento cuando se cierra una ventana para que se refresquen las que estaban por debajo. Busca algún tutorial sobre el tema (no puedo recomendarte uno, no uso wx). > Ya he visto la FAQ, pero no se que es eso de los snippets. y por lo tanto no > se subir el código, te agradeceria que me dieras alguna indicacioón sobre > esto otro también. Servicios para compartir snippets hay muchos. Los más famosos son pastebin y gist, pero tienes una lista aquí: http://alternativeto.net/software/pastebin/ Estos servicios permiten responder a un snippet con otro o, en el caso de gist, controlarlo directamente desde git. > Gracias. Sólo una última cosa: evita hacer "top-posting" (http://es.wikipedia.org/wiki/Top-posting), dificulta saber qué parte del correo estás respondiendo. De nada sirve añadir "literalmente" el correo ya recibido con anterioridad. -- Hyperreals *R: http://ch3m4.org/blog Quarks, bits y otras criaturas infinitesimales From joe en joedicastro.com Wed Oct 31 13:38:06 2012 From: joe en joedicastro.com (joe di castro) Date: Wed, 31 Oct 2012 13:38:06 +0100 Subject: [Python-es] Mision de esta lista In-Reply-To: <2516210.ASNfJpeoxN@cristina> References: <2163020.3H3bVHb2Kx@cristina> <2516210.ASNfJpeoxN@cristina> Message-ID: <50911BAE.9000107@joedicastro.com> On 31/10/12 12:44, Jaime Perea wrote: > On Miércoles, 31 de octubre de 2012 12:07:03 lasizoillo escribió: > >> El día 31 de octubre de 2012 11:39, Jaime Perea escribió: > >> > Está bien que los mensajes se puedan entender. Mira lo siguiente, > intento > >> > usar el gcc sobre un fichero que no existe. > >> > > >> > cristina: gcc lll.c > >> > gcc: error: lll.c: No existe el archivo o el directorio > >> > gcc: error fatal: no hay ficheros de entrada > >> > compilación terminada. > >> > > >> > cristina es mi laptop, va sobre kubuntu. Aquí se dice muy claro que tipo > >> > de > >> > error he cometido y eso es útil para cualquier persona que quiera > compilar > >> > algo, no necesariamente un programador experto. > >> > > >> > O sea, si la gente que trabaja en algo tan básico y bajo nivel como el > >> > compilador de C, se han molestado en posibilitar la traducción, es que > >> > será > >> > útil, digo yo. De hecho gettext y similares existe desde hace mucho > >> > tiempo y no sólo sirve para los mensajes, > >> > > >> > Una de las razones más básicas del éxito de python, es que es cómodo y > >> > fácil de programar. Facilitarle a la gente el acceso a la programación > >> > es bueno. > >> Solo hay un problema con tu propuesta. ¿Cómo se traducen las > >> siguientes excepciones? > >> http://hg.python.org/cpython/file/9f464eff218a/Lib/gettext.py > >> > >> No se me ocurre ninguna forma limpia de romper las dependencias > >> circulares entre gettext y excepciones con tu planteamiento. Aunque > >> también es verdad que no le he dedicado tiempo a pensarlo, prefiero > >> dedicarle tiempo a aprender inglés ;-) > >> > >> Un saludo, > >> > >> Javi > >> _______________________________________________ > >> Python-es mailing list > >> Python-es en python.org > >> http://mail.python.org/mailman/listinfo/python-es > >> FAQ: http://python-es-faq.wikidot.com/ > > > > No hay que equivocarse, estamos hablando de cosas distintas. > > > > Yo lo que creo es que hay varios niveles en lo que te puedes involucrar > en programación python. El más básico es el de gente que quiere hacer un > script corto para resolver un problema. Hay varios ejemplos, en > particular todo aquello que se refiere a gente que usa python como > herramienta de cálculo o análisis, algo así como al nivel de matlab > octave. Para este usuario, python (con scipy/numpy/matplotlib) es una > bendición (incluso para el bolsillo) y por lo tanto no veo ninguna razón > que impida facilitarles el trabajo. > > > > Hay más casos posibles, pero tienen que ver siempre con el uso de python > como un lenguaje de programación fácil de aprender, fácil de utilizar y > productivo desde el primer momento. Tanto, que incluso le perdonamos que > no se pueda compilar. > > > > Más saludos > > > > -- > > Jaime D. Perea Duarte. > > Linux registered user #10472 > > > > Dep. Astrofisica Extragalactica. > > Instituto de Astrofisica de Andalucia (CSIC) > > Apdo. 3004, 18080 Granada, Spain. > > > > > > > > _______________________________________________ > Python-es mailing list > Python-es en python.org > http://mail.python.org/mailman/listinfo/python-es > FAQ: http://python-es-faq.wikidot.com/ > Cualquiera que vaya a emplear Python como herramienta para cálculos científicos va a encontrarse el escollo del Inglés en muchísimas más partes que en Python, eso por seguro. Aún digo más, cualquiera que se arriesgue a estudiar una ingeniería sin un dominio básico del inglés (y por básico, digo básico, no lo que entienden algunos españoles por básico) no sé que demonios tiene en la cabeza. Pero aún más, Windows el sistema operativo más difundido en el planeta se tiró muchos años lanzando pantallazos azules de la muerte, que daba igual que estuviesen en Inglés o en Japones, eran tan cripticos que no había dios que los entendiese, ni los técnicos, que teníamos que tirar de soporte. Y no digamos nada cuando te sale un volcado de pila... lo sigo manteniendo, los errores llevan toda la vida de dios siendo en inglés y no ha pasado nada... en los 80 veías a oficinistas entrados en los 40 manejándose con el MS-DOS y hoy parece que lo queremos todo fácil, cuando es más fácil que nunca buscar información y herramientas de traducción. Entiendo tu punto de vista... pero el esfuerzo que supone y sobre si no se hace un correcto mantenimiento, no creo que valga la pena. Sitios como Codecademy están plagados de usuarios de habla hispana, y están en Inglés. Saludos varios :) From reingart en gmail.com Wed Oct 31 15:37:00 2012 From: reingart en gmail.com (Mariano Reingart) Date: Wed, 31 Oct 2012 11:37:00 -0300 Subject: [Python-es] Mision de esta lista In-Reply-To: <50911BAE.9000107@joedicastro.com> References: <2163020.3H3bVHb2Kx@cristina> <2516210.ASNfJpeoxN@cristina> <50911BAE.9000107@joedicastro.com> Message-ID: Mariano Reingart http://www.sistemasagiles.com.ar http://reingart.blogspot.com 2012/10/31 joe di castro : > On 31/10/12 12:44, Jaime Perea wrote: >> On Miércoles, 31 de octubre de 2012 12:07:03 lasizoillo escribió: >> >>> El día 31 de octubre de 2012 11:39, Jaime Perea escribió: >> >>> > Está bien que los mensajes se puedan entender. Mira lo siguiente, >> intento >> >>> > usar el gcc sobre un fichero que no existe. >> >>> > >> >>> > cristina: gcc lll.c >> >>> > gcc: error: lll.c: No existe el archivo o el directorio >> >>> > gcc: error fatal: no hay ficheros de entrada >> >>> > compilación terminada. >> >>> > >> >>> > cristina es mi laptop, va sobre kubuntu. Aquí se dice muy claro que tipo >> >>> > de >> >>> > error he cometido y eso es útil para cualquier persona que quiera >> compilar >> >>> > algo, no necesariamente un programador experto. >> >>> > >> >>> > O sea, si la gente que trabaja en algo tan básico y bajo nivel como el >> >>> > compilador de C, se han molestado en posibilitar la traducción, es que >> >>> > será >> >>> > útil, digo yo. De hecho gettext y similares existe desde hace mucho >> >>> > tiempo y no sólo sirve para los mensajes, >> >>> > >> >>> > Una de las razones más básicas del éxito de python, es que es cómodo y >> >>> > fácil de programar. Facilitarle a la gente el acceso a la programación >> >>> > es bueno. >> >>> Solo hay un problema con tu propuesta. ¿Cómo se traducen las >> >>> siguientes excepciones? >> >>> http://hg.python.org/cpython/file/9f464eff218a/Lib/gettext.py >> >>> >> >>> No se me ocurre ninguna forma limpia de romper las dependencias >> >>> circulares entre gettext y excepciones con tu planteamiento. Aunque >> >>> también es verdad que no le he dedicado tiempo a pensarlo, prefiero >> >>> dedicarle tiempo a aprender inglés ;-) >> >>> >> >>> Un saludo, >> >>> >> >>> Javi >> >>> _______________________________________________ >> >>> Python-es mailing list >> >>> Python-es en python.org >> >>> http://mail.python.org/mailman/listinfo/python-es >> >>> FAQ: http://python-es-faq.wikidot.com/ >> >> >> >> No hay que equivocarse, estamos hablando de cosas distintas. >> >> >> >> Yo lo que creo es que hay varios niveles en lo que te puedes involucrar >> en programación python. El más básico es el de gente que quiere hacer un >> script corto para resolver un problema. Hay varios ejemplos, en >> particular todo aquello que se refiere a gente que usa python como >> herramienta de cálculo o análisis, algo así como al nivel de matlab >> octave. Para este usuario, python (con scipy/numpy/matplotlib) es una >> bendición (incluso para el bolsillo) y por lo tanto no veo ninguna razón >> que impida facilitarles el trabajo. >> >> >> >> Hay más casos posibles, pero tienen que ver siempre con el uso de python >> como un lenguaje de programación fácil de aprender, fácil de utilizar y >> productivo desde el primer momento. Tanto, que incluso le perdonamos que >> no se pueda compilar. >> >> >> >> Más saludos >> >> >> >> -- >> >> Jaime D. Perea Duarte. >> >> Linux registered user #10472 >> >> >> >> Dep. Astrofisica Extragalactica. >> >> Instituto de Astrofisica de Andalucia (CSIC) >> >> Apdo. 3004, 18080 Granada, Spain. >> >> >> >> >> >> >> >> _______________________________________________ >> Python-es mailing list >> Python-es en python.org >> http://mail.python.org/mailman/listinfo/python-es >> FAQ: http://python-es-faq.wikidot.com/ >> > > Cualquiera que vaya a emplear Python como herramienta para cálculos > científicos va a encontrarse el escollo del Inglés en muchísimas más > partes que en Python, eso por seguro. Aún digo más, cualquiera que se > arriesgue a estudiar una ingeniería sin un dominio básico del inglés (y > por básico, digo básico, no lo que entienden algunos españoles por > básico) no sé que demonios tiene en la cabeza. Ok, entonces Python según tu criterio no puede usarse antes de la educación formal superior (que por cierto, las materias iniciales de programación se ven en paralelo con las de inglés técnico...). Como mínimo me parece muy sesgado. ¿Que tendríamos que hacer? ¿Dejar de fomentar Python en las escuelas primarias y secundarias? http://robots.linti.unlp.edu.ar/index.php?title=P%C3%A1gina_principal (fijate la última foto) > > Pero aún más, Windows el sistema operativo más difundido en el planeta > se tiró muchos años lanzando pantallazos azules de la muerte, que daba > igual que estuviesen en Inglés o en Japones, eran tan cripticos que no > había dios que los entendiese, ni los técnicos, que teníamos que tirar > de soporte. Y no digamos nada cuando te sale un volcado de pila... lo > sigo manteniendo, los errores llevan toda la vida de dios siendo en > inglés y no ha pasado nada... en los 80 veías a oficinistas entrados en > los 40 manejándose con el MS-DOS y hoy parece que lo queremos todo > fácil, cuando es más fácil que nunca buscar información y herramientas > de traducción. Me parece que se están mezclando las cosas. Un volcado interno del núcleo no tiene ni siquiera mucho sentido mostrarlo al usuario, y traducirlo es técnicamente casi imposible. Que una herramienta sea más fácil de usar no implica que sea de menor calidad, sinceramente no entiendo tu postura. Que opcionalmente se muestren los mensajes de error en español o traduzcamos la documentación, no quiere decir que estamos yendo en contra de la profesión o precarizando las habilidades técnicas, al contrario, creo que estamos facilitando que haya nuevos programadores y ampliando la diversidad del lenguaje. > Entiendo tu punto de vista... pero el esfuerzo que supone y sobre si no > se hace un correcto mantenimiento, no creo que valga la pena. Con todo respeto, con ese criterio entonces cerremos esta lista y PyAr y borremos todos los recursos en Español... > Sitios como Codecademy están plagados de usuarios de habla hispana, y > están en Inglés. En mi humilde opinión, tenemos que fomentar que haya más materiales y recursos en nuestro idioma, no al contrario. Sds Mariano Reingart http://www.sistemasagiles.com.ar http://reingart.blogspot.com From reingart en gmail.com Wed Oct 31 15:47:27 2012 From: reingart en gmail.com (Mariano Reingart) Date: Wed, 31 Oct 2012 11:47:27 -0300 Subject: [Python-es] Mision de esta lista In-Reply-To: References: <5090EDA1.7040008@joedicastro.com> <2163020.3H3bVHb2Kx@cristina> Message-ID: 2012/10/31 Kiko : > > > El 31 de octubre de 2012 11:39, Jaime Perea escribió: > >> Está bien que los mensajes se puedan entender. Mira lo siguiente, intento >> usar el gcc sobre un fichero que no existe. >> >> cristina: gcc lll.c >> gcc: error: lll.c: No existe el archivo o el directorio >> gcc: error fatal: no hay ficheros de entrada >> compilación terminada. >> >> cristina es mi laptop, va sobre kubuntu. Aquí se dice muy claro que tipo >> de >> error he cometido y eso es útil para cualquier persona que quiera compilar >> algo, no necesariamente un programador experto. >> >> O sea, si la gente que trabaja en algo tan básico y bajo nivel como el >> compilador de C, se han molestado en posibilitar la traducción, es que >> será >> útil, digo yo. De hecho gettext y similares existe desde hace mucho >> tiempo y >> no sólo sirve para los mensajes, >> >> Una de las razones más básicas del éxito de python, es que es cómodo y >> fácil >> de programar. Facilitarle a la gente el acceso a la programación es >> bueno. > > > Por esta misma razón (llevándolo al absurdo), porqué no traducir for, while, > if, class,... :-) Porque Python (el lenguaje en sí) no es el ingles. Las palabras claves en general son tomadas del ingles, pero no así su gramática, puntuación, etc. > El tema es que el que se mete a programar debería de aprender inglés sí o sí > para poder desenvolverse correctamente (al igual que el que se mete en > ciencia sabe que el lenguaje estándar de facto es el inglés). Dicho esto, > debería establecerse una relación esfuerzo/rentabilidad de la traducción y > si se considera que la relación es aceptable, perfecto. Esto me parece > aceptable para, por ejemplo, el tutorial de python, algunos libros,... Para > lo que comenta Mariano, por ejemplo, no lo veo tan claro. Es mucho más difícil traducir la documentación que los mensajes de error, si vamos al analizarlo desde el punto de vista de esfuerzo/rentabilidad. Además, por ej, el tutorial no se utilizaría tan frecuente como los mensajes, que aparecen constantemente y sí implican una pérdida de productividad en el aula (por ej. por demoras, distracción, pérdida de concentración, etc.), al menos en mi caso. Nadie los obliga a traducir, y nadie los obliga incluso a usar la traducción (si leen bien la propuesta van a ver que es opcional y se puede deshabilitar o directamente no instalar). Sds Mariano Reingart http://www.sistemasagiles.com.ar http://reingart.blogspot.com From s_48k en yahoo.com Wed Oct 31 15:48:10 2012 From: s_48k en yahoo.com (Spectrum Cuarenta y ocho k) Date: Wed, 31 Oct 2012 07:48:10 -0700 (PDT) Subject: [Python-es] Mision de esta lista In-Reply-To: References: <2163020.3H3bVHb2Kx@cristina> <2516210.ASNfJpeoxN@cristina> <50911BAE.9000107@joedicastro.com> Message-ID: <1351694890.86420.YahooMailNeo@web163001.mail.bf1.yahoo.com> Aquí un voto para que todo en el lenguaje esté en inglés. Manuales en español para aprender a usar Python? claro, pero el resto en inglés. Y por un simple motivo, la comunidad Pyhton es mundial, y nos guste o no, el leguaje oficioso en la informática es el inglés. De hecho, si no recuerdo mal, en algún manual de estilo de Python habla de este tema y sugiere que todo el mundo ponga comentarios y mensajes en inglés para que cualquier persona en el mundo pueda entenderlo. Como ejemplo, cuando hacía taekwondo, le preguntamos al profesor que porqué teníamos que aprendernos los nombres de las técnicas en coreano, que era una putada. La respuesta fue la siguiente: hay que aprenderlo así, para que si vas a otro lugar puedas entenderte. Da igual que vayas a Francia, a Madagascar o a Iran, podrás seguir una clase de taekwondo. Pues el tema de la programación es lo mismo. Si un chino hace una librería buenísima, pero me lanza los mensajes en mandarín (simplificado, pero mandarín XD), la llevo clara. ------------ próxima parte ------------ Se ha borrado un adjunto en formato HTML... URL: From reingart en gmail.com Wed Oct 31 15:56:49 2012 From: reingart en gmail.com (Mariano Reingart) Date: Wed, 31 Oct 2012 11:56:49 -0300 Subject: [Python-es] Mision de esta lista In-Reply-To: References: <5090EDA1.7040008@joedicastro.com> <2163020.3H3bVHb2Kx@cristina> Message-ID: 2012/10/31 lasizoillo : > El día 31 de octubre de 2012 11:39, Jaime Perea escribió: >> Está bien que los mensajes se puedan entender. Mira lo siguiente, intento >> usar el gcc sobre un fichero que no existe. >> >> cristina: gcc lll.c >> gcc: error: lll.c: No existe el archivo o el directorio >> gcc: error fatal: no hay ficheros de entrada >> compilación terminada. >> >> cristina es mi laptop, va sobre kubuntu. Aquí se dice muy claro que tipo de >> error he cometido y eso es útil para cualquier persona que quiera compilar >> algo, no necesariamente un programador experto. >> >> O sea, si la gente que trabaja en algo tan básico y bajo nivel como el >> compilador de C, se han molestado en posibilitar la traducción, es que será >> útil, digo yo. De hecho gettext y similares existe desde hace mucho tiempo y >> no sólo sirve para los mensajes, >> >> Una de las razones más básicas del éxito de python, es que es cómodo y fácil >> de programar. Facilitarle a la gente el acceso a la programación es bueno. >> > > Solo hay un problema con tu propuesta. ¿Cómo se traducen las > siguientes excepciones? > http://hg.python.org/cpython/file/9f464eff218a/Lib/gettext.py Hasta donde sé, el módulo gettext.py esta implementado en python, y no es lo que se usaría para traducir las excepciones. Si ves en el parche, Py_GETTEXT es un macro a nivel C que llama a la funcion gettext directamente (librería en C), no pasa por este modulo. > No se me ocurre ninguna forma limpia de romper las dependencias > circulares entre gettext y excepciones con tu planteamiento. Aunque > también es verdad que no le he dedicado tiempo a pensarlo, prefiero > dedicarle tiempo a aprender inglés ;-) Suerte que tu puedes dedicarle tiempo a aprender ingles, muchos de mis alumnos y colegas no :-( (y en muchos casos es más complejo por un tema de recursos económicos, habilidades, contexto, etc., aún para aquellos que han tenido algunas materias de ingles en su educación...) Sds Mariano Reingart http://www.sistemasagiles.com.ar http://reingart.blogspot.com From reingart en gmail.com Wed Oct 31 16:11:12 2012 From: reingart en gmail.com (Mariano Reingart) Date: Wed, 31 Oct 2012 12:11:12 -0300 Subject: [Python-es] Mision de esta lista In-Reply-To: <1351694890.86420.YahooMailNeo@web163001.mail.bf1.yahoo.com> References: <2163020.3H3bVHb2Kx@cristina> <2516210.ASNfJpeoxN@cristina> <50911BAE.9000107@joedicastro.com> <1351694890.86420.YahooMailNeo@web163001.mail.bf1.yahoo.com> Message-ID: 2012/10/31 Spectrum Cuarenta y ocho k : > > Aquí un voto para que todo en el lenguaje esté en inglés. Manuales en > español para aprender a usar Python? claro, pero el resto en inglés. Y por > un simple motivo, la comunidad Pyhton es mundial, y nos guste o no, el > leguaje oficioso en la informática es el inglés. De hecho, si no recuerdo > mal, en algún manual de estilo de Python habla de este tema y sugiere que > todo el mundo ponga comentarios y mensajes en inglés para que cualquier > persona en el mundo pueda entenderlo. Nadie dice lo contrario. El código principal será siendo escrito en Ingles, tanto los mensajes como comentarios. Solo se agrega la localización por fuera del código y modificando un par de archivos. > Como ejemplo, cuando hacía taekwondo, le preguntamos al profesor que porqué > teníamos que aprendernos los nombres de las técnicas en coreano, que era una > putada. La respuesta fue la siguiente: hay que aprenderlo así, para que si > vas a otro lugar puedas entenderte. Da igual que vayas a Francia, a > Madagascar o a Iran, podrás seguir una clase de taekwondo. Pues el tema de > la programación es lo mismo. Si un chino hace una librería buenísima, pero > me lanza los mensajes en mandarín (simplificado, pero mandarín XD), la llevo > clara. No se plantea que se traduzcan las palabras clave o nombres de las excepciones, solo los mensajes de error, y no en el código en si, por lo que si un chino escribe una biblioteca nueva, el mensaje te aparecerá en inglés hasta que alguien lo traduzca. Incluso se están analizando alternativas de hacerlo en paralelo (teniendo la versión original en inglés y/o un código de error, etc.) Igualmente, por mas que te salga el mensaje en ingles, si el usuario no sabe inglés, difícilmente pueda reportar su problema por ej al bug tracker (¿como escribiría el issue en ingles?), y difícilmente se pueda comunicar con otros colaboradores de la comunidad internacional (¿como escribiría los mails en ingles?). En cambio, podríamos cooperar entre todos para tener una comunidad más abarcativa y más diversa, que seguramente se va a ver beneficiada por una mayor cantidad de usuarios de diferentes niveles e idiomas. Ahi es donde nuestras comunidades locales se deben hacer fuerte y ayudar a las personas que no saben inglés, no expulsandolas diciendole "Anda a aprender ingles porque sino sos un mal programador...". Me parece que es un mensaje errado, algo mejor sería "Te ayudamos a dar los primeros pasos de programación con un Python localizado en tu idioma, pero considera aprender inglés para comunicarte mejor con el resto de la comunidad internacional". Las personas pueden elegir aprender ingles, o también pueden elegir otros lenguajes que ya están internacionalizados como .NET o PHP (en menor medida). Con todo respeto, me parece que oponerse sin leer la propuesta o por cuestión de gustos no sería lo más conveniente en estos casos. Sds Mariano Reingart http://www.sistemasagiles.com.ar http://reingart.blogspot.com From kikocorreoso en gmail.com Wed Oct 31 16:35:04 2012 From: kikocorreoso en gmail.com (Kiko) Date: Wed, 31 Oct 2012 16:35:04 +0100 Subject: [Python-es] Mision de esta lista In-Reply-To: References: <2163020.3H3bVHb2Kx@cristina> <2516210.ASNfJpeoxN@cristina> <50911BAE.9000107@joedicastro.com> <1351694890.86420.YahooMailNeo@web163001.mail.bf1.yahoo.com> Message-ID: El 31 de octubre de 2012 16:11, Mariano Reingart escribió: > 2012/10/31 Spectrum Cuarenta y ocho k : > > > > Aquí un voto para que todo en el lenguaje esté en inglés. Manuales en > > español para aprender a usar Python? claro, pero el resto en inglés. Y > por > > un simple motivo, la comunidad Pyhton es mundial, y nos guste o no, el > > leguaje oficioso en la informática es el inglés. De hecho, si no recuerdo > > mal, en algún manual de estilo de Python habla de este tema y sugiere que > > todo el mundo ponga comentarios y mensajes en inglés para que cualquier > > persona en el mundo pueda entenderlo. > > Nadie dice lo contrario. > El código principal será siendo escrito en Ingles, tanto los mensajes > como comentarios. > Solo se agrega la localización por fuera del código y modificando un > par de archivos. > > > Como ejemplo, cuando hacía taekwondo, le preguntamos al profesor que > porqué > > teníamos que aprendernos los nombres de las técnicas en coreano, que era > una > > putada. La respuesta fue la siguiente: hay que aprenderlo así, para que > si > > vas a otro lugar puedas entenderte. Da igual que vayas a Francia, a > > Madagascar o a Iran, podrás seguir una clase de taekwondo. Pues el tema > de > > la programación es lo mismo. Si un chino hace una librería buenísima, > pero > > me lanza los mensajes en mandarín (simplificado, pero mandarín XD), la > llevo > > clara. > > No se plantea que se traduzcan las palabras clave o nombres de las > excepciones, solo los mensajes de error, y no en el código en si, por > lo que si un chino escribe una biblioteca nueva, el mensaje te > aparecerá en inglés hasta que alguien lo traduzca. > > Incluso se están analizando alternativas de hacerlo en paralelo > (teniendo la versión original en inglés y/o un código de error, etc.) > > Igualmente, por mas que te salga el mensaje en ingles, si el usuario > no sabe inglés, difícilmente pueda reportar su problema por ej al bug > tracker (¿como escribiría el issue en ingles?), y difícilmente se > pueda comunicar con otros colaboradores de la comunidad internacional > (¿como escribiría los mails en ingles?). > En cambio, podríamos cooperar entre todos para tener una comunidad más > abarcativa y más diversa, que seguramente se va a ver beneficiada por > una mayor cantidad de usuarios de diferentes niveles e idiomas. > > Ahi es donde nuestras comunidades locales se deben hacer fuerte y > ayudar a las personas que no saben inglés, no expulsandolas diciendole > "Anda a aprender ingles porque sino sos un mal programador...". > Me parece que es un mensaje errado, algo mejor sería "Te ayudamos a > dar los primeros pasos de programación con un Python localizado en tu > idioma, pero considera aprender inglés para comunicarte mejor con el > resto de la comunidad internacional". > > Las personas pueden elegir aprender ingles, o también pueden elegir > otros lenguajes que ya están internacionalizados como .NET o PHP (en > menor medida). > > Con todo respeto, me parece que oponerse sin leer la propuesta o por > cuestión de gustos no sería lo más conveniente en estos casos. > > Yo no creo que nadie se oponga, simplemente hay gente a la que le puede parecer poco interesante hacer una traducción de determinadas cosas. No lo tomes como un ataque hacia ese proyecto en concreto. Por la misma regla de tres, ¿si enseñamos a los alumnos un lenguaje (python), que hay de malo en que a la vez aprendan otro (inglés) aprovechando que python está escrito en inglés? Se matan dos pájaros de un tiro y el inglés es algo que les será útil en su futuro, quizá más que python :-P Saludos. ------------ próxima parte ------------ Se ha borrado un adjunto en formato HTML... URL: From monobot.soft en gmail.com Wed Oct 31 17:04:40 2012 From: monobot.soft en gmail.com (monoBOT) Date: Wed, 31 Oct 2012 16:04:40 +0000 Subject: [Python-es] Mision de esta lista In-Reply-To: References: <2163020.3H3bVHb2Kx@cristina> <2516210.ASNfJpeoxN@cristina> <50911BAE.9000107@joedicastro.com> <1351694890.86420.YahooMailNeo@web163001.mail.bf1.yahoo.com> Message-ID: No creo que sea necesario la traducción del lenguaje a otros idiomas, como ya se ha mencionado no tiene sentido, el lenguaje nace en clave inglesa (for, if, then, etc...) y así debería de quedar. Otra cosa es crear un intérprete "españolizado" para que los peques puedan entrar en la programación desde un prisma mas sencillo. Que aprendan la lógica sin tener que aprender la palabra anglosajona. Me parece un proyecto interesante, pero teniendo en cuenta que la ambición del mismo ha de ser determinada, intentar traducir todo el lenguaje es como tener que reescribirlo entero. mis 2 céntimos El 31 de octubre de 2012 15:11, Mariano Reingart escribió: > 2012/10/31 Spectrum Cuarenta y ocho k : > > > > Aquí un voto para que todo en el lenguaje esté en inglés. Manuales en > > español para aprender a usar Python? claro, pero el resto en inglés. Y > por > > un simple motivo, la comunidad Pyhton es mundial, y nos guste o no, el > > leguaje oficioso en la informática es el inglés. De hecho, si no recuerdo > > mal, en algún manual de estilo de Python habla de este tema y sugiere que > > todo el mundo ponga comentarios y mensajes en inglés para que cualquier > > persona en el mundo pueda entenderlo. > > Nadie dice lo contrario. > El código principal será siendo escrito en Ingles, tanto los mensajes > como comentarios. > Solo se agrega la localización por fuera del código y modificando un > par de archivos. > > > Como ejemplo, cuando hacía taekwondo, le preguntamos al profesor que > porqué > > teníamos que aprendernos los nombres de las técnicas en coreano, que era > una > > putada. La respuesta fue la siguiente: hay que aprenderlo así, para que > si > > vas a otro lugar puedas entenderte. Da igual que vayas a Francia, a > > Madagascar o a Iran, podrás seguir una clase de taekwondo. Pues el tema > de > > la programación es lo mismo. Si un chino hace una librería buenísima, > pero > > me lanza los mensajes en mandarín (simplificado, pero mandarín XD), la > llevo > > clara. > > No se plantea que se traduzcan las palabras clave o nombres de las > excepciones, solo los mensajes de error, y no en el código en si, por > lo que si un chino escribe una biblioteca nueva, el mensaje te > aparecerá en inglés hasta que alguien lo traduzca. > > Incluso se están analizando alternativas de hacerlo en paralelo > (teniendo la versión original en inglés y/o un código de error, etc.) > > Igualmente, por mas que te salga el mensaje en ingles, si el usuario > no sabe inglés, difícilmente pueda reportar su problema por ej al bug > tracker (¿como escribiría el issue en ingles?), y difícilmente se > pueda comunicar con otros colaboradores de la comunidad internacional > (¿como escribiría los mails en ingles?). > En cambio, podríamos cooperar entre todos para tener una comunidad más > abarcativa y más diversa, que seguramente se va a ver beneficiada por > una mayor cantidad de usuarios de diferentes niveles e idiomas. > > Ahi es donde nuestras comunidades locales se deben hacer fuerte y > ayudar a las personas que no saben inglés, no expulsandolas diciendole > "Anda a aprender ingles porque sino sos un mal programador...". > Me parece que es un mensaje errado, algo mejor sería "Te ayudamos a > dar los primeros pasos de programación con un Python localizado en tu > idioma, pero considera aprender inglés para comunicarte mejor con el > resto de la comunidad internacional". > > Las personas pueden elegir aprender ingles, o también pueden elegir > otros lenguajes que ya están internacionalizados como .NET o PHP (en > menor medida). > > Con todo respeto, me parece que oponerse sin leer la propuesta o por > cuestión de gustos no sería lo más conveniente en estos casos. > > Sds > > Mariano Reingart > http://www.sistemasagiles.com.ar > http://reingart.blogspot.com > _______________________________________________ > Python-es mailing list > Python-es en python.org > http://mail.python.org/mailman/listinfo/python-es > FAQ: http://python-es-faq.wikidot.com/ > -- *monoBOT* Visite mi sitio(Visit my site): monobotblog.alvarezalonso.es ------------ próxima parte ------------ Se ha borrado un adjunto en formato HTML... URL: From ricardo.cardenes en gmail.com Wed Oct 31 17:18:57 2012 From: ricardo.cardenes en gmail.com (=?ISO-8859-1?Q?Ricardo_C=E1rdenes?=) Date: Wed, 31 Oct 2012 16:18:57 +0000 Subject: [Python-es] Mision de esta lista In-Reply-To: References: <2163020.3H3bVHb2Kx@cristina> <2516210.ASNfJpeoxN@cristina> <50911BAE.9000107@joedicastro.com> <1351694890.86420.YahooMailNeo@web163001.mail.bf1.yahoo.com> Message-ID: Si es para peques, se podría hacer como con LOGO en su día. Yo recuerdo haber empezado a programar guiando a la tortuga diciéndole "AVANZA 50" y no "FORWARD 50" :) No es tanto trabajo como parece. 2012/10/31 monoBOT : > No creo que sea necesario la traducción del lenguaje a otros idiomas, como > ya se ha mencionado no tiene sentido, el lenguaje nace en clave inglesa > (for, if, then, etc...) y así debería de quedar. Otra cosa es crear un > intérprete "españolizado" para que los peques puedan entrar en la > programación desde un prisma mas sencillo. Que aprendan la lógica sin tener > que aprender la palabra anglosajona. > > Me parece un proyecto interesante, pero teniendo en cuenta que la ambición > del mismo ha de ser determinada, intentar traducir todo el lenguaje es como > tener que reescribirlo entero. > > mis 2 céntimos > > > > > El 31 de octubre de 2012 15:11, Mariano Reingart > escribió: > >> 2012/10/31 Spectrum Cuarenta y ocho k : >> > >> > Aquí un voto para que todo en el lenguaje esté en inglés. Manuales en >> > español para aprender a usar Python? claro, pero el resto en inglés. Y >> > por >> > un simple motivo, la comunidad Pyhton es mundial, y nos guste o no, el >> > leguaje oficioso en la informática es el inglés. De hecho, si no >> > recuerdo >> > mal, en algún manual de estilo de Python habla de este tema y sugiere >> > que >> > todo el mundo ponga comentarios y mensajes en inglés para que cualquier >> > persona en el mundo pueda entenderlo. >> >> Nadie dice lo contrario. >> El código principal será siendo escrito en Ingles, tanto los mensajes >> como comentarios. >> Solo se agrega la localización por fuera del código y modificando un >> par de archivos. >> >> > Como ejemplo, cuando hacía taekwondo, le preguntamos al profesor que >> > porqué >> > teníamos que aprendernos los nombres de las técnicas en coreano, que era >> > una >> > putada. La respuesta fue la siguiente: hay que aprenderlo así, para que >> > si >> > vas a otro lugar puedas entenderte. Da igual que vayas a Francia, a >> > Madagascar o a Iran, podrás seguir una clase de taekwondo. Pues el tema >> > de >> > la programación es lo mismo. Si un chino hace una librería buenísima, >> > pero >> > me lanza los mensajes en mandarín (simplificado, pero mandarín XD), la >> > llevo >> > clara. >> >> No se plantea que se traduzcan las palabras clave o nombres de las >> excepciones, solo los mensajes de error, y no en el código en si, por >> lo que si un chino escribe una biblioteca nueva, el mensaje te >> aparecerá en inglés hasta que alguien lo traduzca. >> >> Incluso se están analizando alternativas de hacerlo en paralelo >> (teniendo la versión original en inglés y/o un código de error, etc.) >> >> Igualmente, por mas que te salga el mensaje en ingles, si el usuario >> no sabe inglés, difícilmente pueda reportar su problema por ej al bug >> tracker (¿como escribiría el issue en ingles?), y difícilmente se >> pueda comunicar con otros colaboradores de la comunidad internacional >> (¿como escribiría los mails en ingles?). >> En cambio, podríamos cooperar entre todos para tener una comunidad más >> abarcativa y más diversa, que seguramente se va a ver beneficiada por >> una mayor cantidad de usuarios de diferentes niveles e idiomas. >> >> Ahi es donde nuestras comunidades locales se deben hacer fuerte y >> ayudar a las personas que no saben inglés, no expulsandolas diciendole >> "Anda a aprender ingles porque sino sos un mal programador...". >> Me parece que es un mensaje errado, algo mejor sería "Te ayudamos a >> dar los primeros pasos de programación con un Python localizado en tu >> idioma, pero considera aprender inglés para comunicarte mejor con el >> resto de la comunidad internacional". >> >> Las personas pueden elegir aprender ingles, o también pueden elegir >> otros lenguajes que ya están internacionalizados como .NET o PHP (en >> menor medida). >> >> Con todo respeto, me parece que oponerse sin leer la propuesta o por >> cuestión de gustos no sería lo más conveniente en estos casos. >> >> Sds >> >> Mariano Reingart >> http://www.sistemasagiles.com.ar >> http://reingart.blogspot.com >> _______________________________________________ >> Python-es mailing list >> Python-es en python.org >> http://mail.python.org/mailman/listinfo/python-es >> FAQ: http://python-es-faq.wikidot.com/ > > > > > -- > monoBOT > Visite mi sitio(Visit my site): monobotblog.alvarezalonso.es > > > _______________________________________________ > Python-es mailing list > Python-es en python.org > http://mail.python.org/mailman/listinfo/python-es > FAQ: http://python-es-faq.wikidot.com/ > From juanlu001 en gmail.com Wed Oct 31 17:31:42 2012 From: juanlu001 en gmail.com (=?ISO-8859-1?Q?Juan_Luis_Cano_Rodr=EDguez?=) Date: Wed, 31 Oct 2012 17:31:42 +0100 Subject: [Python-es] Mision de esta lista In-Reply-To: References: <2163020.3H3bVHb2Kx@cristina> <2516210.ASNfJpeoxN@cristina> <50911BAE.9000107@joedicastro.com> <1351694890.86420.YahooMailNeo@web163001.mail.bf1.yahoo.com> Message-ID: Me gustaría a mí también aportar mis dos céntimos a la conversación. Antes de nada quiero puntualizar que yo también estoy trabajando por la comunidad Python en castellano[1][2], así que digamos que no soy simplemente un observador que critica sin saber. El comentario de Mariano sobre dejar de enseñar Python en las escuelas primarias por estar en inglés me parece fuera de lugar. De hecho, los niños deberían aprender inglés bastante antes que Python. Y, con el debido respeto, la foto queda muy bien pero son niños mirando un robot; para leer una traza de error, ya esté en tu lengua materna o no, hacen falta una serie de estructuras mentales que estos niños no tienen porqué tener aún. Hay compañeros míos de universidad (ingeniería aeronáutica) que no saben interpretar una traza de error. Por otro lado, los argumentos de que para hacerlo a medias o medio mal, mejor no lo hagamos, no me parecen bien tampoco. No se va a incluir nada en la CPython que esté a medias, de eso podemos estar seguros. En todo caso habrá que ver si es útil, factible, empezarlo y, eventualmente, terminarlo. Creo que estamos exagerando un poco las cosas: todos sabemos que no hace falta hablar como Shakespeare para programar. Es un subconjunto del inglés *muy* reducido el que hay que comprender para interpretar una traza de error, o para saber que _if_ es _si_. Estoy de acuerdo con lo que han dicho arriba de que aprender el lenguaje con recursos, tutoriales, libros en español es fabuloso y genial, pero que traducir el lenguaje en sí (o una parte como es este caso las trazas de error) nos situaría en una especie de gueto, nos aislaría. El ejemplo del taekwondo me parece muy ilustrativo, y seguro que hay muchos más. Y el tema de informar de errores también funciona a la inversa: no todos los programadores saben español. Y si me sale una traza en español e informo del error, ¿se supone que solo lo tienen que solucionar programadores hispanos? No tiene sentido. Compartir todos el mismo lenguaje nos ayuda a cooperar con la comunidad internacional, que es bastante mayor que la comunidad hispana. Y dicho todo esto, si hay gente que quiere emplear su tiempo en traducir las trazas no seré yo quien se enfade o les impida hacerlo, pero creo que hay cosas más importantes ahora mismo. No solo los recursos que mencionaba el autor del hilo original, estoy pensando también en la traducción ya un poco obsoleta del tutorial de Python (2.6?), y también en la blogosfera python-hispana, que produce bastante poco contenido original en castellano (algunos blogueros del planet prefieren escribir en inglés, ¿por qué?). Un saludo a todos. [1]: http://pybonacci.wordpress.com/ [2]: http://wiki.python.org/moin/SpanishLanguage#pageinfo ------------ próxima parte ------------ Se ha borrado un adjunto en formato HTML... URL: From s_48k en yahoo.com Wed Oct 31 18:11:58 2012 From: s_48k en yahoo.com (Spectrum Cuarenta y ocho k) Date: Wed, 31 Oct 2012 10:11:58 -0700 (PDT) Subject: [Python-es] Mision de esta lista Message-ID: <1351703518.40457.YahooMailNeo@web163006.mail.bf1.yahoo.com> Si lo dices por mi, me he leído todo el hilo completo y no es que me oponga, es que mi opinión (apoyada en mis convicciones) es distinta, simple y llanamente. Por otra parte, el vocabulario que hay que controlar para entender los errores no es muy extenso. Con el ejemplo del chino, me refería a la '''documentación de la librería''' ------------ próxima parte ------------ Se ha borrado un adjunto en formato HTML... URL: From joe en joedicastro.com Wed Oct 31 18:18:55 2012 From: joe en joedicastro.com (joe di castro) Date: Wed, 31 Oct 2012 18:18:55 +0100 Subject: [Python-es] Mision de esta lista In-Reply-To: References: <2163020.3H3bVHb2Kx@cristina> <2516210.ASNfJpeoxN@cristina> <50911BAE.9000107@joedicastro.com> <1351694890.86420.YahooMailNeo@web163001.mail.bf1.yahoo.com> Message-ID: <50915D7F.7050704@joedicastro.com> On 31/10/12 17:31, Juan Luis Cano Rodríguez wrote: > Me gustaría a mí también aportar mis dos céntimos a la conversación. > > Antes de nada quiero puntualizar que yo también estoy trabajando por la > comunidad Python en castellano[1][2], así que digamos que no soy > simplemente un observador que critica sin saber. > > El comentario de Mariano sobre dejar de enseñar Python en las escuelas > primarias por estar en inglés me parece fuera de lugar. De hecho, los > niños deberían aprender inglés bastante antes que Python. Y, con el > debido respeto, la foto queda muy bien pero son niños mirando un robot; > para leer una traza de error, ya esté en tu lengua materna o no, hacen > falta una serie de estructuras mentales que estos niños no tienen porqué > tener aún. Hay compañeros míos de universidad (ingeniería aeronáutica) > que no saben interpretar una traza de error. > > Por otro lado, los argumentos de que para hacerlo a medias o medio mal, > mejor no lo hagamos, no me parecen bien tampoco. No se va a incluir nada > en la CPython que esté a medias, de eso podemos estar seguros. En todo > caso habrá que ver si es útil, factible, empezarlo y, eventualmente, > terminarlo. > > Creo que estamos exagerando un poco las cosas: todos sabemos que no hace > falta hablar como Shakespeare para programar. Es un subconjunto del > inglés *muy* reducido el que hay que comprender para interpretar una > traza de error, o para saber que _if_ es _si_. > > Estoy de acuerdo con lo que han dicho arriba de que aprender el lenguaje > con recursos, tutoriales, libros en español es fabuloso y genial, pero > que traducir el lenguaje en sí (o una parte como es este caso las trazas > de error) nos situaría en una especie de gueto, nos aislaría. El ejemplo > del taekwondo me parece muy ilustrativo, y seguro que hay muchos más. > > Y el tema de informar de errores también funciona a la inversa: no todos > los programadores saben español. Y si me sale una traza en español e > informo del error, ¿se supone que solo lo tienen que solucionar > programadores hispanos? No tiene sentido. Compartir todos el mismo > lenguaje nos ayuda a cooperar con la comunidad internacional, que es > bastante mayor que la comunidad hispana. > > Y dicho todo esto, si hay gente que quiere emplear su tiempo en traducir > las trazas no seré yo quien se enfade o les impida hacerlo, pero creo > que hay cosas más importantes ahora mismo. No solo los recursos que > mencionaba el autor del hilo original, estoy pensando también en la > traducción ya un poco obsoleta del tutorial de Python (2.6?), y también > en la blogosfera python-hispana, que produce bastante poco contenido > original en castellano (algunos blogueros del planet prefieren escribir > en inglés, ¿por qué?). > > Un saludo a todos. > > [1]: http://pybonacci.wordpress.com/ > [2]: http://wiki.python.org/moin/SpanishLanguage#pageinfo > > > _______________________________________________ > Python-es mailing list > Python-es en python.org > http://mail.python.org/mailman/listinfo/python-es > FAQ: http://python-es-faq.wikidot.com/ > Después de esta magnífica respuesta del compañero Juan Luís, poco queda que añadir. En cuanto a fomentar los recursos en nuestro idioma, el que me conozca un poco sabe perfectamente que no soy precisamente de los que no impulsan la difusión de Python en la comunidad hispana. De todos modos, Mariano, entiendo tu diatriba acerca de la situación que te toca vivir: la falta de recursos, falta de tiempo, habilidades, etc. Entiendo que en ciertas partes somos más "afortunados" que en otros lados, en teoría. Y digo en teoría porque vivo en un país en el que se estudia el Inglés durante bastante tiempo en edad escolar, muchos lo han estudiado durante muchos años y la gran mayoría no tiene ni puta idea del idioma, un nivel deleznable del mismo. Y no es por falta de recursos, créeme, al menos hasta hace poco. Sobran bibliotecas, academias, escuelas oficiales de idiomas, etc... ¿Sabes como aprendí yo inglés? Con un diccionario Larousse de bolsillo (unos 20? en el 92) y traduciendo canciones de grupos de Rock y Heavy Metal (saque la mejor nota posible en la asignatura dos años seguidos sin casi atender en clase). Te puedo asegurar que cuando llegué a estudiar informática tenia un nivel más que suficiente para enfrentarme a lecturas técnicas. Lo he ido mejorando mucho durante años con series extranjeras, leyendo muchos libros en Inglés (técnicos y literatura) y con el trabajo diario (no me quedaba otra). Lo que te quiero decir con todo esto, es que si se quiere, se puede. Los medios disponibles no te van a ahorrar el esfuerzo personal, nunca, en cambio el esfuerzo personal puede suplir la ausencia de muchos medios. Y te digo más, en países donde la escasez de medios y la educación es deficiente la necesidad de aprender Inglés es aún más imperiosa. Porque es la lengua que te va a abrir innumerables puertas en el futuro, en la educación, en el trabajo, ... ¿Por qué crees que la India se ha convertido en uno de las países donde más se deriva el desarrollo de aplicaciones a terceros? ¿Crees que son mejores programadores que los demás? En absoluto, pero cuentan con la tremenda ventaja del idioma, donde el Inglés es una lengua oficial (junto a la mano de obra barata, por supuesto). Y han sabido aprovechar esa ventaja para vender su "talento" y fomentarlo aún más. Pero es que esto ocurre en casi todas las ciencias e ingenierias, es la lengua vehicular, como en su día fue el francés en otros ámbitos y antiguamente el latín. Y ya no te digo nada en los negocios internacionales. Si yo trabajando en España he tenido que "arreglarme" hablando con personas en fránces, italiano, portugués y por supuesto Inglés. Una cosa es fomentar tu idioma y otra es nadar contracorriente. A lo mejor, lo que mejor que puedes hacer con esos alumnos es ayudarles a traducir esos mensajes de error (tampoco es para tanto) y enriquecer, quien sabe si no incentivar, el conocimiento del inglés. Y los críos a esa edad son como una esponja, lo absorben todo. Saludos From jza en oooes.org Wed Oct 31 18:38:46 2012 From: jza en oooes.org (Alexandro Colorado) Date: Wed, 31 Oct 2012 11:38:46 -0600 Subject: [Python-es] Mision de esta lista In-Reply-To: <50915D7F.7050704@joedicastro.com> References: <2163020.3H3bVHb2Kx@cristina> <2516210.ASNfJpeoxN@cristina> <50911BAE.9000107@joedicastro.com> <1351694890.86420.YahooMailNeo@web163001.mail.bf1.yahoo.com> <50915D7F.7050704@joedicastro.com> Message-ID: Uff creo que esta discusion se ha ido por otro lado. Tambien creo que muchos tienen puntos validos, sin embargo el software libre creo que es de hacer, mas que de discutir. Personalemente pienso que mas de intentar probar que el compañero Mariano no beneficiara a nadie. Creo que es mejor emplear este tiempo, en crear o apoyar un proyecto de localizacion sobre Python alternativo. Hacerse un screencast en Youtube que pueda explicar bien los conceptos de python y hacer un video curso. O ayudar a traducir los libros que mencione anteriormente. He visto que aun falta actualizarse las versiones hispanas de: - Byte of Python - How to think like a computer scientist - Doma la serpiente - Think Python - Python no muerde, yo si Estos libros, los que tienen la fortuna de haber sido traducidos aun estan en versiones antiguas para python 2.4 o incompletas. Creo que una mejor propuesta seria empujar estos proyectos. Yo por lo menos puedo apuntar a las personas al SVN de Google Code donde existe: http://code.google.com/p/swfk-es/ Apenas han liberado la version 0.4 cuando la version actual en ingles va por la 0.7 http://code.google.com/p/swfk/ Lo mismo puedo hablar de Think like a computer scientist, donde esta abandonado aun: http://www.openbookproject.net/thinkcs/archive/python/thinkcspyesp3e_abandonado/ Original: http://www.openbookproject.net/thinkcs/ Y finalmente Think Python donde esta en un repositorio de SourceForge Original: http://www.greenteapress.com/thinkpython/ Proyecto hispano: http://sourceforge.net/projects/httlcseifspa/ On 10/31/12, joe di castro wrote: > On 31/10/12 17:31, Juan Luis Cano Rodríguez wrote: >> Me gustaría a mí también aportar mis dos céntimos a la conversación. >> >> Antes de nada quiero puntualizar que yo también estoy trabajando por la >> comunidad Python en castellano[1][2], así que digamos que no soy >> simplemente un observador que critica sin saber. >> >> El comentario de Mariano sobre dejar de enseñar Python en las escuelas >> primarias por estar en inglés me parece fuera de lugar. De hecho, los >> niños deberían aprender inglés bastante antes que Python. Y, con el >> debido respeto, la foto queda muy bien pero son niños mirando un robot; >> para leer una traza de error, ya esté en tu lengua materna o no, hacen >> falta una serie de estructuras mentales que estos niños no tienen porqué >> tener aún. Hay compañeros míos de universidad (ingeniería aeronáutica) >> que no saben interpretar una traza de error. >> >> Por otro lado, los argumentos de que para hacerlo a medias o medio mal, >> mejor no lo hagamos, no me parecen bien tampoco. No se va a incluir nada >> en la CPython que esté a medias, de eso podemos estar seguros. En todo >> caso habrá que ver si es útil, factible, empezarlo y, eventualmente, >> terminarlo. >> >> Creo que estamos exagerando un poco las cosas: todos sabemos que no hace >> falta hablar como Shakespeare para programar. Es un subconjunto del >> inglés *muy* reducido el que hay que comprender para interpretar una >> traza de error, o para saber que _if_ es _si_. >> >> Estoy de acuerdo con lo que han dicho arriba de que aprender el lenguaje >> con recursos, tutoriales, libros en español es fabuloso y genial, pero >> que traducir el lenguaje en sí (o una parte como es este caso las trazas >> de error) nos situaría en una especie de gueto, nos aislaría. El ejemplo >> del taekwondo me parece muy ilustrativo, y seguro que hay muchos más. >> >> Y el tema de informar de errores también funciona a la inversa: no todos >> los programadores saben español. Y si me sale una traza en español e >> informo del error, ¿se supone que solo lo tienen que solucionar >> programadores hispanos? No tiene sentido. Compartir todos el mismo >> lenguaje nos ayuda a cooperar con la comunidad internacional, que es >> bastante mayor que la comunidad hispana. >> >> Y dicho todo esto, si hay gente que quiere emplear su tiempo en traducir >> las trazas no seré yo quien se enfade o les impida hacerlo, pero creo >> que hay cosas más importantes ahora mismo. No solo los recursos que >> mencionaba el autor del hilo original, estoy pensando también en la >> traducción ya un poco obsoleta del tutorial de Python (2.6?), y también >> en la blogosfera python-hispana, que produce bastante poco contenido >> original en castellano (algunos blogueros del planet prefieren escribir >> en inglés, ¿por qué?). >> >> Un saludo a todos. >> >> [1]: http://pybonacci.wordpress.com/ >> [2]: http://wiki.python.org/moin/SpanishLanguage#pageinfo >> >> >> _______________________________________________ >> Python-es mailing list >> Python-es en python.org >> http://mail.python.org/mailman/listinfo/python-es >> FAQ: http://python-es-faq.wikidot.com/ >> > > Después de esta magnífica respuesta del compañero Juan Luís, poco queda > que añadir. > > En cuanto a fomentar los recursos en nuestro idioma, el que me conozca > un poco sabe perfectamente que no soy precisamente de los que no > impulsan la difusión de Python en la comunidad hispana. > > De todos modos, Mariano, entiendo tu diatriba acerca de la situación que > te toca vivir: la falta de recursos, falta de tiempo, habilidades, etc. > > Entiendo que en ciertas partes somos más "afortunados" que en otros > lados, en teoría. Y digo en teoría porque vivo en un país en el que se > estudia el Inglés durante bastante tiempo en edad escolar, muchos lo han > estudiado durante muchos años y la gran mayoría no tiene ni puta idea > del idioma, un nivel deleznable del mismo. Y no es por falta de > recursos, créeme, al menos hasta hace poco. Sobran bibliotecas, > academias, escuelas oficiales de idiomas, etc... > > ¿Sabes como aprendí yo inglés? Con un diccionario Larousse de bolsillo > (unos 20? en el 92) y traduciendo canciones de grupos de Rock y Heavy > Metal (saque la mejor nota posible en la asignatura dos años seguidos > sin casi atender en clase). Te puedo asegurar que cuando llegué a > estudiar informática tenia un nivel más que suficiente para enfrentarme > a lecturas técnicas. Lo he ido mejorando mucho durante años con series > extranjeras, leyendo muchos libros en Inglés (técnicos y literatura) y > con el trabajo diario (no me quedaba otra). Lo que te quiero decir con > todo esto, es que si se quiere, se puede. Los medios disponibles no te > van a ahorrar el esfuerzo personal, nunca, en cambio el esfuerzo > personal puede suplir la ausencia de muchos medios. > > Y te digo más, en países donde la escasez de medios y la educación es > deficiente la necesidad de aprender Inglés es aún más imperiosa. Porque > es la lengua que te va a abrir innumerables puertas en el futuro, en la > educación, en el trabajo, ... ¿Por qué crees que la India se ha > convertido en uno de las países donde más se deriva el desarrollo de > aplicaciones a terceros? ¿Crees que son mejores programadores que los > demás? En absoluto, pero cuentan con la tremenda ventaja del idioma, > donde el Inglés es una lengua oficial (junto a la mano de obra barata, > por supuesto). Y han sabido aprovechar esa ventaja para vender su > "talento" y fomentarlo aún más. > > Pero es que esto ocurre en casi todas las ciencias e ingenierias, es la > lengua vehicular, como en su día fue el francés en otros ámbitos y > antiguamente el latín. Y ya no te digo nada en los negocios > internacionales. Si yo trabajando en España he tenido que "arreglarme" > hablando con personas en fránces, italiano, portugués y por supuesto > Inglés. Una cosa es fomentar tu idioma y otra es nadar contracorriente. > > A lo mejor, lo que mejor que puedes hacer con esos alumnos es ayudarles > a traducir esos mensajes de error (tampoco es para tanto) y enriquecer, > quien sabe si no incentivar, el conocimiento del inglés. Y los críos a > esa edad son como una esponja, lo absorben todo. > > Saludos > > _______________________________________________ > Python-es mailing list > Python-es en python.org > http://mail.python.org/mailman/listinfo/python-es > FAQ: http://python-es-faq.wikidot.com/ > -- Alexandro Colorado PPMC Apache OpenOffice http://es.openoffice.org From jza en oooes.org Wed Oct 31 18:41:40 2012 From: jza en oooes.org (Alexandro Colorado) Date: Wed, 31 Oct 2012 11:41:40 -0600 Subject: [Python-es] Mision de esta lista In-Reply-To: References: <2163020.3H3bVHb2Kx@cristina> <2516210.ASNfJpeoxN@cristina> <50911BAE.9000107@joedicastro.com> <1351694890.86420.YahooMailNeo@web163001.mail.bf1.yahoo.com> <50915D7F.7050704@joedicastro.com> Message-ID: En multimedia esta el proyecto de localizacion de Khan Academy donde estan traduciendo los videos de Khan, y este hizo ultimamente uno de Computer Science usando Python como lenguaje original: http://www.khanacademy.org/cs Proyecto hispano: http://www.youtube.com/user/KhanAcademyEspanol -- Alexandro Colorado PPMC Apache OpenOffice http://es.openoffice.org From joe en joedicastro.com Wed Oct 31 19:21:22 2012 From: joe en joedicastro.com (joe di castro) Date: Wed, 31 Oct 2012 19:21:22 +0100 Subject: [Python-es] Mision de esta lista In-Reply-To: References: <2163020.3H3bVHb2Kx@cristina> <2516210.ASNfJpeoxN@cristina> <50911BAE.9000107@joedicastro.com> <1351694890.86420.YahooMailNeo@web163001.mail.bf1.yahoo.com> <50915D7F.7050704@joedicastro.com> Message-ID: <50916C22.50705@joedicastro.com> On 31/10/12 18:38, Alexandro Colorado wrote: > Uff creo que esta discusion se ha ido por otro lado. Tambien creo que > muchos tienen puntos validos, sin embargo el software libre creo que > es de hacer, mas que de discutir. > > Personalemente pienso que mas de intentar probar que el compañero > Mariano no beneficiara a nadie. Creo que es mejor emplear este tiempo, > en crear o apoyar un proyecto de localizacion sobre Python > alternativo. > > Hacerse un screencast en Youtube que pueda explicar bien los conceptos > de python y hacer un video curso. O ayudar a traducir los libros que > mencione anteriormente. > > He visto que aun falta actualizarse las versiones hispanas de: > - Byte of Python > - How to think like a computer scientist > - Doma la serpiente > - Think Python > - Python no muerde, yo si > > Estos libros, los que tienen la fortuna de haber sido traducidos aun > estan en versiones antiguas para python 2.4 o incompletas. > > Creo que una mejor propuesta seria empujar estos proyectos. Yo por lo > menos puedo apuntar a las personas al SVN de Google Code donde existe: > http://code.google.com/p/swfk-es/ > Apenas han liberado la version 0.4 cuando la version actual en ingles > va por la 0.7 > http://code.google.com/p/swfk/ > > Lo mismo puedo hablar de Think like a computer scientist, donde esta > abandonado aun: > http://www.openbookproject.net/thinkcs/archive/python/thinkcspyesp3e_abandonado/ > Original: http://www.openbookproject.net/thinkcs/ > > Y finalmente Think Python donde esta en un repositorio de SourceForge > Original: http://www.greenteapress.com/thinkpython/ > Proyecto hispano: http://sourceforge.net/projects/httlcseifspa/ > Bueno, yo inicie esta bifurcación de la conversación, simplemente porque no entendía el objetivo de Mariano. Conversar, dialogar, discutir, llamado como queráis, ayuda a comprender, a compartir puntos de vista, a abrir la mente, a hacer amigos, a abrir colaboraciones, etc... Jamás en mi vida he visto un dialogo correcto y respetuoso que haya sido una perdida de tiempo, siempre he sacado algo de valor del mismo, aunque simplemente sea el no oxidar nuestras capacidades en la dialéctica. Hay quienes pues simplemente, pues prefieren no leer lo que no les interesa o que temen que el iniciar una conversación como esta acabará necesariamente como el rosario de la aurora. Diría más, algunos simplemente interpretan el simple hecho de llevar la contraria porque no se está de acuerdo, como un acto de agresividad, lo cual es bastante lamentable porque es algo que está degradando y acabando con el antiguo, noble y apreciado arte de la conversación. Con esto quiero decir, que en la elaboración de esta "película" no se ha matado a ningún gatito ni se ha hecho sufrir innecesariamente a ningún tipo de animal :) Todos hemos sido correctos y a parte de un intercambio de distintos puntos de vista no he visto nada "punible" ni nada merecedor de la tijera de la censura xD Y ciertamente, me entristece cada vez más, ver como se "teme" o "incordia" el realizar este tipo de intercambios intelectuales entre varias personas. Y hay grandes episodios de la historia epistolar que se han dato entre el intercambio de pareceres de algunos de los personajes más trascendentales de la humanidad, y son aún objeto de estudio. Hoy día parece que es imposible llevar a cabo grandes conversaciones que superen unos pocos bloques de menos de 140 caracteres. Ahora entiendo la motivación de Mariano, no la comparto, pero la entiendo. Yo puedo estar equivocado, por supuesto. Y mucho menos es mi propósito impedir de algún modo que Mariano consiga su objetivo, es muy libre de llevarlo a cabo, y es muy posible que encuentre a quien le apoye y lo ayude a conseguirlo. Eso es el SL. Desde luego, comprendo el incordio que mea culpa mediante, hemos creado al desviar el tema principal de tu conversación. Sobre el mismo, creo que sería una buena idea que iniciaras un hilo nuevo solicitando directamente la ayuda para unirse a grupos de traducción de material Python. Yo ahora mismo he cubierto mi cupo de ayuda a la comunidad hispana con los frentes que actualmente tengo abiertos, pero comparto en cierto punto tu inquietud y espero que consigas que te ayuden en la labor. Saludos varios de un "gran conversador" :) From s_48k en yahoo.com Wed Oct 31 19:21:17 2012 From: s_48k en yahoo.com (s_48k) Date: Wed, 31 Oct 2012 19:21:17 +0100 Subject: [Python-es] Mision de esta lista Message-ID: <615671.72308.bm@smtp112-mob.biz.mail.ukl.yahoo.com> Pues yo creo que la lista es el lugar para debatir de estos temas. Un lugar dedicado a python y con gente interesada y con conocimientos del tema. Yo personalmente siento que no he perdido ni un minuto de los que he dedicado a leer la lista. -----Mensaje original----- De: Alexandro Colorado Enviado: 31/10/2012 18:39 Para: La lista de python en castellano Asunto: Re: [Python-es] Mision de esta lista Uff creo que esta discusion se ha ido por otro lado. Tambien creo que muchos tienen puntos validos, sin embargo el software libre creo que es de hacer, mas que de discutir. Personalemente pienso que mas de intentar probar que el compañero Mariano no beneficiara a nadie. Creo que es mejor emplear este tiempo, en crear o apoyar un proyecto de localizacion sobre Python alternativo. Hacerse un screencast en Youtube que pueda explicar bien los conceptos de python y hacer un video curso. O ayudar a traducir los libros que mencione anteriormente. He visto que aun falta actualizarse las versiones hispanas de: - Byte of Python - How to think like a computer scientist - Doma la serpiente - Think Python - Python no muerde, yo si Estos libros, los que tienen la fortuna de haber sido traducidos aun estan en versiones antiguas para python 2.4 o incompletas. Creo que una mejor propuesta seria empujar estos proyectos. Yo por lo menos puedo apuntar a las personas al SVN de Google Code donde existe: http://code.google.com/p/swfk-es/ Apenas han liberado la version 0.4 cuando la version actual en ingles va por la 0.7 http://code.google.com/p/swfk/ Lo mismo puedo hablar de Think like a computer scientist, donde esta abandonado aun: http://www.openbookproject.net/thinkcs/archive/python/thinkcspyesp3e_abandonado/ Original: http://www.openbookproject.net/thinkcs/ Y finalmente Think Python donde esta en un repositorio de SourceForge Original: http://www.greenteapress.com/thinkpython/ Proyecto hispano: http://sourceforge.net/projects/httlcseifspa/ On 10/31/12, joe di castro wrote: > On 31/10/12 17:31, Juan Luis Cano Rodríguez wrote: >> Me gustaría a mí también aportar mis dos céntimos a la conversación. >> >> Antes de nada quiero puntualizar que yo también estoy trabajando por la >> comunidad Python en castellano[1][2], así que digamos que no soy >> simplemente un observador que critica sin saber. >> >> El comentario de Mariano sobre dejar de enseñar Python en las escuelas >> primarias por estar en inglés me parece fuera de lugar. De hecho, los >> niños deberían aprender inglés bastante antes que Python. Y, con el >> debido respeto, la foto queda muy bien pero son niños mirando un robot; >> para leer una traza de error, ya esté en tu lengua materna o no, hacen >> falta una serie de estructuras mentales que estos niños no tienen porqué >> tener aún. Hay compañeros míos de universidad (ingeniería aeronáutica) >> que no saben interpretar una traza de error. >> >> Por otro lado, los argumentos de que para hacerlo a medias o medio mal, >> mejor no lo hagamos, no me parecen bien tampoco. No se va a incluir nada >> en la CPython que esté a medias, de eso podemos estar seguros. En todo >> caso habrá que ver si es útil, factible, empezarlo y, eventualmente, >> terminarlo. >> >> Creo que estamos exagerando un poco las cosas: todos sabemos que no hace >> falta hablar como Shakespeare para programar. Es un subconjunto del >> inglés *muy* reducido el que hay que comprender para interpretar una >> traza de error, o para saber que _if_ es _si_. >> >> Estoy de acuerdo con lo que han dicho arriba de que aprender el lenguaje >> con recursos, tutoriales, libros en español es fabuloso y genial, pero >> que traducir el lenguaje en sí (o una parte como es este caso las trazas >> de error) nos situaría en una especie de gueto, nos aislaría. El ejemplo >> del taekwondo me parece muy ilustrativo, y seguro que hay muchos más. >> >> Y el tema de informar de errores también funciona a la inversa: no todos >> los programadores saben español. Y si me sale una traza en español e >> informo del error, ¿se supone que solo lo tienen que solucionar >> programadores hispanos? No tiene sentido. Compartir todos el mismo >> lenguaje nos ayuda a cooperar con la comunidad internacional, que es >> bastante mayor que la comunidad hispana. >> >> Y dicho todo esto, si hay gente que quiere emplear su tiempo en traducir >> las trazas no seré yo quien se enfade o les impida hacerlo, pero creo >> que hay cosas más importantes ahora mismo. No solo los recursos que >> mencionaba el autor del hilo original, estoy pensando también en la >> traducción ya un poco obsoleta del tutorial de Python (2.6?), y también >> en la blogosfera python-hispana, que produce bastante poco contenido >> original en castellano (algunos blogueros del planet prefieren escribir >> en inglés, ¿por qué?). >> >> Un saludo a todos. >> >> [1]: http://pybonacci.wordpress.com/ >> [2]: http://wiki.python.org/moin/SpanishLanguage#pageinfo >> >> >> _______________________________________________ >> Python-es mailing list >> Python-es en python.org >> http://mail.python.org/mailman/listinfo/python-es >> FAQ: http://python-es-faq.wikidot.com/ >> > > Después de esta magnífica respuesta del compañero Juan Luís, poco queda > que añadir. > > En cuanto a fomentar los recursos en nuestro idioma, el que me conozca > un poco sabe perfectamente que no soy precisamente de los que no > impulsan la difusión de Python en la comunidad hispana. > > De todos modos, Mariano, entiendo tu diatriba acerca de la situación que > te toca vivir: la falta de recursos, falta de tiempo, habilidades, etc. > > Entiendo que en ciertas partes somos más "afortunados" que en otros > lados, en teoría. Y digo en teoría porque vivo en un país en el que se > estudia el Inglés durante bastante tiempo en edad escolar, muchos lo han > estudiado durante muchos años y la gran mayoría no tiene ni puta idea > del idioma, un nivel deleznable del mismo. Y no es por falta de > recursos, créeme, al menos hasta hace poco. Sobran bibliotecas, > academias, escuelas oficiales de idiomas, etc... > > ¿Sabes como aprendí yo inglés? Con un diccionario Larousse de bolsillo > (unos 20? en el 92) y traduciendo canciones de grupos de Rock y Heavy > Metal (saque la mejor nota posible en la asignatura dos años seguidos > sin casi atender en clase). Te puedo asegurar que cuando llegué a > estudiar informática tenia un nivel más que suficiente para enfrentarme > a lecturas técnicas. Lo he ido mejorando mucho durante años con series > extranjeras, leyendo muchos libros en Inglés (técnicos y literatura) y > con el trabajo diario (no me quedaba otra). Lo que te quiero decir con > todo esto, es que si se quiere, se puede. Los medios disponibles no te > van a ahorrar el esfuerzo personal, nunca, en cambio el esfuerzo > personal puede suplir la ausencia de muchos medios. > > Y te digo más, en países donde la escasez de medios y la educación es > deficiente la necesidad de aprender Inglés es aún más imperiosa. Porque > es la lengua que te va a abrir innumerables puertas en el futuro, en la > educación, en el trabajo, ... ¿Por qué crees que la India se ha > convertido en uno de las países donde más se deriva el desarrollo de > aplicaciones a terceros? ¿Crees que son mejores programadores que los > demás? En absoluto, pero cuentan con la tremenda ventaja del idioma, > donde el Inglés es una lengua oficial (junto a la mano de obra barata, > por supuesto). Y han sabido aprovechar esa ventaja para vender su > "talento" y fomentarlo aún más. > > Pero es que esto ocurre en casi todas las ciencias e ingenierias, es la > lengua vehicular, como en su día fue el francés en otros ámbitos y > antiguamente el latín. Y ya no te digo nada en los negocios > internacionales. Si yo trabajando en España he tenido que "arreglarme" > hablando con personas en fránces, italiano, portugués y por supuesto > Inglés. Una cosa es fomentar tu idioma y otra es nadar contracorriente. > > A lo mejor, lo que mejor que puedes hacer con esos alumnos es ayudarles > a traducir esos mensajes de error (tampoco es para tanto) y enriquecer, > quien sabe si no incentivar, el conocimiento del inglés. Y los críos a > esa edad son como una esponja, lo absorben todo. > > Saludos > > _______________________________________________ > Python-es mailing list > Python-es en python.org > http://mail.python.org/mailman/listinfo/python-es > FAQ: http://python-es-faq.wikidot.com/ > -- Alexandro Colorado PPMC Apache OpenOffice http://es.openoffice.org _______________________________________________ Python-es mailing list Python-es en python.org http://mail.python.org/mailman/listinfo/python-es FAQ: http://python-es-faq.wikidot.com/ ------------ próxima parte ------------ Se ha borrado un adjunto en formato HTML... URL: